Kvpy Print

You might also like

Download as pdf or txt
Download as pdf or txt
You are on page 1of 407

CL MEDIA (P) LTD.

CL Media (P) Ltd.


A-45, Mohan Cooperative Industrial Area,
Near Mohan Estate Metro Station,
New Delhi - 110044
: G.K. Publications (P) Ltd.
A-45, Mohan Cooperative Industrial Area,
978-93-88426-67-1 Near Mohan Estate Metro Station,
New Delhi - 110044

-93-87444-84-3 For product information :


Visit or email to
PREFACE
7. A solid hemisphere is attached to the top of a
cylinder having the same radius as that of the
1. The number of pairs (a, b) of positive real numbers
cylinder. If the height of the cylinder were doubled
satisfying a4 + b4 < 1 and a2 + b2 > 1 is
(keeping both radii fixed), the volume of the entire
(a) 0 (b) 1 system would have increased by 50%. By what
(c) 2 (d) more than 2 percentage would the volume have increased if
2. The number of real roots of the polynomial the radii of the hemisphere and the cylinder were
equation x4 – x2 + 2x – 1 = 0 is doubled (keeping the height fixed)?

(a) 0 (b) 2 (a) 300% (b) 400%

(c) 3 (d) 4 (c) 500% (d) 600%


3. Suppose the sum of the first m terms of an 8. Consider a triangle PQR in which the relation
arithmetic progression is n and the sum of its first QR2 + PR2 = 5 PQ2 holds. Let G be the point of
intersection of medians PM and QN. Then
n terms is m, where m n. Then the sum of the
first (m + n) terms of the arithmetic progression is QGM is always
(a) less than 45°
(a) 1 – mn (b) mn – 5
(b) obtuse
(c) –(m + n) (d) m + n
(c) a right angle
4. Consider the following two statements:
(d) acute and larger than 45
I. Any pair of consistent linear equations in two
variables must have a unique solution. 9. Let a, b, c be the side - lengths of a triangle, and
l, m, n be the lengths of its medians. Put
II. There do not exist two consecutive integers.
l m n
the sum of whose squares is 365. K .Then as a, b, c vary, K can assume
a b c
Then every value in the interval
(a) both I and II are true 1 2 1 4
(a) , (b) ,
(b) both I and II are false 4 3 2 5
(c) I is true and II is false 3 4 5
(c) ,1 (d) ,
(d) I is false and II is true 4 5 4
5. The number of polynomials p(x) with integer 10. Let x0, y0 fixed real numbers such that x02 + y20 > 1.
coefficients such that the curve y = p(x) passes If x, y are arbitrary real numbers such that
through (2,2) and (4,5) is x2 y2 1 , then the minimum value of (x – x0)2
(a) 0 + (y – y0)2 is
(b) 1 2
(c) more than 1 but finite (a) x02 y02 1 (b) x02 + y02 –1
(d) infinite (c) (|x0| +|y0|–1)2 (d) (|x0| + |y0|)2 –1
6. The median of all 4 - digit numbers that are 11. Let PQR be a triangle in which PQ = 3. From the
divisible by 7 is vertex R, draw the altitude RS to meet PQ at S.
(a) 5497 Assume that RS 3 and PS = QR. Then PR
(b) 5498.5 equals

(c) 5499.5 (a) 5 (b) 6


(d) 5490 (c) 7 (d) 8
12. A 100 mark examination was administered to a
(c)
class of 50 students. Despite only integer marks
being given, the average score of the class was
47.5. Then, the maximum number of students who V0
could get marks more than the class average is
(a) 25 (b) 35
(c) 45 (d) 49
13. Let s be the sum of the digits of the number
0°C 10°C
152 × 518 in base 10. Then
(d)
(a) s < 6 (b) 6 s 140
(c) 140 s 148 (d) s 148
V0
14. Let PQR be an acute - angled triangle in which
PQ < QR. From the vertex Q draw the altitude
QQ1. the angle bisector QQ2 and the median QQ3,
with Q1, Q2, Q3 lying on PR. Then
(a) PQ1 < PQ2 < PQ3 (b) PQ2 < PQ1 < PQ3
0°C 10°C
(c) PQ1 < PQ3 < PQ2 (d) PQ3 < PQ1 < PQ2 17. A very large block of ice of the size of a volleyball
15. All the vertices of a rectangle are of the form court and of uniform thickness of 8 m is floating
(a,b) which a, b integers satisfying the equation on water. A person standing near its edge wishes
(a – 8)2 – (b – 7)2 = 5. Then the perimeter of the
to fetch a bucketful of water using a rope. The
rectangle is
smallest length of rope required for this is about
(a) 20 (b) 22
(a) 3.6 m (b) 1.8 m
(c) 24 (d) 26
(c) 0.9 m (d) 0.4 m
18. A box filled with water has a small hole on its side
16. A block of wood is floating on water at 0°C with near the bottom. It is dropped from the top of a tower.
volume V0 above water. When the temperature As it falls. a camera attached on the side of the box
of water increases from 0 to 10°C, the change in records the shape of the water stream coming out of
the volume of the block that is above water is
the hole. The resulting video will show
best described schematically by the graph
(a) the water coming down forming a parabolic
(a) stream.
(b) the water going up forming a parabolic stream.
V0 (c) the water coming out in a straight line.
(d) no water coming out.
19. An earthen pitcher used in summer cools water in
it essentially by evaporation of water from its
0°C 10°C porous surface. If a pitcher carries 4 kg of water
and the rate of evaporation is 20 g per hour,
(b) temperature of water in it decreases by T in two
hours. The value of T is close to (ratio of latent of
V0 evaporation to specific heat of water is 540°C)
(a) 2.7°C
(b) 4.2°C
(c) 5.4°C
0°C 10°C (d) 10.8°C
20. Two plane mirrors are kept on a horizontal table (b) The lunar eclipse would occur roughly every
making an angle with each other as shown month if the orbits of earth and moon were
schematically in the figure. The angle is such perfectly coplanar.
that any ray of light reflected after striking both (c) The moon appears red during the eclipse
the mirrors returns parallel to its incident path. because the blue light is absorbed in earth’s
For this to happen, the value of should be atmosphere and red is transmitted.
(d) A lunar eclipse can occur only on a full moon
day.
25. Many exoplanets have been discovered by the
transit method, wherein one monitors a dip in
the intensity of the parent star as the exoplanet
moves in front of it. The exoplanet has radius R
and the parent star has radius 100 R. If I0 is the
intensity observed on earth due to the parent star,
then as the exoplanet transits,
(a) the minimum observed intensity of the parent
star is 0.9 I0.
(a) 30° (b) 45° (b) the minimum observed
(c) 60° (d) 90° (c) the minimum observed intensity of the parent
21. A certain liquid has a melting point of – 50° C and slar 0.999 I0
a boiling point of 150° C.A thermometer is designed (d) the minimum observed intensity of the parent
with this liquid and its melting and boiling points star is 0.9999 I0.
are designated as 0° L and 100° L. The melting 26. A steady current I is set up in a wire whose cross
and boiling points of water on this scale are - sectional area decreases in the direction of the
flow of the current. Then, as we examine the
(a) 25° L and 75° L, respectively.
narrowing region
(b) 0° L and 100° L, respectively. (a) the current density decreases in value.
(c) 20° L and 70° L, respectively. (b) the magnitude of the electric field increases.
(d) 30° L and 80° L, respectively. (c) the current density remains constant.
22. One can define an alpha - Volt ( V) to be the (d) the average speed of the moving charges
energy acquired by an particle when it is remains constant.
accelerated by a potential of 1 Volt. For this 27. Select the correct statement about rainbow:
problem you may take a proton to be 2000 times (a) We can see a rainbow in the western sky in
heavier than an electron. Then the late afternoon.
1eV (b) The double rainbow has red on the inside and
(a) 1 V = (b) 1 V = 2 eV violet on the outside.
4000
(c) 1 V = 8000 eV (d) 1 V = 1 eV (c) A rainbow has an arc shape since the earth is
round.
23. In a particle accelerator, a current of 500 A is
carried by a proton beam in which each proton (d) A rainbow on the moon is violet on the inside
and red on the outside.
has speed of 3x107 m/s. The cross sectional area
of the beam is 1.50 mm2. The charge density in 28. Remote sensing satellites move in an orbit that
this beam in Coulomb/m3 is close to is at an average height of about 500 km from the
surface of the earth. The camera onboard one
(a) 10–8 (b) 10–7 such satellite has a screen of area A on which the
(c) 10–6 (d) 10–5 images captured by it are formed. If the focal
24. Which of the following is NOT true about the total length of the camera lens is 50 cm, then the
terrestrial area that can be observed from the
lunar eclipse?
satellite is close to
(a) A lunar eclipse can occur on a new moon and
(a) 2 × 103 A (b) 106 A
full moon day.
(c) 1012 A (d) 4 × 1012 A
29. Letters, A, B, C and D are written on a cardboard (c)
as shown in the picture.
45

A V(m/s)

D B
C 0 0
Height (m) 101
The cardboard is kept at a suitable distance behind (d)
a transparent empty glass of cylindrical shape. If
45
the glass is now filled with water, one sees an
inverted image of the pattern on the cardboard
V(m/s)
when looking through the glass. Ignoring
magnification effects, the image would appear as
0 0 101
A A Height (m)

(a) B D (b) D B
31. The number of water molecules in 250 mL of
C
C
water is closest to

C [Given: Density of water is 1.0 g mL–1; Avogadro’s


A number = 6.023 × 1023]
B D (a) 83.6 × 1023 (b) 13.9 × 1023
(c) (d)
(c) 1.5 × 1023 (d) 33.6 × 1023
B D
A C
32. Among the following the correct statement is
(a) pH decreases when solid ammonium chloride
30. If a ball is thrown at a velocity of 45 m/s in vertical is added to a dilute aqueous solution of NH3
upward direction, then what would be the velocity
(b) pH decreases when solid sodium acetate is
profile as function of height? Assume g = 10 m/s2. added to a dilute aqueous solution of acetic
(a) acid
(c) pH decreases when solid NaCl is added to a
45 dilute aqueous solution of NaOH
(d) pH decreases when solid sodium oxalate is
V(m/s) added to a dilute aqueous solution of oxalic acid
33. The solubility of BaSO4 in pure water (in g L–1) is
closest to
[Given: K sp for BaSO 4 is 1.0 × 10 –10 at 25°C.
0 0 101
Height (m) Molecular weight of BaSO4 is 233 g mol–1]
(a) 1.0 × 10–5 (b) 1.0 × 10–3
(b) (c) 2.3 × 10–5 (d) 2.3 × 10–3
34. Among the following, the INCORRECT
45 statement is
(a) No two electrons in an atom can have the
V(m/s) same set of four quantum numbers
(b) The maximum number of electrons in the shell
with principal quantum number, n, is equal
0 0 to n2 + 2
Height (m) 101
(c) Electrons in an orbital must have opposite spin
(d) In the ground state, atomic orbitals are filled
in the order of their increasing energies
35. A container of volume 2.24 L can withstand a that reacts most readily with AgNO3 to give a
maximum pressure of 2 atm at 298 K before precipitate is
exploding. The maximum amount of nitrogen (in g)
(a) Cla (b) Clb
that can be safely put in this container at this
temperature is closest to (c) Clc (d) Cld
(a) 2.8 (b) 5.6 40. Among the following sets, the most stable ionic
(c) 1.4 (d) 4.2 species are
36. The compound shown below
(a) and
O NO2

(b) and

can be readily prepared by Friedel - Crafts reaction


between and
(c)
(a) benzene and 2 - nitrobenzoyl chloride
(b) benzyl chloride and nitrobenzene
(c) nitrobenzene and benzoyl chloride (d) and
(d) benzene and 2 - nitrobenzyl chloride
37. The correct statement about the following 41. The correct order of energy of 2 s orbitals in H,
Li, Na and K, is
compounds
(a) K < Na < Li < H (b) Na < Li < K < H
(c) Na < K < H < Li (d) H < Na < Li < K
Br Br 42. The hybridization of xenon atom in XeF4 is
X Y (a) sp3 (b) dsp2
is (c) sp3 d2 (d) d2 sp3
(a) both are chiral 43. The formal oxidation numbers of Cr and Cl in the
(b) both are achiral ions Cr2 O72– and ClO3–, respectively, are

(c) X is chiral and Y is achiral (a) +6 and +7 (b) +7 and +5


(c) +6 and +5 (d) +8 and +7
(d) X is achiral and Y is chiral
44. A filter paper soaked in salt X turns brown when
38. The most acidic proton and the strongest
exposed to HNO3 vapor. The salt X is
nucleophilic nitrogen in the following compound
(a) KCl (b) KBr
O
(c) KI (d) K2 SO4
b c CH3 45. The role of haemoglobin is to
N N
H H (a) store oxygen in muscles
a
N (b) transport oxygen to different parts of the body
H (c) convert CO to CO2
respectively, are (d) convert CO2 into carbonic acid
(a) Na - H; Nb (b) Nb - H; Nc
(c) Na - H; No (d) Nc - H; Na
46. Which ONE of the following molecules is a
39. The chlorine atom of the following compound
secondary metabolite?
c
Cl (a) Ethanol (b) Lactate
b (c) Penicillin (d) Citric Acid
Cld Cl 47. Lecithin is a
(a) carbohydrate (b) phospholipid
a
O Cl (c) nucleoside (d) protein
48. The water potential ( P) of pure water at standard 55. Which ONE of the following statements is TRUE
temperature and atmospheric pressure is about trypsinogen?
(a) 0 (b) 0.5 (a) It is activated by enterokinase
(c) 1.0 (d) 2.0 (b) It is activated by renin
49. Action potential in neurons is generated by a rapid (c) It is activated by pepsin
influx of (d) It does not need activation
(a) chloride ions (b) potassium ions 56. Which ONE of the following organisms respire
(c) calcium ions (d) sodium ions through the skin?
(a) Blue whale (b) Salamander
50. Erythropoietin is produced by
(c) Platypus (d) Peacock
(a) heart (b) kidney
57. Which ONE of the following human cells lacks a
(c) bone marrow (d) adrenal gland
nucleus?
51. Tendrils are modifications of
(a) Neutrophil
(a) stem or leaf (b) stem only (b) Neuron
(c) leaf only (d) aerial roots only (c) Mature erythrocyte
52. Which ONE of the following combinations of (d) Keratinocyte
biomolecules is present in the ribosomes?
58. The first enzyme that the food encounters in
(a) RNA, DNA and protein human digestive system is
(b) RNA, lipids and DNA (a) pepsin (b) trypsin
(c) RNA and protein (c) chymotrypsin (d) amylase
(d) RNA and DNA 59. Glycoproteins are formed in which ONE of the
53. Which ONE of the following proteins does NOT following organelles?
play a role in skeletal muscle contraction? (a) Peroxisome (b) Lysosome
(a) Actin (b) Myosin (c) Golgi apparatus (d) Mitochondria
(c) Troponin (d) Microtubule 60. An example of nastic movement (external
stimulus - dependent movement) in plants is
54. Which ONE of the following reactions is catalyzed
by high - energy ultraviolet radiation in the (a) folding - up of the leaves of Mimosa pudica
stratosphere? (b) climbing of tendrils
(a) O2 + O O3 (b) O2 O+O (c) growth of roots from seeds
(c) O3 + O3 3O2 (d) O + O O2 (d) growth of pollen tube towards the ovule

63. The number of solid cones with integer radius


and integer height each having its volume
61. What is the sum of all natural numbers n such numerically equal to its total surface area is
that the product of the digits of n (in base 10) is
(a) 0 (b) 1
equal to is equal to n2 – 10 n – 36?
(c) 2 (d) infinite
(a) 12 (b) 13
64. Let ABCD be a square. An arc of a circle with A
(c) 124 (d) 2612 as center and AB as radius is drawn inside the
62. Let m (respectively, n) be the number of 5 - digit square joining the points B and D. Points P on
integers obtained by using the digits 1, 2, 3, 4, 5 AB, S on AD, Q and R on arc BD are taken such
with repetitions (respectively, without repetitions) that PQRS is a square. Further suppose that PQ
such that the sum of any two adjacent digits is areaPQRS
and RS are parallel to AC. Then is
m areaABCD
odd. Then is equal to
n 1 1
(a) (b)
(a) 9 (b) 12 8 5
(c) 15 (d) 18 1 2
(c) (d)
4 5
65. Suppose ABCD is a trapezium whose sides and 2
k x a
height are integers and AB is parallel to CD. If 70. A potential is given by V(x) = for x < 0
2
the area of ABCD is 12 and the sides are distinct,
2
then |AB – CD| k x a
and V(x) = for x > 0.
(a) is 2 2
The schematic variation of oscillation period (T)
(b) is 4
for a particle performing periodic motion in this
(c) is 8 potential as a function of its energy E is:
(d) cannot be determined from the data (a) T

66. A coffee maker makes coffee by passing steam


through a mixture of coffee powder, milk and
water. If the steam is mixed at the rate of 50 g E
per minute in a mug containing 500 g of mixture,
then it takes about to seconds to make coffee at (b) T
70 °C when the initial temperature of the mixture
is 25 °C, The value of t0 is close to (ratio of latent
heat of evaporation to specific heat of water is
540 °C and specific heat of the mixture can be
taken to be the same as that of water) E
(a) 30 (b) 45
(c) T
(c) 60 (d) 90
67. A person in front of a mountain is beating a drum
at the rate of 40 per minute and hears no distinct
echo. If the person moves 90 m closer to the
mountain, he has to beat the drum at 60 per
minute to not hear any distinct echo. The speed E
of sound is
(d) T
(a) 320 ms-1 (b) 340 ms-1
(c) 360 ms-1 (d) 380 ms-1
68. A glass beaker is filled with water up to 5 cm. It
is kept on top of a 2 cm thick glass slab. When a
coin at the bottom of the glass slab is viewed at
the normal incidence from above the beaker, its E
apparent depth from the water surface is d cm.
Value of d is close to (the refractive indices of
water and glass are 1.33 and 1.50, respectively)
71. Among the following, the species with identical
(a) 2.5 (b) 5.1 bond order are
(c) 3.7 (d) 6.0 (a) CO and O22– (b) O1– and CO
69. A proton of mass m and charge e is projected from (c) O22– and B2 (d) CO and N2+
a very large distance towards an particle with
72. The quantity of heat (in J) required to raise the
velocity v. Initially, particle is at rest, but it is
temperature of 1.0 kg of ethanol from 293.45 K
free to move. If gravity is neglected, then the to the boiling point and then change the liquid to
minimum separation along the straight line of vapor at that temperature is closest to
their motion will be
[Given: Boiling point of ethanol 351.45 K
e2 5 e2 Specific heat capacity of liquid ethanol 2.44 J g–1 K–1
(a) 0
mv2 (b) mv2
0
4 4 Latent heat of vaporization of ethanol 855 J g–1]
2 e2 4 e2 (a) 1.42 × 102 (b) 9.97 × 102
(c) mv2 (d) mv2
4 0
4 0 (c) 1.42 × 105 (d) 9.97 × 105
73. A solution of 20.2 g of 1, 2 - dibromopropane in 77. A cross was carried out between two individuals
MeOH upon heating with excess Zn produces 3.58 g heterozygous for two pairs of genes was carried
of an unsaturated compound X. The yield (%) of out. Assuming segregation and independent
X is closest to assortment, the number of different genotypes
[Atomic weight of Br is 80] obtained respectively would be
(a) 18 (b) 85 (a) 4 and 9 (b) 6 and 3
(c) 89 (d) 30 (c) 9 and 4 (d) 11 and 4
74. The lower stability of ethyl anion compared to 78. If the H concentration of an aqueous solution is
+

methyl anion and the higher stability of ethyl 0.001 M, then the pOH of the solution would be
radical compared to methyl radical, respectively, (a) 0.001 (b) 0.999
are due to (c) 3 (d) 11
(a) + I effect of the methyl group in ethyl anion 79. Consider the following vision defects listed in
and p - orbital conjugation in ethyl radical Columns I & II and the corrective measures in
(b) - I effect of the methyl group in ethyl anion Column III. Choose the correct combination.
and * conjugation in ethyl radical
Column I Column II Column III
(c) + I effect of the methyl group group in both
P. Hypermetropia i. near - sightedness a. convex lens
cases
Q. Myopia ii. Far - sightedness b. concave lens
(d) + I effect of the methyl group in ethyl anion
and * conjugation in ethyl radical (a) P - ii - b
75. The F - Br - F bond angles in BrF5 and the Cl - P (b) Q - i - b
- Cl bond angles in PCl5, respectively, are (c) P - i - a
(a) identical in BrF5 but non - identical in PCl5 (d) Q - i - a
(b) identical in BrF5 and identical in PCl5 80. Which ONE of the following properties causes the
(c) non - identical in BrF5 but identical in PCl5 plant tendrils to coil around a bamboo stick?
(d) non - identical in BrF5 and non - identical in PCl5 (a) Tendril has spines
(b) The base of the tendril grows faster than the
tip
76. If the genotypes determining the blood groups of (c) Part of the tendril in contact with the bamboo
a couple are IA IO and IA IB, then the probability of
stick grows at a slower rate than the part away
their first child having type O blood is
from it.
(a) 0 (b) 0.25
(d) The tip of the tendril grows faster than the
(c) 0.50 (d) 0.75 base

1. (d) 2. (b) 3. (c) 4. (b) 5. (a) 6. (b) 7. (c) 8. (c) 9. (c) 10. (a)
11. (c) 12. (d) 13. (b) 14. (a) 15. (a) 16. (a) 17. (c) 18. (d) 19. (c) 20. (d)
21. (a) 22. (b) 23. (d) 24. (a) 25. (d) 26. (b) 27. (b) 28. (c) 29. (d) 30. (a)
31. (a) 32. (a) 33. (d) 34. (b) 35. (b) 36. (a) 37. (c) 38. (b) 39. (a) 40. (d)
41. (a) 42. (c) 43. (c) 44. (c) 45. (b) 46. (c) 47. (b) 48. (a) 49. (d) 50. (b)
51. (a) 52. (c) 53. (d) 54. (b) 55. (a) 56. (b) 57. (c) 58. (d) 59. (c) 60. (a)
61. (b) 62. (c) 63. (b) 64. (d) 65. (b) 66. (b) 67. (c) 68. (b) 69. (b) 70. (b)

71. (c) 72. (d) 73. (b) 74. (a) 75. (d) 76. (a) 77. (c) 78. (d) 79. (b) 80. (c)
5. No. polynomial with integral coefficient will pass
through (2, 2) (4, 5)
1. (x, y) R+ R+
a0 + 2a1 + 22a2 + ... + 2nan = 2
x2 + y2 > 1, x4 + y4 < 1
a0 + 4a1 + 42a2 + ... + 4nan = 5
x4 + y4 < 1 x < 1, y < 1
Pair has no integral solution.
y
6. Total 4 digit number divisible by 7 = {1001, 1008,
–––, 9996}
9996 = 1001 + (n – 1) 7
x n = 1286
643th term in Br2 form m middle
T643 = 5495, T644 = 5495 + 7
Median = 5495 + 3.5 = 5498.5
In shaded region we can find infinite points 2 3
7. Let V = r2h r
3 1 3 3
, just satisfies x 2 + y 2 = 1, ,0.5
2 2 2 3V 2 2 3
Given, = r 2h r
2 3
satisfies both in equations.
2. f(x) = x4 – x2 + 2x – 1, 2r
Solving h =
3
f( ) , f(– )
4 3
f(– 1) = – 1 V= r
3
g(x) = f (x) = 4x3 – 2x + 1
2 2 24 r 3
g (x) = 2(6x2 – 1) = 0 Later volume V2 = 2r h (2r)3 =
3 3
x= 1 V2 V
6 100% = 500%
V
1 1 9. Minimum distance will be along common normal.
g g 0
6 6
g(x) has one real root which gives only one A
extremem of f(x), and f(– 1) is negative so f(x) cuts B
x-axis twice.
m
3. 2a (m 1)d n ...(i)
2
n A(x0, y0) B(x, y)
2a (n 1)d m ...(ii)
2
AB2 = x02 y02 1
m n
Sm n 2a m n 1 d 10. R
2
Solving (i) & (ii) we get x
3
Sm + n = – (m + n)
4. x + y = 1 P x Q
S 3–x
2x + 2y = 2 2
x2 = 3 (3 x) 2
This pair has infinite solution.
x2 + (x + 1)2 = 365 x=2
x(x + 1) = 14.13 2
PR = x2 3 7
x = 13
15. (a – 8)2 – (b – 7)2 = 5 2 –2 =
(a + b – 15) (a – b – 1) = 5.1
90
a b 15 5 a b 15 1 2
21. 0°L x1 x2 100°L
a b 1 1 (11,9)
a b 1 5 (11,5)

a b 15 5 a b 15 1 –50°c 0°c 100°c 150°c


a b 1 1 a b 15 5
(5,5) (5,9) x1 0 0 50 50
Side of triangle are {4, 6} 100 150 ( 50) 200
Perimeter = 2 · (4 + 6) = 20 x1 = 25° L
16. x 2 0 100 50 150
100 200 200
x2 = 75° L
Volume of wooden block is V and V0 is above water 22. 1 V = (2e) × 1 V = 2 eV
therefore, (V – V0) wg = V g 23. i = neAV
= q AV where q is the charge density
V – V0 = V
w 6
i 500 10 5
q 6 7
10
V0 V w
V V Av 1.5 10 3 10
w w 24. In lunar eclipse earth is in between moon and
Hence, graph will be, sun which can happen on full moon day. Therefore
(A) is incorrect.
2
Imin A a 100R R2
25. I 2
0.9999
V0 0 A 100R
26. Current remains same at every point along the
length. But the resistance increase as the wire gets
4°c 10° c narrow. Therefore potential difference increases for
0° c a given interval that is, E increases.
17. Length of rope required = height of ice block above 28.
water f

H 8 R
m 0.9m
9 9
r
18. Vefflax 2g H 2 g a H
D
If a = g Vefflax = 0 No water will come out
19. Heat loss in evaporation of mass dm = Ldm r f 50 10 2
10 3
Therefore mc T = Ldm R D 500
2
Ldm L dm A r2 r 6
T 10
mc C m A R2 R
A
20 10 3 2 A 6
106 Ampere
540 = 5.4°C 10
4
29. A cylinderial convex lens will invert the image
20. Net deviation, = 2 – 2
along its axis.
30. v2 = u2 – 2gh
31. PH2O = 1 g/ml.
250ml. of H2O = 250g of H2O 61. P(n) = n2 – 10n – 36
250 P (n) = 2n – 5 > 0
6.023 1023 molecules
18 n > 5, P(n) is increasing
= 83.65 × 1023 molecules
P(n) 0 n (n – 10) > 36 n 13
32. NH4Cl is acidic salt so on adding to basic NH3, it
makes it acidic by decreasing its ph. Since P(n) is increasing then P(n) 81
33. For BaSO4, KSP = S2 Now, n – 10n – 36 2
81
moles n – 10n – 45
2
0
and S K SP 10 10 10 5
litre 10 460
S = 10–5 × 233 gm/litre n
2
S = 2.33 × 10–3 gm/litre n 15
34. The max. no. of electrons in a shell = 2n2 For two digit only solution in P(13) = 3
W Suppose number is 3 digit
35. PV = nRT or PV RT
M
then, 0 P(n) 93
PMV 2 28 2.24
W 5.6 P(n) 729
RT 0.0821 298
36. COCl NO2 Also, P(n) P(100) > 729
NO2 O
Anhyd. m 33 22
+ 62. = 9
AlCl3 n 3 4
37. Y has two same ethyl groups so achiral. r 2h
63. = r2 r r2 h2
39. As Carbocation formed will be most srable. 3

40. rh = 3r 3 r2 h2
r 2 h2 9r 2 6r 2 h 9r 2 9 h2
r 2 h2 6h 9 h2
Aromatic
+ Now,
Aromatic
42. Xef4 Hybridisation is Sp3d2 h = f(r)
43. Oxidation no. of Cr is + 6 9h2
r2 2
Oxidation no. of Cl is + 5 h 6h
44. KBr + HNO3 KNO3 + NO + Br2 + H2O 9 h2
r 9
Salt(X) (Brown Coloured) h(h 6)
49. When a stimulus is applied at a site on the polarised 3h 3
membrane, the membrane at that site becomes r
h(h 6) 6
permeable is Na+. This leads to a rapid influx of 1
h
Na+.
Here, h 6 (h = 8)
50. Erythropoietin is produced by juxta glomerutar
cells of kidney. 3
r 6
55. Trypsinogen is an inactive enzyme secreted by 3
1
pantereas which is activated by the enzyme 4
enterokinase, secreted by intestinal mucosa. 66. Heat loss by steam = Heat gained by mixture
56. Amphibians respire through, gills, lungs and 50 × to × l + 50 × C× (100°– 70°)
through skin. Salamander is an amphibians.
= 500 ×C × (70 – 25°)
58. The food first ecounters the enzyme–Salivary
amylase from salivary glands in buccal cavity. L
50 to 30 500 45
That helps in converting starch into maltose. C
500 45 500 45 and speed has max at x = 0 given by
to min
50 540 30 50 570 1 1
mv 20 Ka 2 E
500 45 2 2
60 s
50 570 2E Ka 2
= 47 s v0
m m
67. Time difference between 2 beats
2
60 s Now, v0 = A = A
1.5 seconds T
40
2d Ka 2
2E
1.5 2 A m m
V T 2 K K 2a 2
v0 2E 2
Now in the second case m 2mE
v
60 d 90 2d 180
2 Hence, graph looks like
60 V V V
Nb Na
1.5V 180 71. Bond order =
1= 2
V V
By solving we get V = 360 m/s 2 10 8
B.O of O2 1
2
d1 d2 5 2
68. dapp = dapp 4 2
1 2 1.33 1.5 B.O of B2 1
2
= 5.08 cm
72. q = mC T
= 5.1 cm
q = 1000 × 2.44 × 58 = 141520J
2
1 2 1 2e Total heat required = 141520 + 855 × 1000
69. In COM frame, v
2 4 0 d = 996520J
4m m 4m = 9.97 × 105J
4m m 5 73. CH3 – CH – CH2(20.2g)
2
1 4m 2e
v2 d 5e2 / 4 0 mv
2
Br Br
2 5 4 0d
Zn (excess)
K 2 ( )CH3–CH = CH2(3.58g)
x a x 0
70. V 2 2 As 202g of 1, 2 dibromopropane gives = 42g
K 2 hydrocarbon
x a x 0
2 20.2gm of 1, 2 dibromopropane gives = 4.2g
Ka 2 hydrocarbon
min potential energy V 0 therefore
2 3.58
So% yield = 100 85.23%
Ka 2 4.2
minimum value of total energy (E) allowed
2 75. Bond angle in Brf5 is 84.8° and in PCL5, there are
Now motion is symmetric for x > 0 & X < 0, lets two types of angles i.e 90° and 120°
consider x > 0 77. Dihybrid cross according to mendel.

Ka 2
We can take X = x – a & E E T
2
Now max value of X occurs when V = E
1 2E
KA 2 E A
2 K 1 2
Ka E
2
(a) (0, 2] (b) (2, 4]
(c) (4, 6] (d) (6, 8]
a b
1. Suppose A is a real matrix with nonzero 6. The number of real solutions x of the equation
c d
1
entries, ad – bc = 0, and A2 = A. Then a + d equals cos2 x sin 2 x 2
cos2 x sec2 x is
(a) 1 (b) 2
1 x

(c) 3 (d) 4 (a) 0 (b) 1


2. On any given arc of positive length on the unit (c) 2 (d) infinite
circle |z| = 1 in the complex plane,
x2 y2
(a) there need not be any root of unity 7. Let 1, a b, be an ellipse with foci F1
a2 b2
(b) there lies exactly one root of unity
and F2. Let AO be its semi-minor axis, where O
(c) there are more than one but finitely many
is the centre of the ellipse. The lines AF1 and
roots of unity
AF2, when extended, cut the ellipse again at points
(d) there are infinitely many roots of unity B and C respectively. Suppose that the triangle
ABC is equilateral. Then the eccentricity of the
2. For 0 , four tangents are drawn at the ellipse is
2
four points (± 3 cos , ± 2 sin ) to the ellipse 1 1
(a) (b)
x 2 y2 2 3
1. If A ( ) denotes the area of the
9 4
1 1
quadrilateral formed by these four tangents, the (c) (d)
3 2
minimum value of A( ) is
8. Let a = cos 1° and b = sin 1°. We say that a real
(a) 21 (b) 24
number is algebraic if it is a root of a polynomial
(c) 27 (d) 30 with integer coefficients. Then
4. Let S x : cos x cos 2x 2 . Then (a) a is algebraic but b is not algebraic
(b) b is algebraic but a is not algebraic
(a) S = (c) both a and b are algebraic
(b) S is a non-empty finite set (d) neither a nor b is algebraic
9. A rectangle with its sides parallel to the x-axis
(c) S is an infinite proper subset of and y-axis is inscribed in the region bounded by
0
the curves y = x 2 – 4 and 2y = 4 – x 2. The
(d) S maximum possible area of such a rectangle is
0
closest to the integer
5. On a rectangular hyperbola x2 – y2 = a2, a > 0
(a) 10 (b) 9
three points A, B, C are taken as follows: A = (–a,
0); B and C are placed symmetrically with respect (c) 8 (d) 7
to the x-axis on the branch of the hyperbola not 10. Let f x x sin x , x . Then
containing A. Suppose that the triangle ABC is
equilateral. If the side-length of the triangle ABC (a) f is differentiable for all x, except at x = n ,
is ka, then k lies in the interval n = 1, 2, 3, .....
(b) f is differentiable for all x, except at x = n ,
1
n = ±1, ±2, ±3, ..... is . The blood test that identifies the disease
5
(c) f is differentiable for all x, except at x = n ,
n = 0, 1, 2, 3, ..... 4
gives the correct result with probability .
(d) f is differentiable for all x, except at x = n , 5
n = 0, ±1, ±2, ±3, ..... Suppose a person is chosen at random from a
group of 30 males and 20 females, and the blood
11. Let f : 1,1 be a function defined by test of that person is found to be positive. What
is the probability that the chosen person is a man?
x2 cos for x 0, 75 3
f x x (a) (b)
for x 0 107 5
0
15 3
The set of points where f is not differentiable is (c) (d)
19 10
(a) x 1,1 : x 0
16. The number of functions f : 0, 1 0, 1
2
(b) x 1,1 : x 0 or x ,n satisfying f x f y x y for all x, y in [0,
2n 1
1] is
2 (a) exactly 1
(c) x 1,1 : x ,n
2n 1 (b) exactly 2
(d) [–1, 1] (c) more than 2, but finite
(d) infinite
12. The value of the integral 1 sin 8x dx is 17. Suppose A is a 3 × 3 matrix consisting of integer
0
entries that are chosen at random from the set
(a) 0 (b) –1
{–1000, – 999, ...., 999, 1000}. Let P be the
(c) –2 (d) –3 probability that either A2 = – I or A is diagonal,
13. Let ln x denote the logarithm of x with respect to where I is the 3 × 3 identity matrix. Then
the base e. Let S be the set of all points
1 1
where the function ln (x – 1) is well-defined. Then (a) P (b) P
2
18
10 1018
the number of functions f : S that are

differentiable, satisfy f x ln x 2 1 for all 52 53 54


(c) P (d) P
x S and f(2) = 0, is 1018 1018 1018
(a) 0 (b) 1
18. Let xk be real numbers such that xk k4 k2 1
(c) 2 (d) infinite
2018
14. Let S be the set of real numbers p such that there for 1 k 2018. Denote N = k. Consider
k 1
is no nonzero continuous function
the following inequalities:
x
f : satisfying f t dt pf x for all 2018 2 2018
0
I. kxk N kxk2
x . Then S is k 1 k 1
(a) the empty set 2
2018 2018
(b) the set of all rational numbers kxk N k2 xk2
II.
(c) the set of all irrational numbers k 1 k 1

(d) the whole set Then


15. The probability of men getting a certain disease (a) both I and II are true
(b) I is true and II is false
1
is and that of women getting the same disease (c) I is false and II is true
2
(d) both I and II are false
19. Let x2 = 4ky,k > 0, be a parabola with vertex A. 24. An ideal gas undergoes change in its state from
Let BC be its latus rectum. An ellipse with centre the initial state I to the final state F via two
on BC touches the parabola at A, and cuts BC at possible paths as shown. Then
points D and E such that BD = DE = EC (B, D, E,
C in that order). The eccentricity of the ellipse is P
1 1
(a) (b)
2 3
2
5 3 1
(c) (d) I F
3 2
20. Let f: [0, 1] [–1, 1] and g : [–1, 1] [0, 2] be two
functions such that g is injective and g ° f : [0, 1]
[0, 2] is surjective. Then (a) there is no change in internal energy along
(a) f must be injective but need not be surjective path 1.
(b) f must be surjective but need not be injective (b) heat is not absorbed by the gas in both paths.
(c) f must be bijective (c) the temperature of the gas first increases and
then decreases for path 2.
(d) f must be a constant function
(d) work done by the gas is larger in path 1.
25. A thermally insulated rigid container of one litre
21. A table has a heavy circular top of radius 1 m and volume contains a diatomic ideal gas at room
mass 20 kg, placed on four light (considered temperature. A small paddle installed inside the
massless) legs placed symmetrically on its container is rotated from the outside such that
circumference. The maximum mass that can be the pressure rises by 10 5 Pa. The change in
kept anywhere on the table without toppling it is internal energy is close to
close to
(a) 0 J (b) 67 J
(a) 20 kg (b) 34 kg
(c) 150 J (d) 250 J
(c) 47 kg (d) 59 kg
26. In a Young’s double slit experiment the
22. Air (density ) is being blown on a soap film amplitudes of the two waves incident on the two
(surface tension T) by a pipe of radius R with its slits are A and 2A. If I0 is the maximum intensity,
opening right next to the film. The film is then the intensity at a spot on the screen where
deformed and a bubble detaches from the film the phase difference between the two interfering
when the shape of the deformed surface is a waves is .
hemisphere. Given that the dynamic pressure on
2 I0
1 2 (a) I0 cos (b) sin 2
the film due to the air blown at speed v is v , 2 3 2
2
the speed at which the bubble is formed is I0 I0
(c) 5 4 cos (d) 5 8 cos
9 9
T 2T
(a) (b) 27. Figures below show water flowing through a
R R
horizontal pipe from left to right. Note that the
pipe in the middle is narrower. Choose the most
4T 8T appropriate depiction of water levels in the
(c) (d)
R R vertical pipes.
23. For an ideal gas the internal energy is given by
5 PV
U C, where C is a constant. The
2
equation of the adibetes in the PV plane will be (a)
(a) P V = constant
5 7
(b) P V = constant
7 5

(c) P V = constant
3 5
(d) P5 V2 = constant
(a) (I) < (III) < (II) < (IV)
(b) (IV) < (II) < (I) < (III)
(c) (II) < (IV) < (III) < (I)
(b) (d) (IV) < (I) < (II) < (III)
30. The current is flowing along the path abcd of a
cube (shown to the left) produces a magnetic field
at the centre of cube of magnitude B. Dashed line
depicts the non-conducting part of the cube.

d d
(c)
a a

c c

b b

(d) Consider a cubical shape shown to the right which


is identical in size and shape to the left. If the
same current now flows in along the path daefgcd,
28. A plank is moving in a horizontal direction with a then the magnitude of magnetic field at the centre
constant acceleration aiˆ . A uniform rough cubical will be
block of side l rests on the plank, and is at rest (a) zero (b) 2B
relative to the plank.
(c) 3B (b) B
y
31. A thin metallic disc is rotating with constant
Block
a
angular velocity about a vertical axis that is
x
Plank perpendicular to its plane and passes through its
centre. The rotation causes the free electrons in
the disc to redistribute. Assume that there is no
l
Let the centre of mass of the block be at 0, external electric or magnetic field. Then
2
(a) a point on the rim of the disc is at a higher
g potential than the centre.
at a given instant. If a , then the normal
10 (b) a point on the rim of the disc is at a lower
reaction exerted by the plank on the block at that potential than the centre
instant acts at (c) a point on the rim of the disc is at the same
potential as the centre.
l
(a) 0, 0) (b) ,0 (d) the potential in the material has an extremum
20
between center and the rim
l l 32. One mole of a monatomic gas and one mole of a
(c) ,0 (d) ,0 diatomic gas are initially in the same state. Both
10 10
gases are expanded isothermally and then
29. Using the Heisenberg uncertainty principle, adiabatically such that they acquire the same final
arrange the following particles in the order of state. Choose the correct statement.
increasing lowest energy possible.
(a) work done by diatomic gas is more than that
(I) an electronic H2 molecule by monatomic gas.
(II) a H atom in a H2 molecule (b) work done by monatomic gas is more than
(III) a proton in the carbon nucleus that by diatomic gas.
(IV) a H2 molecule within a nanotube (c) work done by both the gases are equal.
(d) change in internal energies of both the gases
are equal.
33. An ideal gas is made to undergo the cyclic process 36. The graph shows the log of activity (log R) of a
shown in the figure below. Let W depict the work radioactive material as a function of time t in
done, U be the change in internal energy of the minutes.
gas and Q be the heat added to the gas. Sign of 10
each of these three quantities for the whole cycle
will be (0 refers to no change)
8
Pressure

Volume 2
0 5 10 15 20 25
(a) –, 0, – (b) +, 0, + time (min)
The half-life (in minutes) for the decay is closest
(c) 0, 0, 0 (d) +, +, +
to
34. Two balls of mass M and 2M are thrown
(a) 2.1 (b) 3.0
horizontally with the same initial velocity v0 from
top of a tall tower and experience a drag force of (c) 3.9 (d) 4.4
–kv (k > 0), where v is the instantaneous velocity. 37. The magnetic field is uniform for y > 0 and points
Then the plane. The magnetic field is uniform and
points out of the plane for y < 0. A proton denoted
by filled circle leaves y = 0 in the -y direction
v0
with some speed as shown below.

Which of the following best denotes the trajectory


of the proton.
(a) the heavier ball will hit the ground further
away than the lighter ball.
(a)
(b) the heavier ball will hit the ground closer than
the lighter ball.
(c) both balls will hit the ground at the same point.
(d) both balls will hit the ground at the same time.
(b)
35. Consider a glass cube slab of dielectric bound by
the planes x = 0, x = a; y = 0, y = b; z = 0, z = c;
with b > a > c. The slab is placed in air and has a
refractive index of n. The minimum value for n
such that all rays entering the dielectric at y = 0 (c)
reach y = b is
(a) 1 (b) 2
(c) 3 (d) 2
(d)
38. The Hitomi satellite recently observed the Lyman OH
alpha emission line (n = 2 to n = 1) of Hydrogen- Cl
like iron ion (atomic number of iron is 26) from and
(a)
the Perseus galaxy cluster. The wavelength of
the line is closest to
(a) 2 Å (b) 1 Å OH
(c) 50 Å (d) 10 Å l
and
39. Assume that the drag force on a football depends (b)
only on the density of the air, velocity of the ball
and the cross-sectional area of the ball. Balls of Cl
different sizes but the same density are dropped
in an air column. The terminal velocity reached OH
and
by balls of masses 250g and 125g are in the ratio: (c)
1 1
(a) (b) l
26 23
OH
1 2 and
(c) (d) (d)
22 23
40. An electrostatic field line leaves at an angle
from point charge q1 and connects with point 43. X and Y
charge – q2 at an angle (q1 and q2 are positive) H Cl
3 H Cl H3C H
(see figure below). If q2 = q and = 30°, then
2 1 Cl
H CH3
H3C CH3 Cl

+q1 –q2 X Y
are
(a) 0 30 (b) 30 (a) enantiomers
(b) diastereomers
(c) 30 60 (d) 60 90
(c) constitutional isomers
(d) conformers
41. The amount (in mol) of bromoform (CHBr 3) 44. The higher stabilities of tert-butyl cation over
produced when 1.0 mol of acetone reacts isopropyl cation, and trans-2-butene over propene,
completely with 1.0 mol of bromine in the respectively, are due to orbital interactions
presence of aqueous NaOH is involving

1 2 (a) and *
(a) (b)
3 3 (b) vacant p and *
(c) 1 (d) 2
(c) * and
42. The following compound
(d) vacant p and *
O 45. Benzaldehyde can be converted to benzyl alcohol
in concentrated aqueous NaOH solution using
(a) acetone
(b) acetaldehyde
(c) formic acid
can readily be prepared by Williamson ether (d) formaldehyde
synthesis by reaction between
46. The major product of the following reaction (a) BCl3 < BF3 < B(OMe)3 < B(NMe2)3
O (b) BF3 < BCl3 < B(OMe)3 < B(NMe2)3
NaBH4
(c) BCl3 < B (NMe2)3 < B(OMe)3 < BF3
CO2H
(d) BCl3 < BF3 < B (NMe2)3 < B(OMe)3
53. Consider the following statements about
is Langmuir isotherm:
O (i) The free gas and adsorbed gas are in dynamic
equilibrium
(a) CO2H
(ii) All adsorption sites are equivalent
(iii)The initially adsorbed layer can act as a
O substrate for further adsorption
OH (iv) The ability of a molecule to get adsorbed at a
(b) given site is independent of the occupation of
neighboring sites.
O The correct statements are
OH (a) (i), (ii), (iii) and (iv)
(c) (b) only (i), (ii), and (iv)
(d) only (i), (ii) and (iii)
54. Among the following, the plot that correctly
CO2H
(d) represents the conductometric titration of 0.05
M H2SO4 with 0.1 M NH4OH is
47. Among the following species, the H-X-H angle (X
= B, N or P) follows the order
(a) PH3 < NH3 < NH4+ < BF3
(b) NH3 < PH3 < NH4+ < BF3
(c) BF3 < PH3 < NH4+ < NH3 (a)
(d) BF3 < NH4+ < NH3 < PH3
48. The ionic radii of Na+, F–, O2–, N3– follow the order Volume of NH4OH
(a) O2– > F– > Na+ > N3–
(b) N3– > Na+ > F– > O2–
(c) N3– > O2– > F– > Na+
(d) Na+ > F– > O2– > N3–
(b)
49. The oxoacid of phosphorus having the strongest
reducing property is
Volume of NH4OH
(a) H3PO3 (b) H3PO2
(c) H3PO4 (d) H4P2O7
50. Among C, S and P, the element(s) that produce
(s) SO2 on reaction with hot conc. H2SO4 is/are
(a) only S (b) only C and S
(c)
(c) only S and P (d) C, S and P
51. The complex that can exhibit linkage isomerism Volume of NH4OH
is
(a) [Co(NH3)5 (H2O)]Cl3
(b) [Co(NH3)5 (NO2)]Cl2
(c) [Co(NH3)5 (NO3)](NO3)2
(d) [Co(NH3)5 Cl]SO4
(d)
52. The tendency of X in BX3 (X = F, Cl, OMe, NMe)
to form a bond with boron follows the order
Volume of NH4OH
55. The correct representation of wavelength-
intensity relationship of an ideal blackbody
radiation at two different temperatures T1 and T2
is

(a) P

T2 T2 > T1

(a) Intensity
V

Wavelength

(b) P
T2 T2 > T1

(b) Intensity
V
T1

Wavelength

(c) P
T2 > T1
T1

(c) Intensity

T2 V

Wavelength

(d) P
T1 T2 > T1

(d) Intensity

T2 V

57. A buffer solution can be prepared by mixing equal


Wavelength volumes of
(a) 0.2 M NH4OH and 0.1 M HCl
56. The pressure (P)-volume (V) isotherm of a van
der Walls gas, at the temperature at which it (b) 0.2 M NH4OH and 0.2 M HCl
undergoes gas to liquid transition, is correctly (c) 0.2 M NaOH and 0.1 M CH3COOH
represented by (d) 0.1 M NH4OH and 0.2 M HCl
58. The plot of total vapour pressure as a function of 60. The average energy of each hydrogen bond in A-
mole fraction of the components of an ideal T pair is x kcal mol–1 and that in G-C pair is y kcal
solution formed by mixing liquids X and Y is mol–1. Assuming that no other interaction exists
between the nucleotides, the approximate energy
required in kcal mol–1 to split the following double
stranded DNA into two single strands is
A–T–A–T–G–C–A–G

(a) T–A–T–A–C–G–T–C
[Each dashed line may represent more than one
0 hydrogen bond between the base pairs]
0 Mole fraction of X 1 (a) 10x + 9y (b) 5x + 3y
(c) 15x + 6y (d) 5x + 4.5 y

61. What is the maximum number of oxygen atoms


that a molecule of hemoglobin can bind?
(a) 2 (b) 4
(b) (c) 8 (d) 16
62. Bt toxin produced by Bacillus thuringiensis does
0 not kill the producer because the toxin is.
0 Mole fraction of X 1
(a) in an inactive protoxin form
(b) rapidly secreted outside
(c) inactivated by an antitoxin
(d) in unfolded form
63. An angiosperm was identified with its endosperm
of 6n. Assuming that this is a self-pollinating
species, which ONE of the following is the correct
(c)
ploidy of the parent?
(a) 3n (b) 4n
0
0 Mole fraction of X 1 (c) 6n (d) 8n
64. Which ONE of the following statements is TRUE
about viruses?
(a) All viruses possess a protein coat around its
genetic material at all stages of their life cycle
(b) All viruses contain RNA as genetic material
(c) All viruses contain DNA as genetic material
(d) (d) All viruses replicate only within the host cell
65. Mitochondrial cristae are infoldings of the
0 (a) outer membrane and they increase the surface
0 Mole fraction of X 1
area
59. On complete hydrogenation, natural rubber (b) outer membrane and they decrease the surface
produces area
(a) polyethylene (c) inner membrane and they increase the surface
(b) ethylene-propylene copolymer area
(c) polyvinyl chloride (d) inner membrane and they decrease the surface
area
(d) polypropylene
66. In biological nitrogen fixation, the enzyme
nitrogenase converts
(a) nitrate to nitrite
(b) atmospheric nitrogen to nitrite (d)
(c) nitrite to ammonia
(d) atmospheric nitrogen to ammonia
67. The graph below represents the absorption
spectrum of a major pigment contributing to 400 500 600 700
photosynthesis.
Wave length, nm
68. Which ONE of the following properties of normal
cell is lost during its transition to cancerous cell?
(a) Glutamine utilization
(b) Contact inhibition
(c) Glucose utilization
(d) Membrane fluidity
69. Which ONE of the following gases is produced
400 500 600 700 during fermentation by yeast?
Wave length, nm (a) CO2 (b) O2
Which ONE of the following best represents the (c) H2 (d) N2
photosynthetic efficiency of the pigment?
70. Serine proteases are called so because they
(a) require free serine for their activity
(b) cleave after serine residues in the substrate
(c) are inhibited by the presence of free serine
(a) (d) have a serine residue at their active site
71. The maximum number of genotypes of the pollens
produced by a tall pea plant with round, yellow
seeds of the genotype TtRrYY, if the three loci
400 500 600 700 are unlinked, would be
Wave length, nm (a) 1 (b) 2
(c) 4 (d) 8
72. Which ONE of the following statements is TRUE
with respect to human ovary?
(a) Estrogen is secreted by Graafian follicles and
progesterone by corpus luteum
(b) Estrogen is secreted by corpus luteum and
(b) progesterone by Graafian follicles
400 500 (c) Both estrogen and progesterone are secreted
600
by Graafian follicles
Wave length, nm
(d) Both estrogen and progesterone are secreted
by corpus luteum
73. Which ONE of the following statements is
INCORRECT with respect to human antibodies?
(a) They can neutralize microbes
(b) They are synthesised by T cells
(c) (c) They are made up of four polypeptide chains
(d) Milk contains antibodies
400 500 600 700
Wave length, nm
74. Concentration (%) of NaCl isotonic to human blood
is
(a) 0.085 – 0.09% (b) 1.7 – 1.8%
(c) 3.4 – 3.6% (d) 0.85 – 0.9% 81. Let R be a rectangle, C be a circle, and T be a
75. Which ONE of the following statements is TRUE triangle in the plane. The maximum possible
about the Golgi apparatus? number of points common to the perimeters of
(a) It is found only in animals R,C, and T is
(b) It is found only in prokaryotes (a) 3 (b) 4
(c) It modifies and targets proteins to the plasma (c) 5 (d) 6
membrane 82. The number of different possible values for the
(d) It is a site for ATP production sum x + y + z, where x, y, z are real numbers
76. Creutzfeldt Jakob Disease (CJD) is a transmissible such that x4 + 4y4 + 16z4 + 64 = 32xyz is
disease caused by a (a) 1 (b) 2
(a) virus (b) bacterium (c) 4 (d) 8
(c) fungus (d) misfolded protein 83. Let be a circle with diameter AB and centre O.
77. A researcher found petrified dinosaur faeces. Let be the tangent to at B. For each point M
Which ONE of the following is unlikely to be found on different from A, consider the tangent t at
in this fossil? M and let it intersect at P. Draw a line parallel
(a) Decayed conifer wood to AB through P intersecting OM at Q. The locus
(b) Bamboo of Q as M varies over is
(c) Cycad (a) An arc of a circle
(d) Giant fern (b) a parabola
78. Which ONE of the pairs of amino-acids contains (c) an arc of an ellipse
two chiral centres? (d) a branch of a hyperbola
(a) Isoleucine and threonine 84. The number of solutions x of the equation sin
(b) Leucine and valine (x + x2) – sin (x2) = sin x in the interval [2, 3] is
(c) Valine and isoleucine (a) 0 (b) 1
(d) Threonine and leucine (c) 2 (d) 3
79. In photosynthetic carbon fixation, which ONE of 85. The number of polynomials p : satisfying
the following reacts with CO2?
(a) Phosphoglycolate 1
p(0) = 0, p(x) > x2 for all x 0, and p 0 is
(b) 3-Phosphoglycerate 2
(c) Ribulose-1, 5-bisphosphate (a) 0
(d) Ribose-5-phosphate (b) 1
80. Match the diseases in column I with the roots of (c) more than 1, but finite
infection is column II. Choose the CORRECT (d) infinite
combination.
86. Suppose the limit
Column I Column II
P. Tuberculosis i. Contaminated food 1 1
L lim n n
dx
n 0
and water 1 x2
Q. Dysentry ii. Inhalation of aerosol
R. Filariasis iii. Contact via skin 1
exists and is larger than . Then
S. Syphilis iv. Sexual intercourse 2
v. Mosquito bite 1
(a) P-ii, Q-i, R-v, S-iv (a) L 2 (b) 2 < L < 3
2
(b) P-ii, Q-i, R-iii, S-v
(c) 3 < L < 4 (d) L 4
(c) P-i, Q-iii, R-v, S-iv
(d) P-ii, Q-iii, R-iv, S-v
87. Consider the set An of points (x, y) such that
Heat flow Electrostatics
0 x n, 0 y n where n, x, y are integers.
Let Sn be the set of all lines passing through at T r V r
least two distinct points form An. Suppose we J r E r
choose a line at random from Sn. Let Pn be the
probability that is tangent to the circle x2 + y2 = We exploit this equivalence to predict the rate
2 .
1 Q of total heat flowing by conduction from the
n 2 1 1 . Then the limit lim Pn is
n n
surfaces of spheres of varying radii, all maintained
(a) 0 (b) 1 .
at the same temperature. If Q Rn , where R is
1 1 the radius, then the value of n is
(c) (d)
2 (a) 2 (b) 1
(c) –1 (d) –2
88. Let f : 0, 1 be an injective continuous
92. An arrangement of spring, strings, pulley and
function that satisfies the condition masses is shown in the figure. The pulley and
–1 < f(0) < f (1) < 1. the strings are massless and M > m. The spring
Then the number of functions g : [–1, 1] [0, 1] is light with spring constant k. If the string
connecting m to the ground is detached, then
such that (g ° f) (x) = x for all x 0,1 is
immediately after detachment
(a) 0
(b) 1
(c) more than 1, but finite
(d) infinite
89. The maximum possible area bounded by the
parabola y x2 x 10 and a chord of the
M
parabola of length 1 is m
1 1
(a) (b)
12 6 (a) the magnitude of the acceleration of m is zero
and that of M is g.
1 1
(c) (d) (b) the magnitude of the acceleration of m is
3 2
(M – m) g/m and that of M is zero.
90. Suppose z is any root of 11z8 + 20iz7 + 10iz – 22 = 0,
(c) the accelerations of both masses are same.
2
where i 1. Then S = z z 1 satisfies (d) the elongation in the spring is (M – m)g/k.
(a) S 3 (b) 3 < S < 7 93. The potential due to an electrostatic charge
distribution is
(c) 7 S 13 (d) S 13
qe r
V r
4 0r
91. In steady state heat conduction, the equations
where is positive. The net charge within a
that determine the heat current J r [heat
flowing per unit time per unit area] and I
sphere centered at the origin and of radius is
temperature T r in space are exactly the same
2q 1
as those governing the electric field E r and (a) (b) 1 q
e e
electrostatic potential V r with the equivalence
given in the Table below: q 1
(c) (d) 1 q
e e
(c)
94. A wheel of radius R is trapped in a mud pit and Entropy
spinning. As the wheel is spinning, it splashes
mud blobs with initial speed u from various points
on its circumference. The maximum height from
the centre of the wheel, to which a mud blob can
reach is

u2 u2 gR2
(a) (b)
2g 2g 2u2 UI,i
UI
u 2
u2 Entropy
(c) (d) R
2g 2g
95. Two rods of copper and iron with the same cross
sectional area are joined at S and a steady current
I flows through the rods as shown in the figure.
S

I Cu Fe I
UII,i UII
(a) UI increases and UII decreases and the total
Choose the most appropriate representation of entropy remains the same.
charges accumulated near the junction S.
(b) UI decreases and UII increases and the total
S entropy remains the same.
+ –
I Cu Fe I (c) UI increases and UII decreases and the total
(a) + –
+ entropy increases.

(d) UI decreases and UII increases and the total
S entropy increases.
+ +
I Cu Fe I 97. The image of an object O due to reflection from
(b) + +
the surface of a lake is elongated due to the ripples
+ +
on the water surface caused by a light breeze.
S This is because the ripples act as tilted mirrors
as shown. Consider the case where O and the
I Cu Fe I observer E are at the same height above the
(c)
surface of the lake. If the maximum angle that
the ripples make with the horizontal is a, the
S angular of the image will be
– –
I Cu Fe I O E
(d) – –
– –
96. Graphs below show the entropy vs energy (U) of
two systems I and II at constant volume. The
B C
initial enrgies of the systems are indicated by
UI, i and UII, i, respectively. Graphs are drawn to
the same scale. The systems are then brought (a) (b)
2
into thermal contact with each other. Assume
that at all times the combined energy of the two (c) 2 (d) 4
systems remains constant, Choose the most 98. A spiral galaxy can be approximated as an
appropriate option indicating the energies of the infinitesimally thin disk of a uniform surface mass
two systems and the total entropy after they density (mass per unit area) located at z = 0. Two
achieve the equilibrium. stars A and B start from rest from heights 2z0
and z0 (z0 << radial extent of the disk), respectively,
and fall towards the disk, cross over to the other
side, and execute periodic oscillations. The ratio
of time periods of A and B is
101. For the electrochemical cell shown below
1 Pt|H 2 (P = 1 atm)|H + (aq., x M)||Cu 2+
(a) 2 (b) 2
2 (aq., 1.0M)|Cu (s)
1 the potential is 0.49 V at 298 K. The pH of the
(c) 1 (d) solution is closest to
22
99. Two mutually perpendicular infinitely long [Given : Standard reduction potential, E° for j
straight conductors carrying uniformly distributed Cu2
charges of linear densities 1 and 2 are positioned is 0.34V
Cu
at a distance r from each other. Gas constant, R is 8.31 J K–1 mol–1
Faraday constant, F is 9.65 × 104 J V–1 mol–1]
(a) 1.2 (b) 8.3
(c) 2.5 (d) 3.2
102. Consider the following reversible first-order
reaction of X at an initial concentration [X]0. The
values of the rate constants are kf = 2 s–1 and
kb = 1 s–1.
kf
X Y
kb
A plot of concentration of X and Y as function of
time is
Force between the conductors depends on r as
[X]0
1 1 [Y]eq
(a) (b) 2
r r
(c) r (d) r0
100. The graph below shows the variation of a force
(a)
(F) with time (t) on a body which is moving in a
straight line. Dependence of force on time is F [X]eq
tn. Initially body is at rest.
t
30 [X]0
[Y]eq

20

(b)
10
[X]eq

0 t
[X]0
0 1 2 3 4 5
t(s)
If the speed of the object is 2 m/s at 3 s, the speed [X]eq
at 4 s will be approximately (in m/s)
[Y]eq
(a) 2.5 (b) 6.5
(c)
(c) 7.8 (d) 3.1

t
[X]0 Ph
Ph
[X]eq
(a) (b)
(d) Me

[Y]eq Ph Ph

t
(c) (d)
103. Nitroglycerine (MW = 227.1) detonates according
to the following equation: Me OH
2C3H5(NO3)3 (1) 3 N2(g) + ½ O2(g) + 6 CO2(g) + 5
H2O(g) 108. Among the following reactions, a mixture of
diastereomers is produced from
The standard molar enthalpies of formation, Hf°
for all the compounds are given below: Me H HBr
(a)
Hf° [C3H5(NO3)3] = –364 kJ/mol
Hf° [CO2(g)] = –393.5 kJ/mol Me H H/Pt
2
Hf° [H2O (g)] = –241.8 kJ/mol
(b)
Hf° [N2 (g)] = 0 kJ/mol
Hf° [O2 (g)] = 0 kJ/mol
Me H HBr
The enthalpy change when 10 g of nitroglycerine
(c)
is detonated is ROOR, hv
(a) –100.5 kJ (b) –62.5 kJ Me H B2H6
(c) –80.3 kJ (d) –74.9 kJ (d)
H2O/NaOH
104. The heating of (NH4)2 Cr2O7 produces another 2

chromium compound along with N 2 gas. The 109. Reaction of phenol with NaOH followed by heating
change of the oxidation state of Cr in the reaction with CO2 under high pressure, and subsequent
is acidification gives compound X as the major
(a) +6 to +2 (b) +7 to +4 product, which can be purified by steam
distillation. When reacted with acetic anhydride
(c) +8 to +4 (c) +6 to +3
in the presence of a trace amount of conc. H2SO4,
105. The complex having the highest spin-only compound X produces Y as the major product.
magnetic moment is Compound Y is
(a) [Fe(CN)6[3– (b) [Fe(H2O)6]2+
O
(c) [MnF6]4– (d) [NiCl4]2–
OH O O
106. Among Ce(4f1 5d1 6s2), Nd (4f4 6s2), Eu (4f7 6s2) and O
Dy (4f10 6s2), the elements having highest and
(a) CO2H (b) O
lowest 3rd ionization energies, respectively, are
(a) Nd and Ce (b) Eu and Ce
(c) Eu and Dy (d) Dy and Nd
O
107. The major product of the following reaction
sequence OH
O
(i) B2H6
Ph
(ii) H2O2/NaOH
(c) (d) O
(iii) conc. H2SO4
Me CO2H O O
is
110. A tetrapeptide is made of naturally occurring
alanine, serine, glycine and valine. If the C-
terminal amino acid is alanine and the N-terminal
amino acid is chiral, the number of possible
sequences of the tetrapeptide is
(a) 12 (b) 8 Product
(c) 6 (d) 4
(ii)
Substrate
111. What is the probability that a human individual
would receive the entire haploid set of
Progress of the reaction
chromosomes from his/her grandfather?
23
1 1
(a) (b)
2 2

2 46
1 1
(c) (d)
2 2 Substrate

112. Which ONE among the following primer pairs (iii)


would amplify the fragment of DNA given below?
Product
5 -CTAGTCGTCGAT-(N)300-GACTGAGCTGAG
CTG-3 .
Progress of the reaction
3 -GATCAGCAGCTA-(N)300-CTGACTCGACTCGA
C-5 .
(a) 5 -CTAGTCGTCGAT-3 and
5 -GACTGAGCTGAGCTG-3
(b) 5 -CTGACTCGACTCGAC-3 and
5 -CTAGTCGTCGAT-3 Subtrate
(c) 5 -CTAGTCGTCGAT-3 and
(iv)
5 -CAGCTCAGCTCAGTC-3 Substrate
(d) 5 -CTAGTCGTCGAT-3 and Product

5 -GTCGAGTCGAGTCAG-3
Progress of the reaction
113. The following graphs with the solid and dotted
(a) (i) only (b) (iii) and (iv)
lines correspond to the reactions without and with
enzyme, respectively. Which of the following (c) (ii) only (d) (i) and (ii)
graph (s) correctly represent the concept of 114. A novel species with double stranded genetic
activation energy? material consists of 5 bases namely P, Q, R, S, T,
with percentages given below.

P Q R S T
Percentage 22 28 22 12 16

Based on the above information, which ONE of


the following inferences is NOT supported by the
Substrate
observations?
(i) (a) S base pairs with T, and Q base pairs with R
(b) S base pairs with Q, and T base pairs with Q
Product
(c) P base pairs with R, and S base pairs with Q
Progress of the reaction (d) P base pairs with R, and T base pairs with Q
115. How many different blood groups are possible in 119. Match the vitamins listed in Column I with their
a diploid species with ABCO blood grouping respective coenzyme forms in Column II. Choose
system involving IA, IB, IC and IO alleles (IO is the CORRECT combination.
recessive and others are co-dominant)? Column I Column II
(a) 4 (b) 6 P. Vitamin B1 i. Thiamine
(c) 7 (d) 8 pyrophosphate
116. Within the exponential phase of growth, if the Q. Vitamin B2 ii. Flavine adenine
initial surface area and the growth rate of a leaf
dinucleotide
are 10 mm2 and 0.015 mm2/hour respectively, the
area of the leaf after 4 days would range from R. Vitamin B6 iii. Methylcobalamin
(a) 10 to 12 mm2 (b) 20 to 24 mm2 S. Vitamin B12 iv. Coenzyme A
(c) 30 to 36 mm2 (d) 40 to 48 mm2 v. Pyridoxal phosphate
117. If the acidic, basic and hydrophobic residues of (a) P-v, Q-iii, R-i, S-iv
proteins are considered to be red, green and blue (b) P-iii, Q-iv, R-ii, S-i
in color, respectively, then a globular protein in (c) P-i, Q-ii, R-v, S-iii
aqueous solution would have (d) P-i, Q-iv, R-ii, S-iii
(a) red and blue on the surface and green at the 120. Two independent experiments related to
core photosynthesis were conducted– one with 18O–
(b) red and green on the surface and blue at the labelled water (experiment P), and the other with
core 14
C–labelled CO2 (experiment Q). Which ONE of
(c) blue on the surface and red and green at the the following options lists the first labelled
core products in experiments P and Q, respectively?
(d) blue and green on the surface and red at the (a) P : O2, Q : 3-Phosphoglycerate
core (b) P : 3-Phosphoglycerate, Q: NADPH
118. A lysosome vesicle of 1 µm diameter has an (c) P : O2, Q : ATP
internal pH of 5.0. The total number of H+ ions (d) P : 3-Phosphoglycerate,
inside this vesicle would range from
Q : 3-Phosphoglycerate
(a) 103 to 104
(b) 104 to 105
(c) 105 to 1010
(d) 1010 to 6.023 × 1023

1. (a) 2. (d) 3. (b) 4. (d) 5. (b) 6. (b) 7. (d) 8. (c) 9. (b) 10. (b)
11. (c) 12. (c) 13. (d) 14. (d) 15. (a) 16. (b) 17. (a) 18. (a) 19. (c) 20. (b)
21. (c) 22. (d) 23. (a) 24. (c) 25. (d) 26. (c) 27. (a) 28. (b) 29. (b) 30. (c)
31. (b) 32. (b) 33. (a) 34. (a) 35. (b) 36. (b) 37. (d) 38. (a) 39. (a) 40. (c)
41. (a) 42. (b) 43. (d) 44. (d) 45. (d) 46. (a) 47. (a) 48. (c) 49. (b) 50. (d)
51. (b) 52. (a) 53. (b) 54. (b) 55. (a) 56. (b) 57. (a) 58. (b) 59. (b) 60. (a)
61. (c) 62. (a) 63. (b) 64. (d) 65. (c) 66. (d) 67. (a) 68. (b) 69. (a) 70. (d)
71. (c) 72. (a) 73. (b) 74. (d) 75. (c) 76. (d) 77. (b) 78. (a) 79. (c) 80. (a)
81. (d) 82. (d) 83. (b) 84. (c) 85. (a) 86. (a) 87. (a) 88. (d) 89. (b) 90. (b)
91. (b) 92. (b) 93. (c) 94. (b) 95. (b) 96. (c) 97. (c) 98. (d) 99. (d) 100. (b)
101. (c) 102. (b) 103. (b) 104. (d) 105. (c) 106. (b) 107. (c) 108. (a) 109. (a) 110. (d)
111. (b) 112. (c) 113. (d) 114. (a) 115. (c) 116. (d) 117. (b) 118. (a) 119. (c) 120. (a)
y
A

30°
x F2
F1 O

B C

P(3cos, 2 sin)

O A

(0, 2)
D A

(–2, 0) (2, 0)
C B
(0,–4)
10. f(x) = |sin x| is not diffrentiable at x = n
f(x) = x|sin x|in differentiable at x = 0
h sinh 0
f (0 ) = lim 0
+
h 0 h
– ( h) sinh 0
f (0 ) = lim 0
h 0 h
11. f : [–1, 1] R
2
f(x) = x cos ; x 0, f(0) = 0
x
is differentiable at x = 0

h2 cos 0
h
f (0) = lim = lim h cos 0 18. Qradratic mean Arithemetic mean
h 0 h h 0 h
2 x12 x22 ... xn2 x1 x2 ... xn
at x = ; (n = 0)
3 n n
2 Now we apply then save for neighted mean
2
h cos 0
3 2 a1 x12 a2 x22 ... an xn2 a1 x1 a2 x2 ... an xn
h
f (2/3) = lim 3 a1 a2 ... an a1 a2 ... an
h 0 h Using the inequality we get (A)
4 2 4h 3
h cos
= lim 9 3 2 3h
h 0 h F
3 4 3
= lim h cos lim cos
h 0 3h 2 h 0 3 3h 2
D E
3 (–2K, K)B C(2K, K)
4 cos
3h 2
lim
h 0 9h
Last part is indeterminate. O

21. Table will start toppling first when m is kept at


the position shown
A
1
R 1
R 2
R
2 mg
2 Mg 3
1 4
At the instant when the table top is about to
topple, N1 & N2 become zero
Therefore torque due to the weights Mg & mg. 2 2
balance each other I0 I1 I2 3 KA

R 1 I0
Therefore, Mg mgR 1 = 9KA2 KA2 =
2 2 9
Now if phase difference is
M 20 20
m 48.3kg
2 1 1.414 1 0.414 I = I1 + I2 + 2 I1I2 cos
4T
22. Excess pressure in side = = KA2 + 4KA2 + 2 KA 2 4KA 2 cos
R
1 = 5KA2 + 4KA2 cos
2
Now pressure inside = PV = KA2 (5 + 4 cos )
2
1 4T I0
Bubble will form when PV2 = 5 4 cos
2 R 9
8T 27. Area of cross section decreases speed increases
V
PR
5PV 5
23. U C = nRT C
2 2
5
U nR T 3
2 1 2

5 v1 = v3 < v2
CV R i.e. gas is diatomic. Now according to Bernoullis theorem as ‘v’
2
increases ‘p’ decreases
7
r p1 = p3 > p2 therefore height of column in 2 will
5 be lower.
7
28. The block does not topple, that means net torque
Adiabatic is, PV 5 = const
on the block is zero. This happens because
or P5 V7 = const Normal shifts toward left. If we observe in the
24. A: isobaric process, frame of plank, FBD will look like
N
2
P ma
I 1 F

V A mg
U changes Wrong x
B: U > 0, W > 0 Q>0 Wrong
C: in path 2 the states initially go to higher –x
isotherms then lower correct D: W2 > W1 2
Wrong Now, balancing torque about A, we get

25. V remains constant l l


in a ma Nx mg , N mg
V P = nR T 2 2
gl al
5 5 5 l a 9l
Now U = nR T V P 10 3 10 5
x= 2 2 1
2 2 2 g 2 g 20
= 250 J l
26. I1 = KA2 where K is a constant If COM is at 0, l 2 , Normal acts at ,0
20
I2 = 4KA2
h 32. PV diagram will be as shown
29. p x =K
4 isothermal
K K P
p> p> adiabatic
x x
monoatomic
p2 K2
E = 2m 2
2m n diatomic
2
K
Now E1 V
10 2
2 10 me Since work done is area under P - V curve. work
done will be less in diatomic
K2 33. Since the cycle is counter clockwise,
E2
10 2
2 10 mp W < O, Q < O, U = 0

34. Drag force, F KV Fx KVx


K2
E3
15 2 Fy = – KVy
2 10 mp
vx
K2
E4
9 2
2 10 2m p
vy
E3 > E1 > E2 > E4
KVx KVy
30. Case 2 can be viewed as the following So ax = ,a y g
m m
dVx KVx
Now a x
+ + dt m
K
dVx K m
dt
dt Vx Vo e
Vx m
1 2 3
magnetic fields by this arrangement is, K kt
dx m
mVo m
B2 Voe X 1 e
dt K
B
As tower is tall , t ,
B1
X1 m1 V0 m 2 V0 m1
BNet B12 B2 2 B32 , B1 = B2 = B3 = B So / Heavier ball will
X2 K K m2

BNet 3B go further.
35. If all ray incident at face 1 have to reach face 2
31. (b) Fe = mer 2
then
= eE = mer 2

Y
a
(2)
R b
C E
(4) (3)
r
Now the force on e is towards the centre. (1) X
Therefore E should be radially outward. That i
means VC > VR
All rays refracted at 1 should under go TIR at 3 & 4 4
c always 10967700 26 2 3
Now + r = 90° ...
1.798 10 10 m 2A
= 90° – r ...(1)
39. Let the drag force F Pa Vb Ac
Therefore decreases as r increases we have to
[MLT–2] = [ML–3]a [LT–1]b [L2]c
ensure that > c , which corresponds to rmax
min
= Ma L–3a+b+2c T–b
which happens when i = 90°
a = 1, b = 2, c = 1
rmax = c F = PAV2
Therefore from (1) At terminal velocity mg = F PAV2 = mg
= 90° – c
mg
V
90° – c > c Ap

c < 45°
V1 m1 A2
Sin < Sin 45° Therefore
c V2 m2 A1
1 1 Now density is same
n 2
n 2 volume mass
1
dN 4 m1 r1
36. R N R(t) = R e– t
r13 2 23
dt o 3 m2 r2
4 3
log R = log Ro – t r2
3
Therefore slope of log R will give the value of .
2
A1 r12
4 10 6 1 n 1 23
From the graph, slope = m A2 r22
24 0 24 4
2 1 1
1 1 V1 3
2 2 23 26
4 4 V2
40.
t1 ln 2 0.693
2 4 0.693
1 q1 q2
4
3
q2 = q1
2.77 min 3 min 2
Therefore more field line will terminate at – q
37. than originating at q1
So the solid angle between the field lines will be
smaller at –q
plane angles will also be smaller at – q <
(Trivial) 30°
41. CH3COCH3 3Br2 4NaOH
1 2 1 1
38. RZ
n12 n 22
CH3COONa CHBr3 3H2O 3NaBr
1 1 1 1
10967700 262 For mole of Br2, Acetone required ml
12 22 3
1
3 So CHBr3 formed ml
= 10967700 × 262 × 3
4
44. Hyperconjugation involves – P overlap.
45.

CHO CHOH
2
C
50%
+ HCHO + HCOONa 81.
NaOH
T
46. NaBh4 don’t reduce – COOH group but reduces
Carbonyl group to secondary alcohol.
47. Angle of PH3 = 90° circle(c) & (T) intersect in 6 points
NH3 = 107° 6 points (d)
NH4+ = 109.5°
x4 4y 4 16z4 64 32xyz
BF3 = 120° 82.
48. The more negative charge on atoms of same
Apply AM GM on these 4 positive num-
period, the large size.
bers.
49. H3PO2 because of lowest oxidation state.
(x, y, z )
50. P + H2SO4 H3PO4 + SO2 + H2O
C + H2SO4 CO2 + 2SO2 + 2H2O x4 4y 4 16z4 64
(x4 × 4y4 × 16z4 × 64)1/4
S + H2SO4 3SO2 + 2H2O 4
51. [Co(NH3)5(NO2)]Cl2 x4 + 4y4 + 16z4 + 64 32xyz ...(ii)
and From (i) & (ii), (ii) is possible only
when x4 = 4y4 = 16z4 = 64
[Co(NH3)5(ONO)]Cl2
54. As we add base dropwise, the OH consumes H+ x 2 2 2 possible values
and form water so on adding NH4OH, NH4+ ions y= 2 possible values
2
are added in solution and each H+ is being replaced
slowly by NH4+ and hence conductance decreases. z= 2 2 possible values
57. (0.2 MNH4OH + 0.1 MHCl) Can behave as buffer x + y + z has 2 × 2 × 2 = 8 possible values
because HCl will acts as limiting reagent.
62. Bt toxin produced from Bacillus thuringiensis is 83. l
produced in protoxin form which gets activated P
M M
in the gilt of insect due to basic pH. M P
63. Endosperm is triploid. P
Then, the haploid cell will have A B, M
6n
= 2n. P
3 C M
M M P
Then, Ploidy of the parent is 4n. P
67. Most of the photosynthesis takes place in blue
and red region of visible spectrum of light the
wavelength of blue is near 400 nm and that of red Take M at different possitions of the circle. A
in near 600-700 nm. Therefore the graph in option parabola is obtained.
A represents the photosynthetic efficiency of the 84. sin (x + x2) – sin(x2) = sinx
pigment.
A B A B
73. Human antibodies are synthesised by B-cells. By sinA – sinB = 2cos sin
2 2
77. Bamboo is recent plant species.
x x x x
2cos x 2 sin 2sin cos 0
2 2 2 2
x x x 1
2sin cos x2 cos 0 85. p (0) = 0 at x = 0, p(x) has minimum.
2 2 2 2
p(0) = 0
A B B A
By cos A cos B 2sin sin y
2 2
y = x2
x x x2 x2
4sin sin sin 0
2 2 2 x

x
sin 0
2

x x2
n n I As p(x) > x2, p(x) = a4 x4 a6 x 6 ......
2 2
x = 2n
1
n = 1, x = 2 > 6 (as 3.14) & p´´ 0 p (x) = x + 4a4x3 + 6a6x5 + .....
2
no value of x [2, 3] is possible
or d 2
For values of x close to 0, p´(x) < x
dx
x x2
sin 0 For x values close to 0 but < 1, the graph of p(x)
2 rises slowly than the graph of x2 & at x = 0, both
+ve value 0. Hence for values of x near 0, p(x)
x x2 >/ x2.
n
2 no such possibility
x x2 2n 88. Now g(x), an inverse an of f(x), maps values of f(0)
for x = 2, x + x2 = 6 to f(1) back to 0 to 1.
for x = 3, x + x2 = 12
we need 2n
y(x)
between 6 & 12
1
n = 1, 2 > 6
f(1)
n = 2, 4 > 12
1 value x(y)
or 0 1
f(0)
x2 1
sin 0
2 g(x)

x2
n n I Now rotate the diagram by 90° & consider x-axis
2 and y-axis & vice versa.
x 2 2n g(x) can assume any value in [–1, f(0)) f(1), 1]]
For n = 1 infinite possibilities for g(x).
x2 = 2 6.28(approx) 2
1 39
2<x<3 89. y x2 x 10 x
2 4
for n = 2
x2 = 4 > 12 This parabola is obtained by shifting the parabola
x>3 39
0.5 units left & units upwards. Hence work
1 value 4
Hence 2 values [2, 3] in y = x2.
y dT
y = x2 = – K 4 r2
1 dr
2
(t, t ) (1/2, 1/4))
Q dr
x dT
4 K r2
1/2
o
Q dr
By integrating, dT
4 K r2
T r
1
By symmetry t =
2 Q
T
Shaded area = required area 4 Kr
1 Q = 4 KrT rn n=1
2 92. Kx = Mg = T2
1 1
x 2 dx
=2× 2 4 when T1 is cut, Net force on m is T2– mg
0

1 1 1 1 T2
2 2
8 8 3 12 6
m Kx
90. 11z8 + 20iz7 + 10iz – 22 = 0 ...(i)
T1
Let z = ix, x R M
f(x) 11x + 20x – 10x – 22 = 0
8 7
...(ii)
(ii) is an equation with real coefficients
[Net force on M is still zero, So am = 0]
f(0) = –22
= Mg – mg = (M – m)g
f(1) = –1
ma = (M – m)g
f(2) > 0
There lies at least a root between 1 & 2 M m
a g
which satisfies (ii) m
(i) has root ix, where x (1, 2) r
93. V qe 1 q in
4 0r 4 0 r
z ix x = lies between 1 & 2
q
1 z 2 putting, r = 1 we get qin =
e
2 94.
1 1 z z 1 z2 z 1 u 2 sin2
2 2g
3 z z 1 7 u R cos

3 5 7
91. Heat current, Height attained by a blob from centre is

u 2Sin 2
H R cos
2g
dr
dH u2
For max, =0 2Sin cos RSin = 0
d 2g
gR
dT Sin = 0 or cos = 2
Q = – KA
dr
Sin = 0 is neglected as it is horizontal. may or may not be constant) Also, Since slope of
1 is more than 2 therefor if U changes by same
gR
cos amount (say U) in both the systems, E1 > E2
u2
(A) If U1 increase by U & U2 decrease by U the
u2 Etotal = E1 + E1 + E2 – E2 = E + E1 – E2 > E
Hmax Sin 2 R cos
2g
So E increase WRONG
2 2 2
u g R gR
1 4
R (B) Similarly B is also WRONG
2g u u2
(C) CORRECT (as seen above)
u2 gR 2 gR 2 u2 gR 2
(D) If U1 decrease & U2 increases,
2g 2u 2 u2 2g 2u 2
E = E1 – E1 + E2 + E2 E – ( E1 – E2) < E
95. S Fe
So E decreases WRONG
97. Using circle concept in plane mirror
O E
Icu = IFe
Jcu = JFe
Now J E ( is conductivity)

cu Ecu Fe EFe I1 A
2 2
cu Fe I2
Ecu EFe combination, we can see that images formed by
If we consider a small volume around s enclosed the two mirrors will be rotated by 2 and
by two parallel circular area. I1 AI2 4 . O, I1, I2 will remain on the circle
EFe A Ecu A EFe Ecu A 0 with centre at A Now OA = I1A = I2A = r
net
If OA & AE are same then E also lies on the circle
q enc 0
I1 AI2 4
So positive charge will accumulate I1EI2 2
2 2
96. Since total energy of the two systems is constant,
Z
98. g Z 2 G 1
R Z2

U1, i

2 G as Z R
and g remain constant for both

H
2
U2, i t1 g H1 2 Z0
2
therefor t H H2 Z0
energy loss by one system will be equal to energy 2 2 2
gained by other And E = E1 + E2 at any instant (E g
99. At a distance x from the shortest distance,
34 34
X V 3 2 2 m
16m 32
r 2
x 2 x
44
Y Therefore V(4) 32
16 34
r
4
4
2 2 6.32m / s
3

Electric field due to 1 is given by 6.5m / s

2
2
E1 .0591 H
2
2 0 x r2 101. Ecell = Ecell log
n Cu 2
therefore force on a small element of 2 is

2 dx 2
dF = 2dx E1 , .0591 H
2 0 x2 r2 log
0.49 = 0.34 2 Cu 2
Component of dF along × axis will get cancelled
out, therefor Net force 2 is
2
0.15 2 H
1 2 dx r log
F net dF cos .0591 1
2 G x4r 2 x2 r2
0.3
[H+]2 = Antilog Anti log 5.076
1 2r dx 1 2r 1 1 2
.0591
2 0 x2 r2 2 0 r 2 0 [H+]2 = 8.39 × 10–6
3 3
1 2
H 8.39 10 2.89 10
So F i.e. independent of r
2 0
PH = – log [H+] = – log (2.89 × 10–3)
100. F tn = 3 – log 2.89 = 3 – 0.46 = 2.54
F = Kt n
103. H°f of reaction = [6 × (– 393.5) + 5 × (–241.8)
Now, F(2) = 2, F(4) = 16 KJ
+ 364 × 2]
K(2)n = 2 K(4)n = 16 mol
1 = – 2842 KJ/mol
By solving we get n = 3 & K
4 Now, for 2 moles of substance energy given =
– 2842 KJ
t3 F t3 OR by 454g of substance energy given = – 2842 KJ
F a
4 m 4m
2842
by lo g of substance energy given 10
v t 454
dv t3 t3
dv dt = – 62.5
dt 4m 4m
0 0
104. (NH4)2 Cr2O7 Cr2O3 + N2 + 4H2O
t4 105. (Mn F6)4– has 5 unpaired electrons so maximum
V
16m spin only magnetic moment.
107. Ph Ph Ph
(i) B2H6 Conc. H2SO4
(ii) H2O2, OH +
OH
Me Me Me

Ph Ph

+
Me Me

– +
109. OH O Na OH
COONa OH
NaOH CO2 COOH
H+
High Pressure

(X)

OCOCH3
COOH

Aspirin
(Y)

115. IA IB IC I0
IA IB AB ICI0 C
IA IC AC ICIC C
I I
B C
BC
I I
A 0
A
I I
A A
A
I I
B 0
B
I I
B B
B
In total, there are 7 blood groups formed.
III. P is a regular polygon if it is cyclic.
1. Suppose BC is a given line segment in the plane Then
and T is a scalene triangle. The number of points (a) I is true and I implies II
A in the plane such that the triangle with vertices (b) II is true
A,B,C (in some order) is similar to triangle T is
(c) III is false
(a) 4 (b) 6
(d) I and III are true
(c) 12 (d) 24
6. Consider the following statements. For any integer
2. The number of positive integers n in the set n,
{2,3,....,200} such that 1/n has a terminating decimal
I. n2 + 3 is never divisible by 17.
expansion is
II. n2 + 4 is never divisible by 17.
(a) 16 (b) 18
Then
(c) 40 (d) 100
(a) both I and II are true
3. If a,b,c are real numbers such that a + b + c = 0
and a2 + b2 + c2 = 1, then (3a + 5b – 8c)2 + (–8a + (b) both I and II are false
3b + 5c)2 + (5a – 8b + 3c)2 is equal to (c) I is false and II is true
(a) 49 (b) 98 (d) I is true and II is false
(c) 147 (d) 294 7. Let S be the set of all ordered pairs (x,y) of positive
4. Let ABC be a triangle and M be a point on side AC integers, with HCF (x,y) = 16 and LCM (x,y) =
closer to vertex C than A. Let N be a point on side 48000. The number of elements in S is
AB such that MN is parallel to BC and let P be a (a) 4 (b) 8
point on side BC such that MP is parallel to AB. (c) 16 (d) 32
If the area of the quadrilateral BNMP is equal 8. Consider the set A of natural numbers n whose
5 units digit is nonzero, such that if this units digit
to th of the area of triangle ABC, then the ratio is erased, then the resulting number divides n.
18
If K is the number of elements in the set A, then
AM/MC equals.
(a) K is infinite
(a) 5 (b) 6
(b) K is finite but K > 100
18 15 (c) 25 K 100
(c) (d)
5 2
(d) K < 25
5. Let n 4 be a positive integer and let 1 , 2 ,......, 9. There are exactly twelve sundays in the period
n
be the lengths of the sides of arbitrary n– sided from january 1 to march 31 in a certain year.
non-degenerate polygon P. Suppose Then the day corresponding to february 15 in that
year is
l1 l2 ln 1 ln
..... n. (a) Tuesday
l2 l3 ln l1
(b) Wednesday
Consider the following statements: (c) Thursday
I. The lengths of the sides of P are equal. (d) not possible to determine from the given data
II. The angles of P are equal.
10. Consider a three-digit number with the following y. If the average marks of all students is z%,
properties: the ratio of the number of girls to the total number
of students is
I. If its digits in units place and tens place are
interchanged, the number increases by 36; z x z y
II. If its digits in units place and hundreds place (a) (b)
y x y x
are interchanged, the number decreases by
198. Now suppose that the digits in tens place z y z x
and hundreds place are interchanged. Then (c) (d)
y x y x
the number.
(a) increases by 180
(b) decreases by 270 16. Particle used in the Rutherford’s scattering
(c) increases by 360 experiment to deduce the structure of atoms
(d) decreases by 540 (a) had atomic number 2 and were fully ionised.
11. Consider four triangles having sides (5,12,9), (b) had atomic number 2 and were neutral.
(5,12,11), (5,12,13) and (5,12,15). Among these, the (c) had atomic number 4 and were fully ionised.
triangle having maximum area has sides (d) had atomic number 4 and were neutral.
(a) (5,12,9) (b) (5,12,11) 17. The number of completely filled shells for the
(c) (5,12,13) (d) (5,12,15) 32
element 165 is
12. In a classroom, one-fifth of the boys leave the class
and the ratio of the remaining boys to girls is 2:3. (a) 1 (b) 2
If further 44 girls leave the class, the ratio of boys (c) 3 (d) 4
to girls is 5 : 2. How many more boys should leave
18. In an experiment on simple pendulum to
the class so that the number of boys equals that
determine the acceleration due to gravity, a
of girls?
student measures the length of the thread as 632
(a) 16 (b) 24 cm and diameter of the pendulum bob as 2.256
(c) 30 (d) 36 cm. The student should take the length of the
13. Let X,Y,Z be respectively the areas of a regular pendulum to be
pentagon, regular hexagon and regular heptagon (a) 64.328 cm (b) 64.36 cm
which are inscribed in a circle of radius 1. Then (c) 65.456 cm (d) 65.5 cm
X Y Z 19. A uniform metallic wire of lenght L is mounted
(a) and X < Y < Z
5 6 7 in two configurations. In configuration I (triangle),
it is an equilateral triangle and a voltage V is
X Y Z applied to corners A and B. In configuration 2
(b) and X > Y > Z (circle), it is bent in the form of a circle, and the
5 6 7
potential v is applied at diameterically opposite
X Y Z points P and Q. The ratio of the power dissipated
(c) and X > Y > Z in configuration 1 to configuration 2 is.
5 6 7

X Y Z
(d) and X < Y < Z
5 6 7
14. The least value of a natural number n such that

n 1 n 1 n n n!
, where ,
5 6 7 r n r !r!
is 2 9
(a) (b)
(a) 12 (b) 13 3 8
(c) 14 (d) 15 5 7
(c) (d)
15. In a Mathematics test, the average marks of boys 4 8
is x% and the average marks of girls is y% with x
20. Six objects are placed at the vertices of a regular 22. A total charge q is divided as q1 and q2 which are
hexagon. The geometric center of the hexagon kept at two of the vertices of an equilateral
is at the origin with objects 1 and 4 on the x-axis triangle of side a. The magnitude of the electric
(see figure). The mass of the kth object is mk = k field E at the third vertex of the triangle is to be
M |cosqk| where i is an integer, M is a constant depicted schematically as a function of x = q1/q.
with dimension of mass, and qk is the angular
Choose the correct figure.
position of the k th verted measured from the
positive x-axis in the counter-clockwise sense. If
the net gravitational force on a body at the
(a) (b)
centroid vanishes, the value of i is

(a) 0 (c) (d)


(b) 1
(c) 2
(d) 3
21. A mirror is placed at an angle of 30° with respect
to y-axis (see figure). A light ray travelling in the
negative y-direction strikes the mirror. The
direction of the reflected ray is given by the vector

23. The refractive index of water in a biology


laboratory tank varies as 1.33 + 0.002/ 2, where
is the wavelength of light. Small pieces of organic
matter of different colours are seen at the bottom
of the tank using a travelling microscope. Then
the image of the orgainc matter appears
(a) deeper for the violet pieces than the green
ones.
(b) shallower for the blue pieces than the orange
ones.
(c) at the same depth for both the blue and orange
pieces.
(a) î
(d) deeper for the green pieces than the red ones.
(b) ˆi 3ˆj

(c) 3iˆ ˆj

(d) ˆi 2ˆj
24. Two students P and Q perform an experiment to
verify Ohm’s law for a conductor with resistance
R. They use a current source and a voltmeter
with least counts of 0.1 mA and 0.1 mV,
respectively. The plots of the variation of voltage
drop (V) across R with current (I) for both are shown
below

(a) M is g(h 0 H w)

r2
(b) N is g(h H )
0 w
R2
(c) M is g H w

w HR 2 0 hr
2

(d) N is g
R2 r2
26. Two cars S 1 and S 2 are moving in coplanar
concentric circular tracks in the opposite sense
with the periods of revolution 3 min and 24 min,
respectively. At time t = 0, the cars are farthest
apart. Then, the two cars will be
(a) closest to each other at t = 12 min and farthest
at t = 18 min.
(b) closest to each other at t = 3 min and farthest
at t = 24 min
(c) closest to each other at t = 6 min and farthest
at t = 12 min
(d) closest to each other at t = 12 min and farthest
at t = 24 min
27. In the circuit shown below, a student performing
Ohm’s law experiment accidently puts the
voltmeter and the ammeter as shown in the
circuit below; the reading in the voltmeter will
The statement which is most likely to be correct be close to
is:
(a) P has only random error (s).
(b) Q has only systematic error (s).
(c) Q has both random and systematic errors.
(d) P has both random and systematic errors.
25. A cylindrical vessel of base radius R and height H
has a narrow neck of height h and radius r at one
end (see figure). The vessel is filled with water
(density w) and its neck is filled with immiscible
oil (density ). Then (a) 0 V (b) 4.8 V
(c) 6.0 V (d) 1.2 V
28. The bhagirathi and the Alaknanda merge at (III)Image (i) has been viewed from the cocave
Deoprayag to form the Ganga with their speeds in side of a plano-concave lens and image (ii) from
the ratio 1 :1.5. The cross-sectional areas of the the planar side of a plano-convex lens.
Bhagirathi, the Alaknanda and the Ganga are in (IV)Image (i) has been viewed from the planar side
the ratio 1 : 2 :3. Assuming streamline flow, the of a plano-concave lens and image (ii) from the
ratio of the speed of Ganga to that of the convex side of a plano-convex lens.
Alaknands is
Which of the above statements are correct ?
(a) 7 : 9 (b) 4 : 3
(a) Only (III)
(c) 8 : 9 (d) 5 : 3
(b) Only (IV).
29. A long cylindrical pipe of radius 20 cm is closed at
(c) Only (III) and (IV).
its upper end and has an airtight piston of
negligible mass as shown. When a 50 Kg mass is (d) All four.
attached to the other end of the piston, it moves
down by a distance l before coming to
31. The IUPAC name for the following compound is
equilibrium. Assuming air to be an ideal gas, l/
L (see figure) is close to (g = 10 ms2 , atmospheric
pressure is 105 Pascal),
(a) 4,6-dimethylheptane
(b) 1,3,5 -trimethylhexane
(c) 2,4-dimethylheptane
(d) 2,4,6—trimethylhexane
32. The stability of carbocations

(I) (CH3)3C

(a) 0.01 (II) (CH3)2C(OCH3)


(b) 0.02
(c) 0.04 (III) CH3CH2CH2CH2
(d) 0.09
30. The word ‘’KVPY’’ is written on a board and viewed (IV) CH3CHCH2CH3
through different lense such that board is at a follows the order
distance beyond the focal length of the lens. (a) III < IV < II < I
(b) III < IV < I < II
(c) IV < III < II < I
(d) IV < III < I < II
33. The acidity of compounds I-IV in water
I. Ethanol
lII. Acetic Acid
III. Phenol
lgnorging magnification effects, consider the lIV. Acetonitrile
following statements follows the order
(I) Image (i) has been viewed from the planar side (a) IV < I < III < II
of a plano-convex lens and image (ii) from the
(b) I < II < III < IV
planar side of a plano-convex lens.
(c) IV < I < II < III
(II) Image (i) has been viewed from the concave
side of a plano-concave lens and image (ii) from (d) IV < III < I < II
the convex side of a plano-convex lens.
34. In the following reaction 40. Which of the following transformations can be
carried out by using HI as a reducing agent, under
acidic conditions ?
[Given : I2(s) 2I– E = 0.54 V]
(i) Cu+ Cu(s) E = 0.52 V
(ii) Cr3+ Cr2+ E = – 0.41 V
the major product is (iii)Fe 3+
Fe2+ E = 0.77 V
(iv) Fe2+ Fe(s) E = – 0.44 V
(a) (i) and (iii) (b) (ii) and (iv)
(c) only (iii) (d) only (ii)
(a) (b)
41. C60 emerging from a source at a speed (v) has a de
Broglie wavelength of 11.0 Å. The value of v (in m
s–1) is closest to
[Planck’s constant h = 6.626 × 10–34 J s]
(a) 0.5 (b) 2.5
(c) (d) (c) 5.0 (d) 30
42. The lattice energies of NaCl, NaF, KCl and RbCl
follow the order
35. The reddish brown precipitate formed in the (a) KCl < RbCl < NaCl < NaF
Fehling’s test for aldehydes (RCHO) is due to the
(b) NaF < NaCl < KCl < RbCl
formation of
(c) RbCl < KCl < NaCl < NaF
(a) Cu (b) Cu2O
(d) NaCl < RbCl < NaF < KCl
(c) CuO (d) (RCOO)2Cu
43. The oxidation states of P atom in POCl3, H2PO3
36. The reducing ability of the metals K, Au, Zn and
and H4P2O6, respectively, are
Pb follows the order
(a) +5, +4, +4 (b) +5, +5, +4
(a) K > Pb > Au > Zn
(c) +4, +4, +5 (d) +3, +4, +5
(b) Pb > K > Zn > Au
44. A solution (5 mL) of an acid X is completely
(c) Zn > Au > K > Pb
neutralized by y mL of 1M NaOH. The same
(d) K > Zn > Pb > Au volume (y ML) of 1M NaOH is required to
37. White phosphorous catches fire in air to produce neutralize 10 mL of 0.6 M of H2SO4 completely.
dense white fumes. This is due to the formation The normality (N) of the acid X is
of (a) 1.2 (b) 2.4
(a) P4O10 (b) PH3 (c) 4.8 (d) 0.6
(c) H3PO3 (d) H3 PO2 45. 1.25 g of a metal (M) reacts with oxygen
38. The maximum number of electrons that can be completely to produce 1.68 g of metal oxide. The
filled in the shell with the principal quantum empirical formula of the metal oxide is
number n = 4 is [molar mass of M and O are 69.7 g mol–1 and 16.0
(a) 64 (b) 26 g mol–1, respectively]
(c) 18 (d) 32 (a) M2O (b) M2O3
39. At a constant pressure P, the plot of volume (V) as (c) MO2 (d) M3O4
a function of temperature (T) for 2 moles of an
ideal gas gives a straight line with a slope 0.328
L K–1. The value of P (in atm) is closest to 46. According to Watson-Crick model, hydrogen
[Gas constant, R = 0.0821 L atm mol –1
K ]
–1 bonding in a double-stranded DNA occurs between
(a) 0.25 (b) 0.5 (a) Adenine and guanine
(c) 1.0 (d) 2.0 (b) Adenine and thymine
(c) Cytosine and adenine
(d) guanine and thymine
47. Which ONE of the following statements about (a) Glucose to ethanol by Saccharomyces.
mitosis is CORRECT ? (b) Lactose to lactic acid by Lactobacillus.
(a) One nucleus gives rise to 4 nuclei (c) Glucose to CO2 and H2O by Saccharomyces.
(b) Homologous chromosomes synapse during (d) Cellulose to glucose by Cellulomonas.
anaphase
55. An amount of 18 g glucose corresponds to
(c) The centromeres separate at the onset of
(a) 1.8 mole (b) 1 mole
anaphase
(c) 0.18 mole (d) 0.1 mole
(d) Non-sister chromatids recombine
56. The number of electrons required to reduce one
48. Gaseous exchange of oxygen and carbon dioxide
molecule of oxygen to water during mitochondrial
between alveolar air and capillaries takes place
oxidation is
by
(a) 4 (b) 3
(a) Active transport
(c) 2 (d) 1
(b) Diffusion
57. Which ONE of the following molecules is derived
(c) Carrier-mediated transport
from pantothenic acid ?
(d) Imbibition
(a) Thiamine pyrophosphate
49. Of the periods listed below, which ONE is the
(b) Nicotinamide adenine dinucleotide phosphate
earliest period when Ostracoderms, the jawless
and finless fishes, appeared? (c) Flavin adenine dinucleotide phosphate
(a) Devonian period (d) Acetyl-CoA
(b) Cambrian period 58. Match the disease given in Column I with the
principal causal organism in Column II and choose
(c) Carboniferous period
the correct combination.
(d) Silurian period
Colum I Column II
50. Scurvy is caused by the deficiency of
(P) AIDS (i) HBV
(a) Nicotinic acid
(Q) Syphilis (ii) Neisseria sp.
(b) Ascorbic acid
(R) Viral hepatitis (iii) Treponema sp.
(c) Pantothenic acid
(S) Gonorrhoea (iv) HIV
(d) Retinoic acid
(a) P-iv, Q-iii, R-i, S-ii
51. Optical activity of DNA is due to its
(b) P-iv, Q-ii, R-i, S-iii
(a) Bases
(c) P-i, Q-ii, R-iv, S-iii
(b) Sugars
(d) P-i, Q-iv, R-ii, S-iii
(c) Phosphate
59. Chromosomes are classified based on the position
(d) Hydrogen bonds of centromere. A chromosome having a terminal
52. The monarch butterfly avoids predators such as centromere is called
birds by (a) metacentric
(a) Changing color frequently (b) telocentric
(b) Flying away from the predator swiftly (c) sub-metacentric
(c) Producing a chemical obnoxious to the (d) acrocentric
predator
60. Which ONE of the following options lists the
(d) Producing ultrasonic waves primary energy source (s) for all forms of life on
53. Filariasis is caused by earth ?
(a) Entamoeba histolytica (a) Light, Inorganic substances
(b) Plasmodium falciparum (b) Inorganic substances, Organic substances
(c) Trypanosoma brucei (c) Light, Organic substances
(d) Wuchereria bancrofti (d) N2, CO2
54. Which ONE of the following conversions does NOT
happen under anaerobic conditions ?
65. Let a, b, c, d be real numbers between – 5 and 5
61. Let ABCD be a trapezium with parallel sides AB such that |a| = 4 5 a , |b| = 4 5 b,
and CD such that the circle S with AB as its
diameter touches CD. Further, the circle S passes |c| = 4 5 c , |d| = 4 5 d,
through the midpoints of the diagonals AC and BD
Then the product abcd is
of the trapezium. The smallest angle of the
trapezium is (a) 11 (b) –11
(c) 121 (d) –121
(a) (b)
3 4
66. Persons A and B are standing on the opposite
(c) (d) sides of a 3.5 m wide water stream which they
5 6 wish to cross. Each one of them has a rigid wooden
plank whose mass can be neglected. However,
a c each plank is only slightly longer than 3 m. So
62. Let S be the set of all points , on the circle
b d they decide to arrange them together as shown
with radius 1 centred at (0,0) where a and b are in the figure schematically. With B (mass 17 kg)
relatively prime integers, c and d are relatively standing, the maximum 2mass of A, who can walk
prime integers (that is HCF (a, b) = HCF (c,d) = over the plank is close to ,
1), and the integers b and d are even. Then the
set S
(a) is empty
(b) has four elements
(c) has eight elements
(d) is infinite
63. Suppose we have two circles of radius 2 each in
the plane such that the distance between their (a) 17 kg. (b) 65 kg.
centres is 2 3. The area of the region common (c) 80 kg. (d) 105 kg.
to both circles lies between 67. Two different liquids of same mass are kept in
(a) 0.5 and 0.6 (b) 0.65 and 0.7 two identical vessels, which are placed in a freezer
(c) 0.7 and 0.75 (d) 0.8 and 0.9 that extracts heat from them at the same rate
64. Let C1, C2 be two circles touching each other causing each liquid to transform into a solid. The
externally at the point A and let AB be the diameter schematic figure below shows the temperature T
of circle C1. Draw a secant BA 3 to circle C2, vs time t plot for the two materials. We denote
intersecting circle C1 at a point A1( A), and circle the specific heat of metrials in the liquid (solid)
C2 at points A2 and A3 . If BA1 = 2, BA2 = 3 and BA3 states to be CL1 (CS1) and CL 2 (CS2) respectively.
= 4, then the radii of circles C 1 and C 2 are
respectively
30 3 30
(a) ,
5 10

5 7 5
(b) ,
2 10

6 6 (a) CL1 < CL2 and CS1 < CS2


(c) , (c) CL1 > CL2 and CS1 > CS2
2 2
(b) CL1> CL2 and CS1 > CS2
10 17 10 (d) CL1 < CL2 and CS1 > CS2
(d) ,
3 30
68. A ray of light originates from inside a glass slab (in meter/second) of the atoms that will emerge
and is incident on its inner surface at an angle at the other end.
as shown,

(a) 75, 25
(b) 100, 50
(c) 300,100
(d) 600, 200
In this experiment the location x of the spot where
the ray hits the screen is recorded. Which of the
following correctly shows the plot of variation of 71. Among the following compouds, E/Z isomerism
x with the angle ? is possible for
(a) 2-methylbut-2-ene
(b) 2-methylbut-1-ene
(c) 3-methylpent-1-ene
(d) 3-methylpent-2-ene
72. In the reaction

(a) A (b) B
x and y, respectively, are
(c) C (d) D
(a) x = CH3OH ; y = Pd/BaSO4 ,quinoline, H2
69. Four identical pendulums are made by made by
(b) x = CH3I ; y = Pd/BaSO4 ,quinoline, H2
attaching a small ball of mass 100 g on a 20 cm
long thread and suspended from the same point. (c) x = CH3I ; y = Na in liq. NH3
Now each ball is given charge Q so that balls move (d) x = CH3OH ; y = Na in liq. NH3
away from each other with each thread making 73. Among the following molecules, the one with the
an angle of 45° from the vertical. The value of Q largest bond angle at the central atom is
1 (a) ClF3
is close to 4 9 109 in SI units
0
(b) POCl3
(c) BCl3
(a) 1 C
(d) SO3
(b) 1.5 C
74. A compound hasthe following composition by weight;
(c) 2 C
Na = 18.60 %, S = 25.80 %, H = 4.02 % and
(d) 2.5 C O = 51.58 % Assuming that all the hydrogen atoms
70. Two parallel discs are connected by a rigid rod of in the compound are part of water of
length L = 0.5 m centrally. Each disc has a slit crystallization, the correct molecular formula of
oppositely placed as shown in the figure. A beam the compound is
of neutral atoms are incident on one of the discs (a) Na2S2O3.3H2O
axially at different velocities v, while the system (b) Na2SO4.5H2O
is rotated at angular speed of 600 rev/second so
that atoms only with a specific velocity emerge at (c) Na2SO4.10H2O
the other end. Calculate the two largest speeds (d) Na2S2O3.5H2O
75. X g of ice at 0 °C is added to 340 g of water at 20° C. (a) 0.1 mg
The final temperature of the resultant mixture is (b) 1 mg
5°C. The value of X (in g) is closest to
(c) 10 mg
[Heat of fusion of ice = 333 J/g ; Specific heat of
(d) 100 mg
water = 4.184 J/g. K]
78. In an in vitro tanslation experiment, poly (UC)
(a) 80.4
RNA template produced poly (Ser-Leu), while poly
(b) 52.8 (AG) RNA template produced poly (Arg-Glu)
(c) 120.6 polypeptide. Which ONE of the following options
(d) 60.3 represents correct inter- pretations of the codons
assignments for Ser, Leu, Arg, and Glu.
(a) Ser – UCU, Leu – CUC, Arg – AGA, Glu –
76. Considering ABO blood grouping system in GAG
humans, during blood transfusion some
(b) Ser – CUC, Leu – GAG, Arg – UCU, Glu –
combinations of blood groups are compatible ( ,
AGA
whereas the others are incompatible (X). Which
ONE of the following options is CORRECT ? (c) Ser – AGA, Leu – UCU, Arg – GAG, Glu –
CUC
(d) Ser – GAG, Leu – AGA, Arg – CUC, Glu –
UCU
79. A single bacterium is actively growing in a
(a) medium that supports its growth to a number of
100 million. Assuming the division time of the
bacterium as 3 hours and the life span of non-
dividing bacteria as 5 hours, which ONE of the
following represents the maximum number of
bacteria that would be present at the end of 15
hour ?
(a) 10
(b)
(b) 64
(c) 24
(d) 32
80. A couple has two sons and two daughters. Only
one son is colour blind and the rest of the siblings
are normal. Assuming colour blindness is sex-
(c) linked, which ONE of the following would be the
phenotype of the parents ?
(a) Mother would be colour blind, father would be
normal.
(b) Father would be colour blind, mother would
be normal.
(c) Both the parents would be normal.
(d) (d) Both the parents would be colour blind.

77. A 25,000 Da protein contains a single binding site


for a molecule (ligand), whose molecular weight
is 2,500 Da. Assuming high affinity and
phsiologically irreversible blinding, the amount of
the ligand required to occupy all the binding sites
in 10 mg protein will be
1. (c) 2. (b) 3. (c) 4. (a) 5. (d) 6. (d) 7. (b) 8. (d) 9. (c) 10. (d)

11. (c) 12. (b) 13. (d) 14. (c) 15. (a) 16. (a) 17. (b) 18. (b) 19. (b) 20. (a)

21. (c) 22. (c) 23. (b) 24. (d) 25. (a) 26. (d) 27. (c) 28. (c) 29. (c) 30. (d)

31. (c) 32. (b) 33. (a) 34. (c) 35. (b) 36. (d) 37. (a) 38. (d) 39. (b) 40. (c)

41. (c) 42. (c) 43. (a) 44. (b) 45. (b) 46. (b) 47. (c) 48. (b) 49. (d) 50. (b)

51. (b) 52. (c) 53. (d) 54. (c) 55. (d) 56. (a) 57. (d) 59.(a) 59. (b) 60. (a)

61. (d) 62. (a) 63. (c) 64. (a) 65. (a) 66. (c) 67. (b) 68. (a) 69. (b) 70. (d)

71. (d) 72. (c) 73. (c) 74. (d) 75. (d) 76. (d) 77. (b) 78. (a) 79. (d) 80. (c)

3. (c) Expanding the expression are get


98 (a2 + b2 + c2) – 98 (ab + bc + ca)
1. (c) Let triangle T is DEF possibilities
1
98 98 = 98 + 49
2

= 147

4. (a)

A can take two positions if ABC DEF AM


To find .....(i)
We can arrange order of A, B, C in 3 ! = 6 MC a
ways
BAC ~ PMC
Total positions which A can take = 6 × 2 = 12
PM BA
2. (b) The required numbers are
CM AC
2, 4, 8, 16, 32, 64, 128,
k BA
5, 25, 125, a a
10, 20, 40, 50, 80, 100, 160, 200 ak
BA
a
1 1 1
Area of ABC = AC × AB 5
2 a a 6
1 1
5
1 ak
= a× ...(ii) 6. (d) n2 + 4 is divisible by 17. Now for n = 9
2 a

Now area of quad. BNMP = Area fo trapezium n2 3 n2 4 1


AMPB – area of AMN ...(iii) not divisible by 17
17 17 17
Also NAM ~ BAC
7. (b) 48000 = 16 × 3000
AN AB = 16 × [31× 23 × 53]
AM AC
As H.C.F. is 16 23 can be selected in 1 way
& 3 × 5 can be seleted in (1+ 1) (3 +1) = 8
1 3

AN ak ways
a a
No of ordred pairs = 8

k 8. (d) Required numbers are = 9 from 11 to 19


AN
a 4 i.e. (22, 24, 26, 28)
Now, area quad. BNMP 3 i.e. (33, 36, 39)
2 i.e. (44, 48)
1 1
= (AB + PM) × AM – AM × AN
2 2 5 i.e. (55, 66, 77, 88, 99)
23
1 ak 1 k
k 9. (c) 1st Jan will be monday as there will be 90
2 a 2 a
days from Jan. 1 to march 31 (Non leap year)

1 2ak k 2
k 2 (If year is leap year then days will be 91 = 13
2 a weaks not possible
15th February will be Thursday
2
ak k 10. (d) Let Three digit number be 100 a + 10 b + c
=k
a
Given 100 a + 10 b + c = 100 a + 10 c + b – 36

5 1 a2k 9b – 9c + 36 = 0
Given k
18 2 a c=b+4

36 a2 b=c–4 ...(i)
5 a Also given 100 a + 10 b + c
2 2 = 100c + 10 b + a + 198
36a – 36 = 5a
2 2 99 a – 99 c = 198
5a – 36a + 36 =0
2 2 a=c+2 ...(ii)
5a – 30a – 6a + 36 =0
Now 100 a + 10 b + c – (100 b + 10 a + c)
5a (a – 6 ) – 6 (a–6 ) = 0
5a = 6 a=6 = 90 (a – b)
= 90 ( c + 2 – c + 4) From eq (i) and (ii)
a 6
Not possible) = 540
5
Hence, value is decrease by 540
11. (c) Clearly area of triangle having sides (5, 12,
13) 3
=3×
2
is greatest (use = s s a s b s c = 2.598 = y

12. (b) Let the numbers of Boys be x 1 360


Area of heptagon = 7 × sin
& Let the number of girls be y 2 7

According to question, = 2.73 = z


14. (c) n–1C + n–1C < nC
4x 5 6 7
5 2 nC < nC
6 7
y 3
n! n!
n 6 6! n 7 7!
2x 1
5y 3 n–6>7

6x n > 13
y ...(i)
5 nmin = 14
15. (a) Given
4x
5 5 Let the number of Boys be B & number of
Also,
girls = G
y 44 2
Sum of marks obtained by boys = B.x
8x = 25 (y – 44)
Sum of marks obtained by girls = G.y
6x Now, given
8x = 25 44 from eq (i)
5
Bx Gy
x = 50 z
B G
y = 60
B(x – z) = G(z – y)
13. (d)
B z y
G x z

Add 1

B z y
1 1
G x z

B G x y
G x z

G z x
B G y x
1 16. (a) -particle bombard during Ruther Ford's
Area of Pentagon = 5 × sin 72° = 2.377 = x
2 scattering experiment.

1 -particles are ionized helium.


Area of hexagon = 6 × sin 60°
2
17. (b) Electronic configuration of sulphur is 1S2, 2S2,
R 1 l/2
2P6, 3S2, 3P4. Hence number of completely Re q
2 2 A
22
filled shells for the elements 165 is 2.
V V
18. (b) Length should be taken up to COM(Centre of i2 4A
Mass) Re q l

2.256 Therefore, ratio of power dissipated in


= 63.2 + configuration 1 to configuration 2 is:
net
2

64.328 i1 9
So, student should take the length of i2 8
pendulum to be 64.3 cm.
20. (a) For Gravitational equilibrium (FNet) 0
by signlicont figures
All Diagonal opposite should have equal mass
net
64.3 cm
2i° M cos 60°
19. (b) 4i° M cos(60° + 180°)
By solving, we get
i=0

21. (c)

l
R
A

l
R1 3
A

1 1 1
r 2
Re q 2R1 R1
Incident Ray = ˆj

l Reflected Ray
2
2R1 3
Re q
3 3A

V V
i1 9A
Re q 2el

3ˆ 1ˆ
Vector i j
2 2
22. (c) 2
2
24

time for meet (closest)

2 1 4
= 2n 1
1 2 3

4 20 28
t sec, 4, , ,12
3 3 3

Here, Enet is given by= E + E 2 n


1 2
time of farthest
q = q1 + q2 1 2

2 2 4
kq1 kq 2 2k 2 q1 q 2 t 2n /
Enet .cos60 3
a a a2
27. (c) Voltmeter has very high Resistane therefore it
is put in parallel. If voltmeter is put in series
k maximum of potential difference will be across
E net q 12 q 22 q1q 2
a voltmeter
28. (c) By equation of continuity
k 2
E net q 12 q q1 q1 q q1
a Area of Bhagirathi A
Area of Alaknanda 2A
2 2 2
k q1 q1 2q1 q1 q Area of Ganga 3A
Enet 12 1
2
aq q q q q q
Now, VB : VAL : VG :
3
k V: V : V1
Enet x2 1 x 2
aq

B = Bhagirathi,AL = Alaknanda, G = Ganga]


k 2
Enet x 1/ 2 3/4
aq Now, By equation of continuity

Hence, minima must be at |x = 1/2| 3


AV + A 2.V 3A.V1
25. (a) Pressure at same height is same. 2

By figure given in question, we get


4
Vganga = V
PM = PN 3
Hence, P = P = 0gH + wgH
M N
3
26. (d) VAlaknanda V
2 9
Vganga 4 8
S1 S2 V
3
V :V =8:9
1 2 ganga Alaknanda

2
1 3
29. (c) 38. (d) 4S, 4P, 4d, 4f can occupy 32 electrons.

V nR
39. (b) = Slope
T P

2 .0821
Slope = 0.5
0.328
40. (c) It has maximum reduction potential.

1
42. (c) U lattice
Size of molecule
Process is isothermal
43. (a) Oxi. number of P in POCl3 = +5
So, P1V1 = P2 V2
Oxi. number of P in H2PO3 = +4
(P0) (A) ( ) PA ( + ) Oxi. number of P in H4P2O6 = +4
P0 Al P0 l 44. (b) Equivalent of NaOH = Equivalent of H2SO4
P A l l l l 1 × 1 × y = 10 × .6 × 2
final

y = 12 ml.
Now, By force equilibrium Now, N × 5 = (1 × 1) × 12
(P – P) A mg 12
0
N= = 2.4
5
P0 l
P0 A mg
l l 1.25 1.68
45. (b)
E E 8
l Solving, E = 23.25
0.04
l
Molar Mass
E=
30. (d) (i) For convex lens If object is placed beyond n factor
focus image is inverted
69.7
(ii) For plano-concave lens or concave lens if So, n–factor = 3
23.25
object is placed beyond focus image is
erected So Empirical formula = M2O3
32. (b) II carbocation is stabilised by resonance while 46. (b) In DNA A = T, and G C
in case of others, stability is 3° > 2° > 1° 47. (c) Number of chromosome becomes double in
33. (a) Correct order will be anaphase as the sister chromatids separates
from centromere.
48. (b) Gaseous exchange between Alveolar air and
capillaries takes place by diffusion.
CH 3COOH > > C2 H5 OH > Acetonitrile 49. (d) Periods are ‘time period’.
50. (b) Ascorbic acid is required for collagen synthesis
during wound healing and also in many
biosynthetic pathways.
52. (c) Some pray may produce toxic substances to
protect themselves from predators.
Hoffmann
34. (c) Bromamide 53. (d) Wuchereria lives in lymphatic vessels and
causes swelling. Generally lower limbs and
scrotum is affected.
+ K2CO3 + H2O 54. (c) Glucose is completely oxidised to CO2 and
H2O during aerobic respiration.
37. (a) P4 5O2 P4 O10
(White fumes) mass in (g) 18
55. (d) Mole = = 0.1 mole
Molecular mass 180
56. (a) O2 + 4e– 4H+ 2H2O even odd odd
57. (d) Vitamin B5 is pantothenic acid and synthesize b2 = a2 + p2
Co-A (co-enzyme A)
= (2k + 1)2 + ( + 1)2
60. (a) Plants use light and some bacteria used
inorganic compounds for synthesis of energy = 4k2 + 4k + 1 + 4 +4 +1
rich compounds.
b2 = 4 (k2 + +k+ )+2 impossilbe
as L.H.S. is multiple of 4 but R.H.S is not
multiple of 4
61. (d)
63. (c) Let two circles are
x2 + y2 = 4 & (x – 2)2 + y2 = 4

equation of common chord is x = 3

Join AN
ANB = 90°

In ANB,

BN
cos =
2r

BN = 2r cos A 3, 1 , B 3, 1
BD = 2BN = 4r cos
So AC1B = 60°
In BQD
AB = 2 & MC1= 3
BQ r
sin = Required area
BD 4r cos
= 2 [area of sector C1AB – ar C1AB]
1
sin 2 = 1 2 1
2 2 2 2 3 = .723
2 3 2
= 15°
Now similarly = 15° = & AC = 4r cos 64. (a)
Trapezium will be isosceles
ADB = 30°

62. (a) circle is x2 + y2 = 1

a2 a
y 1 x
b2 b
BM = A1M = 1
1 A1A2 = 1
y b2 a2
b
1
As y is retional so A 2N A3N
2
b2 – a2 = p2
Let radius of C1 is r1
Let radius of C2 is r2
5 170x
1 10m × 170x m
PM = r12 1, QN = r22 10 5
4
Maximum value is near to 2.5 infact shift by
QNB ~ PMB less than 2.5

1
r22
4 BN 7/2
r 2
1 BM 1
1

4r 2 = 49r 2 – 48 ...(i)
Also, in QNB
BQ2 = BN2 + NQ2 170
Maximum vlaue of m = × 2.5 = 85
5
49 1
2r1 r2 r22 Hence, from the given options
4 4
Option (c) (approx) is correct
2
r1 r1 r2 3 ...(ii)
67. (b)Let Refrigerater extract Q joul/per second
Solve (i) & (ii)
Q.t ms (Tf – T)
6 30 3 30 Higher the specific heat, Higher the slope
r1 & r2
5 5 10
68. (a) This is case of Total Internal Reflector
65. (a) Given |a| = 4 5 a, (90– ) is incident angle. As increases thus
incident angle decreases. Initially Ray will be
squaring Reflected
a2 = 4 – 5 a At C angle will be TIR afterwards Refraction
a4 + 16 – 8a2 = 5 – a takes place.
69. (b)
a4 – 8a2 + a + 11 = 0
Similarly squaring other given equations
& solving we can say that a,b, –c, –d are roots
of x4 – 8x2 + x + 11 = 0
product of roots
ab (–c) (–d) = 11
abcd = 11
66. (c) Fe
Here, tan
mg
45
kQ2
2
20 10 2
2
tan 45 3
1000 10 10
Hence, value of Q
N = 10 m
20 12
N × 0.5 = 170 x 10 1.5 c
9
70. (d) Time gap between two disc should be given 18.6
74. (d) Moles of Na = 0.823
as: 23

3 5 25.8
t , , Moles of S = 0.832
32

Hence, 51.58
Moles of O = 3.2216
16
0.5
V1 600 m / s 4.02
Moles of H = 4.02
2 1
600

Na 1
0.5
V2 200 m / s S 1
3
600
2 O 4 = Simplest mole ratio
H 5
C2H5 H C2H5 CH3
So formula is Na2S2O3. 5H2O
71. (d) C=C and C=C
76. (d) Blood group O Universal Donor
CH3 CH3 CH3 H
Blood group AB Universal recipient
E – form Z – form
10 25000
77. (b)
72. (c) CH3 C C H NaNH2
CH3 C C Na x 2500

10 10
x 1 mg
CH 3 I x 1
78. (a) One codon has 3 nitrogenous bases.
CH 3 CH CH CH 3
15
Na / liq.NH3
CH3 C C CH3 NaI 79. (d) Number of divisions = =5
3
Number of bacterial cells = 25 = 32.
80. (c) In a male child father contribute ‘y’
F O
Cl chromosome and X-chromosomes comes from
F Cl mother only. and mother contain two
73. (c) Cl –B S X-chromosomes. Hence a normal son
Cl F O O indicates that mother contributed a normal
X-chromosome (xx) i.e. mother is a carrier.
(120°) (180°) (120°)

O
P
Cl
Cl
Cl
(109.5°)
6. Let ABCD be a trapezium, in which AB is parallel
to CD, AB = 11, BC = 4, CD = 6 and DA = 3. The
1. Suppose the quadratic polynomial P(x) = ax2 + bx distance between AB and CD is
+ c has positive coefficients a, b, c in arithmetic
(a) 2
progression in that order. If P(x) = 0 has integer
roots and then + + equals (b) 2.4
(a) 3 (b) 5 (c) 2.8
(c) 7 (d) 14 (d) not determinable with the data
2 . The number of digits in the decimal expansion of 7 . The points A, B, C, D, E are marked on the
165516 is circumference of a circle in clockwise direction
such that ABC = 130º and CDE = 110º. The
(a) 16 (b) 17 measure of ACE in degrees is
(c) 18 (d) 19 (a) 50º (b) 60º
3 . Let t be real number such that t = at + b for 2
(c) 70º (d) 80º
some positive integers a and b. Then for any
choice of positive integers a and b, t3 is never 8 . Three circles of radii 1, 2 and 3 units respectively
equal to touch each other externally in the plane. The
circumradius of the triangle formed by joining
(a) 4t + 3 (b) 8t + 5 the centers of the circles is
(c) 10t + 3 (d) 6t + 5 (a) 1.5 (b) 2
4 . Consider the equation (1+ a b)2 = 3(1+ a b2), (c) 2.5 (d) 3
where a b are real numbers. Then
9 . Let P be a point inside a triangle ABC with ABC
(a) there is no solution pair (a b) = 90º. Let P1 and P2 be the images of P under
reflection in AB and BC respectively. The distance
(b) there are infinitely many solution pairs (a b) between the circumcenters of triangles ABC and
P1 P P2 is
(c) there are exactly two solution pairs (a b)
(d) there is exactly one solution pair (a b) AB AP BP CP
(a) (b)
2 3
5. Let a1, a2, ....... a100 be non-zero real numbers such
that a1 + a2 + ....... +a100 = 0, Then AC AB BC AC
(c) (d)
2 2
100 100
(a) a i 2a i 0 and ai 2 ai
0
i 1 i 1
10. Let a and b be two positive real numbers such
that a 2b 1. Let A1 and A2 be, respectively, the
100 100
(b) i 1
a i 2a i 0 and i 1
ai 2 ai
0 areas of circles with radii ab3 and b2. Then the
A1
100 ai 100 ai maximum possible value of is
(c) i 1
ai 2 0 and i 1
ai 2 0 A2

1 1
(d) the sign of
100
a 2ai or
100
a 2 ai
depends on (a) (b)
i 1 i i 1 i 16 64
the choice of ai ‘s 1 1
(c) (d)
16 2 32
11. There are two candles of same length and same
size. Both of them burn at uniform rate. The first
16. A person walks 25.0º north of east for 3.18 km.
one burns in 5 hours and the second one burns
How far would she have to walk due north and
in 3 hours. Both the candles are lit together. After
then due east to arrive at the same location?
how many minutes the length of the first candle
is 3 times that of the other? (a) towards north 2.88 km and towards east 1.34
km
(a) 90
(b) towards north 2.11 km and towards east 2.11
(b) 120 km
(c) 135 (c) towards north 1.25 km and towards east 1.93
km
(d) 150
(d) towards north 1.34 km and towards east 2.88
12. Consider a cuboid all of whose edges are integers km
and whose base is square. Suppose the sum of
all its edges is numerically equal to the sum of 17. The length and width of a rectangular room are
the areas of all its six faces. Then the sum of all measured to be 3.95 0.05 m and 3.05 0.05 m,
respectively, the area of the floor is
its edges is.
(a) 12.05 0.01 m2
(a) 12 (b) 18
(b) 12.05 0.005 m2
(c) 24 (d) 36
(c) 12.05 0.34 m2
13. Let A1, A 2 .... , A m be non-empty subsets of
{1,2,3,..... ,100} satisfying the following conditions: (d) 12.05 0.40 m2

(1) the numbers |A1|, |A2|,... , |Am| are distinct; 18. A car goes around uniform circular track of radius
R at a uniform speed v once in every T seconds.
(2) A1, A2 , ... , Am are pairwise disjoint. The magnitude of the centripetal acceleration is
ac. If the car now goes uniformly around a larger
(Here |A| denotes the number of elements in circular track of radius 2R and experiences a
the set A.) Then the maximum possible value of centripetal acceleration of magnitude 8ac then
m is its time period is
(a) 13 (b) 14 (a) 2T (b) 3T
(c) 15 (d) 16 (c) T/2 (d) 3/2 T

14. The number of all 2-digit numbers n such that n 19. The primary and the secondary coils of a
is equal to the sum of the square of digit in its transformer contain 10 and 100 turns,
tens place and the cube of the digit in units place respectively. The primary coil is connected to a
is battery that supplies a constant voltage of 1.5
volts. The voltage across the secondary coil is
(a) 0 (b) 1
(a) 1.5 V (b) 0.15 V
(c) 2 (d) 4
(c) 0.0 V (d) 15 V
15. Let f be a function defined on the set of all positive 20. Water falls down a 500.0 m shaft to reach a
integers such that f (xy) = f (x) + f (y) for all positive turbine which generates electricity. How much
integers x, y. If f (12) = 24 and f (8) = 15, the value water must fall per second in order to generate
of f (48) is 1.00 × 10 9 Watts of power? (Assume 50%
efficiency of conversion and g = 10m/s2)
(a) 31
(a) 250 m3
(b) 32
(b) 400 m3
(c) 33
(c) 500 m3
(d) 34
(d) 200 m3
21. The diagram below shows two circle loops of wire
(A and B) centred on and perpendicular to the x-
axis, and oriented with their planes parallel to
each other. The y-axis passes vertically through
loop A (dashed line). There is a current IB in loop
B as shown. Possible actions which we might
perform on loop A are: (c) (d)

23. In an experiment , setup A consists of two parallel


wires which carry currents in opposite directions
as shown in the figure. A second setup B is
identical to setup A, except that there is a metal
plate between the wires

(i) Move A to the right along x axis closer to B


(ii) Move A to the left along x axis away from B
(iii) As viewed from above, rotate A clockwise
about y axis
(iv) As viewed from above, rotate A anticlockwise Let FA and FB be the magnitude of the force
about y axis between the two wires in setup A and setup B,
respectively.
Which of these actions will induce a current in A
only in the direction shown. (a) FA > FB 0
(a) Only (i) (b) Only (ii) (b) FA < FB
(c) Only (i) and (iv) (d) Only (ii) and (iii) (c) FA = FB 0
22. A rigid ball rolls without slipping on a surface (d) FA > FB = 0
shown below.
24. In the circuit, wire 1 is of negligible resistance,
Then

Which one of the following is the most likely


representation of the distance traveled by the
ball vs time graph?

(a) Current will flow through wire 1 if 1 2

(b) Current will flow through wire 1 if 1


/R1 2
/
R2
(c) Current will flow through wire 1 if ( 1+ 2)/(R1+
(a) (b) R2) ( 1– 2) / (R1–R2)
(d) No current will flow through wire 1.
25. The radius of a nucleus is given by r0 A1/3 where
r0 = 1.3 × 10-15 m and A is the mass number of the
nucleus, the Lead nucleus has A = 206. the 31. One mole of one of the sodium salts listed below,
electrostatic force between two protons in this having carbon content close to 14.3% produces 1
nucleus is approximately mole of carbon dioxide upon heating (atomic mass
(a) 102 N (b) 107 N Na = 23, H = 1, C = 12, O = 16). The salt is
(c) 10 N
12
(d) 1017 N (a) C2H5COONa (b) NaHCO3
26. A hollow lens is made of thin glass and in the (c) HCOONa (d) CH3COONa
shape of a double concave lens. It can be filled
with air, water of refractive index 1.33 or CS2 of 32. Among formic acid, acetic acid, propanoic acid
refractive index 1.6. It will act as a diverging lens and phenol, the strongest acid in water is
if it is
(a) formic acid (b) acetic acid
(a) filled with air and immersed in water.
(b) filled with water and immersed in CS2. (c) propanoic acid (d) phenol
(c) filled with air and immersed in CS2. 33. According to Graham’s Law, the rate of diffusion
(d) filled with CS2 and immersed in water. of CO,O2, N2 and CO2 follows the order:
27. A stone thrown down with a seed u takes a time (a) CO = N2 > O2 > CO2
t1 to reach the ground , while another stone,
thrown upwards from the same point with the (b) CO = N2 > CO2 > O2
same speed, takes time t2. The maximum height
(c) O2 > CO = N2 > CO2
the second stone reaches from the ground is
(a) ½ gt1 t2 (b) g/8 (t1 + t2)2 (d) CO2 >O2 >CO = N2

(c) g/8 (t1 – t2)2 (d) ½ gt22 34. The major product formed when 2-butene is
reacted with O3 followed by treatment with Zn/
28. An electric field due to a positively charged long H2O is
straight wire at a distance r from it is proportional
to r–1 in magnitude. Two electrons are orbiting (a) CH3COOH (b) CH3CHO
such a long straight wire in circular orbits of radii
(c) CH3CH2OH (d) CH2=CH2
1 Å and 2 Å. The ratio of their respective time
periods is 35. The IUPAC name for the following compound is
(a) 1 : 1 (b) 1 : 2 CH3-CH2-CH2- CH2-C-CH2-CH2-CH3
(c) 2 : 1 (d) 4 : 1 ||
CH2
29. Two particles of identical mass are moving in
circular orbits under a potential given by V(r) = (a) 2-propylhex-1-ene
Kr-n, where K is a constant. If the radii of their (b) 2-butylpent-1-ene
orbits are r1, r2 and their speeds are v1, v2,
respectively, then (c) 2-propyl-2- butylethene

(a) v12 r1n v22 r2n (b) v12 r1 n v22 r2 n (d) Propy1-1-butylethene
36. The major products obtained in the reaction of
(c) v12 r1 v22 r2 (d) v12 r12 n
v22 r22 n
oxalic acid with conc. H2SO4 upon heating are
30. Mercury is often used in clinical thermometers. (a) CO, CO2, H2O (b) CO, SO2, H2O
Which one of the following properties of mercury
is not a reason for this? (c) H2S, CO, H2O (d) HCOOH, H2S, CO
(a) The coefficient of the thermal expansion is 37. LiOH reacts with CO2 to form Li2CO3(atomic mass
large. of Li = 7). The amount of CO2 (in g) consumed by
(b) It is shiny. 1g of LiOH is closest to
(c) It is a liquid at room temperature. (a) 0.916 (b) 1.832
(d) It has high density.
(c) 0.544 (d) 1.088
38. The oxidation number of sulphur is +4 in
(a) H2S
(b)
(b) CS2
(c) Na2SO4
(d) Na2SO3
39. Al2O3 reacts with
(a) only water
(b) only acids
(c) only alkalis (c)

(d) both acids and alkalis


40. The major product formed in the oxidation of
acetylene by alkaline KMnO4 is
(a) ethanol
(b) acetic acid
(c) formic acid
(d)
(d) oxalic acid
41. In a closed vessel, an ideal gas at 1 atm is heated
from 27º C to 327º C. the final pressure of the gas
wil approximately be
(a) 3 atm (b) 0.5 atm
(c) 2 atm (d) 12 atm
42. Among the element Li,N,C and Be, one with the
largest atomic radius is 45. The graph that depicts Einstein photoelectric
effect for a monochromatic source of frequency
(a) Li (b) N above the threshold frequency is
(c) C (d) Be
43. A redox reaction among the following is
(a) CdCl2 + 2KOH Cd (OH)2 + 2KCl
(b) BaCl2 + K2SO4 aSO4 + 2KCl (a) (b)
(c) CaCO3 CaO + CO2
(d) 2 Ca + O2 CaO
44. The electronic configuration which obeys Hund’s
rule for the ground state of carbon atom is

(a)
(c) (d)
54. Bombyx mori (silk worm) belongs to the order

46. What is the length of human DNA containing (a) Lepidoptera (b) Diptera
6.6 × 109 bp? (c) Hymenoptera (d) Coleoptera
(a) 22 nm (b) 0.22 mm 55. The source of mammalian hormone “Relaxin” is
(c) 2.2 m (d) 22 m (a) ovary
47. The Diptheria, Pertussis, Tetanus (DPT) vaccine (b) stomach
consists of
(c) intestine
(a) live attenuated strains of Diptheria, Pertussis,
Tetanus (d) pancreas

(b) toxoid of Diptheria, Tetanus, and heat killed 56. Which one of the following animals is a connecting
whole cells of Pertussis link between reptiles and mammals?

(c) whole cell lysate of Diptheria, Pertussis, (a) Platypus


Tetanus
(b) Bat
(d) heat killed strains of Diptheria, Pertussis,
(c) Armadillo
Tetanus
(d) Frog
48. Which of the following is NOT an enzyme?
57. What is the number of chromosomes in an
(a) Lipase (b) Amylase
individual with Turner’s syndrome?
(c) Trypsin (d) Bilirubin
(a) 44 (b) 45
49. The pH of the avian blood is maintained by
(c) 46 (d) 47
(a) HCO3– (b) H2PO4–
58. Chipko movement in the year 1974 in Garhwal
(c) CH3COO –
(d) Cl– Himalayas involved

50. Podocyte layer that provides outer lining to the (a) protecting tigers
surface of glomerular capillaries are found in
(b) preventing soil erosion by planting trees
(a) bowman’s capsule
(c) preventing pollution by closing down
(b) loop of Henle industries

(c) renal artery (d) hugging trees to prevent the contractors from
felling them
(d) ureter
59. Which of the following amino acids is NOT
51. If a dsDNA has 20% adenine, what would be its involved in gluconeogenesis ?
cytosine content ?
(a) Alanine
(a) 20% (b) 30%
(b) Lysine
(c) 40% (d) 80%
(c) Glutamate
52. Which one of the following is incapable of curing
Pellagra? (d) Arginine

(a) Niacine (b) Nicotine 60. Which of the following entities causes syphilis?

(c) Nicotinamide (d) Tryptophan (a) Treponema pallidum

53. In Escherichia coli, how many codons code for (b) Neisseria gonorrhoea
the standard amino-acids?
(c) HIV
(a) 64 (b) 60
(d) Hepatitis B
(c) 61 (d) 20
65. If a 3-digit number is randomly chosen, what is
the probability that either the number itself or
61. Suppose a is a positive real number such that some permutation of the number (which is a
a5– a3 + a = 2. Then 3-digit number) is divisible by 4 and 5?
(a) a6 < 2 (b) 2 < a6 < 3 1 29
(a) (b)
45 180
(c) 3 < a6 < 4 (d) 4 a6
11 1
62. Consider the quadratic equation nx2 7 nx + n (c) (d)
60 4
= 0, where n is a positive integer. Which of the
following statements are necessarily correct ?
I. For any n, the roots are distinct.
66. Which one of the following four graphs best depict
II. There are infinitely many values of n for the variation with x of the moment of inertia I of
which both roots are real. a uniform triangular lamina about an axis parallel
to its base at a distance x from it
III. The product of the roots is necessarily an
integer.
(a) III only (b) I and III only
(c) II and III only (d) I, II and III
63. Consider a semicircle of radius 1 unit constructed
on the diameter AB, and let O be its centre. Let
C be a point on AO such that AC : CO = 2:1.
Draw CD perpendicular to AO with D on the
semicircle. Draw OE perpendicular to AD with E
on AD. Let OE and CD intersects at H. Then DH (a) (b)
equals
1 1
(a) (b)
5 3
1 5 1
(c) (d)
2 2
64. Let S1 be the sum of areas of the squares whose
sides are parallel to coordinate axes. Let S2 be (c) (d)
the sum of areas of the slanted squares as shown
in the figure. Then S1 / S2 is

(a) 2 (b) 2 67. A rectangular block is composed of three different


1 glass prisms (with refractive indices 1, 2 and 3)
(c) 1 (d) as shown in the figure below. A ray of light
2 incident normal to the left face emerges normal
to the right face. Then the refractive indices are
related by

(a) 2
1
2
2 2 2
3
(b) 2
1
2
2
2
3

(c) 2
1
2
3 2 2
2 (d) 2
2
2
3 2 2
1
68. A uniform metal plate shaped like a triangle ABC 73. Given
has a mass of 540 gm. the length of the sides AB,
BC and CA are 3 cm, 5 cm and 4 cm, respectively. NO(g) + O3 (g) NO2 (g) + O2 (g)
The plate is pivoted freely about the point A . H = –198.9 kJ/mol
What mass must be added to a vertex , so that
the plate can hang with the long edge horizontal? O3(g) g) H = –142.3 kJ/mol

(a) 140 gm at C (b) 540 gm at C O2 (g) g H = +495.0 kJ/mol

(c) 140 gm at B (d) 540 gm at B The enthalpy change ( H) for the following
reaction is
69. A 20gm bullet whose specific heat is 5000 J / (kg-
ºC) and moving at 2000 m/s plunges into a 1.0 kg NO (g) + O (g) g
block of wax whose specific heat is 3000 J /(kg-ºC). (a) –304.1 kJ/mol
Both bullet and wax are at 25 ºC and assume that
(i) the bullet comes to rest in the wax and (ii) all its (b) +304.1 kJ/mol
kinetic energy goes into heating the wax. Thermal
temperature of the wax in ºC is close to (c) -403.1 kJ/mol

(a) 28.1 (b) 31.5 (d) +403.1 kJ/mol

(c) 37.9 (d) 42.1 74. A 1.85 g sample of an arsenic-containing pesticide


was chemically converted to AsO43– (atomic mass
70. A"V" shaped rigid body has two identical uniform of As = 74.9) and titrated with Pb2+ to form Pb3
arms. What must be the angle between the two (AsO4)2. If 20 mL of 0.1 M Pb2+ is required to reach
arms so that when the body is hung from one the equivalence point, the mass percentages of
end, the other arm is horizontal ? arsenic in the pesticide sample is closest to
(a) cos–1 (1/3) (b) cos–1 (1/2) (a) 8.1 (b) 2.3
(c) cos–1 (1/4) (d) cos–1 (1/6) (c) 5.4 (d) 3.6
75. When traded with conc. HCl2 , MnO2 yields gas
(X) which further reacts with Ca(OH2) to generate
71. In the following reactions, X,Y and Z are a white solid (Y) reacts with dil. HCl to produce
the same gas X. the solid Y is
(a) CaO (b) CaCl2
(c) Ca(OCl)Cl (d) CaCO3

(a) X = CH3Cl;Y = anhydrous AlCl3; Z = HNO3 76. The atmospheric pressure is 760 mm Hg at the
sea level. Which of the following ranges is nearest
+ H2SO4 to the partial pressure of CO2 in mm Hg?
(b) X = CH3COCl;Y = anhydrous AlCl3; Z = HNO3 (a) 0.30–0.31 (b) 0.60–0.61
+ H2SO4 (c) 3.0–3.1 (d) 6.0–6.1
(c) X = CH3Cl;Y = conc. H2SO4; Z = HNO3 + H2SO4 77. A breeder crossed a pure bred tall plant having
white flowers to a pure bred short plant having
(d) X = CH3Cl; Y = dil. H2SO4; Z = HNO3
blue flowers. He obtained 202 F1 progeny and
72. 2,3-dibromobutane can be converted to 2-butyne found that they are all tall having white flowers.
in two-step reaction using Upon selfing these F 1 plants, he obtained a
progeny of 2160 plants. Approximately, how
(a) (i) HCl and (ii) NaH many of these are likely to be short and having
(b) (i) alcoholic KOH and (ii) Na NH2 blue flowers?

(c) (i) Na and (ii) NaOH (a) 1215 (b) 405

(d) (i) Br2 and (ii) NaH (c) 540 (d) 135
78. Match the different types of heart given in column
A with organisms given in the column B. Choose
the correct combination.
Column A Column B
P. Neurogenic heart i. Human
Q. Bronchial heart ii. King crab
R. Pulmonary heart iii. Shark (a) P and P (b) P and S

(a) P-ii, Q-iii, R-i (c) P and R (d) R and R

(b) P-iii, Q-ii, R-i 80. Match the enzymes in Group I with the reactions
in Group II. Select the correct combination.
(c) P-i, Q-iii, R-ii Group I Group II
(d) P-ii, Q-i, R-iii P. Hydrolase i. Inter-conversion of optical
isomers
79. Given below are the four schematics that describe
Q. Lyase ii. Oxidation and reduction of
the dependence of the rate of an enzymatic two substrates
reaction on temperature. Which of the following
combinations is true for thermophilic and R. Isomerase iii. Joining of two compounds
psychrophilic organisms? S. Ligase iv. Removal of a chemical
group from a substrate
v. Transfer of a chemical
group from one substrate
to another
(a) P-iv, Q-ii, R-iii, S-i
(b) P-v, Q-iv, R-i, S-iii
(c) P-iv, Q-i, R-iii, S-v
(d) P-i, Q-iv, R-v, S-ii

1. (c) 2. (c) 3. (b) 4. (d) 5. (a) 6. (b) 7. (b) 8. (c) 9. (c) 10. (b)

11. (d) 12. (c) 13. (a) 14. (c) 15. (d) 16. (d) 17. (c) 18. (c) 19. (c) 20. (b)

21. (a) 22. (d) 23. (c) 24. (d) 25. (a) 26. (d) 27. (b) 28. (b) 29. (a) 30. (d)

31. (b) 32. (a) 33. (a) 34. (b) 35. (a) 36. (a) 37. (a) 38. (d) 39. (d) 40. (d)

41. (c) 42. (a) 43. (d) 44. (a) 45. (c) 46. (c) 47. (b) 48. (d) 49. (a) 50. (a)

51. (b) 52. (b) 53. (c) 54. (a) 55. (a) 56. (a) 57. (b) 58. (d) 59. (b) 60. (a)

61. (c) 62. (b) 63. (c) 64. (a) 65. (b) 66. (a) 67. (c) 68. (a) 69. (c) 70. (a)

71. (a) 72. (b) 73. (a) 74. (c) 75. (c) 76. (a) 77. (d) 78. (a) 79. (a) 80. (b)
8. (c) Formed triangle will be right angle whose
sides are 3, 4, 5
2. (c) 165 516 Length of hypotenuse
circumradius = 2.5
= 16 × 164 × 516 2
= 16 × 1016
It is 18 digit number 9. (c)
3. (b) t2 = at + b ; a, b I+ t3 = at2 + bt
= a(at + b) + bt
= a2t + bt + ab
t3 = (a2 + b)t + ab , check possibility for a, b
from options.
From option (a) a2 + b = 4
ab = 3 possible
From option (b) a2 + b = 8
ab = 5 not possible M is circumcentre of ABC
From option (c) a2 + b = 10
a b
ab = 3 possible Co-ordinate of M is ,
2 2
From option (d) a2 + b = 6
ab = 5 possible & N is circumcentre of ABC
4. (d) (1 + a + b)2 = 3 (1 + a2 + b2) N = (0, 0) = B (Mid-point of P1 and P2).
1 a + a b + 1 b = 12 + a2 + b2 AC
1=a=b MN =
2
exactly one pair.
10. (b) a + 2b 1 (given)
A1 = a b 2 6
6. (b)
and A2 = b4

A1
a 2b2
A2
E F and 1 a + 2b 2(AM GM)

1 2 2ab
In AED, a + h = 9
2 2
....(1) 1 4 2ab
and in CFB, (6 – a) + h = 16
2 2
....(2) 1
we solving equation (i) and (ii) we get, h = 2.4 a2b2
64
11. (d) Let V1 and V2 are rates of burning for both
7. (b)
candles respectively and L is the length of
each candle

L L
V1 = and V2 =
5 3
After time ‘t’, let their lengths be 1
and 2
resp

L
1
=L– t
5
L North
and 2
=L– t 16. (d) B
3
25°
Now, 1
=3 2
(Given)
P
L L 3.18 KM
L t L 3
5 3

5 t
3 t
5
4t = 10
t = 2.5 hrs = 150 min.
12. (c) Let sides of cuboid a, a, h
So, 4a + 4h + 4a = 2(a2 + ah + ah)
2a + 2h + 2a = a2 + 2ah
a2 – 4a = 2h (1 – a) P
sin 25
(a2 – 1) + 1 – 4 (a – 1) – 4 = 2h (1 – a) 3.18
P = 3.18 sin 25º
(a – 1) (a + 1) – 4 (a – 1) – 3 = 2h (1 – a)
=1.34 KM along north
3 B = 3.18 cos 25º
2h = 4 a 1
a 1 = 2.88 KM along east
So a = 2 and h = 2 are the only integral 17.(c) A = = 3.95 ± 0.05
solution. b = 3.05 ± 0.05
13. (a) The possibility is dA = db + b d
|A1| = 1 ; |A2| = 2 ; |A3| = 3 ;........... , |Am| dA ldb bdl
=m A lb lb
1 + 2 + 3 + ....... + m d” 100{ Because all are dA db dl
disjoint} A lb l
m m 1 0.05 0.05
100 3.05 3.95
2
= 0.016 + 0.012
m < 14
= 0.028 × 12.05
14 set will have the same size as that of one
th
dA = 0.33
of the previous sets So, m = 13
12.05 0.34
15. (d) f (x y) = f (x) + f (y)
18. (c) Circular track
f (x) = logax So, f (12) = 24
loga 12 = 24
12 = a24 and f (8) = 15
loga 8 = 15
8 = a15 2 = a5
f (48) = loga 48 = loga 12 + loga 4
= loga 12 + loga 22
= 24 + 2 5
= 34
2 R' 24. (d)
Time period =
V'

2 2R I1 I2
4V
R
V current through wire 1 = 0 because I1 and I2
complete there loop.
= (T/2)
25. (a) r = r0 A1/3r0 = 1.3 × 10–15
19. (c) Since the voltage production is based upon
A.C. supply and this voltage is D.C which is Electrostatic force
constant. Therefore, no flux will change in 1 q1 q 2
secondary and no voltage will be induced. F
4 0 r2
20. (b) Pout = 1 × 109 h = 500 m
= 50% g = 10 m/s
Pout
Pin =

109
0.5
Pin = 2 × 109
9 109 1.6 10 19 1.6 10 19
mgh F
= 2 × 109 r0 2 A 2 / 3
time

9 109 1.6 10 19
1.6 10 19

2 109 2 F 2 2/ 3
m/t = 106 1.3 10 30
206
10 500 5
= 4 × 105 23.04 1039 10 38

= 400 m3 1.69 34.81


21. (a) According to Lenz’s Law option (a) is correct.
22. (d) 23.04 10
1.69 34.81

= 3.91 Newton
= 0.039 × 102
26. (d) air
=1
water
= 1.33

es2 = 1.6

23. (c) In setup B, A metal is placed, due to which


metal may get magnetized and it may also
Concave lens
exert force on current carrying wire but force
between two wire remain same however net
1 n2 1 1
force on wire may get charge due to magnetic 1
f n1 R1 R2
field produced by magnetized metal.
1 n2 1 1 U2
1 Max height = h +
f n1 R1 R2 2g

for diverging lens f must be – ve. U2


h
n2 2g
for this >1
n1
1 2 U2
Ut1 gt1
n2 > n 1 2 2g
Lens should be filled with liquid which has
more refractive index in comparison to liquid g 1 2 1 g2 2
t1 t 2 t1 gt1 t2 t1
in which lens is immersed. 2 2 2g 4
Ans (D) is the correct option as
g gt12 g 2
t2 t1 t1 t2 t1
es2
> water 2 2 8

1 t2 t1
2
27. (b) – h = – Ut1 + g t1 2 g 2 2
2 t 2 t1 t1 t1
2 4
1
h = Ut1 + g t1 2..... (1)
2
g 4t 2 t1 4t12 4t12 t 22 t1 2 2t1 t 2
2 4

g t12 t2 2 2t1 t 2
2 4

2
g t1 t2
2 4

1 g
h Ut 2 gt 22 ....(2) 2
2 t1 t2
8
1 1 2 Hence correct Answer is (B).
Ut1 Ut2 gt 2 2 gt1 (from 1 & 2)
2 2
28. (b)

K
E
r
F = qE
for 1st electron
g
U(t + t ) = (t – t ) × (t1 + t2 ) 1 mv12
2 2 1
(q)
r1 r1
g q = mv 2
U t2 t1
2
q V1 2 r2 n
v12
M V2 2 r1 n

V 2r n = V 2 r n

q
v1 30. (d) high density is not the reason for its uses in
M clinical thermometers.

q 12
Similarly v2 = 31. (b) % of C in NaHCO3 = 100 = 14.28
M 84
32. (a) Formic Acid
v1 v2
1
R1 1 33. (a) r
M
R2 2
(i) O3
34. (b) CH3 CH CH CH3 (ii)H2 O, Zn
2CH3CHO
2nR1
T1 v1 2
T2 2nR 2 35. (a) CH3 CH2 CH2 CH2 C CH2 CH2 CH3
6 4 3 ||
v2 5
CH2
1

2nR1 v2
v1 2nR 2 COOH
36. (a) | H 2 SO 4
CO2 CO H 2O
COOH
R1 v2 1
v1 v2
v1 R2 2 37. (a) 2LiOH CO2 Li2CO3 H2 O
29. (a) V(r) = Kr –n
48g LiOH requires CO2 = 44 g

dV 44
gravitational field = E = 1 g LiOH requires CO2 = = 0.916 g
dr 48

d n
38. (d) Na 2SO3 Oxidation number of S = +4
K r
dr
39. (d) Al2 O3 HCl AlCl3
= (– K) (– n) r– n – 1

Kn Al 2O3 NaOH NaAlO2


=
rn 1
40. (d) CH Alk. KMnO4
COOH
force on mass = E × M, where M = mass of ||| |
body CH COOH
MV1 2
ME1 =
r1 P1 T1
41. (c)
P2 T2
MV2 2
ME2 = 1 600
r2 P2 = 2
300
V1 2 r1 E1 42. (a) Along a period, radii goes on decreasing except
V2 2 r2 E2 noble gases

0 0 2 1
V1 2 r1 Kn r2 n 1 43. (d) Ca O Ca O
2
V2 2 r2 r1 n 1 Kn
45. (c) Photoelectric current × Intensity of light
49
46. (c) Length of human DNA So n
4
3.4 10 10 So n can be {1, 2, 3, ..... 12}
= Number of bases ×
dis tan ce between two Clearly product of the roots is 1
con sec utive base pairs
63. (c)
= 6.6 × 109 × 3.4 × 10–18
= 2.2 m.
47. (b) DPT consist of – diphtheria and tetanus toxoid
and heat killed – Pertussis cells.
48. (d) Bilirubin – Bile pigment
Rest 3 are enzymes:
lipase – lipid digestion
Amylase – starch digestion
Trypsin – protein digestion
49. (a) HCO3– maintain the pH of avian blood. ‘E’ is mid point of AD.
50. (a) Cells of squamous epithelium of Bowman’s 1
capsule are called podocyte. cos 2 =
3
51. (b) Given A = 20%
A = T = 20% 1
2 cos2 –1=
A + T = 40% G+C 100 – 40 60% 3
Hence, G= C = 30% 2
52. (b) Niacine, Nicotinamide and Tryptophan can cos2 =
3
cure pellagra.
53. (c) These are 64 codons. 3 are stop codons. 2
cos =
So 64 – 3 61. 3
55. (a) Ovary produces ‘Relaxin’ during parturition.
2 1
56. (a) Connecting link between reptile and sin = 1
mammals is platypus. 3 3
57. (b) 44 + XO 45 chromosomes
ED 1
59. (b) Lysine is not involved in gluconeogenesis. 1 3
60. (a) Syphilis is caused by treponema pallidum.
DH = ED sec

1 3
61. (c) a5 – a3 + a = 2 ; a R+ 3 2
Let f(a) = a5 – a3 + a – 2 ; {Note f (a) > 0 a
R} 1
for a = 3
6
a=3 1/6
= 1.2 {use calculator} 2
we get f(1.2) < 0 and at a = 4 1/6
f(4 ) > 0
1/6

a2 a2
so one root in a (3, 4) 64. (a) S1 = a2 + ......
4 16
62. (b) D = 49n – 4n2
= n(49 – 4n) a2 4a 2
D 0 for any n I . So roots are distinct
+
1 3
1
For roots to be real D 0 4
a2 a2 a2 a2 4a 2 67. (c)
S2 = ..... 1
2 8 32 6
1
4
S1
2
S2
65. (b) We need 3-digit number which is divisible by
4 & 5 both.
i.e.their last two digits are
00, 20, 40, 60 & 80
Now,ending with 00 are ‘9’.
{100, 200, ……, 900}.
If digit repeat other than ‘0’ then they are
{220, 440, 660, 880}
but 220 numbers can be permuted according
to the condition as {220, 202}
So, there are ‘8’ other favorable cases.
If the number have no digit repeated like 320. for surface AB:
320 can be permuted in 4 ways. sin 45º = sin … (1)
1 2 1
{302, 230, 320, 203} for surface AC:
So, such numbers are 8 × 4 × 4 = 128
2
sin ( – 1
)= 3
sin 45
Total favorable = 9 + 8 + 128 = 145
3
sin 45º = 2
cos 1
… (2)
145 29 = 90º
So, required prob. =
900 180 Squaring and adding equation (1) & (2)
66. (a) 2 2
2
1 3
2
2 2

1
+ 3
=2 2

68. (a) Uniform metal plate shape like a triangle

I = ICOM + Mx2
First it will decrease because x is increasing
and axis is coming closer to COM axis. After
Passing COM axis, M & I will again increase
I is minimum about the axis passing
through COM if we compare I about other
parallel axis

CM of triangular plate is on the median. If


we put a mass say m1 on C it will produce
torque about A which balance the torque
produce mg about A. Thus plate will can be
in equilibrium position m1g × 4 cos 37
= mg × y
4 For one arm to remain horizontal the net
m1g × 4 × = mg × y torque about O must be zero.
5
for this OP = OQ
5
m1 = m × y × l
16
OQ = cos
2
m1 5
y from figure
m 16
AE = AC + CE
m1 AE = cos + OQ
y<3 1
m
m1 < m l l
= cos + cos
m1 < 540 g 2 2

from given option Ans. (A) l


cos =
69. (c) MB = 20 × 10–3 Kg 3
CB = 5000 J / Kg-ºC hence = cos–1 (1/3)
V = 2000 M/s correct Answer is (A)
Mw = 1 Kg
CH3 CH3
Cw = 3000 J / Kg – ºC NO2
Anhyd. C o nc. HNO3
Tf = 25º C = 298 K 71. (a) +CH3Cl AlCl3
Conc. H2 SO4

1
MV2 = Mw Cw Tw + MB CB TB
2
Br Br
1
= MBV2 = Mw Cw ( Tw) + MB CB TB | |
2 72. (b) CH3 CH CH CH3
1
× 20 × 10–3 × 4 × 106 (i) alc. KOH CH3 C C CH3
2
(ii) alc. NaNH2
= ( T) {1 × 3000 + 20 × 10–3 × 5000}
40 × 103 = T {3000 + 100) 73. (a) NO O3 NO 2 O2 H = – 198.9 ...(i)

40 103 3
T= O3 O2 H = –142.3 ...(ii)
3100 2
T = 12.9 O3 2O 2 H = –495 ...(iii)
Tf – 25 = 12.9
Tf = 25 + 12.9 = 37.9ºC NO 0 NO 2 H = ?
70. (a) V shaped uniform rigid body H = (i) – (ii) – (iii)/2

495
H = –198.9 – (–142.3) – = –304.1 kJ/mole
2

2 3
74. (c) Number of moles of Pb3 ASO4 2 10
2 3
= .00133
So, moles of Arsenic = .00133
Weight of As = .00133 × 74.9 = .0996 g

.0996
% of As = 100 = 5.4
1.85

75. (c) MnO2 HCL MnCl2 Cl 2 X


Dihybrid ratios = TW : Tw : tW : tw
Ca OH 2
Cl2 CaOCl 2 Y
9 : 3:3 :1
CaOCl2 dil. HCl CaCl2 Cl2 H2 O Number of short and blue flower plants will
be
76. (a) PCO2 0.30 – 0.31 mm Hg
1
77. (d) TT WW × tt ww × 2160 135.
16
79. (a) Enzymes are made up of proteins. They have
Tt Ww 202 plants. a temperature optima at which their activity
is maximum. It decreasing both above and
Selfing below the optimum temperature.
(thermophiles– live in very high
temperature, while psychrophiles live in –
2160 plants. 20°C to +106°C temperature)
5. The largest non-negative integer k such that 24k
divides 13! is -
1 . Two distinct polynomials f(x) and g(x) are defined
as follows : (a) 2 (b) 3
f(x) = x2 + ax + 2; g(x) = x2 + 2x + a. (c) 4 (d) 5
If the equations f(x) = 0 and g(x) = 0 have a 6. In a triangle ABC, points X and Y are on AB and
common root then the sum of roots of the AC, respectively, such that XY is parallel to BC.
equation f(x) + g(x) = 0 is - Which of the two following equalities always hold ?
(Here [PQR] denotes the area of triangle PQR)-
1 I. [BCX] = [BCY]
(a) (b) 0
2 II. [ACX].[ABY] = [AXY].[ABC]
1 (a) Neither I nor II (b) I only
(c) (d) 1
2 (c) II only (d) Both I and II
2. If n is the smallest natural number such that n + 7. Let P be an interior point of a triangle ABC. Let
2n + 3n + ..... + 99n is a perfect square, then the Q and R be the reflections of P in AB and AC,
number of digits in n2 is - respectively. If Q, A, R are collinear then A
equals -
(a) 1 (b) 2
(a) 30º (b) 60º
(c) 3 (d) more than 3
(c) 90º (d) 120º
3. Let x, y, z be positive reals. Which of the following
implies x = y = z ? 8. Let ABCD be a square of side length 1, and a
circle passing through B and C, and touching AD.
I. x3 + y3 + z3 = 3xyz
The radius of is -
II. x3 + y2z + yz2 = 3xyz
III. x3 + y2z + z2x = 3xyz 3 1
(a) (b)
IV. (x + y + z) = 27 xyz
3 8 2
(a) I, IV only (b) I, II, IV only 1 5
(c) I, II and III only (d) All of them (c) (d)
2 8
4. In the figure given below, a rectangle of perimeter
9. Let ABCD be a square of side length 1. Let P,Q,R,S
76 units is divided into 7 congruent rectangles :
be points in the interiors of the sides AD, BC, AB,
CD, respectively, such that PQ and RS intersect
3 3
at right angles. If PQ = then RS equals -
4
2
(a)
3

3 3
(b)
4
What is the perimeter of each of the smaller
rectangles ? 2 1
(c)
(a) 38 (b) 32 2
(c) 28 (d) 19
(d) 4 2 2
10. In the figure given below, if the areas of the two
regions are equal then which of the following is
true ? 16. In an experiment, mass of an object is measured
by applying a known force on it, and then
measuring its acceleration. If, in the experiment,
the measured values of applied force and the
measured acceleration are F = 10.0 ± 0.2 N and
a = 1.00 ± 0.01 m/s2, respectively, the mass of the
object is -
(a) 10.0 kg (b) 10.0 ± 0.1 kg
(a) x = y (b) x = 2y (c) 10.0 ± 0.3 kg (d) 10.0 ± 0.4 kg
(c) 2x = y (d) x = 3y 17. A hollow tilted cylindrical vessel of negligible mass
rest on a horizontal plane as shown. The diameter
11. A man standing on a railway platform noticed that of the base is a and the side of the cylinder makes
a train took 21 seconds to cross the platform (this an angle with the horizontal. Water is then
means the time elapsed from the moment the slowly poured into the cylinder. The cylinder
engine enters the platform till the last topples over when the water reaches a certain
compartment leaves the platform) which is 88 height h, given by
metres long, and that it took 9 seconds to pass
him. Assuming that the train was moving with
uniform speed, what is the length of the train in
meters ?
(a) 55 (b) 60
(c) 66 (d) 72
12. The least positive integer n from which (a) h = 2a tan (b) h = a tan2
1 a
3
n 1 3
n is - (c) h = a tan (d) h = tan
12 2
18. An object at rest at the origin begins to move in
(a) 6 (b) 7
the +x direction with a uniform acceleration of 1
(c) 8 (d) 9 m/s2 for 4s and then it continues moving with a
13. Let n > 1 be an integer. Which of the following uniform velocity of 4 m/s in the same direction.
sets of numbers necessarily contains a multiple The x-t graph for object’s motion will be -
of 3 ?
(a) n19 – 1, n19 + 1 (b) n19, n38 – 1
(c) n38, n38 + 1 (d) n38, n19 – 1
14. The number of distinct primes dividing 12! + 13!
(a) (b)
+ 14! is -
(a) 5 (b) 6
(c) 7 (d) 8
15. How many ways are there to arrange the letters
of the word EDUCATION so that all the following
three conditions hold ?
(c) (d)
the vowels occur in the same order (EUAIO)
the consonants occur in the same order
(DCTN) 19. If the axis of rotation of the earth were extended
no two consonants are next to each other into space then it would pass close to -
(a) 15 (a) the moon
(b) 24 (b) the sun
(c) 72 (c) the pole star
(d) 120 (d) the centre of mass of all the planets in the
solar system
20. Methane is greenhouse gas because - 26. A light bulb of resistance R =16 is attached in
(a) it absorbs longer wavelengths of the series with an infinite resistor network with
electromagnetic spectrum while transmitting identical resistances r as shown below. A 10 V
shorter wavelengths. battery drives current in the circuit. What should
(b) it absorbs shorter wavelengths of the be the value of r such that the bulb dissipates
electromagnetic spectrum while transmitting about 1 W of power.
longer wavelengths
(c) it absorbs all wavelengths of the
electromagnetic spectrum
(d) it transmits all wavelengths of the
electromagnetic spectrum
21. A parachutist with total weight 75 kg drops
vertically onto a sandy ground with a speed of 2 (a) 14.8 (b) 29.4
ms–1 and comes to a halt over a distance of 0.25 (c) 7.4 (d) 3.7
m. The average force from the ground on her is 27. A ball is launched from the top of Mt. Everest
close to - which is at elevation of 9000 m. The ball moves
(a) 600 N (b) 1200 N in circular orbit around earth. Acceleration due
(c) 1350 N (d) 1950 N to gravity near the earth’s surface is g. The
22. The beta particles of a radioactive metal originate magnitude of the ball’s acceleration while in orbit
from - is -
(a) the free electrons in the metal (a) close to g/2
(b) the orbiting electrons of the metal atoms (b) zero
(c) the photons released from the nucleus (c) much greater than g
(d) the nucleus of the metal atoms (d) nearly equal to g
23. An optical device is constructed by fixing three 28. A planet is orbiting the sun is an elliptical orbit.
identical convex lenses of focal lengths 10 cm each Let U denote the potential energy and K denote
inside a hollow tube at equal spacing of 30 cm the kinetic energy of the planet at an arbitrary
each. One end of the device is placed 10 cm away point on the orbit. Choose the correct statement-
from a point source. How much does the image (a) K < |U| always
shift when the device is moved away from the (b) K > |U| always
source by another 10 cm ?
(c) K = |U| always
(a) 0 (b) 5 cm
(d) K = |U| for two positions of the planet in the
(c) 15 cm (d) 45 cm
orbit
24. An isosceles glass prism with base angles 40º is
29. One mole of ideal gas undergoes a linear process
champed over a tray of water in a position such
as shown in figure below. Its temperature
that the base is just dipped in water. A ray of
expressed as function of volume V is -
light incident normally on the inclined face suffers
total internal reflection at the base. If the
refractive index of water is 1.33 then the condition
imposed on the refractive index of the glass is-
(a) < 2.07 (b) > 2.07
(c) < 1.74 (d) > 1.74
25. A point source of light is moving at a rate of 2 cm-
s–1 towards a thin convex lens of focal length 10
cm along its optical axis. When the source is 15 P0 V0 P0 V
cm away from the lens the image is moving at- (a) (b)
R R
(a) 4 cm-s–1 towards the lens 2
(b) 8 cm-s–1 towards the lens P0 V V P0 V V
(c) R 1 (d) R 1 V0
(c) 4 cm-s–1 away from the lens V0
(d) 8 cm-s–1 away from the lens
30. The international space station is maintained in a 38. The major products of the following reaction
nearly circular orbit with a mean altitude of
ZnS (s) + O2 (g) heat are
330 km and a maximum of 410 km. An astronaut is
floating in the space station’s cabin. The acceleration (a) ZnO and SO2 (b) ZnSO4 and SO3
of astronaut as measured from the earth is - (c) ZnSO4 and SO2 (d) Zn and SO2
(a) zero 39. If Avogadro’s number is A0, the number of sulphur
(b) nearly zero and directed towards the earth atoms present in 200 mL of 1N H2SO4 is
(c) nearly g and directed along the line of travel (a) A0/5 (b) A0/2
of the station (c) A0/10 (d) A0
(d) nearly g and directed towards the earth 40. The functional group present in a molecule having
the formula C12O9 is
(a) carboxylic acid (b) anhydride
31. The percentage of nitrogen by mass in (c) aldehyde (d) alcohol
ammonium sulphate is closest to (atomic masses 41. A sweet smelling compound formed by reacting
H = 1, N =- 14, O = 16, S = 32) acetic acid with ethanol in the presence of
(a) 21% (b) 24% hydrochloric acid is
(c) 36% (d) 16% (a) CH3COOC2H5 (b) C2H5COOH
32. Mendeleev’s periodic law states that the properties (c) C2H5COOCH3 (d) CH3OH
of elements are a periodic function of their 42. Among Mg, Cu, Fe, Zn, the metal that does not
(a) reactivity of elements produce hydrogen gas in reaction with
(b) atomic size hydrochloric acid is
(c) atomic mass (a) Cu (b) Zn
(d) electronic configuration (c) Mg (d) Fe
33. Maximum number of electrons that can be 43. The maximum number of isomeric ethers with
accommodated in the subshell with azimuthal the molecular formula C4H10O is
quantum number l = 4, is (a) 2 (b) 3
(a) 10 (b) 8 (c) 4 (d) 5
(c) 16 (d) 18 44. The number of electrons required to reduce
34. The correct order of acidity of the following chromium completely in Cr2O2 to Cr3+ in acidic
compounds is medium, is
(a) 5 (b) 3
(c) 6 (D 2
45. At constant pressure, the volume of a fixed mass
of a gas varies as a function of temperature as
shown in the graph

(a) 1 > 2 > 3 (b) 1 > 3 > 2


(c) 3 > 1 > 2 (d) 3 > 2 > 1
35. Reaction of 2-butene with acidic KMnO4 gives
(a) CH3CHO (b) HCOOH
(c) CH3CH2OH (d) CH3COOH
36. The gas released when baking soda is mixed with
vinegar, is
(a) CO (b) CO2
(c) CH4 (d) O2
37. The element which readily forms an ionic bond The volume of the gas at 300°C is larger than
has the electronic configuration that at 0°C by a factor of
(a) 1s22s22p3 (b) 1s22s22p1 (a) 3 (b) 4
(c) 1s 2s 2p
2 2 2
(d) 1s 2s 2p 3s
2 2 6 1 (c) 1 (d) 2
53. Deficiency of which one of the following vitamins
can cause impaired blood clotting ?
46. Excess salt inhibits bacterial growth in pickles by (a) Vitamin B (b) Vitamin C
(a) endosmosis (b) exosmosis (c) Vitamin D (d) Vitamin K
(c) oxidation (d) denaturation 54. Which one of the following is detrimental to soil
47. Restriction endonucleases are enzymes that are fertility ?
used by biotechnologists to - (a) Saprophytic bacteria
(a) cut DNA at specific base sequence (b) Nitrosomes
(b) join fragments of DNA (c) Nitrobacter
(c) digest DNA from the 3 end (d) Pseudomonas
(d) digest DNA from the 5 end 55. In which one of the following phyla is the body
48. Enzyme X extracted from the digestive system segmented ?
hydrolyses peptide bonds. Which of the following (a) Porifera (b) Platyhelminthes
are probable candidates to be enzyme X ?
(c) Annelida (d) Echinodermata
(a) Amylase (b) Lipase
56. Widal test is prescribed to diagnose
(c) Trypsin (d) Maltase
(a) Typhoid (b) Pneumonia
49. A person with blood group AB has
(c) Malaria (d) Filaria
(a) antigen A and B on RBCs and both anti-A and
57. Which, among grass, goat, tiger and vulture, in a
anti-B antibodies in plasma
food chain, will have the maximum concentration
(b) antigen A and B on RBCs, but neither anti-A of harmful chemicals in its body due to
nor anti-B antibodies in plasma contamination of pesticides in the soil ?
(c) no antigen on RBCs but both anti-A and anti- (a) Grass since it grows in the contaminated soil
B antibodies in plasma
(b) Goat since it eats the grass
(d) antigen A on RBCs and anti-B antibodies in
(c) Tiger since it feeds on the goat which feeds
plasma
on the grass
50. Glycolysis is the breakdown of glucose to pyruvic
(d) Vulture since it eats the tiger, which in turn
acid. How many molecules of pyruvic acid are
eats the goat, which eats the grass
formed from one molecule of glucose ?
58. Considering the average molecular mass of a base
(a) 1
to be 500 Da, what is the molecular mass of a
(b) 2 double stranded DNA of 10 base pairs ?
(c) 3 (a) 500 Da
(d) 4 (b) 5 kDa
51. The process of transfer of electrons from glucose (c) 10 kDa
to molecular oxygen in bacteria and mitochondira
(d) 1 kDa
is known as -
59. Which of the following pairs are both
(a) TCA cycle
polysaccharides ?
(b) Oxidative phosphorylation
(a) Cellulose and glycogen
(c) Fermentation
(b) Starch and glucose
(d) Glycolysis
(c) Cellulose and fructose
52. Which one of the following cell types is a part of
(d) Ribose and sucrose
innate immunity ?
60. Which one of the following is a modified leaf ?
(a) Skin epithelial cells
(a) Sweet potato
(b) B cells
(b) Ginger
(c) T lymphocytes
(c) Onion
(d) Liver cells
(d) Carrot
61. A triangular corner is cut from a rectangular piece 66. A girl sees through a circular glass slab (refractive
of paper and the resulting pentagon has sides 5, index 1.5) of thickness 20 mm and diameter 60
6, 8, 9, 12 in some order. The ratio of the area of cm to the bottom of a swimming pool. Refractive
the pentagon to the area of the rectangle is - index of water is 1.33. The bottom surface of the
slab is in contact with the water surface.
11 13
(a) (b)
18 18
15 17
(c) (d)
18 18
62. For a real number x, let [x] denote the largest
integer less than or equal to x, and let {x} = x –
[x]. The number of solutions x to the equation [x] The depth of swimming pool is 6m. The area of
{x} = 5 with 0 x 2015 is - bottom of swimming pool that can be seen through
(a) 0 (b) 3 the slab is approximately -
(c) 2008 (d) 2009 (a) 100 m2 (b) 160 m2
63. Let ABCD be a trapezium with AD parallel to BC. (c) 190 m2 (d) 220 m2
Assume there is a point M in the interior of the 67. 1 Kg of ice at – 20ºC is mixed with 2 Kg of water
segment BC such that AB = AM and DC = DM. at 90ºC. Assuming that there is no loss of energy
Then the ratio of the area of the trapezium to to the environment, what will be the final
the area of triangle AMD is - temperature of the mixture ? (Assume latent heat
(a) 2 of ice = 334.4 KJ/Kg, specific heat of water and
(b) 3 ice are 4.18 kJ/(kg.K) and 2.09 kJ/(kg.K),
(c) 4 respectively.)
(d) not determinable from the data (a) 30ºC (b) 0ºC
64. Given area three cylindrical buckets X, Y, Z whose (c) 80ºC (d) 45ºC
circular bases are of radii 1, 2, 3 units, 68. A rigid body in the shape of a “V” has two equal
respectively. Initially water is filled in these arms made of uniform rods. What must the angle
buckets upto the same height. Some water is then between the two rods be so that when the body is
transferred from Z to X so that they both have suspended from one end, the other arm is
the same volume of water. Some water is then horizontal ?
transferred between X and Y so that they both 1
cos–1 1 (b) cos–1
have the same volume of water. If hY, hZ denote
3 2
the heights of water at this stage in the buckets
hY 1 1
Y, Z respectively, then the ratio equals- (c) cos–1 (d) cos–1
hZ 4 6
4 69. A point object is placed 20 cm left of a convex lens
(a) (b) 1 of focal length f = 5 cm (see the figure). The lens
9
is made to oscillate with small amplitude A along
9 81
(c) (d) the horizontal axis. The image of the object will
4 40 also oscillate along the axis with
65. The average incomes of the people in two villages
are P and Q, respectively. Assume that P Q. A
person moves from the first village to the second
village. The new average incomes are P’ and Q’,
respectively. Which of the following is not
possible?
(a) P’ > P and Q’ > Q (b) P’ > P and Q’ < Q
(c) P’ = P and Q’ = Q (d) P’ < P and Q’ < Q
(a) amplitude A/9, out of phase with the oscillation 74. The Amount of Ag (atomic mass = 108) deposited
of the lens. at the cathode when a current of 0.5 amp is passed
(b) amplitude A/3, out of phase with the through a solution of AgNO3 for 1 hour is closest
oscillations of the lens. to
(c) amplitude A/3, in phase with the oscillations (a) 2 g (b) 5 g
of the lens (c) 108 g (d) 11 g
(d) amplitude A/9, in phase with the oscillations 75. The major produced of the reaction is -
of the lens
70. Stoke’s law states that the viscous drag force F
experienced by a sphere of radius a, moving with
a speed v through a fluid with coefficient of
viscosity , is given by F = 6 av
If this fluid is flowing through a cylindrical pipe
of radius r, length and a pressure difference of
(a) I (b) II
P across its two ends, then the volume of water
V which flows through the pipe in time t can be (c) III (d) IV
written as
a
V p b 76. Genomic DNA is digested with Alu, I, a restriction
k rc ,
t l enzyme which is a four base-pair cutter. What is
where k is a dimensionless constant. Correct the frequency with which it will cut the DNA
values of a, b and c are - assuming a random distribution of bases in the
genome ?
(a) a = 1, b = –1, c = 4
(a) 1/4 (b) 1/24
(b) a = –1, b = 1, c = 4
(c) 1/256 (d) 1/1296
(c) a = 2, b = –1, c = 3
77. If rice is cooked in a pressure cooker on the
(d) a = 1, b = –2, c = –4
Siachen glacier, at sea beach, and on Deccan
plain, which of the following is correct about the
time taken for cooking rice ?
71. When 262 g of xenon (atomic mass = 131) reacted
(a) Gets cooked faster on the Siachen glacier
completely with 152 g of fluoride (atomic mass =
19), a mixture of XeF2 and XeF6 was produced. (c) Gets cooked faster on Deccan plain
The molar ration XeF2 : XeF6 is (b) Gets cooked faster at sea beach
(a) 1 : 2 (b) 1 : 4 (d) Gets cooked at the same time at all the three
(c) 1 : 1 (d) 1 : 3 places
72. Reaction of ethanol with conc. sulphuric acid at 78. A few rabbits are introduced in an un-inhabited
170 ºC produces a gas which is then treated with island with plenty of food. If these rabbits breed
bromine is carbon tetrachloride. The major in the absence of any disease, natural calamity
product obtained in this reaction is and predation, which one of the following graphs
best represents their population growth ?
(a) 1,2-dibromoethane
(b) Ethylene glycol
(c) Bromoethane
(d) Ethyl sulphate
(a) (b)
73. When 22.4 L of C4H8 at STP is burnt completely,
89.6 L of CO2 gas at STP and 72g of water are
produced.
The volume of the oxygen gas at STP consumed
in the reaction is closest to
(a) 89.6 L (b) 112 L (c) (d)
(c) 134.4 L (d) 22.4 L
79. What is the advantage of storing glucose as 80. A line is drawn from the exterior of an animal
glycogen in animals instead of as monomeric cell to the centre of the nucleus, crossing
glucose ? through one mitochondrion. What is the
(a) Energy obtained from glycogen is more than minimum number of membrane bilayers that
that from the corresponding glucose monomers the line will cross ?
(b) Glucose present as monomers within the cell (a) 4
exerts more osmotic pressure than a single (b) 3
glycogen molecule, resulting in loss of water (c) 8
from the cells.
(d) 6
(c) Glucose present as monomers within the cell
exerts more osmotic pressure than a single
glycogen molecule, resulting in excess water
within the cells.
(d) Glycogen gives more rigidity to the cells.

1. (c) 2. (c) 3. (b) 4. (c) 5. (b) 6. (d) 7. (c) 8. (d) 9. (b) 10. (b)

11. (c) 12. (c) 13. (b) 14. (a) 15. (a) 16. (c) 17. (c) 18. (b) 19. (c) 20. (a)

21. (c) 22. (d) 23. (a) 24. (b) 25. (d) 26. (a) 27. (d) 28. (a) 29. (c) 30. (d)

31. (a) 32. (c) 33. (d) 34. (c) 35. (d) 36. (b) 37. (d) 38. (a) 39. (c) 40. (b)

41. (a) 42. (a) 43. (b) 44. (c) 45. (d) 46. (b) 47. (a) 48. (c) 49. (b) 50. (b)

51. (b) 52. (a) 53. (d) 54. (d) 55. (c) 56. (a) 57. (d) 58. (c) 59. (a) 60. (c)

61. (d) 62. (d) 63. (b) 64. (d) 65. (c) 66. (b) 67. (a) 68. (a) 69. (a) 70. (a)

71. (c) 72. (a) 73. (c) 74. (a) 75. (a) 76. (c) 77. (d) 78. (a) 79. (c) 80. (a)
4. (c)
1 . (c) Let ‘ ’ be the common root
2
+a +2=0 ....(i)
and 2 + 2 + a = 0 ...(ii)
from eq. (i) and (ii) we get
(a – 2) + 2 – a = 0
= 1 is common root.
12 + a + 2 = 0 a = – 3.
Now f(x) + g(x) = 0
2(3a) + 2(a + b) = 76
2x2 + (a + 2) x + (a + 2) = 0
2x2 – x – 1 = 0 4a + b = 38 ...(i)
and 3a = 4b ...(ii)
–b 1
Solving eq. (i) and (ii) and we get
a 2
16a + 3a = 38 × 4
1 19 a = 38 × 4
Sum of roots = .
2 a=8
2. (c) n + 2n + 3n + ... + 99n = k 2
b=6
n(1 + 2 + 3 + ... + 99) = k2 Now, perimeter of smaller rectangle
99 100 = 2(a + b) = 2 (8 + 6) = 28.
n k2
2 5. (b) Let 13! = 2 .3 .m n

n × 99 × 50 = k 2
Where m is maximum possible value & n is
n = 11.2 = 22 also maximum possible value
n = 484
2
13 13 13 13
So m = .......
No. of digits in n2 is 3. 2 4 8 16
= 6 + 3 + 1 = 10
x3 y3 z3
3. (b) I. = (x3y3z3)1/3
3 13 13 13
n= .......
Hence x = y = z {AM = GM} 3 9 27

x3 y 2 z yz 2 =2+1=3
II. (x y z )
3 3 3 1/3
(AM = GM) So, 13! = 210.33. .
3
= 2.(23.3) (23.3) (23.3) . = 2.(24)3 .
III. x3 + y2z + z2x = 3xyz
k=3
3 y 2z yz 2 2 6. (d)
x z x 1/ 4
2 2 x 4 y 4 z4
4 4

3xyz xyz
4 2
(which is not not possible)

x y z
IV. (xyz)1/3 Clearly ar (BCX) = ar (BCY) { s between
3 parallel lines & same base}
(x + y + z)3 = 27 xyz. [BCX] = [BCY]
(I) is true
1 10. (b)
(II) ar ( ACX) = AC.AX sin A
2
1
ar( ABY) = AB.AY sin A.
2
ar ( AXY) = 1 AX.AY sin A
2
1
ar ( ABC) = AB.AC sin A.
2
Clearly [ACX].[ABY] = [AXY].[ABC] 1
Area of Ist figure = x × 2y + (2y + y) x.
(II) is true. 2
7. (c) 7xy
=
2

2 + 2 = 180º
+ = 90º
A = 90º
8. (d)
Area of IInd figure
1
= 2x.2y – 2y. 2y. = 4xy – y2
2
area (I) = area (II)

Let O be centre of circle. 7xy


= 4xy – y2
2
OM = radius = r
2y2 = xy
2
2 1
Now, r 2 1 r 2y = x .
2
11. (c)
5
2r
4
8r = 5
5
r
8
9. (b) Let speed of train be v m/s.
v × 21 = 88 +
21 v = 88 +
and
v×9=

PQM RSN
l
3 3 21 × = 88 +
RS = PQ = 9
4
= 66 sec.
1 17. (c)
12. (c) (n + 1)1/3 – n1/3 <
12
Taking cube on both the side
(n + 1) – n – 3 (n + 1)1/3 n1/3 ((n + 1) – n1/3 )
3
1
< .
12
1 1
1 – 3 n1/3 (n + 1)1/3 × < 12 3
(COM at mid pt of filled cylinder)
12
(12)3 – 1 < 3.(12)2 n1/3 (n + 1)1/3 BC BC h
sin ; AC ; AC
1727 AC sin sin
< n1/3 (n + 1)1/3 a
3 144
1727
3
cos = 2
n(n + 1) > h
3 144
2sin
n (n + 1) > 63.88
a sin a sin
n=8 cos =
h cos
14. (a) 12! + 13! + 14!
= 12!(1 + 13 + 14 × 13) sin
h = a tan tan
= 12! × 196 cos
Prime nos. are 2, 3, 5, 7, 11 18. (b)
Total = 5
16. (c) Given that,
Force F = 10.0 ± 0.2 N, a = 1.00 ± 0.01 m/s2
F
F = ma m=
a
1 2
m = 10.0 x= at parabolic for 0 to 4 sec
1.00 2
m = 10.0 kg 1
[at t = 4 sec x = × (1) (4)2 = 8m]
For error (F = ma) 2
m1a1 F–1 = const. then after (v = 4 m/s) v = 4
v=4
dm da dF
0
m a F dx
4
[By taking log and differentiate] dt
x t
m F a
dx 4dt
m F a max 8 4

m 0.2 0.01 x – 8 = 4 (t – 4)
m 10.0 1.00 x = 4t – 8 (straight line)
3
m m
100
3
m 10kg
100
m 0.3 kg
mass m = (10.0 ± 0.3 kg)
19. (c) Pole star is a visible star that is
approximately aligned with the axis of rotation
of earth.
20. (a) Absorbs infrared radiation thus it absorbs
w
longer wavelength of EMwave spectrum sin 40º >
D
while transmitting shorter wavelength.
21. (c) Weight of Parachutist = 75 kg >
w
D
D
1
K.E. = 0 – mv2 > 2.07
2
25. (d)
1
K.E. = – 75 (2)2
2
K.E. = – 150 J
Now, total work done by forces = – 150 J
Given that,
– F. x = –150 J
f = 10 cm
150
F= (avg force) u = – 15cm, f = + 10 cm
x
By mirror formula,
150
F= 1 1 1
0.25
v u f
F = 600 N (upward direction)
fu
v
u f
10 15
v
15 10
FR – mg = F v = + 30 cm
FR = F + mg dv v 2 du
Now,
FR = 600 + 750 = 1350 N dt u 2 dt
(resistive force by ground) 2
dv 30
23. (a) (+ 2 cm/s)
dt 15
dv
= + 8 cm/s(away from lens)
dt
26. (a)

Distance between object to image in both case


Let assume Req = PQ = x
is 90 cm. Because object is at same position
so image also be at same position in both
cases.
24. (b)

rx
ReqPQ = r +
r x
r2 rx rx
x=
r x
For Total internal Refelction rx + x2 = r2 + 2rx
40º > c
x2 – rx – r2 = 0
sin 40º > sin c r r2 4r 2
x=
r 2
sin 40º >
D
r 1 5 P0
P = P0 – V ......(1)
V0
2
Power in bulb = 1 watt By Ideal gas eqn.
i2R = 1 PV = RT .....(2)

i2 × 16 = 1 from (1) and (2)

1 RT P0
i= ampere P0 V
4 V V0
10 P0 V P0 V 2 P0 V2
T V
Also, i R R PQ R RV0 R V0

1 10 P0 V V
T 1
4 r R V0
16 1 5
2 GM
30. (d) g =
r r 2
16 1 5 40 1 5 24 R h
2 2
As h << R
r = 14.8
27. (d) At earth surface acceleration due to gravity GM
g towards the earth
is given by, R2
GM 28
g= 31. (a) % of Nitrogen = 100 = 21.21 %
R2 132
Now, At height = 9000 m, Radius of orbit of 33. (d) l = 4
ball is 6400 + 9 km
Number of orbitals = 2l + 1
radius r > R
Max. number of electrons = 2(2l + 1)
Radius is almost equal to radius of earth.
= 18
Now, (v) orbital velocity of ball is given by =
34. (c) NO2 is electron with drawing and –OCH3 group
GM has +R – effect.
r
35. (d) CH3 CH CH CH3 Alk.KmnO 4
2CH3 COOH
v2 GM
Acceleration =
r r2 36. (b) CH3COOH NaHCO3
CH3COONa H2O CO2
As r is very near to R
37. (d) Largest size and one valence electron to loose
GM
Acceleration = g
R2 38. (a) ZnS O2 ZnO SO2
28. (a) Planet sun system is bounded system 39. (c) Moles of S = 0.1
Total energy of the system is negative Atoms of S = .1 A0
Total energy = Kinetic energy + Potential
energy 40. (b) Anhydride of COOH
TE = KE + PE COOH COOH
K – |U| {PE is negative here}
as TE is negative COOH
|U|>K HOOC
COOH
29. (c)
41. (a) CH3 COOH C2 H5OH H
CH3 COOC 2 H 5 H 2O

42. (a) Cu is less reactive than hydrogen


43. (b) CH 3 CH 2 CH2 O CH 3 and

CH 3CH 2 O CH 2CH 3 and


CH3
61. (d)
CH – O – CH3
CH3

44. (c) Cr2 O72 14H 6e 2Cr 3


7H2O

Vgas at 300 C 500


45. (d) 2
Vgas at 0 C 250 Clearly x = 4 = 12 – 8
and y = 3
46. (b) Excessive salt in pickle causes exosmosis and
kill the bacteria. Now area of rectangle = 12 × 9 = 108
47. (a) Restriction endonuclease cut the DNA Area of pentagon = (12 × 9) – area of triangle
molecule by identifying a specific pallindromic 1
sequence. = 108 (– × 3 × 4) = 102
2
The DNA molecule is cut at a site which is
little away from the center of pallindromic Area of pentagon 102 17
sequence. Area of rec tan gle 108 18
48. (c) Since peptide bonds are found in protein and 63. (b)
trypsin as a protein digesting enzyme.
49. (b) In a person with blood group AB. Both
antigen A and B are present on RBC. But
both antibodies (anti-A, anti-B) are absent.
Glucose Pyrubic Acid
50. (b) (6c) (8c)
2 Pyrubic acid molecules are produced by Area of ABCD 3 1 3 2 3( 1 2 )3
each glucose molecule during Glycolysis. Area of AMD 1 2 ( 1 2 ) 1
51. (b) Electron transport system oxidative
phoshorylation x1 x2 .... x n
65. (c) P
52. (a) Innate immunity is the general defense of n
the body. y1 y2 .... y m
Macro phages, skin, Hcl in digestive juice kill Q
m
or resist infection.
x1 + x2 + ..... + xn = nP
53. (d) Vitamin K - Helps in synthesis of blood
and y1 + y2 +.....+ ym = mQ
clotting factors. Hence help in cloating.
Now if a person moves from Ist village to IInd
54. (d) Pseudomonas denitrificans converts nitrogen village then
compounds into nitrogen (N2)
56. (a) Serological test for typhoid. x1 x 2 .... x n 1
P'
57. (d) Maximum concentration of harmful n 1
substances will be in vulture. As it will be on
the top of the food chain and in n n
biomagnification toxic substance increases at nP – x = (n – 1)P P = P
n 1 n 1
successive trophic level.
58. (c) Give 10bp total bases 2 × 10 y1 y2
.... y m x n
and Q'
20 m 1
Molar mass of one base 500da mQ + xn = (m + 1)Q
Hence for 20 bases 20 × 500 If P = P
10,000 da. n xn
10 Kda. P 1
n 1 n 1
59. (a) Cellulose polymer of -glucose xn
P
Glycogen polymer of -glucose n 1 n 1
60. (c) In Onion modified leaves store food.
P xn 68. (a)
and when Q = Q
xn = Q = Q
In that case P = Q (which is not possible)
66. (b)

Using snell law When centre of mass of system and Hinged


point lie on one line then only system can
4
1 × sin 90 = sin r remain in equilibrium in given position.
3
AB = cos
3
sin r = l
4 AP = cos
2 2
3
tan r = AB
7 cos
2 AP
3 18
x = 6 tan r = 6.8 AB AP cos
7 7 2
(D) diameter = 2x + 0.6 2 × 6.8 + 0.6 = 14.2 l 2
2 cos = cos
D 2
3.14 14.2 2 2
Area = m2 160m 2
4 4 1 cos
2 cos
67. (a) 2
4cos = 1 + cos
3cos = 1
1
cos =
3
1 1
cos
3
69. (a)

Total heat gain is given by


= 20 × 2.09 + 334.4 KJ = 376.2 kJ
Total heat loss is given by = 752.4 kJ By mirror’s formula,
Now, Heat gain required = 752.4 – 376.2 1 1 1
376.2 kJ f v u
376.2 = ms T 1 1 1
376.2 = 3 × 4.18 × T v f u
T = 30 centigrate fu
v
Tfinal = 30ºC f u
v f 262
m 71. (c) Moles of Xe = 2
u f u 131
As lens is oscillating with small amplitude A.
Image will oscillate with m2A 152
Moles of F = 8
19
When lens move left then O will come near
to lens thus I will go away. Thus image is
2Xe 4F2 XeF2 XeF6
oscillating out of phase with respect to lens.
Moles of XeF2 = 1
5
m= Moles of XeF6 = 1
5 20
Conc.
72. (a) C2 H5OH CH2 = CH2 Br2
5 1 H2 SO4
15 3
CH2 CH2
2
1 A | |
Amplitude of image = A
3 9 Br Br
a
V p 22.4
70. (a) k b c
r , 73. (c) Moles of C4 H8 1
t l 22.4
Here,
C4 H8 6O2 4CO2 4H2 O
V volume
P pressure So volume of O2 required = 6 × 22.4 = 134.4L
coefficient of viscosity MIT
74. (a) W =
and 96500n
r radius
108 0.5 3600
By Using dimensional analysis W= =2g
96500 1
[M0L3T–1] = [M1L–2T–2]a [M1L–1T–1]b [L]c
+
[M0L3T–1] = [Ma+b L–2a–b+c T–2a–b]
75. (a) H H2 O
OH
a+b=0 ...(1)
–2a – b + c = 3 ...(2) 76. (c) Frequency of Alu I to cut DNA
a = –b 1 1
=
–2a – b = – 1 ...(3)
44 256
put the value of – 2a – b = –1 in equation (2)
Type of bases Number of base pairs in Alu I
–1 + c = 3
c=4 77. (d) In pressure cooker, there will be no effect of
changing the place as it is an isolated system
put a = – b in equation (3) hence atmospheric pressure wont play any
2b – b = –1 b = –1 role in it.
and a = 1 79. (c) Glucose Osmotically active, while
glycogen is inactive
Now, we get a = 1, b = – 1 and c = 4
80. (a) Cell membrane, mitochondrial membrane
Hence option (A) is correct. twice, nuclear evolop.
7 . A semi-circle of diameter 1 unit sits at the top of
a semi-circle of diameter 2 units. The shaded
1 . Let r be a root of the equation x2 + 2x + 6 = 0. The region inside the smaller semi-circle but outside
value of (r + 2) (r + 3) (r + 4) (r + 5) is equal to- the larger semi-circle is called a lune. The area
(a) 51 (b) – 51 of the lune is-
(c) –126 (d) 126
2. Let R be the set of all real numbers and let f be a
function from R to R such that
1
f (x) x f (1 x) 1,
2
for all x R. Then 2f(0) + 3f(1) is equal to-
(a) 2 (b) 0
(c) –2 (d) –4
3. The sum of all positive integers n for which 3 3
(a) (b)
6 4 4 24
13 23 (2n)3
12 22 n2 3 3
(c) (d)
is also an integer is 4 12 4 8
8. The angle bisectors BD and CE of a triangle ABC
(a) 8 (b) 9
are divided by the incentre I in the ratios 3 : 2
(c) 15 (d) Infinite and 2 : 1 respectively. Then the ratio in which I
4. Let x and y be two 2-digit numbers such that y is divides the angle bisector through A is-
obtained by reversing the digits of x. Suppose they (a) 3 : 1 (b) 11 : 4
also satisfy x2 – y2 = m2 for some positive integer (c) 6 : 5 (d) 7 : 4
m. The value of x + y + m is-
9. Suppose S 1 and S2 are two unequal circles;
(a) 88 (b) 112 AB and CD are the direct common tangents to
(c) 144 (d) 154 these circles. A transverse common tangent PQ
5 . Let p(x) = x2 – 5x + a and q(x) = x2 – 3x + b, where cuts AB in R and CD in S. If AB = 10, then RS is-
a and b are positive integers. Suppose hcf (p(x),
q(x)) = x – 1 and k(x) = lcm(p(x), q(x)). If the
coefficient of the highest degree term of k(x) is 1,
the sum of the roots of (x – 1) + k(x) is-
(a) 4 (b) 5
(c) 6 (d) 7
6. In a quadrilateral ABCD, which is not a trapezium,
it is known that DAB = ABC = 60º. Moreover,
CAB = CBD. Then-
(a) AB = BC + CD
(a) 8
(b) AB = AD + CD
(b) 9
(c) AB = BC + AD (c) 10
(d) AB = AC + AD (d) 11
10. On the circle with center O, points A, B are such 15. Suppose a2, a3, a4, a5, a6, a7 are integers such
that OA = AB. A point C is located on the tangent that
at B to the circle such that A and C are on the 5 a 2 a 3 a 4 a 5 a 6 a7
opposite sides of the line OB and AB = BC. The
7 2! 3! 4! 5! 6! 7!
line segment AC intersects the circle again at F.
Then the ratio BOF : BOC is equal to- where 0 aj < j for j = 2, 3, 4, 5, 6, 7. The sum
a2 + a3 + a4 + a5 + a6 + a7 is-
(a) 8 (b) 9
(c) 10 (d) 11

16. In the following displacement (x) vs time (t) graph


at which among P, Q and R is the object’s speed
increasing?
x
P

Q
(a) 1 : 2 (b) 2 : 3
(c) 3 : 4 (d) 4 : 5 (0, 0) t
R
11. In a cinema hall, the charge per person is
Rs. 200. On the first day, only 60% of the seats
were filled. The owner decided to reduce the price
by 20% and there was an increases of 50% in the
number of spectators on the next day. The (a) R only
percentage increase in the revenue on the second (b) P only
day was-
(c) Q and R only
(a) 50 (b) 40
(d) P, Q, R
(c) 30 (d) 20
17. A box, when hung from a spring balance shows a
12. The population of cattle in a farm increases so reading of 50 kg. If the same box is hung from
that the difference between the population in year the same spring balance inside an evacuated
n + 2 and that in year n is proportional to the chamber, the reading on the scale will be
population in year n + 1. If the populations in
(a) 50 kg because the mass of the box remains
years 2010, 2011 and 2013 were 39, 60 and 123,
unchanged.
respectively, then the population in 2012 was-
(b) 50 kg because the effect of the absence of the
(a) 81 (b) 84
atmosphere will be identical on the box and
(c) 87 (d) 90 the spring balance.
13. The number of 6-digit numbers of the form ababab (c) less than 50 kg because the weight of the
(in base 10) each of which is a product of exactly 6 column of air on the box will be absent.
distinct primes is-
(d) more than 50 kg because the atmosphere
(a) 8 (b) 10 buoyancy force will be absent.
(c) 13 (d) 15 18. Two positively charged spheres of masses m1, and
14. The houses on one side of a road are numbered m2 are suspended from a common point at the
using consecutive even numbers. The sum of the ceiling by identical insulating massless strings of
numbers of all the houses in that row is 170. If length . Charges on the two spheres are q1 and
there are at least 6 houses in that row and a is q2, respectively. At equilibrium both strings make
the number of the sixth house, then- the same angle with the vertical. Then
(a) 2 a 6 (a) q1 m1 = q2 m2
(b) 8 a 12 (b) m1 = m2
(c) 14 a 20 (c) m1 = m2 sin
(d) 22 a 30 (d) q2 m1 = q1 m2
19. A box when dropped from a certain height reaches 24. A charged particle initially at rest at O, when
the ground with a speed . When it slides from released follows a trajectory as shown. Such a
rest from the same height down a rough inclined trajectory is possible in the presence of
plane inclined at an angle 45° to the horizontal, it
reaches the ground with a speed /3. The coefficient
of sliding friction between the box and the plane is
(acceleration due to gravity is 10 ms–2)
8
(a) (a) electric field of constant magnitude and
9 varying direction.
1 (b) magnetic field of constant magnitude and
(b)
9 varying direction
2 (c) electric field of constant magnitude and
(c)
3 constant direction.
1 (d) electric and magnetic fields of constant
(d) magnitudes and constant directions which are
3
parallel to each other
20. A thin paper cup filled with water does not catch
fire when placed over a flame. This is because 25. Two equal charges of magnitude Q each are placed
at a distance d apart. Their electrostatic energy
(a) the water cuts off oxygen supply to the paper
is E. A third charge – Q/2 is brought midway
cup.
between these two charges. The electrostatic
(b) water is an excellent conductor of heat. energy of the system is now
(c) the paper cup does not become appreciably (a) – 2E (b) –E
hotter than the water it contains.
(c) 0 (d) E
(d) paper is a poor conductor of heat.
26. A bar magnet falls with its north pole pointing
21. Ice is used in a cooler in order to cool its contents. down through the axis of a copper ring. When
Which of the following will speed up the cooling viewed from above, the current in the ring will
process? be
(a) Wrap the ice in a metal foil. (a) clockwise while the magnet is above the plane
(b) Drain the water from the cooler periodically. of the ring, and counter clockwise while below
(c) Put the ice as single block. the plane of the ring.
(d) Crush the ice. (b) counter clockwise throughout.
22. The angle of a prism is 60°. When light is incident (c) counter clockwise while the magnet is above
at an angle of 60° on the prism, the angle of the plane of the ring, and clockwise while
emergence is 40°. The angle of incidence i for below the plane of the ring.
which the light ray will deviate the least is such (d) clockwise throughout.
that 27. Two identical bar magnets are held perpendicular
(a) i < 40° to each other with a certain separation, as shown
(b) 40° < i < 50° below. The area around the magnets is divided
(c) 50° < i < 60° into four zones.

(d) i > 60°


23. A concave lens made of material of refractive
index 1.6 is immersed in a medium of refractive
index 2.0 The two surfaces of the concave lens
have the same radius of curvature 0.2 m. The
lens will behave as a
(a) divergent lens of focal length 0.4 m.
Given that there is a neutral point it is located in
(b) divergent lens of focal length 0.5 m.
(a) Zone I (b) Zone II
(c) convergent lens of focal length 0.4 m.
(c) Zone III (d) Zone IV
(d) convergent lens of focal length 0.5 m.
28. A large number of random snap shots using a
camera are taken of a particle in simple harmonic
motion between x = – x0 and x = + x0 with origin 31. The isoelectronic pair is :
x = 0 as the mean position . A histogram of the (a) CO, N2 (b) O2, NO
total number of times the particle is recorded (c) C2, HF (d) F2, HCl
about a given position (Event no.) would most
32. The numbers of lone pair and bond pairs in
closely resemble
hydrazine are, respectively :
(a) 2 and 4 (b) 2 and 6
(c) 2 and 5 (d) 1 and 5
(a)
33. The volume of oxygen at STP required to burn
2.4 g of carbon completely is :
(a) 1.12 L (b) 8.96 L
(c) 2.24 L (d) 4.48 L
(b) 34. The species that exhibits the highest Rf value in
a thin layer chromatogram using a nonpolar
solvent on a silica gel plate is :

(c)
(a) (b)

(c) (d)
(d)

29. In 1911, the physical Ernest Rutherford discovered 35. The number of C–C sigma bonds in the compound
that atoms have a tiny , dense nucleus by shooting
positively charged particles at a very thin gold
foil. A key physical property which led Rutherford
to use gold was that it was
(a) electrically conducting
(b) highly malleable
(a) 16 (b) 17
(c) shiny
(c) 18 (d) 11
(d) non-reactive
36. If the radius of the hydrogen atom is 53 pm, the
30. Consider the following statements : radius of the He+ ion is closest to :
(I) All isotopes of an elements have the same (a) 108 pm (b) 81 pm
number of neutrons.
(c) 27 pm (d) 13 pm
(II) only one isotope of an element can be stable
37. The diamagnetic species is :
and non-radioactive .
(a) NO (b) NO2
(III) All elements have isotopes.
(c) O2 (d) CO2
(IV) All isotopes of Carbon can form chemical
compounds with Oxygen-16 38. The pH of 0.1 M aqueous solution of NaCl,
CH3COONa and NH4Cl will follow the order:
The correct option regarding an isotope is
(a) NaCl < CH3COONa < NH4Cl
(a) (III) and (IV) only.
(b) NH4Cl < NaCl < CH3COONa
(b) (II), (III) and (IV) only.
(c) NH4Cl < CH3COONa < NaCl
(c) (I), (II) and (III) only
(d) NaCl < NH4Cl < CH3COONa
(d) (I) (III) and (IV) only
39. At room temperature, the average speed of 45. The first ionization enthalpies for three elements
Helium is higher than that of Oxygen by a factor are 1314, 1680 and 2080 kJ mol–1, respectively.
of : The correct sequence of the element is:
6 (a) O, F and Ne (b) F, O and Ne
(a) 2 2 (b) (c) Ne, F and O (d) F, Ne and O
2
(c) 8 (d) 6
40. Ammonia is NOT produced in the reaction of :
46. Individuals of one kind occupying a particular
(a) NH4Cl with KOH
geographic area at a given time are called
(b) AlN with water
(a) community (b) population
(c) NH4Cl with NaNO2
(c) species (d) biome
(d) NH4Cl with Ca(OH)2
47. What fraction of the assimilated energy is used
41. The number of isomers which are ethers and in respiration by the herbivores?
having the molecular formula C4H10O, is:
(a) ~10 percent (b) ~60 percent
(a) 2 (b) 3
(c) ~30 percent (d) ~80 percent
(c) 4 (d) 5
48. Athletes are often trained at high altitude because
42. The major product of the reaction of 2-butene with
(a) training at high altitude increases muscle
alkaline KMnO4 solution is:
mass
(b) training at high altitude increases the number
(a) (b) of red blood cells
(c) there is less chance of an injury at high altitude
(d) athletes sweat less at high altitude
(c) (d) 49. In human brain, two cerebral hemispheres are
connected by a bundle of fibers which is known as
43. Among the compounds I-IV, the compound having (a) Medulla oblongata (b) cerebrum
the lowest boiling point is: (c) cerebellum (d) corpus callosum
50. Which of the following hormones is produced by
(I)
the pancreas?
(a) Prolactin
(II) (b) Glucagon
(c) Leutinizing hormone
(III)
(d) Epinephrine
51. The stalk of a plant leaf is derived from which
(IV) one of the following types of plant tissue?
(a) Sclerenchyma (b) Parenchyma
(a) I (b) II (c) Chlorenchyma (d) Collenchyma
(c) III (d) IV 52. Which of the following muscle types CANNOT
44. Of the following reactions be used voluntarily?
(i) A B, G° = 250 kJ mol–1 (a) Both striated and smooth
(b) Both cardiac and striated
(ii) D E, G° = –100 kJ mol–1
(c) Both smooth and cardiac
(iii) F G, G° = –150 kJ mol–1
(d) Cardiac, striated and smooth
(iv) M N, G° = 150 kJ mol–1 53. The pulmonary artery carries
the reaction with the largest equilibrium constant (a) deoxygenated blood to the lungs
is (b) oxygenated blood to the brain
(a) (i) (b) (ii) (c) oxygenated blood to the lungs
(c) (iii) (d) (iv) (d) deoxygenated blood to the kidney
54. Both gout and kidney stone formation is caused 58. Which one of the following options is true in
by photosynthesis?
(a) calcium oxalate (b) uric acid (a) CO2 is oxidized and H2O is reduced
(c) creatinine (d) potassium chloride (b) H2O is oxidized and CO2 is reduced
55. The auditory nerve gets its input from which of (c) Both CO2 and H2O are reduced
the following? (d) Both CO2 and H2O are oxidized
(a) The sense cells of the cochlea 59. Human mature red blood cells (RBCs) do NOT
(b) Vibration of the last ossicle contain
(c) Eustachian tube (a) Iron
(d) Vibration of the tympanic membrane (b) Cytoplasm
56. Which of the following organelles contain circular (c) Mitochondria
DNA? (d) Haemoglobin
(a) Peroxisomes and Mitochondria 60. A person was saved from poisonous snake bite by
(b) Mitochondria and Golgi complex antivenom injection. Which of the following
(c) Chloroplasts and Lysosomes immunity explains this form of protection?
(d) Mitochondria and Chloroplast (a) Naturally acquired active immunity
57. A reflex action does NOT involve (b) Artificially acquired active immunity
(a) neurons (b) brain (c) Naturally acquired passive immunity
(c) spinal cord (d) muscle fiber (d) Artificially acquired passive immunity

63. The number of integers a in the interval


[1, 2014] for which the system of equations
61. Let a, b, c be non-zero real numbers such that a
+ b + c = 0; let q = a2 + b2 + c2 and r = a4 + b4 + c4. x2 y2
x y a, 4
Then- x 1 y 1
(a) q2 < 2r always has finitely many solutions is-
(b) q2 = 2r always (a) 0 (b) 1007
(c) q2 > 2r always (c) 2013 (d) 2014
(d) q2 – 2r can take both positive and negative 64. In a triangle ABC with A = 90º, P is a point on
values BC such that PA : PB = 3 : 4. If AB = 7 and AC
1947
1 = 5 then BP : PC is-
62. The value of n 1947 is equal to
n 0 2 2 (a) 2 : 1
487 (b) 4 : 3
(a) (c) 4 : 5
21945
(d) 8 : 7
1946 65. The number of all 3-digit numbers abc (in base
(b)
21947 10) for which (a × b × c) + (a × b) + (b × c) + (c × a)
+ a + b + c = 29 is
1947
(c) (a) 6
21947 (b) 10
1948 (c) 14
(d)
21947 (d) 18
66. A uniform square wooden sheet of side a has its
center of mass located at point O as shown in the
figure on the left. A square portion of side b of
this sheet is cut out to produce as L-shaped sheet
as shown in the figure on the right.

If the refractive index of the material of the prism


is , then

(a) > 5 (b) 3 < < 5

The centre of mass of the L-shaped sheet lies at (c) 2 < < 3 (d) < 2
the point P (in the diagram) when 70. Consider the circuit shown below where all
resistors are of 1 k .
a 5 1 a 5 1
(a) (b)
b 2 b 2
a 3 1 a 3 1
(c) (d)
b 2 b 2
67. A machine is blowing spherical soap bubbles of If a current of magnitude 1 mA flows through the
different radii filled with helium gas. It is found resistor marked X, what is the potential difference
that if the bubbles have a radius smaller than 1 measured between points P and Q?
cm, then they sink to the floor in still air. Larger (a) 21 V (b) 68 V
bubbles float in the air. Assume that the thickness (c) 55 V (d) 34 V
of the soap film in all bubbles is uniform and equal.
Assume that the density of soap solution is
same as that of water (= 1000 kg m–3) The density
71. 10 moles of a mixture of hydrogen and oxygen
of helium inside the bubbles and air are 0.18 kg
gases at a pressure of 1 atm at a constant volume
m–3 and 1.23 kg m –3, respectively. Then the
and temperature, react to form 3.6 g of liquid
thickness of the soap film of the bubbles is (note
water. The pressure of the resulting mixture will
1 m = 10–6 m)
be closest to :
(a) 0.50 m (b) 1.50 m
(a) 1.07 atm (b) 0.97 atm
(c) 7.00 m (d) 3.50 m
(c) 1.02 atm (d) 0.92 atm
68. An aluminum piece of mass 50 g initially at
72. The ammonia evolved from 2 g of a compound in
300°C is dipped quickly and taken out of 1 kg of
Kjeldahl’s estimation of nitrogen neutralizes 10
water , initially at 30°C. if the temperature of the
mL of 2 M H2SO4 solution. The weight percentage
aluminum piece immediately after being taken
of nitrogen in the compound is :
out of the water is found to be 160°C, what is the
temperature of the water then ? (specific heat (a) 28 (b) 14
capacities of aluminum and water are 900 Jkg–1 (c) 56 (d) 7
K–1 and 4200 Jkg–1, respectively.) 73. Complete reaction of 2.0 g of calcium (at wt. = 40)
(a) 165°C (b) 45°C with excess HCl produces 1.125 L of H 2 gas.
(c) 31.5°C (d) 28.5°C Complete reaction of the same quantity of another
metal ‘‘M’’ with excess HCl produces 1.85 L of H2
69. A ray of light incident parallel to the base PQ of
gas under identical conditions. The equivalent
an isosceles right-angled triangular prism PQR
weight of ‘‘M’’ is closest to:
suffers two successive total internal reflections
at the face PQ and QR before emerging reversed (a) 23 (b) 9
in direction as shown. (c) 7 (d) 12
74. A compound X formed after heating coke with
lime reacts with water to give Y which on passing
over red-hot iron at 873 K produces Z. The 76. In which of the following cellular compartment(s)
compound Z is: do respiratory reactions occur?
(a) Cytoplasm and endoplasmic reticulum
(a) (b) Mitochondria and Golgi complex
(c) Mitochondria and cytoplasm
(b) (d) Mitochondria only
77. A woman heterozygous for color blindness
(c) marries a color blind man. What would be the
ratios of carrier daughters, color blind daughters,
normal sons and color blind sons in the F1
(d) generation?
(a) 1 : 2 : 2 : 1
75. In the following reaction sequence
(b) 2 : 1 : 1 : 2
(c) 1 : 1 : 1 : 1
(d) 1 : 1 : 2 : 2
78. Two semi-permeable bags containing 2% sucrose
are placed in two beakers, ‘P’ containing water
and ‘Q’ containing 10% sucrose. Which one of the
following outcomes is true?
(a) Bag in ‘P’ becomes flaccid due to exosmosis
X and Y are, respectively
(b) Bag in ‘P’ becomes turgid due to endosmosis
(c) Bag in ‘Q’ becomes turgid due to endosmosis
(d) Concentration of sucrose remains unchanged
(a)
in both
79. Children suffering from phenylketonuria are given
food low in phenylalanine and supplemented with
tyrosine. This is because they
(b) (a) are unable to utilize phenylalanine
(b) do not require phenylalanine
(c) have increased tyrosine anabolism
(d) have increased tyrosine catabolism
80. Two bottles were half filled with water from Ganga
(c) (‘P’) and Kaveri (‘Q’) and kept under identical
airtight conditions for 5 days. The oxygen was
determined to be 2% in bottle (‘P’) and 10% in
bottle (‘Q’). What could be the cause of this
difference?
(a) Ganga is more polluted than Kaveri
(d) (b) Both the rivers are equally polluted
(c) Kaveri is more polluted than Ganga
(d) Kaveri has more minerals than Ganga
1. (c) 2. (c) 3. (a) 4. (d) 5. (d) 6. (c) 7. (b) 8. (b) 9. (c) 10. (b)
11. (d) 12. (b) 13. (c) 14. (c) 15. (b) 16. (a) 17. (d) 18. (b) 19. (a) 20. (c)
21. (d) 22. (b) 23. (d) 24. (a) 25. (b) 26. (c) 27. (a) 28. (c) 29. (b) 30. (a)
31. (a) 32. (c) 33. (d) 34. (a) 35. (b) 36. (c) 37. (d) 38. (b) 39. (a) 40. (c)
41. (b) 42. (d) 43. (c) 44. (c) 45. (a) 46. (b) 47. (c) 48. (b) 49. (d) 50. (b)
51. (d) 52. (c) 53. (a) 54. (b) 55. (a) 56. (a) 57. (b) 58. (b) 59. (c) 60. (d)
61. (b) 62. (a) 63. (c) 64. (a) 65. (c) 66. (b) 67. (d) 68. (c) 69. (a) 70. (d)
71. (b) 72. (a) 73. (d) 74. (a) 75. (a) 76. (c) 77. (c) 78. (b) 79. (a) 80. (a)

6n2 2n 1
2 6n(2n 1) 6n(n 1 n)
1
2 . (c) f (x) x f (1 x) 1
n(n 1)(2n 1) (n 1) n 1
2
1 6 n2 6
f (1 x) 1 x f (1 1 x ) 1 6n 6n 6 n 1 Integer
2 n 1 n 1
3 The values of ‘n’ which are satisfies above
f (1 x) x f (x) 1
2 equation is n = 1, 2, 5
1 f (x) 3 sum = 1 + 2 + 5 = 8
x f (x) 1 4 . (d) x = ab = 10a + b
1 2
x
2 and y = ba = 10b + a 1 a, b 9
3 3 x 1 x2 – y2 = m2
1 f (x) x x2 f (x) x
2 4 2 2 Now, (10a + b)2 – (10b + a)2 = m2
1 1 100a2 + b2 + 20ab – (100b2 + a2 + 20ab) = m2
f (x) x x 2 x 99(a2 – b2) = m2
4 2
9 × 11 × (a – b) (a + b) = m2
f x 4x 4x 2 1 2x 1
a=6
4 2
and b = 5
f x 4x 4x 2 1 4x 2 m2 = 9 × 11 × 1 × 11 m = 3 × 11
2 4x x + y + m = 10a + b + 10b + a + 33
f (x)
4x 4x 2
1 = 11(a + b) + 33 = 11(11) + 33
= 121 + 33 = 154
2 0 2 4 5 . (d) p(x) = x2 – 5x + a, q(x) = x2 – 3x + b
Now, 2f (0) 3f (1) 2 3
0 0 1 4 4 1 {where, a, b N}
3( 2) HCF (p(x), q(x)) = x – 1
4 4 6 2
1 So x – 1 is root of both p(x) and q(x)

2
p(1) = 1 – 5 + a = 0 a=4
2n(2n 1) q(1) = 1 – 3 + b = 0 b=2
2
2 4 n2 2n 1
3 . (a) p(x) = x2 – 5x + 4 = (x – 1) (x – 4)
n(n 1)(2n 1) 4n(n 1)(2n 1)
6 6 q(x) = x2 – 3x + 2 = (x – 1) (x – 2)
Now k(x) is lcm (p(x), q(x)) = (x – 1) (x – 2) (x – I divides AF
4) b+c:a
(x – 1) + k(x) = (x – 1) + (x – 1) (x – 2) (x – 4)
I divides BD
= (x – 1) [1 + x2 – 6x + 8]
c+a:b
= (x – 1) (x2 – 6x + 9)
= (x – 1) (x – 3)2 c a 3
Hence roots are 1, 3, 3 b 2
Sum of roots = 1 + 3 + 3 = 7 I divides CE in
6 . (c) a+b:c

a b 2
c 1
c a 3 a b 2
;
b 2 c 1
2c + 2a = 3b
a + b = 2c
3a
a + b + 2a = 3b a+ = 2c
2
5
ABC ~ BPD 3a = 2b a = 2c
2
x b a
x a b 3a 5
x x b c= a
2 4
AB = AD + BC
3a 5a
7 . (b) b c 2 4 6 5 11
Now
a a 4 4

9 . (c)

Required area
2
1
2 60 3 2
= (1)2 1
2 360 4

3 3
=
8 6 4 4 24
AR = PR = 10 – x
8 . (b)
and PQ = 10 – 2x
Now AB = CD = 10
CD = CS + SD = y + SD
= y + SP + PQ
10 = y + y + 10 – 2x
y=x
Now, RS = SP + PQ + QR
= y + 10 – 2x + x = 10 + y – x = 10
10. (b) 100000 10101(ab) < 1000000
9.9 ab < 99
‘ab’ number can be obtained as product of
ordered pairs
(2, 5); (2, 11); (2, 17); (2, 19); (2, 23); (2, 29); (2,
31); (2, 41); (2, 43); (2, 47); (5, 11); (5, 17); (5,
19)
Total numbers = 13
14. (c) Let maximum house is ‘n’ ; sum of first ‘n’
even natural numbers = n2 + n
Let first ‘m’ even natural numbers are left in
OAB should be equilateral
numbering the houses.
Now OBC = 90º and AB = BC
(n2 + n) – (m2 + m) = 170
BAC = BCA = 15º
n2 – m2 + n – m = 170
BOF = 30º & BOC = 45º
(n – m) (n + m + 1) = 170
{ OBC is right isosceles}
n – m = 10 n – m = 10 ...(i)
BOF 30 2 n + m + 1 = 17 n + m = 16 ...(ii)
{ BOF = 2 BAF}
BOC 45 3
Solving equation (i) and (ii) we get
11. (d) Let total seats = 100
n = 13; m = 3
Revenue collected on first days = 200 × 60
n 13
= Rs. 12000
If first term is a – 10 then sixth term will be a
and Revenue collected on second day = 90 ×
160 n
[2(a – 10) + 2 (n – 1)] = 170
= Rs. 14400 2
n[a + n – 11] = 170
2400
Required % increase = 100 % 170
12000 a= + 11 – n
= 20% n
12. (b) Year wise distribution n = 10 (only will make ‘a’ integer)
2010 39 170
a= 11 10 17 + 11 – 10 = 18
2011 60 10
2012 x 16. (a) Magnitude of slope is increasing at point R.
2013 123 Magnitude of slope of displacement-time
Let proportionality constant is ‘k’ graph represents speed
According to the question 17. (d) In an evacuated chamber, in absence of air,
buoyancy force due to air on box is absent.
(123 – 60) x 123 – 60 = kx
18. (b)
63 = kx … (i)
and (x – 39) 60
x – 39 = 60k … (ii)
solving equation (i) & (ii) we get
x2 – 39x = 60 × 63
x = 84
13. (c) 100000 ababab < 1000000
for equilibrium of m1
105a + 104b + 103a + 100b + 10a + b < 1000000
T1cos = m1 g
a(105 + 103 + 10) + b(104 + 102 + 1) 1000000
T1sin = F
100000 (104 + 102 + 1) (10a + b) < 1000000
F 21. (d) Surface area of Ice get increases by crushing
tan = … (1) and cooling due to ice occur due to convection
m1g
process which is proportional to area.
For equilibrium of m2
T2 cos = m2 g 23. (d)
T2 sin =F
F
tan = … (2)
m1g
from (1) & (2)
1 n1 n2 1 1
m1 = m2
f n1 R1 R2
19. (a) R1 = – 0.2 ; R2 = 0.2
n2 = 1.6 ; n1 = 2.0
1 1.6 2 1 1
f 2 0.2 0.2
( 0.4) 2
2 ( 0.2)

When box is dropped from a height h, then 1 2


speed at ground is v, therefore using f 1
mechanical energy conservation ƒ = 0.5 metre
1 Converging lens as ƒ is positive.
mgh = mv2 … (i)
2 25. (b)
when body slides on rough inclined plane,
friction force will also act
f= N= mg cos
kQ Q kQ2
Applying work-energy theorem Energy E = … (1)
d d
2 Third charge is put between them
1 v
mgh – fs = m 0
2 3
2
h 1 v
mgh f m
sin 2 3
h
sin
s Energy of system
2
h 1 mv
mgh mg cos … (ii) kQ Q kQ Q kQ Q
sin 2 9
d d 2 d 2
from equation (i) & (ii)
2 2
1
mgh 1 cot mgh kQ 2 kQ2 kQ2
9
putting = 45º, cot = 1 d d d

1 kQ 2
1– =
9 d

8 From (1)
= Energy of system = – E
9
20. (c) specific heat of water is very high
It temperature rises by small amount.
26. (c) 29. (b) Very thin foil can be made only of highly
malleable material.
30. (a) All elements have isotopes. All isotopes of
carbon can form chemical compounds with
oxygen-16.
31. (a) Electrons in CO = 6 + 8 = 14
Electrons in N2 = 7 × 2 = 14

32. (c) N N
H H H H
Magnet is approaching sing due to which 5 bond pair & 2 lone pair
downward flux through ring is increasing.
According to lenz law induced current is 2.4
33. (d) Moles of Carbon = 0.2
anticlockwise or counter clockwise. 12
Volume of 0.2 mole at S.T.P. = 0.2 × 22.4
= 4.48 L
34. (a) Least polar compound, the highest Rf
35. (b)

12
36. (c) rHe 53 Pm 26.5 Pm
2
37. (d) CO2 has no unpaired electrons.

38. (b) CH3COONa Basic Salt


When magnet is below the plane of ring and
moving away from ring flux in downward NH4 Cl Acidic Salt
decreasing due to which induced current is
NaCl Neutral Salt
clockwise.
27. (a) Magnetic filed lines are in opposite direction
MO
in zone 1 so Neutral point is located in zone 1. 39. He 2

O2 MH
e

He 2 2 O
2

40. (c) NH4 Cl NaNO2 NH4 NO2 NaCl

41. (b) CH3 CH2 CH2 OCH3 and CH3 – CH – O – CH3


|
CH3

and CH3CH2 O CH2CH3

Cold
a/k
42. (d) CH3 CH CH CH3 KMnO4

CH3 – CH – CH – CH3
28. (c) Speed of particle doing SHM decrease as it go | |
away from mean position. Time during which OH OH
particle remain in extreme position will be 43. (c) All others can form H-bond but ether cannot.
longer.
G 974 974 487
44. (c) log K = Sum =
2.303RT 21947 2 21945 21945
45. (a) As Ne is most stable, so has maximum first x2 y2
ionisation enthalpy. 63. (c) x + y = a; 4
x 1 y 1
48. (b) At high altitude RBC count es. Where a [1, 2014]
49. (d) Corpus callosum attaches both cerebral 1 1
hemispheres of mammals. Now, x + 1 + +y+1+ =4
x 1 y 1
50. (b) cells of pancrease secrete Glucogen.
52. (c) Cardiac and smooth muscles are involuntary. 1 1
(x – 1) + + (y – 1) + =0
53. (a) Pulmonary artery carry deoxygenated blood. (x 1) (y 1)
56. (a) Mitochondria and chloroplast both contain 1 1
circular DNA. x 1 y 1 0
x 1 y 1
57. (b) Reflex action is controlled by spinal cord.
58. (b) In photosynthesis water is oxidised while CO2 1 1
is reduced to sugar. (a – 2) + =0
(x 1) (y 1)
59. (c) Nucleus, mitochondria and ER all three are
absent in a nature RBC. y 1 x 1
a z
60. (d) Antivenom gives ‘Artificially’ acquired passive x 1 y 1
immunity. It contain performed antibodies.
(a 2)
(a – 2) + =0
(x 1)(y 1)
61. (b) a + b + c = 0 1
q = a2 + b2 + c2 and r = a4 + b4 + c4 (given) (a – 2) 1 =0
xy 1 a
q2 – 2r = (a2 + b2 + c2)2 – 2(a4 + b4 + c4) a 2 [for a = 2 infinitely many solutions]
= 2a2b2 + 2b2c2 + 2a2c2 – a4 – b4 – c4 xy + 1 – a + 1 = 0
= 2a2c2 + 2b2c2 – (a2 – b2)2 – c4 x(a – x) – a + 2 = 0
= c2[2ab + a2 + b2 – c2] x2 + ax – (2 – a) = 0
= c2[(a + b)2 – c2] D = b2 – 4ac = a2 + 4(2 – a) = a2 – 4a + 8
= c2[(a + b + c) (a + b – c)] D>0
[ a + b + c = 0] There two real solutions.
=0 a 2 and a [1, 2014]
1947 Total 2014 – 1 = 2013 values
1
62. (a) Total terms = 1948
n 0 2n 21947 64. (a)
1
T1
1 21947
1
T1948
1947
2 21947
1
T1 T1948
21947
1
Similarly, T2 + T1947 = = T3 + T1946 =
21947 5 5 7
tan = ; sin = ; cos =
and so on… 7 12 12
1948 Now by using sine rule in APB,
Total 974 pairs
2 3x 4x 7
sin sin sin(180 ( ))
4 4 5 20 m1 x1 m 2 x2
sin sin Xcm =
3 3 12 27 m1 m2
m1 is the mass of square wooden sheet of side
7
cos a & m2 is the mass of removed square portion
27 of side b.
and 3x sin( ) 7 sin x-coordinate of C.O.M. of remaining
L-shaped sheet. is areal mass density
7 7 20 m1 = a2, m2 = b2
3x 7
27 6 27
a b
a2 b2
1 2 2
3x 1 Xcm =
3 a2 b2
1
x 1 a3 b3
3 Xcm =
2 a2 b2
BP 4x 4 2
PC 12 4x 6 4 1 1 a3 b3
similarly Ycm =
65. (c) abc + ab + bc + ca + a + b + c = 29 2 a 2 b2
abc + ab + bc + ca + a + b + c + 1 = 30 centre of mass lies on point P (b, b)
(a + 1) (b + 1) (c + 1) = 30 Xcm = b & Ycm = b
Product can be 2 × 3 × 5 digits are 1, 2, 4 so 1 a 2 b2 ab
possible number = 3! = 6 b
2 a b
Product can be 1 × 5 × 6 digits are 0, 4, 5 so a2 + b2 + ab = 2ab + 2b2
possible number = 2 × 2! = 4
a2 = ab + b2
Product can be 1 × 3 × 10 digits are 0, 2, 9
so possible number = 2 × 2! = 4 2
a a
1
Total numbers = 6 + 4 + 4 = 14 b b
a
66. (b) Let x =
b
x2 – x – 1 = 0
1 1 4
x=
2
5 1
x=
2
a 5 1
b 2

67. (d)

Let X is thickness of soap film for equilibrium.


Gravity force = buoyancy force
4 2
3
2
2
10 0.18 4 10 x 1000
3
4 2 3
10 (1.23)
3
2 4 6 i1 R i1
4 10 (x) 1000 10 (1.08) i
3 3R 3
105 x 0.36 i1 = 3i
5
x 0.36 10
6
x 0.36 10 m
68. (c) Calorimetry principle says that
Heat lost = Heat gain
Heat loss by aluminum = Heat gain by water 3
i1 = i2 ×
3 8
50 10 900 (300 160) 1 4200 (T 30)
6300 = 4200(T – 30) 8
i2 = i
3 1
1.5 = T – 30
T = 31.5ºC 8
3i 8i
3
69. (a)

8
i2 i3
21

45 + r > c … (1)
45 – r > c … (2)
90º > 2 c
45º > c … (3)
sin 45º > sin 21 21
c i3 i2 8i 21i
8 8
1 1
2 13R
Vp VQ i3 R
21
2
34R
taking equation 2 only 21i
21
45 – c> r , sin( 45 – c) > sin r
34iR
1 1 sin 45
cos c sin c 34 × 1 × 10–3 × 1 × 103 = 34 volt
2 2
71. (b) 2H2 + O2 2H2O
u2 1 1 1 2
1 2. 5 Initially x moles (10 – x) moles 0 moles

a
Ans. is > 5 as this is common solution After reaction (x – a) 10 x a
2
70. (d) R = 1 k i = 1 mA = 1 × 10–3 A
3.6
Now a = 0.2
18
a – C – Ph
New moles = x – a + 10 – x – = 9.7 75. (a) CH2 – CH 2– CH – Ph (i) alc. KOH Ph – C =
| |
2 Br Br (ii) alc. NaNH2 HgSO4 / dil.H2SO4

O
P1 x1 ||
Now, or P2 = 0.97 Ph – C – CH3
P2 x2
Glycolysis In cytoplasm
14 17 4 10
72. (a) WN = = 0.56 g 76. (c) Respiration Kreb cycle Mitochondria
17 1000
ETS Mitochondria
0.56 77. (c) Color blindness X - linked recessive
% of N = 100 = 28
2
x xc xc y
40
73. (d) Equivalent mass of Ca = = 20
2
Carrier
daughter xc y
w
ECa Equivalents of released by Ca
x x xc xy Normal son
w Equivalents of H2 released by Metal M
EM

2 xc xc xc xc y
Colour blind son
20 1.125
2 1.85
x Colour blind
Daughter
1.125 79. (a) pku is an autosomal recessive disorder. In
x 20 12 this disorder the enzyme which converts
1.85
phenyl alanine to tyrosine (ENZYME - phenyl
alanin hydroxi lase) becomes non functional.
74. (a) CaO C CaC2
So, now phenyl alanine is converted into
phenyl pyrubic acid. Which causes mental
red hot retardation. Hence children suffering from
Ca OH 2
C2 H 2 iron tube PKU should be given Tyrosin in their diet
and low phenyl alanin.
80. (a) Disolved oxygen (DO) in high in pure water
compared to polluted water.
So, more DO means loss polluted.
respectively. Which of the following triangles
CANNOT be similar to ABC?
1 . Let x, y, z be three non-negative integers such (a) Triangle ABD (b) Triangle BCE
that x + y + z = 10. The maximum possible value
(c) Triangle CAF (d) Triangle DEF
of xyz + xy + yz + zx is
7 . Tangents to a circle at points P and Q on the
(a) 52 (b) 64
circle intersect at a point R. If PQ = 6 and PR = 5
(c) 69 (d) 73 then the radius of the circle is
2 . If a, b are natural numbers such that 2013 + a2 =
b2, then the minimum possible value of ab is (a) 13 (b) 4
3
(a) 671 (b) 668 15 16
(c) (d)
(c) 658 (d) 645 4 5
3 . The number of values of b for which there is an 8 . In an acute-angled triangle ABC, the altitudes
isosceles triangle with sides of length b + 5, 3b –2 from A, B, C when extended intersect the circumcircle
and 6 – b is again at points A1, B1, C1, respectively. If
(a) 0 (b) 1 ABC = 45° then A1B1C1 equals
(c) 2 (d) 3 (a) 45° (b) 60°
4 . Let a, b be non-zero real numbers. Which of the (c) 90° (d) 135°
following statements about the quadratic equation 9 . In a rectangle ABCD, points X and Y are the
is neccesarily true? midpoints of AD and DC, respectively. Lines BX
ax2 + (a + b)x + b = 0 and CD when extended intersect at E, lines BY
and AD when extended intersect at F. If the area
b of ABCD is 60 then the area of BEF is
x = –1 and x = are the roots.
a (a) 60 (b) 80
(I) It has at least one negative root (c) 90 (d) 120
(II) It has at least one positive root. 10. In the figure given below, ABCDEF is a regular
(III)Both its roots are real. hexagon of side length 1, AFPS and ABQR are
(a) (I) and (II) only squares. Then the ratio Area (APQ)/ Area (SRP)
equals
(b) (I) and (III) only
(c) (II) and (III) only 2 1
(a)
(d) All of them 2
5 . Let x, y, z be non-zero real numbers such that (b) 2

x y z y z x 3 3
7 and 9, then (c)
y z x x y z 4
(d) 2
x3 y3 z3
3 is equal to 11. A person X is running around a circular track
y3 z3 x3 completing one round every 40 seconds. Another
(a) 152 (b) 153 person Y running in the opposite direction meets
X every 15 second. The time, expressed in
(d) 154 (d) 155
seconds, taken by Y to complete one round is
6 . In a triangle ABC with A < B < C, points D,
(a) 12.5 (b) 24
E, F are on the interior of segments BC, CA, AB,
(c) 25 (d) 55
12. The least positive integer n for which 17. A ball is thrown horizontally from a height with a
certain initial velocity at time t = 0. The ball
n 1 n 1 0.2 is bounces repeatedly from the ground with the
(a) 24 (b) 25 coefficient of restitution less than 1 as shown.
(c) 26 (d) 27
13. How many natural numbers n are there such that
n!+10 is a perfect square?
(a) 1
(b) 2
(c) 4
(d) infinitely many Neglect air resistance and taking the upward
14. Ten points lie in a plane so that no three of them direction as positive, which figure qualitatively
are collinear. The number of lines passing through depicts the vertical component of the ball’s
exactly two of these points and dividing the plane velocity (Vy) as a function of time (t)?
into two regions each containing four of the
remaining points is
(a) 1
(b) 5
(c) 10
(d) dependent on the configuration of points (a)
15. In a city, the total income of all people with salary
below Rs. 10000 per annum is less than the total
income of all people with salary above Rs. 10000
per annum. If the salaries of people in the first
group increases by 5% and the salaries of people
in the second group decreases by 5% then the
average income of all people
(a) increases
(b) decreases (b)
(c) remains the same
(d) cannot be determined from the data

16. A man inside a freely falling box throws a heavy


ball towards a side wall. The ball keeps on
bouncing between the opposite walls of the box.
We neglect air resistance and friciton. Which of
the following figures depicts the motion of the (c)
centre of mass of the entire system (man, the
ball and the box)?

(a) (b) (c) (d)


(d)
18. A tall tank filled with water has an irregular shape 21. The refractive index of a prism measured using
as shown. The wall CD makes an angle of 45° three lines of a mercury vapour lamp. If , and
with the horizontal; the wall AB is normal to the are the measured refractive indices for these
base BC. The lengths AB and CD are much green, blue and yellow lines respectively, then
smaller than the height h of water (figure not to (a) > > (b) > >
scale).
(c) > > (d) > >
22. A horizontal parallel beam of light passes through
a vertical convex lens of focal length 20 cm and is
then reflected by a tilted plane mirror so that it
converges to a point I. The distance PI is 10 cm.

Let P1, P2 and P3 be the pressures exerted by


M is a point at which the axis of the lens intersects
the water on the wall AB, base BC and the wall
the mirror. The distance PM is 10 cm. The
CD respectively. Density of water is and g is
angle which the mirror makes with the horizontal
acceleration due to gravity. Then, approximately
is
1 (a) 15° (b) 30°
(a) P1 = P2 = P3 (b) P1 = 0, P3 = P
2 2 (c) 45° (d) 60°
1 23. In a car a rear view mirror having a radius of
(c) P1 = P3 = P (d) P1 = P3 = 0, P2 = h g
2 2 curvature 1.50 m forms a virtual image of a bus
19. The accompanying graph of position x versus time located 10.0 m from the mirror. The factor by
t represents the motion of a particle. If p and q which the mirror magnifies the size of the bus is
are both positive constants, the expression that close to
best describes the acceleration of the particle is (a) 0.06
(b) 0.07
(c) 0.08
(d) 0.09
24. Consider the circuit shown in the figure below :

(a) a = – p – qt (b) a = –p + qt
(c) a = p + qt (d) a = p – qt
20. Two stones of mass m1 and m2 (such that m1 > m2)
are dropped t time apart from the same height
towards the ground. At a later time t the
difference in their speed is V and their mutual
separation is S. While both stones are in flight
(a) V decreases with time and S increases with
time All the resistors are identical. The ratio I/I’ is
(b) Both V and S increase with time (a) 8
(c) V remains constant with time and S (b) 6
decreases with time (c) 5
(d) V remains constant with time and S (d) 4
increases with time
25. The figure shows a bar magnet and a metallic 29. An ideal gas filled in a cylinder occupies volume
coil. Consider four situations. V. The gas is compressed isothermally to the
(I) Moving the magnet away from the coil. volume V/3. Now the cylinder valve is opened and
the gas is allowed to leak keeping temperature
(II) Moving the coil towards the magnet.
same. What percentage of the number of
(III)Rotating the coil about the vertical diameter. molecules escape to bring the pressure in the
(IV)Rotating the coil about its axis. cylinder back to its original value.
(a) 66% (b) 33%
(c) 0.33% (d) 0.66%
An emf in the coil will be generated for the 30. An electron enters a chamber in which a uniform
following situations. magnetic field is present as shown
(a) (I) and (II) only
(b) (I), (II) and (IV) only
(c) (I), (II), and (III) only
(d) (I), (II), (III), and (IV)
26. A current of 0.1 A flows through a 25 resistor
represented by the circuit diagram. The current
in the 80 resistor is
An electric field of appropriate magnitude is also
applied so that the electron travels undeviated
without any change in its speed thorugh the
chamber. We are ignoring gravity. Then, the
direction of the electric field is
(a) opposite to the direction of the magnetic field
(b) opposite to the direction of the electron’s
motion
(c) normal to the plane of the paper and coming
(a) 0.1 A out of the plane of the paper
(b) 0.2 A (d) normal to the plane of the paper and into the
plane of the paper
(c) 0.3 A
(d) 0.4 A
27. Solar energy is incident normally on the earth’s 31. The molecule having a formyl group is
surface at the rate of about 1.4 kW m -2. The
(a) acetone (b) acetaldehyde
distance between the earth and the sun is 1.5 ×
1011 m. Energy (E) and mass (m) are related by (c) acetic acid (d) acetic anhydride
Einstein equation E=mc2 where c (3 × 108 ms-1) 32. The structure of cis-3-hexene is
is the speed of light in free space. The decrease
in the mass of the sun is (a) (b)
(a) 109kg s-1
(c) (d)
(b) 1030kg s-1
33. The number of sp2 hybridized carbon atoms in
(c) 1026 kg s-1
(d) 1011 kg s-1
28. If the current through a resistor in a circuit
increases by 3%, the power dissipated by the (a) 3 (b) 5
resistor (c) 4 (d) 6
(a) increases approximately by 3% 34. The number of valence electrons in an atom with
(b) increases approximately by 6% electronic configuration 1s2 2s2 2p6 3s2 3p3 is
(c) increases approximately by 9% (a) 2 (b) 3
(d) decreases approximately by 3% (c) 5 (d) 11
35. The pair of atoms having the same number of 42. The order of reactivity of K, Mg, Au and Zn with
neutrons is water is
(a) 126 C,12
24
Mg (b) 23 19
11 Na,9 F (a) K > Zn > Mg > Au (b) K > Mg > Zn > Au
23 24 23 39
(c) 11 Na,12 Mg (d) 11 Na,19 K (c) K > Au > Mg > Zn (d) Au > Zn > K > Mg
36. Which of the following molecules has no dipole 43. Which of the following is an anhydride ?
moment ?
(a) CH3Cl (b) CHCl3
(a) (b)
(c) CH2Cl2 (d) CCl4
37. The decay profiles of three radioactive species A,
B and C are given below :
(c) (d)

44. Which of the following metals will precipitate


copper from copper sulphate solution?
(a) Hg (b) Sn
(c) Au (d) Pt
45. The radii of the first Bohr orbit of H (rH), He+
(rHe+) and Li2+ (rLi2+) are in the order
(a) rHe+ > rH > rLi2+ (b) rH < rHe+ < rLi2+
(c) rH > rHe > r
+
Li
2+
(d) rHe+ < rH+ < rLi2+

These profiles imply that the decay constants kA,


kB and kC follow the order 46. The Bowman’s capsule, a part of the kidney is
the site of
(a) kA > kB > kC (b) kA > kC > kB
(a) filtration of blood constituents
(c) kB > kA > kC (d) kC > kB > kA
(b) re-absorption of water and glucose
38. A specific volume of H2 requires 24 s to diffuse
(c) formation of ammonia
out of a container. The time required by an equal
volume of O 2 to diffuse out under identical (d) formation of urea
conditions, is 47. In human brain the sensation of touch, pain and
(a) 24 s (b) 96 S temperature is controlled by the
(c) 384 S (d) 192 s (a) parietal lobe of cerebrum
39. Acetic acid reacts with sodium metal at room (b) limbic lobe of cerebrum
temperature to produce (c) temporal lobe of cerebrum
(a) CO2 (b) H2 (d) frontal lobe of cerebrum
(c) H2O (d) CO 48. A pathogan which can not be cultured in an
artificial medium is,
40. The equilibrium constant, KC 3C2H2 g C6H6 g
(a) protozoan (b) virus
is 4 L mol . If the equilibrium concentration of
2 –2
(c) becterium (d) fungus
benzene is 0.5 mol L–1, that of accetylene in mol
49. Meiosis I and Meiosis II are characterised by the
L–1 must be
separation of,
(a) 0.025 (b) 0.25
(a) homologous chromosomes; sister chromatids
(c) 0.05 (d) 0.5
(b) sister chromatids; homologous chromosomes
41. The weight percent of sucrose (formula weight
(c) centromere; telomere
= 342 g mol-1) in an aqueous solution is 3.42. The
density of the solution is 1 g mL-1, the concentration (d) telomere; centromere
of sucrose in the solution in mol L-1 is 50. People suffering from albinism cannot synthesize
(a) 0.01 (b) 0.1 (a) suberin (b) melanin
(c) 1.0 (d) 10 (c) keratin (d) collagen
51. Short sightedness in humans can be corrected by 56. Which one of the following class of animals
using consitutes the largest biomass on earth?
(a) concave lens (b) convex lens (a) Insects (b) Fish
(c) cylindrical lens (d) plain glass (c) Mammals (d) Reptilians
52. A person with blood group “A” can (a) donate blood 57. In the digestive system, the pH of the stomach
to, and (b) receive blood from, and the intestine, respectively are,
(a) (a) persons with blood group “AB”, and (b) (a) alkaline; acidic (b) acidic; alkaline
persons with any blood group
(c) acidic; neutral (d) acidic; acidic
(b) (a) person with blood group “A” or “AB”, and
(b) “A” or “O” blood groups 58. The major nitrogenous excretory product in
mammals is,
(c) (a) person with blood group “B” or “AB”, and
(b) “B” or “O” blood groups (a) amino acids (b) ammonia
(d) (a) person with any blood group, and (b) “O” (c) urea (d) uric acid
blood group only 59. Which of the following plant traits (characters) is
53. Animal cells after removal of nuclei still contained NOT an adaptatioin to dry (Xeric) habitats?
DNA. The source of this DNA is (a) Sunken stomata on leaves
(a) nucleosomes (b) mitochondria (b) Highly developed root system
(c) peroxisomes (d) lysosome (c) Thin epidermis without a cuticle on stem and
54. Which one of the following combinations is found leaves
in DNA? (d) Small leaves and photosynthetic stem
(a) Guanine and guanidine 60. Biological diversity increases with the productivity
(b) Guanidine and cytosine of an ecosystem. In which of the following habitats
(c) Guanine and cytosine do we see the greatest diversity of species?
(d) Adenine and guanidine (a) Tropical dry grasslands
55. Which one of the following is NOT a mode of (b) Temperate deciduous forests
asexual reproduction? (c) Alpine grasslands
(a) Binary fission (b) Multiple fission (d) Tropical evergreen forests
(c) Budding (d) Conjugation

63. Let ABCD be a square and let P be point on


segment CD such that DP : PC = 1 : 2. Let Q be a
61. Let a, b, c, d, e be natural numbers in an point on segment AP such that BQP = 90°. Then
arithmetic progression such that a + b + c + d + e the ratio of the area of quadrilateral PQBC to the
is the cube of an integer and b + c + d is square of area of the square ABCD is
an integer. The least possible value of the number
31 37
of digits of c is (a) (b)
60 60
(a) 2 (b) 3
39 41
(c) 4 (d) 5 (c) (d) 3
60 60
62. On each face of a cuboid, the sum of its perimeter
64. Suppose the height of a pyramid with a square
and its area is written. Among the six numbers
base is decreased by p% and the lengths of the
so written, there are three distinct numbers and
sides of its square base are increased by p%
they are 16, 24 and 31. The volume of the cuboid
(where p > 0). If the volume remains the same,
lies between
then
(a) 7 and 14
(a) 50 < p < 55
(b) 14 and 21
(b) 55 < p < 60
(c) 21 and 28
(c) 60 < p < 65
(d) 28 and 35
(d) 65 < p < 70
65. There are three kinds of liquids X, Y, Z,. Three Ignore gravity. Balls will be received by Q
jars J1, J2, J3 contain 100 ml of liquids X, Y, Z, (a) one every 2.5 s in case (I) and one every 3.3 s
respectively. By an operation we mean three steps in case (II)
in the following order:
(b) one every 2 s in case (I) and one every 4 s in
- stir the liquid in J1 and transfer 10 ml from J1 case (II)
into J2 ;
(c) one every 3.3 s in case (I) and one every 2.5 s
- stir the liquid in J2 and transfer 10 ml from J2 in case (II)
into J3 ;
(d) one every 2.5 s in case (I) and one every 2.5 s
- stir the liquid in J3 and transfer 10 ml from J3 in case (II)
into J1 ;
68. A 10.0 W electrical heater is used to heat a
After performing the operation four times, let x, container filled with 0.5 kg of water. It is found
y, z be the amounts of X, Y, Z, respectively, in J1. that the temperature of the water and the
Then container rise by 3 K in 15 minutes. The container
(a) x > y > z (b) x > z > y is then emptied, dried, and filled with 2 kg of an
(c) y > x > z (d) z > x > y oil. It is now observed that the same heater raises
the temperature of the container-oil system by 2
K in 20 minutes. Assuming no other heat losses
in any of the processes, the specific heat capacity
66. Two identical uniform rectangular blocks (with
of the oil is
longest side L) and a solid sphere of radius R are
to be balanced at the edge of a heavy table such (a) 2.5 × 103 JK–1 kg–1
that the centre of the sphere remains at the (b) 5.1 × 103 JK–1 kg–1
maximum possible horizontal distance from the (c) 3.0 × 103 JK–1 kg–1
vertical edge of the table without toppling as
(d) 1.5 × 103 JK–1 kg–1
indicated in the figure.
69. A ray of light incident on a transparent sphere at
an angle /4 and refracted at an angle r, emerges
from the sphere after suffering one internal
reflection. The total angle of deviation of the ray
is
3
(a) 4r (b) 4r
2 2
5
(c) r (d) 4r
4 2
70. An electron with an initial speed of 4.0 × 106 ms–1
If the mass of each block is M and of the sphere is is brought to rest by an electric field. The mass
M/2, then the maximum distance x that can be and charge of an electron are 9 × 10–31 kg and
achieved is 1.6 × 10–19C, respectively. Identify the correct
statement
(a) 8L/15 (b) 5L/6
(a) The electron moves from a region of lower
(c) (3L/4 + R) (d) (7L/15 + R)
potential to higher potential through a
67. Two skaters P and Q are skating towards each potential difference of 11.4 V.
other. Skater P throws a ball towards W every 5
(b) The electron moves from a region of higher
s such that it always leaves her hand with speed
potential to lower potential through a potential
2 ms–1 with respect to the ground. Consider two
difference of 11.4 V.
cases:
(I) P runs with speed 1 ms–1 towards Q while Q (c) The electron moves from a region of lower
remains stationary potential to higher potential through a
potential difference of 45 V.
(II) Q runs with speed 1 ms–1 towards P while P
remains stationary. (d) The electron moves from a region of higher
potential to lower potential through a potential
Note that irrespective of speed of P, ball always difference of 45 V
leaves P’s hand with speed 2 ms–1 with respect
to the ground.
71. The degree of dissociation of acetic acid (0.1 mol 76. Sister chromatids of a chromosome have
L-1) in water (K of acetic acid is 10-5) is (a) different genes at the same locus
(a) 0.01 (b) 0.5 (b) different alleles of the same gene at the same
(c) 0.1 (d) 1.0 locus
72. Compound ‘X’ on heating with Zn dust gives (c) same alleles of the same gene at the same locus
compound ‘Y’ which on treatment with O 3 (d) same alleles at different loci
followed by reaction with Zn dust gives 77. A diabetic individual becomes unconscious after
propionaldehyde. The structure of ‘X’ is self-administering insulin. What should be done
immediately to revive the individual?
(a) Provide him sugar
(a) (b)
(b) Give him high dose of insulin
(c) Provide him salt solution
(d) Provide him lots of water
(c) (d) 78. A regular check on the unborn baby of a lady
towards the end of her pregnancy showed a heart
rate of 80 beats per minute. What would the doctor
73. The amount of metallic Zn (Atomic weight = 65.4)
infer about the baby’s heart condition from this?
required to react with aqueous sodium hydroxide
(a) Normal heart rate
to produce 1 g of H2, is
(b) Faster heart rate
(a) 32.7 g (b) 98.1 g
(c) Slower heart rate
(c) 65.4 g (d) 16.3 g
(d) Defective brain function
74. Natural abundances of 12C and 13C isotopes of
carbon are 99% and 1%, respectively. Assuming 79. Three uniformly watered plants i, ii and iii were
they only contributes to the mol. wt. of C2F4, the kept in 45% relative humidity, 45% relative
humidity with blowing wind and 95% relative
percentage of C2F4 having a molecular mass of
humidity, respectively. Arrange these plants in
101 is
the order (faster to slowest) in which they will
(a) 1.98 dry up.
(b) 98 (a) i = ii, iii (b) ii, i, iii
(c) 0.198 (c) iii, ii, i (d) iii, i = ii
(d) 99 80. Many population colonising a new habitat show a
75. 2, 3-Dimethylbut-2-ene when reacted with logistic population growth pattern over time, as
bromine forms a compound which upon heating shown in the figure below.
with alcoholic KOH produce the following major
product.

(a)

(b)

(c) In such a population, the POPULATION growth


rate
(a) stays constant over time
(b) increases and then reaches an asymptote
(d)
(c) decreases over time
(d) increases to a maximum and then decrease
1. (c) 2. (c) 3. (c) 4. (b) 5. (c) 6. (a) 7. (c) 8. (c) 9. (c) 10. (d)
11. (b) 12. (c) 13. (a) 14. (b) 15. (b) 16. (a) 17. (b) 18. (a) 19. (d) 20. (d)
21. (b) 22. (a) 23. (b) 24. (a) 25. (c) 26. (c) 27. (a) 28. (b) 29. (a) 30. (d)
31. (a) 32. (c) 33. (a) 34. (c) 35. (c) 36. (d) 37. (d) 38. (b) 39. (b) 40. (d)
41. (b) 42. (b) 43. (a) 44. (b) 45. (c) 46. (a) 47. (a) 48. (b) 49. (a) 50. (b)
51. (a) 52. (*) 53. (b) 54. (c) 55. (d) 56. (a) 57. (b) 58. (c) 59. (c) 60. (d)
61. (b) 62. (d) 63. (d) 64. (c) 65. (b) 66. (a) 67. (a) 68. (a) 69. (a) 70. (d)
71. (c) 72. (c) 73. (a) 74. (a) 75. (b) 76. (c) 77. (a) 78. (c) 79. (b) 80. (b)

11 1 11
Sides , , sides can't be negative.
1. (c) Taking three no’s. 2 2 2
x + 1, y + 1, z + 1 Case (III) 3b – 2 = 6 – b
AM GM. 4b = 8
b=2
x 1 y 1 z 1
x 1 y 1 z 1 1/
So sides are 7, 4, 4.
3
3
Case I and Case II are valid.
10 3
3 4. (b) ax2 + (a + b)x + b = 0
xyz + xy + yz + zx + 11
3 b
(x + 1) (ax + b) = 0 roots are –1,
3 a
13
– 11 xyz + xy + yz + zx 5. (c) a3 + b3 + c3 – 3abc
3
= [a + b + c] [(a + b + c)2 – 3(ab + bc + ca)]
For x = 3, y = 3 and z = 4, we get maximum
= [7] [(7)2 - 3(9)]
value.
= 7(49 – 27) = 7 × 22 = 154
2. (c) (b – a) (b + a) = 2013 = 3 × 11 × 61
6. (a)
Value of ab is minimum when
b – a = 33
… (i)
andb + a = 61
… (ii) A< B< C
Solving eq. (i) and (ii), we get In ABD greatest angles is D which is
greater than C ABD is not similar to
a = 14 and b = 47
ABC.
ab = 14 × 47 = 658
7. (c)
3. (c) Case (I)
If b + 5 = 3b – 2
7
b=
2
17 17 5
So sides are , ,
2 2 2
Case (II)
1
b+5=6–b b=
2
4 n! + 10 =
In RCP cos =
5
3
and In PCO cos =
r
4 3 than exponent of 2 is 1 so it is not a perfact
5 r square.
15 14. (b)
r
4
8. (c)

15. (b) Let total number of people with salary below


Rs. 10000 per annum is x and salary is A.
BCH = BCA1 = 45° Let total number of people with salary above
C1CA1 = C1B1A1 = 90° Rs. 10000 per annum is y and salary is B then.
x XA – YB < 0
9. (c) A B
105 95
X A Y B
y average after 100 100
average before XA YB
E D C 5 XA YB
1
100 XA YB
16. (a) Centre of Mass will go downwards
F 17. (b) Vy= Uy – gt, hence correct option is (b).
Let AB = x 18. (a) All are nearly at same height hence
and BC = y P1 = P2 = P3
and area of rectangle ABCD = 60 19. (d) From graph we see that V first increases then
Now XED XBA decreases Hence a is earliar positive then
and DFY CBY negative
Now, BEF = XDY + XED + DFY + DEF a = p – qt
= 60 + DEF 20. (d) V = Const & S increases with time
1 C
= 60 + xy 21. (b) ( )=B+ 2 +...
2
= 60 + 30 = 90 Hence, > >
10. (d)
22. (a)
11. (b)
12. (c)
13. (a) If n = 1, 2, 4, 5, then (n! + 10) is not a perfect
square
If n = 3, n! + 10 is a perfact square
If n > 5
23. (b) Here, we have = m.(3 × 108)2
u = – 10 m 4 22 1.5
2
1.4 10 9
R = 1.5 m m= 109 kg/s
7 9
1 1 2 1 2 1 So, mass of sun is decreased by 109 kg/s.
v u R v 1.5 10 28. (b) P = I2R
By solving, we get
P 2 I
6%
1 1 2 P I
v 10 1.5 Hence, the power dissipated by resistor
increase approximately by 6%.
30
v 29. (a) We know that,
43
P1 V1 P2 V2
v 30
m 0.07 n1 n2
u 43 10
24. (a) Here, V1 = V and V2 = V/3
V
P1 V P1 .
3
n1 n2

n1
n2
3

2
Now, of Gas will come out to make the
3
presence P1.
So 66.66% is correct option.

30. (d) qE q V B 0
I
Now, Ratio 8 Hence, dirction of electric field is into the
I'
plane of paper.
25. (c) No EMF Induce if ring rotate about its own
axis ( = 0)
O
Hence, I, II & IV are correct ||
31. (a) C H is formyl group so it is contained in
26. (c) acetaldehyde.
33. (a) All doubly bonded carbon can be SP2 hybrid
so total of three are there.
34. (c) Valence electrons are last shell electrons.
35. (c) Number of neutrons in Na = 23 – 11 = 12
Number of neutrons in Mg = 24 – 12 = 12
36. (d) CCl4 has zero dipole moment.
20 60 4
0.1 25 I2 20 I2
20 60 20
rO 2 1
I2 = 0.2A 38. (b) 2

rH 32 4
Hence, I through 80 is given by, 2

0.1 + 0.2 = 0.3A


27. (a) E Radiated by Sun, 1
So time required by O2 = 24 = 6 sec.
4
E = 4 r2 × 1.4 kW = mC2
Now, E = 4 × (1.5 × 1011)2 × 1.4 × 103
39. (b) CH3COOH Na CH3COONa H2 2 a
63. (d) CP = a PD =
3 3
0.5
40. (d) K 3 1
C2 H 2 Let PAD = tan = (In APD)
3
Now, DAP = QBA =
3 0.5 1 1
So C2 H 2 or C2 H 2 0.5 Area of PQBC
4 8 2 Required ratio =
a2
3.42 1000 a2 area of ADP area of AQB
41. (b) Molarity = 0.1
342 100
a2
42. (b) According to their reduction potential
1 3 41
1
44. (b) As E°red.of Sn is less than that of Cu 6 20 60
2
1 1 2 1 2 100 p 100 p
45. (c) r , So order is rH rHe rLi2 64. (c) x y x
z 3 3 100 100
46. (a) Ultrafilteration of blood occure in Bowman's p2 + 100p – 1002 = 0
capsule.
47. (a) Parietal lobe central sensations of touch pain p = 12500 50
and temperature.
60 < p < 65
48. (b) Virus can be cultured only in living medium.
65. (b)After one operation amount of x, y, z in J,
49. (a) separation of Homologous chromosoms -
Meiosis - I separation of chromatids - Meiosis respectively are
- II 2 2
1 1 100
50. (b) Melanin is responsible for skin colour. 90 10 , 100 ,
11 11 11
51. (a) Image is formed before Retina. Hence,
diversity lenses are required. Similarly we can find after four operations
52. (*) A person with blood group A can donate blood amount of x,y, z in J1.
to A and A, B and can receive blood from A 66. (a) Here, 2 + S System lie above edge of 1,
and O.
53. (b) Mitochondria contain DNA M L
then, y M y 0
54. (c) G C form a base pair in DNA. 2 2
55. (d) Configuration is a type of sexual y L L
reproduction in bacteria. Where DNA is y y
exchanged between two bacteria. 2 2 3
56. (a) Insects.
57. (b) pH of stomach = 1.8 – 2.0 Acidic
Intestine = 7.4 – 7.6 Alkaline
58. (c) Ammonia is converted to urea in liver cells.
59. (c) Xeric plants require thick cuticle to prevent
water loss.

62. (d) 2(a + b) + ab = 16 ... (i)


2(b + c) + bc = 24 ... (ii) Now 1 + 2 + S centre of mass will lie above
2(c + a) + ca = 31 ... (iii) the table
From eq. (ii), eq. (iii) 3M L L L
x M x 0
(a – b) (2 + c) = 7 ... (iv) 2 3 3 2
From eq. (ii) and eq. (iv) 3x L L L
x 0
4a = 2 + 5b ... (v) 2 2 3 2
By solving eq. (i) and (v), we get 5x 4L 4L 2 8L
x
b = 2 a = 3, c = 5 2 3 5 3 15
Volume = 30
67. (a) (I) Let initial distance between P & Q is x 31 2
1 9 10 4 106
V 45V
2 1.6 10 19
45 V from higher to lower potential.
x
at t1 a receive the ball. Ka 10 5 2
2 71. (c) x 10
Next ball C .1
x 5 72. (c) Br Br
t2 5 |
2
5 CH3 CH 2 CH CH CH2 CH3
t
2 Zn dust
(II) in second case
at t = 0 P throws the ball. CH 3 CH 2 CH CH CH 2CH 3 ZnBr2

Ozonolysis

2CH 3CH 2 CHO


x
Here, t1 .
73. (a) 2NaOH Zn Na 2 ZnO2 H2
3
Now, next ball
For 2 g Hydrogen, Zinc required = 65.4 g
x 5
t2 5
3 65.4
For 1 g Hydrogen, Zinc required =
10 2
t
3 = 32.7 g
68. (a) Pt = mwSw T + mcsc T
75. (b) CH3 CH3 CH3 CH 3
10 × 15 × 60 = 0.5 × 4200 × 3 + mcsc × 3 Br2in | |
C=C CH3– C – C – CH 3
9000 = 6300 + mcsc3 CCl4
CH3 CH3 | |
mcsc = 900 J/k. Now, for oil Br Br
10 × 20 × 60 = 2 × S0 × 2 + 900 × 2
Alc. KOH
12000 - 1800 = 4 S0
10200 CH2= C – C = CH 2
S0 = = 2.51 × 103 J/kg-k | |
4 CH3 CH 3
69. (a)
76. (c) Sister chromatids contain same alleles of the
same gene at the same locus because they
are identical DNA molecules.
77. (a) When the blood sugar is low after
administering the insulin, brain gets
inadequate sugar. Hence the person should
be given sugar/glucose.
78. (c) Infants has high heart rate.
For example, embryonic pulse for a 45 days
Here,
old embryo will be 160 – 200 beats per
minute.
S1 r S2 = – 2r S3 r
4 4 dN N
Now, total angle of deviation of ray is, 80. (b) rN 1
dt K
3
S = S1 + S2 + S3 = 4r When population (N) approach carrying
2 capacity (K)
1 2
70. (d) qV = mv2 V = 1 mv dN
2 2 q will
dt
7 . A regular octagon is formed by cutting congruent
isosceles right-angled triangles from the corners
1 . Let f(x) be a quadratic polynomial with f(2) = 10 of a square. If the square has side- length 1, the
and f(2) = – 2. Then the coefficient of x in f(x) is– side-length of the octagon is –
(a) 1 (b) 2 2 1
(c) 3 (d) 4 (a) (b) 2 1
2
2 . The square root of
5 1 5 1
(c) (d)
(0.75)3 4 3
0.75 (0.75)2 1 is –
1 (0.75) 8. A circle is drawn in a sector of a larger circle of
(a) 1 (b) 2 radius r, as shown in the adjacent figure. The
smaller circle is tangent to the two bounding radii
(c) 3 (d) 4
and the arc of the sector. The radius of the small
3 . The sides of a triangle are distinct positive circle is –
integers in an arithmetic progression. If the
smallest side is 10, the number of such triangles
is –
(a) 8 (b) 9
(c) 10 (d) Infinitely many
4. 60
r
a a b a b c a b c d a
, then r r
3 4 5 6 b 2 c 3d (a) (b)
is – 2 3
1 2 3r r
(a) (b) 1 (c) (d)
2 5 2
(c) 2 (d) Not determinable 9. In the figure AHKF, FKDE and HBCK are unit
squares; AD and BF intersect in X. Then the ratio
22 42 62 (2n)2 of the areas of triangles AXF and ABF is –
5 . For to exceed 1.01, the
12 32 52 (2n 1)2
maximum value of n is –
(a) 99
(b) 100
(c) 101
(d) 150
6 . In triangle ABC, let AD, BE and CF be the
internal angle bisectors with D, E and F on the
sides BC, CA and AB respectively. Suppose AD,
BE and CF concur at I and B, D, I, F are concyclic, 1 1
(a) (b)
then IFD has measure – 4 5
(a) 15° (b) 30° 1 1
(c) (d)
(c) 45° (d) Any value 90° 6 8
10. Suppose Q is a point on the circle with centre P 15. Let S = {1, 2, 3, …, 40} and let A be a subset of S
and radius 1, as shown in the figure; R is a such that no two elements in A have their sum
point outside the circle such that QR = 1 and divisible by 5. What is the maximum number of
QRP = 2°. Let S be the point where the segment elements possible in A?
RP intersects the given circle. Then measure of (a) 10 (b) 13
RQS equals –
(c) 17 (d) 20

16. A clay ball of mass m and speed v strikes another


metal ball. of same mass m, which is at rest. They
stick together after collision. The kinetic energy
of the system after collision is –

mv2 mv2
(a) (b)
2 4

(a) 86° (b) 87° (c) 2mv2 (d) mv 2


(c) 88° (d) 89° 17. A ball falls vertically downward and bounces off a
horizontal floor. The speed of the ball just before
11. Observe that, at any instant, the minute and hour
reaching the floor (u1) is equal to the speed just
hands of a clock make two angles between them
after having contact with the floor (u2) ; u1 = u2.
whose sum is 360°. At 6 : 15 the difference between
The corresponding magnitudes of accelerations
these two angles is –
are denoted respectively by a1 and a2. The air
(a) 165° (b) 170° resistance during motion is proportional to speed
(c) 175° (d) 180° and is not negligible. If g is acceleration due to
12. Two workers A and B are engaged to do a piece of gravity, then–
work. Working alone, A takes 8 hours more to (a) a1 < a2
complete the work than if both worked together. (b) a1 > a2
On the other hand, working alone, B would need
(c) a1 = a2 g
1 (d) a1 = a2 = g
4 hours more to complete the work than if both
2 18. Which of the following statements is true about
worked together. How much time would they take the flow of electrons in an electric circuit?
to complete the job working together? (a) Electrons always flow from lower to higher
(a) 4 Hours (b) 5 Hours potential
(c) 6 Hours (d) 7 Hours (b) Electrons always flow from higher to lower
13. When a bucket is half full, the weight of the bucket potential
and the water is 10 kg. When the bucket is two - (c) Electrons flow from lower to higher potential
thirds full, the total weight is 11kg. What is the except through power sources
total weight, in kg, when the bucket is completely (d) Electrons flow from higher to lower potential,
full – except through power sources
1 19. A boat crossing a river moves with a velocity v
(a) 12 (b) 12
2 relative to still water. The river is flowing with a

2 v
(c) 12 (d) 13 velocity with respect to the bank. The angle
3 2
14. How many ordered pairs of (m, n) integers satisfy with respect to the flow direction with which the
boat should move to minimize the drift is–
m 12
? (a) 30º
12 n
(b) 60º
(a) 30 (b) 15
(c) 150º
(c) 12 (d) 10
(d) 120º
20. In the Arctic region hemispherical houses called 24. Following figures show different combinations of
Igloos are made of ice. It is possible to maintain a identical bulb(s) connected to identical
temperature inside an Igloo as high as 20º C because battery(ies). Which option is correct regarding the
(a) Ice has high thermal conductivity total power dissipated in the circuit–
(b) Ice has low thermal conductivity
(c) Ice has high specific heat
(d) Ice has higher density than water
21. In the figure below, PQRS denotes the path
followed by a ray of light as it travels through
P Q
three media in succession. The absolute refractive
indices of the media are 1, 2 and 3 respectively.
(The line segment RS’ in the figure is parallel to
PQ).

R S

(a) P < Q < R < S (b) P < Q < R = S


(c) R < Q < P < S (d) P < R < Q < S
25. A circular metallic ring of radius R has a small
(a) > > (b) = < gap of width d. The coefficient of thermal
1 2 3 1 3 2
expansion of the metal is in appropriate units.
(c) 1 < 2 < 3 (d) 1 < 3 < 2 If we increase the temperature of the ring by a
22. A ray of white light is incident on a spherical water amount T, then width of the gap –
drop whose center is C as shown below. When (a) Will increase by an amount d T
observed from the opposite side, the emergent light
(b) Will not change
(c) Will increase by an amount (2 R -d) T
(d) Will decrease by an amount d T
C 26. A girl holds a book of mass m against a vertical
wall with a horizontal force F using her finger so
that the book does not move. The frictional force
(a) Will be white and will emerge without on the book by the wall is –
deviating (a) F and along the finger but pointing towards
(b) Will be internally reflected the girl
(c) Will split into different colors such that the (b) F upwards where is the coefficient of static
angles of deviation will be different for all colors friction
(d) Will split into different colors such that the (c) mg and upwards
angles of deviation will be the same for all (d) Equal and opposite to the resultant of F and
colors mg
23. A convex lens of focal length 15 cm is placed in 27. A solid cube and a solid sphere both made of
front of a plane mirror at a distance 25 cm from same material are completely submerged in
the mirror. Where on the optical axis and from water but to different depths. The sphere and
the centre of the lens should a small object be the cube have same surface area. The buoyant
placed such that the final image coincides with force is–
the object? (a) Greater for the cube than the sphere
(a) 15 cm and on the opposite side of the mirror (b) Greater for the sphere than the cube
(b) 15 cm and between the mirror and the lens (c) Same for the sphere and the cube
(c) 7.5 cm and on the opposite side of the mirror (d) Greater for the object that is submerged
(d) 7.5 cm and between the mirror and the lens deeper
238 214
33. The number of electrons plus neutrons in
28. 92 U atom disintegrates to 84 Po with a half life 40
19 K
is –
of 4.5 × 109 years by emitting six alpha particles
and n electrons. Here n is – (a) 38 (b) 59
(a) 6 (b) 4 (c) 39 (d) 40
(c) 10 (d) 7 34. Among the following, the most basic oxide is –
29. Which statements about the Rutherford model of (a) Al2O3 (b) SiO2
the atom is NOT true? (c) P2O 5 (d) Na2O
(a) There is a positively charged center in an atom 35. By dissolving 0.35 mole of sodium chloride in
called the nucleus water, 1.30 L of salt solution is obtained. The
(b) Nearly all the mass of an atom resides in the molarity of the resulting solution should be
nucleus reported as –
(c) Size of the nucleus is completely to the atom (a) 0.3 (b) 0.269
(d) Electrons occupy the space surrounding the (c) 0.27 (d) 0.2692
nucleus 36. Among the quantities, density ( ), temperature (T),
30. A girl brings a positively charged rod near a thin enthalpy (H), heat capacity (Cp), volume (V) and
neutral stream of water from a tap. She observes pressure (P), a set of intensive variables are
that the water stream bends towards her. Instead, (a) ( , T, H) (b) (H, T, V)
if she were to bring a negatively charged rod near (c) (V, T, Cp) (d) ( , T, P)
to the steam, it will– 37. The value of ‘x’ in KAl(SO4)x·12H2O is –
(a) Bend in the same direction (a) 1 (b) 2
(b) Bend in the opposite direction (c) 3 (d) 4
(c) Not bend at all 38. Among the following substituted pyridines, the
(d) Bend in the opposite direction above and below most basic compound is –
and rod

(a) (b)
31. The weight of calcium oxide formed by burning
20 g of calcium in excess oxygen is –
(a) 36 g (b) 56 g
(c) 28 g (d) 72 g
32. The major products in the reaction (c) (d)

NaOH
Br3CCHO are –
39. The major product in the following reaction is –
H3 C C C H 2HBr (excess)
(a)

(a) (b)

(b)
(c) (d)

40. The major product in the following reaction at


(c) 25° C is –
CH3CH2 NH2
CH3COOH
(d) (a) CH3CONHCH2CH3
(b) CH3CH=NCH2CH3
(c) NH3+CH2CH3·CH3COO–
(d) CH3CON=CHCH3
41. A reaction with reaction quotient Q C and
equilibrium constant K C, will proceed in the (c)
direction of the products when –
(a) QC = KC (b) QC < KC PV
(c) QC > KC (d) QC = 0
42. Acetylsalicylic acid is a pain killer and is commonly
known as –
(a) paracetamol T

(b) ibuprofen at constant P


(c) aspirin
(d) penicillin V
43. The molecule which does not exhibits strong
hydrogen bonding is – (d)
(a) methyl amine
(b) diethyl ether –273°C
T
(c) acetic acid
(d) glucose
44. The following two compounds are – 46. A smear of blood from a healthy individual is
stained with a nuclear stain called hematoxylin
and then observed under a light microscope.
Which of the following cell type would be highest
in number ?
(a) neutrophils (b) lymphocytes
(c) eosinophils (d) monocytes
47. Which of the following biological phenomenon
(a) geometrical isomers involves a bacteriophage?

(b) positional isomers (a) transformation (b) conjugation

(c) functional group isomers (c) translocation (d) transduction

(d) optical isomers 48. In which compartment of a cell does the process
of glycolysis takes place?
45. The graph that does not represent the behaviour
of an ideal gas is – (a) golgi complex (b) cytoplasm
(c) mitochondria (d) ribosomes
at constant T 49. Huntington’s disease is a disease of the –
(a) nervous system (b) circulatory system
(c) respiratory system (d) excretory system
P
50. A cell will experience the highest level of
(a) endosmosis when it is kept in –
(a) distilled water (b) sugar solution
V (c) salt solution (d) protein solution
51. When the leaf of the ‘touch-me-not’ (chui-mui,
at constant P Mimosa pudica) plant is touched, the leaf droops
(b) because –
P
(a) a nerve signal passes through the plant
(b) the temperature of the plant increases
(c) water is lost from the cells at the base of the
leaf
1/V (d) the plant dies
52. If you are seeing mangroves around you, which 57. Which one of the following organelles can
part of India are you visiting? synthesize some of its own proteins ?
(a) Western Ghats (a) lysozome (b) golgi apparatus
(b) Thar desert (c) vacuole (d) mitochondrion
(c) Sunderbans 58. Maltose is a polymer of –
(d) Himalayas (a) one glucose and one fructose molecule
53. Myeloid tissue is a type of – (b) one glucose and one galactose molecule
(a) haematopoietic tissue (c) two glucose molecules
(b) cartilage tissue (d) two fructose molecules
(c) muscular tissue 59. The roots of some higher plants get associated
with a fungal partner. The roots provide food to
(d) areolar tissue
the fungus while the fungus supplies water to the
54. The heat of an amphibian is usually – roots. The structure so formed is known –
(a) two chambered (a) lichen
(b) three chambered (b) anabaena
(c) four chambered (c) mycorrhiza
(d) three and half chambered (d) rhizobium
55. Gigantism and acromegaly are due to defects in 60. Prehistoric forms of life are found in fossils. The
the function of the following gland – probability of finding fossils of more complex
(a) adrenals (b) thyroid organisms –
(c) pancreas (d) pituitary (a) increases from lower to upper strats
56. The pH of 10–8 M HCl solution is – (b) decreases from lower to upper strata
(a) 8 (b) close to 7 (c) remains constant in each stratum
(c) 1 (d) 0 (d) uncertain

63. In a triangle ABC, it is known that AB = AC.


Suppose D is the mid-point of AC and BD = BC = 2.
Then the area of the triangle ABC is –
a 2 b
61. Let a, b, c be positive integers such that (a) 2
b 2 c
is a rational number, then which of the following (b) 2 2
is always an integer?
(c) 7
2 2 2 2
2a b a 2b
(a) (b) (d) 2 7
2 2 2
2b c b 2 c2
64. A train leaves Pune at 7 : 30 am and reaches
Mumbai at 11 : 30 am. Another train leaves
a2 b2 c2 a2 b2 c2
(c) (d) Mumbai at 9 : 30 am and reaches Pune at 1 :00 pm.
a b c a c b
Assuming that the two trains travels at constant
62. The number of solutions (x, y, z) to the system of speeds, at what time do the two trains cross each
equations other –
x + 2y + 4z = 9, 4yz + 2xz + xy = 13, xyz = 3 (a) 10 : 20 am
Such that at least two of x, y, z are integers is – (b) 11 : 30 am
(a) 3 (b) 5 (c) 10 : 26 am
(c) 6 (d) 4 (d) Data not sufficient
65. In the adjacent figures, which has the shortest shooting point. Assuming that all trajectories are
path – contained in the same plane, how far will the
other piece fall from the shooting point? (Take
g = 10 ms–2 and neglect air resistance)–
(a) 63 m or 144 m (b) 72 m or 9 m
(c) 28 m or 72 m (d) 63 m or 117 m
68. A block of mass m is sliding down an inclined
plane with constant speed. At a certain instant
t 0 , its height above the ground is h. The
coefficient of kinetic friction between the block
and the plane is . If the block reaches the ground
at a later instant tg, then the energy dissipated
by friction in the time interval (tg – t0) is–
(a) Fig 1 (b) Fig 2
(c) Fig 3 (d) Fig 4

66. In the circuit shown, n identical resistors R are


connected in parallel (n > 1) and the combination
is connected in series to another resistor R0. In
the adjoining circuit n resistors of resistance R
are all connected in series along with R0–

mgh
R (a) mgh (b)
sin

n mgh
R0 (c) mgh (d)
cos
R 69. A circular loop of wire s in the same plane as an
R
infinitely long wire carrying a constant current i.
Four possible motions of the loop are marked by
N, E, W, and S as shown. –
E

n
R0 R R R

A clockwise current is induced in the loop when


E
loop is pulled towards
The batteries in both circuits are identical and
(a) N (b) E
net power dissipated in the n resistors in both
circuits is same. The ratio R0/R is (c) W (d) S
(a) 1 (b) n 70. 150 g of ice is mixed with 100 g of water at
temperature 80º C. The latent heat of ice is
(c) n 2 (d) 1/n
80 cal/g and the specific heat of water is 1 cal/g– ºC.
67. A firecracker is thrown with velocity of 30 ms–1 Assuming no heat loss to the environment, the
in a direction which makes an angle of 75º with amount of ice which does not melt is –
the vertical axis. At some point on its trajectory,
(a) 100 g (b) 0 g
the firecracker splits into two identical pieces in
such a way that one piece fall 27 m far from the (c) 150 g (d) 50 g
M1 M2 M3 M4
71. Upon fully dissolving 2.0 g of a metal in sulfuric
acid, 6.8 g of the metal sulfate is formed. The
equivalent weight of the metal is –
(a) 13.6 g (b) 20.0 g
(c) 4.0 g (d) 10.0 g
72. Upon mixing equal volumes of aqueous solutions
of 0.1 M HCl and 0.2 M H2SO4, the concentration
of H+ in the resulting solution is–
(a) 0.30 mol/L (b) 0.25 mol/L
(c) 0.15 mol/L (d) 0.10 mol/L
73. The products X and Y in the following reaction (a) Li, Na, K, and Rb (b) Rb, K, Na, and Li
sequence are – (c) Na, K, Li, and Rb (d) Rb, Li, Na, and K
75. The number of moles of Br2 produced when two
moles of potassium permanganate are treated with
excess potassium bromide in aqueous acid
medium is –
(a) 1 (b) 3
(c) 2 (d) 4

(a) 76. A baby is born with the normal number and


distribution of rods, but no cones in his eyes. We
would expect that the baby would be –
(a) colour blind (b) night blind
(c) blind in both eyes (d) blind in one eye
77. In mammals, pleural membranes cover the lungs
as well as insides of the rib cage. The pleural fluid
(b)
in between the two membranes –
(a) dissolves oxygen for transfer to the alveoli
(b) dissolves CO2 for tranfer to the blood
(c) provides partial pressure
(d) reduces the friction between the ribs and the
lungs
(c)
78. At which phase of the cell cycle, DNA polymerase
activity is at its highest?
(a) Gap 1 (G1) (b) Mitotic (M)
(c) Synthetic (S) (d) Gap 2 (G2)
79. Usain Bolt, an Olympic runner, at the end of a
100 meter sprint, will have more of which of the
(d) following in his muscles?
(a) ATP (b) Pyruvic acid
(c) Lactic acid (d) Carbon dioxide
74. A plot of the kinetic energy (½mv2) of ejected 80. Desert temperature often varies between 0 to
electrons as a function of the frequency ( ) of 50°C. The DNA polymerase isolated from a camel
incident radiation for four alkali metals (M1, M2, living in the desert will be able to synthesize DNA
M3, M4) is shown below. most efficiently at –
The alkali metals M 1, M 2 , M 3 and M 4 are, (a) 0° C (b) 37° C
respectively – (c) 50° C (d) 25° C
1. (c) 2. (b) 3. (b) 4. (a) 5. (d) 6. (b) 7. (b) 8. (b) 9. (b) 10. (d)
11. (a) 12. (c) 13. (d) 14. (a) 15. (c) 16. (b) 17. (a) 18. (c) 19. (d) 20. (b)
21. (d) 22. (a) 23. (a) 24. (d) 25. (a) 26. (c) 27. (b) 28. (b) 29. (c) 30. (a)
31. (c) 32. (a) 33. (c) 34. (d) 35. (b) 36. (d) 37. (b) 38. (c) 39. (b) 40. (a)
41. (b) 42. (b) 43. (b) 44. (b) 45. (b,c) 46. (a) 47. (d) 48. (b) 49. (a) 50. (a)
51. (c) 52. (c) 53. (a) 54. (b) 55. (d) 56. (b) 57. (d) 58. (c) 59. (c) 60. (a)
61. (d) 62. (b) 63. (c) 64. (b) 65. (b) 66. (a) 67. (d) 68. (c) 69. (b) 70. (d)
71. (b) 72. (b) 73. (b) 74. (b) 75. (*) 76. (a) 77. (d) 78. (c) 79. (c) 80. (d)

10, 10 + d , 10 + 2d
and sum of a + b > c
1 . (c) Let f x ax2 bx c
20 + d > 10 + 2d
Now f (2) a(2)2 b(2) c : 4 a 2b c 10 > d
= 10 ...(i) As the d is minimum, Hence total possibility
2 of d is 9
f ( 2) a( 2) b( 2) c : 4 a 2b c
=–2 (ii) a a b a b c a b c d
4 . (a) k (Let)
Solving eq. (i) and (ii) we get 3 4 5 6
b=3 a = 3k, a + b = 4k
2 . (b) Let x = 0.75 b = 4x – a = k
Hence question, Similarly, c = k add = k
3
x a 3k 1
x2 x 1 P
1 x b 2 c 3d k 2 k 3k 2
x3 x2 x 1 22 12 22 32 n2
P 5 . (d)
1 x 12 22 32 (2n)2 22 12 22 32 n2
3
x x3 1
P 22 n(n 1)(2n 1)
1 x
= 6 1.0
1
P 2n(2n 1)(4 n 1) 22 n(n 1)(2n 1)
1 x
6 6
1 4
P 2n 2
3 1 1.01
1
4 2n 1
P 2 3
1 1.01
3 . (b) 2n 1
2n 1 300
n 150.5
6 . (b)

As a, b, c are in AP (criteria to form any


triangle)
A C Equation of BF
AIC 180 (y – 2) = 2(x – 1)
2
y – 2 = 2x – 2
B y = 2x ...(i)
90
2 equation of AD

B (y – 1) =
Now, 90 + + B = 180º
2
B = 60º x + 2y = 4 ...(ii)
This will be case of equilateral triangle Solving equation (i) and (ii) we get,
IFD 30º (x, y) =
7 . (b)
1 8
1 2
ar AFX 2 5 1
ar ABF 1 5
1 2
2
10. (d)

According to question,
2
1 x
2 x2
2
2
1 x 2 x2
x 2 1
8 . (b)
By Cosine rule
2
QS 2 2 cos 2
QS = 2 sin1°
Now by sine rule in RQS,
sin 2
sin
2sin 1
Let the radius of smaller circle be x = 89º
Here, RQS = 180º – (2º +89º) = 180º – 91º = 89º

In OAP, = cosec 30° = 11. (a) We want to find here angle between minute
hand and hour hand at 6 : 15
OP = x cosec 30°
and OQ = r = x + x cosec 30º
r
x=
3
9 . (b)

Hour hand covers 30º in 60 minute.


Then in 15 minute it covers = 7.5º
So angle between both hand at 6 : 15 is
90º + 7.5 = 97.5º
Another angle is 360º – 97.5º = 262.5º
Hence difference is 262.5º – 97.5º = 165º
12. (c) If A & B work together take time t hr. If A 15. (c) A {1, 2, 6, 7, 11, 12, 16, 17, 21, 22, 26, 27, 31,
work alone takes t + 8 hr. to complete the work 32, 36, 37} and One of the element which is
1 multiple of 5
Work/hr by A is .
t 8 B {3, 4, 8, 9, 13, 14, 18, 19, 23, 24, 28, 29, 33,
1 34, 38, 39} and One of the element which is
Similarly for B work/hr is multiple of 5
9
t
2 16. (b) Applying the law of conservation of momentum,
When A & B work together there work/hr mv + 0 = (2m) v’
1 1 v
=
t 8 9 v’ =
t 2
2
Work will be done in t hr. 1
K.E = (2m)v’2
2
1 1
t 1 17. (a)
t 8 9
t
2
Now by solving we get
t=6
13. (d) Here weight will be measured in two criteria’s
weight of bucket w + weight of water ( r2h ×
density) FR
Downward- a1 = g –
In, First condition, m
h FR
w r2 10 Upward :- a2 = g +
...(i) m
2
In second condition, a2 < a1
2 19. (d)
w r2 h 11 ...(ii)
3
Subtracting eq. (i) from eq. (ii) we get
2 1
r2 h 1 To minimize the drifting
3 2
vw 1
h sin
r2 1 vbw 2
6
r2h 6 90º + = 120º
2
r h 20. (b) Ice has low thermal conductivity
Also, w 10
2 So no exchange of heat outside surrounding.
w 3 10 21. (d) 1 < 3 < 2
22. (a) Perpendicular incidence so no deviation.
w=7

Hence total weight is w r 2 h = 7 + 6 = 13


23. (a)
m 12
14. (a)
12 n
144
mn 144 n 24. (d) Let R = resistance of each bulb.
3
We want to find here total number of divisors
of 144
144 = 24·32
Total divisors are (4 + 1) (2 + 1) = 15
But negative pairs are also possible
total divisor = 30
25. (a) Will increases by an amount d T 38. (c) Because of Hyperconjugation of Methyl group.
26. (c)
39. (b) CH 3 C C H

Markovnikov HBr (excess)


Addition
Br

CH3 C CH3

f = mg Br
Vcube a3
27. (b) buyount force B = V lg,
Vsphere 4 40. (a) CH3COOH CH3CH2 NH2
R3
3
H2 O
but it is given 6 a2 4 R2
CH3CONHC2 H5
Vcube
so,
Vsphere 6 41. (b) If QC < KC, reaction moves forward to attain
28. (b) 238 214 equilibrium.
92 U 84 Po 6 ne
92 = 84 + 12 – n 43. (b) diethyl ether don't have any Hydrogen Bond.
n=4 44. (b) 1–Butene and 2–butene are positional isomers.
29. (c) Size of the nucleus is completely to the atom
46. (a) Most abundant WBC are neutrophils (50 –
29. (a) Due to induction, bend in same direction 70% of WBC)
31. (c) 2Ca + O2 2Ca0 47. (d) Transduction: Genetic recombination with
the help of a virus/bacteriophase.
20 1
Moles of calcium = 48. (b) Glycolysis Glucose 2 × Pyrubic acit + AT
40 2

Occurs in Cytoplasm
1
So moles of CaO formed =
2 49. (a) Huntington's Disease – Neurodegenerative
genetic disorder, affects muscles
m coordination.
Now, moles =
M
50. (a) W for distilled water is zero. Hence will
show maximum endosmosis.
1 m
, So m 28g
2 56 51. (c) Charge in turgor pressure causes leaf drop
in mimosa pudica.
32. (a) CBr3 CHO NaOH
Haloform Re action
CHBr3 HCOONa 52. (c) In india mangrover are found in Sundarbans.
33. (c) Number of electrons = 19 – 1 = 18 53. (a) Myeloid Tissue - mainly found in red bone
marrow and produce blood cells, hence the
Number of neutrons = 21 name haematopoietic tissue.
34. (d) As Na2O forms NaOH on adding in water and 54. (b) Amphibian Heart is 3 - chambered.
NaOH is strong base so Na2O most basic.
(2 atria and 1 ventricle)
0.35 55. (d) Over secretion of growth hormone by
35. (b) M = 0.269
1.3 Pituitary, causes Gigantism and this
Syndrome is Acromegaly
36. (d) Density, Temperature and Pressure are
intensive properties. 57. (d) Mitochondria contain DNA, RNA and
ribosome's.
37. (b) As K has one valency and Al has 3 valency, so
value of x will be 2.
58. (c) Maltose is made up of two molecules of AB2
glucose. AB2 4 8
4
60. (a) Evolution of simple organism took place
earlier and complex organism evolved later. AB2
4
Complex organism fossil are found in upper 4
strata (Recent fossilization) AB2 = 8
AB = 2 2

a 2 b b 2 c Now Area 1 7 7 square unit.


61. (d)
b 2 c b 2 c
2ab 2ac 2b2 bc
2 2
2b c
b 2a c 2 b2 ac
2 2
2b c
Is rational when b2 = ac i.e. a, b, c are in GP 64. (b) First train from Pune to Mumbai takes
Here given option (A), (B), (C) does not satisfy 4 hrs.
the criteria Second train from Mumbai to Pune takes
But option (D) always satisfy 3.5 hrs.
62. (b) x + 2y + 4z = 9 ; Speed of train from Pune to Mumbai is
2xy + 8yz + 4xz = 26 ; d
V1
(x) (2y) (4z) = 24 4
Speed of train from Mumbai to Pune is
Say roots of P3 – 9P2 + 26P – 24 = 0 are x, 2y
and 4z d
V2
Here (P – 2) (P2 –7P + 12) = 0 3.5
Distance traveled by first train till 9:30 is
(P – 2) (P –3) (P – 4) = 0
now 4z = 4 ; 2y = 2 ; x = 3 then (3, 1, 1) d d
x 2
4 2
3 Say now both trains meet after time from 9:30
x = 2 ; 2y = 4 ; 4z = 3 then 2, 2,
4
d
x
1 x 2 4x 3.5 3.5x
x = 3; 2y = 4 ; 4z = 2 then 3, 2,
2 d/4 d d 2 d
3 15x 3.5d 30x 7d
x = 2; 2y = 3 ; 4z = 4 then 2, , 1
2 time taken by first train from 9: 30 is
3 4x 7 14
x = 4; 2y = 2 ; 4z = 3 then 4, 1, 4 = 56 min
4 d 30 15
Hence there are five solutions. Then both train meet at 9.30 + 56 min
63. (c) = 10 : 26 am
65. (b) A(1, 1); B(2, 3); C(5, 2); D(3, 0)
For Fig 1

AB 1 4 5

BC 9 1 10
We know
CD 4 4 8
AB2 + BC2 = 2(CD2 + BD2)
then AB + BC +CD = 8.22
2 AB2
AB 4 2 4 For Fig 2
4
70. (d) Heat loss by water = heat gain by ice.
AB 1; BC 2; CD 10 ; DE 8
100 × 1 × 80 = m × 80
AB + BC + CD + DE= 8.99
m = 100 gm ice melt
Is max distance
Remaining ice = 50 g
For Fig 3
71. (b) Equivalents of Metal = Equivalent of Metal
AB 5 ; BC 2; CD 5 ; DE 5 Sulphate
AB + BC + CD + DE = 8.70
For Fig 4 2 6.8
E E 48
AB 1; BC 2; CD 13 ; DE 5
AB + BC + CD + DE = 8.841 Solving, E = 20

nE E 0.1V 2X.2V
66. (a) i1 , i2 72. (b) H 0.25
nR0 R R0 nR mix. V V

nE2R nE2 R
P1 , P2 73. (b) NO2 NH2 Br
2 2
nR0 R R0 nR
Sn+ (i) NaNO/HCl
2

P1 = P2 HCl (ii) Cu Br
Hence R0/R = 1 (X) (Y)
67. (d)
75. (Bonus)
KMnO4 KBr H2SO4 MnSO4 K2SO4
Br2 H2O
Balanced equations
2KMnO4 + 10KBr + 8H2SO4 2MnSO4 +
5Br2 + 6K2SO4 + 8H2O
Under influence of constant force centre of Eq.of KMnO4 = Eq.of Br
mass follows its original path
Mole × n = Mole × n
1 2 × 5 = Mole × 2
30 30
R 2 45 m 5 moles of Br2 are formed
10
76. (a) Total colour blindness occur when 2 or all 3
m 27 mx types of cone pigments are missing. Vision
45 in such person becomes monochromatic like
m m
black & white television.
x = 63 m, 117 m 77. (d) Pleural fluid reduces the friction between the
68. (c) Wf Wmg K.E. , ( K.E. 0) ribs and the lungs.
Wf Wmg 78. (c) DNA replication occurs in s-phase of cell cycle.
Hence DNA polymorase will be maximum
Wf = mgh active in s-phase.
Energy dissipated = mgh 79. (c) During 100 meter sprint He needs to run fast,
hence his oxygen will become a limiting factor
69. (b) This is in accordance with Lenz’s law so anaerobic respiration will produce lactic
acid.
80. (d) Camel is a warm blooded animal, and
optimum temperature for DNA synthesis is
37°C.
6. Let ABC be a triangle with B = 90º. Let AD be the
bisector of A with D on BC. Suppose AC = 6 cm
1. Suppose a, b, c are three distinct real numbers. and the area of the triangle ADC is 10 cm2. Then
x b x c x c x a the length of BD in cm is equal to
Let P ( x)
a b a c b c b a 3 3
(a) (b)
x a x b 5 10
+
c a c b
When simplified, P(x) becomes 5 10
(c) (d)
3 3
(a) 1
(b) x 7. A piece of paper in the shape of a sector of a circle
(see Fig. 1) is rolled up to form a right-circular
x2 a b c ab bc ca cone (see Fig. 2). The value of the angle is.
(c)
a b b c c a
(d) 0
2. Let a, b, x, y be real numbers such that a2 + b2 = 81,
x2 + y2 = 121 and ax + by = 99. Then the set of all
possible values of ay – bx is -
9 9
(a) 0, (b) 0,
11 11
9 10 9
(c) {0} (d) , 0 (a) (b)
11 13 13
5 6
1 1 1 (c) (d)
3. If x + = a, x2 + 3 = b, then x3 + is - 13 13
x x x2
8. In the adjoining figure AB = 12 cm, CD = 8 cm,
(a) a3 + a2 – 3a – 2 – b (b) a3 – a2 – 3a + 4 – b
BD = 20 cm; ABD = AEC = EDC = 90º. If
(c) a3 – a2 + 3a – 6 – b (d) a3 + a2 + 3a – 16 – b BE = x, then
4. Let a, b, c, d be real numbers such that a b = 2,
b c = 3, c d = 4. Then the sum of all possible
values of a d is
(a) 9 (b) 18
(c) 24 (d) 30
5. Below are four equations in x. Assume that
0 < r < 4. Which of the following has the largest
solution for x ? (a) x has two possible values whose difference
is 4
x x
r r (b) x has two possible values whose sum is 28
(a) 5 1 9 (b) 5 1 9
17 (c) x has only one value and x 12
(d) x cannot be determined with the given
x
x 1 information
(c) 5 1 2r 9 (d) 5 1 9
r
9. Three circles each of radius 1, touch one another 15. There are 30 questions in a multiple-choice test.
externally and they lie between two parallel lines. A student gets 1 mark for each unattempted
The minimum possible distance between the lines question, 0 mark for each wrong answer and
is 4 marks for each correct answer. A student
answered x question correctly and scored 60. Then
(a) 2 + 3 (b) 3 + 3 the number of possible value of x is
1 (a) 15 (b) 10
(c) 4 (d) 2 +
3 (c) 6 (d) 5

10. The number of distinct prime divisors of the


number 5123 – 2533 – 2593 is 16. A simple pendulum oscillates freely between
(a) 4 (b) 5 points A and B.
(c) 6 (d) 7
11. Consider an incomplete pyramid of balls on a
square base having 18 layers; and having 13 balls
on each side of the top layer. Then the total
number N of balls in that pyramid satisfies
(a) 9000 < N < 10000 (b) 8000 < N < 9000
(c) 7000 < N < 8000 (d) 10000 < N < 12000 We now put a peg (nail) at some point C as shown.
12. A man wants to reach a certain destination. One- As the pendulum moves from A to the right, the
sixth of the total distance is muddy while half the string will bend at C and the pendulum will go
distance is tar road. For the remaining distance to its extreme points D. Ignoring friction, the
he takes a boat. His speed of traveling in mud , point D
in water, on tar road is in the ratio 3 : 4 : 5. The (a) will lie on the line AB
ratio of the durations he requires to cross the
(b) will lie above the line AB
patch of mud, stream and tar road is
(c) will lie below the line AB
1 4 5 (d) will coincide with B
(a) : : (b) 3 : 8 : 15
2 3 2 17. A small child tries to moves a large rubber toy
(c) 10 : 15 : 18 (d) 1 : 2 : 3 placed on the ground. The toy does not move but
13. A frog is presently located at the origin (0, 0) in gets deformed under her pushing force F which
the xy-plane. It always jumps from a point with is obliquely upward as shown. Then
integer coordinates to a point with integer
coordinates moving a distance of 5 units in each
jump. What is the minimum number of jumps
required for the frog to go from (0, 0) to (0, 1) ?
(a) 2
(b) 3
(c) 4
(d) 9 (a) the resultant of the pushing force F , weight
14. A certain 12-hour digital clock displays the hour of the toy, normal force by the ground on the
and the minute of a day. Due to a defect in the toy and the frictional force is zero
clock whenever the digit 1 is supposed to be (b) the normal force by the ground is equal and
displayed it displays 7. What fraction of the day opposite to the weight of the toy
will the clock show the correct time?
(c) the pushing force F of the child is balanced
1 5 by the equal and opposite frictional force
(a) (b)
2 8
(d) the pushing force F of the child is balanced
3 5 by the total internal force in the toy generated
(c) (d)
4 6 due to deformation
18. A juggler tosses a ball up in the air with initial 23. Light enters an isosceles right triangular prism
speed u. At the instant it reaches its maximum at normal incidence through face AB and
height H, he tosses up a second ball with the same undergoes total internal reflection at face BC as
initial speed. The two balls will collide at a height shown below.
H H
(a) (b)
4 2
3H 3
(c) (d) H
4 4
19. On a horizontal frictionless frozen lake, a girl
(36 kg) and a box (9 kg) are connected to each
other by means of a rope. Initially they are 20 m
apart. The girl exerts a horizontal force on the
box, pulling it towards her. How far has the girl
The minimum value of the refractive index of the
traveled when she meets the box-
prism is close to
(a) 10 m
(a) 1.10 (b) 1.55
(b) Since there is no friction, the girl will not move
(c) 1.42 (d) 1.72
(c) 16 m
24. A convex lens is used to form an image of an object
(d) 4m on a screen. If the upper half of the lens is
20. The following three objects (1) a metal tray, (2) a blackened so that it becomes opaque, then
block of wood, and (3) a woolen cap are left in a (a) only half of the image will be visible
closed room overnight. Next day the temperature
(b) the image position shifts towards the lens
of each is recorded as T1, T2 and T3 respectively.
The likely situation is (c) the image position shifts away from the lens
(a) T1 = T2 = T3 (b) T3 > T2 > T1 (d) the brightness of the image reduces
(c) T3 = T2 > T1 (d) T3 > T2 = T1 25. A cylindrical copper rod has length L and
resistance R. If it is melted and formed into
21. We sit in the room with windows open. Then
another rod of length 2L, the resistance will be
(a) air pressure on the floor of the room equals
(a) R (b) 2R
the atmospheric pressure but the air pressure
on the ceiling is negligible (c) 4R (d) 8R
(b) air pressure is nearly the same on the floor, 26. Two charges + Q and –2Q are located at points A
the walls and the ceiling and B on a horizontal line as shown below:
(c) air pressure on the floor equals the weight of
the air column inside the room (from floor to
ceiling) per unit area
(d) air pressure on the walls is zero since the The electric field is zero at a point which is located
weight of air acts downward at a finite distance
22. A girl standing at point P on a beach wishes to (a) on the perpendicular bisector of AB
reach a point Q in the sea as quickly as possible. (b) left of A on the line
She can run at 6 km h–1 on the beach and swim (c) between A and B on the line
at 4 km h–1 in the sea. She should take the path (d) right of B on the line
27. A 750 W motor drivers a pump which lifts 300
litres of water per minute to a height of 6 meters.
The efficiency of the motor is nearly (take
acceleration due to gravity to be 10 m/s2)
(a) 30%
(b) 40%
(c) 50%
(a) PAQ (b) PBQ (d) 20%
(c) PCQ (d) PDQ
28. Figure below shows a portion of an electric circuit 33. The set of principal (n), azimuthal ( ) and magnetic
with the currents in amperes and their directions. (m1) quantum numbers that is not allowed for
The magnitude and direction of the current in the electron in H-atom is
the portion PQ is (a) n = 3, = 1, m1 = –1
(b) n = 3, = 0, m1 = 0
(c) n = 2, = 1, m1 = 0
(d) n = 2, = 2, m1 = –1
34. At 298 K, assuming ideal behaviour, the average
kinetic energy of a deuterium molecule is
(a) two times that of a hydrogen molecule
(b) four times that of a hydrogen molecule
(c) half of that of a hydrogen molecule
(d) same as that of a hydrogen molecule
35. As isolated box, equally partitioned contains two
(a) 0 A (b) 3 A from P to Q ideal gases A and B as shown
(c) 4 A from Q to P (d) 6A from Q to P
214
29. A nucleus of lead Pb82 emits two electrons
followed by an alpha particle. The resulting
nucleus will have
When the partition is removed, the gases mix.
(a) 82 protons and 128 neutrons The changes in enthalpy ( H) and entropy ( S)
(b) 80 protons and 130 neutrons in the process, respectively, are
(c) 82 protons and 130 neutrons (a) zero, positive (b) zero, negative
(d) 78 protons and 134 neutrons (c) positive, zero (d) negative, zero
30. The number of air molecules in a (5 m × 5 m × 4 m) 36. The gas produced from thermal decomposition of
room at standard temperature and pressure is of (NH4)2Cr2O7 is
the order of (a) oxygen (b) nitric oxide
(a) 6 × 1023 (b) 3 × 1024 (c) ammonia (d) nitrogen
(c) 3 × 1027 (d) 6 × 1030 37. The solubility curve of KNO3 in water is shown
below.

31. Two balloons A and B containing 0.2 mole and


0.1 mole of helium at room temperature and 2.0
atm, respectively, are connected. When
equilibrium is established, the final pressure of
He in the system is
(a) 1.0 atm (b) 1.5 atm
(c) 0.5 atm (d) 2.0 atm
32. In the following set of aromatic compounds

the correct order of reactivity toward Friedel-


The amount of KNO3 that dissolves in 50 g of
Crafts alkylation is
water at 40°C is closest to
(a) i > ii > iii > iv (b) ii > iv > iii > i
(a) 100 g (b) 150 g
(c) iv > ii > iii > i (d) iii > i > iv > ii
(c) 200 g (d) 50 g
38. A compound that shows positive iodoform test is 45. The major product formed in the reaction
(a) 2-pentanone (b) 3-pentanone
(c) 3- pentanol (d) 1-pentanol is
39. After 2 hours the amount of a certain radioactive
substance reduces to 1/16th of the original amount
(the decay process follows first-order kinetics). The
half-life of the radioactive substance is
(a) 15 min (b) 30 min
(c) 45 min (d) 60 min
40. In the conversion of zinc ore to zinc metal, the
process of roasting involves (i) (ii)
(a) ZnCO3 ZnO (b) ZnO ZnSO4
(c) ZnS ZnO (d) ZnS ZnSO4
41. The number of P-H bond(s) in H3PO2, H3PO3 and
H3PO4, respectively, is
(a) 2, 0, 1 (b) 1, 1, 1
(c) 2, 0, 0 (d) 2, 1, 0 (iii) (iv)
42. When chlorine gas is passed through an aqueous (a) i (b) ii
solution of KBr, the solution turns orange brown (c) iii (d) iv
due to the formation of
(a) KCl (b) HCl
(c) HBr (d) Br2 46. If parents have free ear lobes and the offspring
43. Among has attached ear lobes, then the parents must be
(a) homozygous (b) heterozygous
(c) co-dominant (d) nullizygous
47. During meiosis there is
(a) one round of DNA replication and one division
(b) two rounds of DNA replication and one division
(c) two rounds of DNA replication and two division
the compound which is not aromatic is
(d) one round of DNA replication and two division
(a) i (b) ii 48. Blood clotting involves the conversion of
(c) iii (d) iv (a) prothrombin to thromboplastin
44. Among the following compounds (b) thromboplastin to prothrombin
(c) fibrinogen to fibrin
(d) fibrin to fibrinogen
49. The gall bladder is involved in
(a) synthesizing bile
(b) storing and secreting bile
(c) degrading bile
(d) producing insulin
50. Which one of the following colours is the LEAST
useful for plant life?
(a) red (b) blue
(c) green (d) violet
2, 3- dimethylhexane is 51. At rest the volume of air that moves in and out
per breath is called
(a) i (b) ii
(a) resting volume (b) vital capacity
(c) iii (d) iv
(c) lung capacity (d) tidal volume
52. How many sex chromosomes does a normal 56. The presence of nutrients in the food can be
human inherit from father? tested. Benedict’s test is used to detect
(a) 1 (b) 2 (a) sucrose (b) glucose
(c) 23 (d) 46 (c) fatty acid (d) vitamin
th
53. In the 16 century, sailors who travelled long 57. Several minerals such as iron, iodine, calcium
distances had diseases related to malnutrition, and phosphorous are important nutrients. Iodine
because they were not able to eat fresh vegetables is found in
and fruits for months at a time. Scurvy is a result (a) thyroxine (b) adrenaline
of deficiency of (c) insulin (d) testosterone
(a) carbohydrates (b) proteins 58. The principle upon which a lactometer works is
(c) Vitamin C (d) Vitamin D (a) viscosity (b) density
54. The following structure is NOT found in plant (c) surface tension (d) presence of protein
cells 59. Mammalian liver cells will swell when kept in
(a) vacuole (b) nucleus (a) hypertonic solution (b) hypotonic solution
(c) centriole (d) endoplasmic reticulum (c) isotonic solution (d) isothermal solution
55. The cell that transfers information about pain to 60. The form of cancer called ‘carcinoma’ is associated
the brain is called a with
(a) neuron (b) blastocyst (a) lymph cells (b) mesodermal cells
(c) histoblast (d) haemocyte (c) blood cells (d) epithelial cells

(a) 65
(b) 55
61. Let f(x) = ax2 + bx + c, where a, b, c are integers.
(c) 45
Suppose f(1) = 0, 40 < f(6) < 50, 60 < f(7) < 70, and
1000t < f(50) < 1000 (t + 1) for some integer t. (d) 35
Then the value of t is 64. Two friends A and B are 30 km apart and they
(a) 2 (b) 3 start simultaneously on motorcycles to meet each
other. The speed of A is 3 times that of B. The
(c) 4 (d) 5 or more distance between them decreases at the rate of 2
62. The expression km per minute. Ten minutes after they start, A’s
22 1 32 1 42 1 (2011)2 1 vehicle breaks down and A stops and waits for B
to arrive. After how much time (in minutes) A
22 1 32 1 4 2 1 (2011)2 1
started riding, does B meet A?
lies in the interval
(a) 15
1
(a) 2010, 2010 (b) 20
2
(c) 25
1 1 (d) 30
(b) 2011 , 2011
2011 2012 65. Three taps A, B, C fill up a tank independently in
1 10 hr, 20 hr, 30 hr, respectively. Initially the tank
(c) 2011, 2011
2 is empty and exactly one pair of taps is open
during each hour and every pair of taps is open
1
(d) 2012, 2012 at least for one hour. What is the minimum
2 number of hours required to fill the tank?
63. The diameter of one of the bases of a truncated (a) 8
cone is 100 mm. If the diameter of this base is
(b) 9
increased by 21% such that it still remains a
truncated cone with the height and the other base (c) 10
unchanged, the volume also increases by 21%. (d) 11
The radius of the other base (in mm) is-
69. A student sees the top edge and the bottom center
C of a pool simultaneously from an angle above
66. An object with uniform density is attached to a
the horizontal as shown in the figure. The
spring that is known to stretch linearly with
refraction index of water which fills up to the top
applied force as shown below.
4 h 7
edge of the pool is . If then cos is
3 x 4
2
(a)
7
8
(b)
3 45
8
When the spring-object system is immersed in a (c)
liquid of density 1 as shown in the figure, the 3 53
spring stretches by an amount x1( > 1). When 8
(d)
the experiment is repeated in a liquid of density 21
2 < 1, the spring stretches by an amount x2. 70. In the following circuit, the 1 resistor dissipates
Neglecting any buoyant force on the spring, the power P. If the resistor is replaced by 9 , the
density of the object is power dissipated in it is
1 x1 2 x2 1 x2 2 x1
(a) P
(a) (b) (b) 3P
x1 x2 x2 x1
(c) 9P
1 x2 2 x1 1 x1 2 x2 P
(c) (d) (d)
x1 x2 x1 x2
3
67. A body of 0.5 kg moves along the positive x-axis
under the influence of a varying force F
(in Newtons) as shown below. 71. An aqueous buffer is prepared by adding 100 ml
of 0.1 mol l–1 acetic acid to 50 ml of 0.2 mol l–1 of
sodium acetate. If pKa of acetic acid is 4.76, the
pH of the buffer is
(a) 4.26 (b) 5.76
(c) 3.76 (d) 4.76
72. The maximum number of structural isomers
possible for the hydrocarbon having the molecular
formula C4H6, is
(a) 12 (b) 3
(c) 9 (d) 5
If the speed of the object at x = 4 m in 3.16 ms–1 73. In the following reaction sequence, X and Y,
then its speed at x = 8 m is respectively are
(a) 3.16 ms–1 (b) 9.3 ms–1
(c) 8 ms–1 (d) 6.8 ms–1
68. In a thermally isolated system, two boxes filled
with an ideal gas are connected by a valve. When (a) H2O2; LiAlH4
the valve is in closed position, states of the box 1 (b) C6H5COOOH; LiAlH4
and 2, respectively, are (1 atm, V, T) and (0.5 atm,
(c) C6H5COOOH; Zn/Hg.HCl
4V, T). When the valve is opened, the final
pressure of the system is approximately (d) Alkaline KMnO4; LiAlH4
(a) 0.5 atm 74. Among (i) [Co(NH 3) 6]Cl 3, (ii) [Ni(NH 3) 6]Cl 2 ,
(iii) [Cr(H2O)6]Cl3, (iv) [Fe(H2O)6]Cl2 the complex
(b) 0.6 atm
which is diamagnetic is
(c) 0.75 atm
(a) i (b) ii
(d) 1.0 atm
(c) iii (d) iv
75. At 783 K in the reaction H2(g) + I2(g) 2HI (g), 77. Vestigial organs such as the appendix exist
the molar concentrations (mol–1) of H2, I2 and HI because
at some instant of time are 0.1, 0.2 and 0.4, (a) they had an important function during
respectively. If the equilibrium constant is 46 at development which is not needed in the adult
the same temperature, then as the reaction (b) they have a redundant role to play if an organ
proceeds with similar function fails
(a) the amount of HI will increase (c) nature cannot get rid of structures that have
(b) the amount of HI will decrease already formed
(c) the amount of H2 and I2 will increase (d) they were inherited from an evolutionary
ancestor in which they were functional
(d) the amount of H2 and I2 will not change
78. Mendel showed that unit factors, now called
alleles, exhibit a dominant/recessive relationship.
In a monohybrid cross, the ............ trait
76. You remove four fresh tobacco leaves of similar disappears in the first filial generation
size and age. Leave “leaf 1” as it is, smear “leaf 2”
(a) dominant (b) co-dominant
with vaseline on the upper surface, “leaf 3” on
the lower surface and “leaf 4” on both the surfaces. (c) recessive (d) semi-dominant
Hang the leaves for a few hours and you observe 79. If a man with an X-linked dominant disease has
that leaf 1 wilts the most, leaf 2 has wilted, leaf 3 six sons with a woman having a normal
wilted less than leaf 2 and leaf 4 remains fresh. complement of genes, then the sons will
Which of the following conclusion is most logical ? (a) not show any symptoms of the disease
(a) tobacco leaf has more stomata on the upper (b) show strong symptoms of the disease
surface (c) three will show a disease symptom, while
(b) tobacco leaf has more stomata on the lower three will not
surface (d) five will show a disease symptom, while one
(c) stomata are equally distributed in upper and will not
lower surfaces 80. In evolutionary terms, an Indian school boys is
(d) no conclusion on stomatal distribution can be more closely related to
drawn from this experiment (a) an Indian frog (b) an American snake
(c) a Chinese horse (d) an African shark

1. (a) 2. (c) 3. (a) 4. (b) 5. (b) 6. (d) 7. (b) 8. (a) 9. (a) 10. (c)

11. (b) 12. (c) 13. (b) 14. (a) 15. (c) 16. (a) 17. (a) 18. (c) 19. (d) 20. (a)

21. (b) 22. (c) 23. (c) 24. (d) 25. (c) 26. (b) 27. (b) 28. (d) 29. (a) 30. (c)

31. (d) 32. (c) 33. (d) 34. (d) 35. (a) 36. (d) 37. (a) 38. (a) 39. (b) 40. (c)

41. (d) 42. (d) 43. (*) 44. (b) 45. (c) 46. (b) 47. (d) 48. (c) 49. (b) 50. (c)

51. (d) 52. (a) 53. (c) 54. (c) 55. (a) 56. (b) 57. (a) 58. (b) 59. (b) 60. (d)

61. (c) 62. (c) 63. (b) 64. (d) 65. (a) 66. (b) 67. (d) 68. (b) 69. (c) 70. (a)

71. (d) 72. (c) 73. (b) 74. (a) 75. (a) 76. (b) 77. (d) 78. (c) 79. (a) 80. (c)
5 . (b) Given 0 < r < 4
In the given options
x b x c x c x a
1 . (a) P ( x) 9
a b a c b c b a (Base)x =
5
x a x b
+ for largest value of x, base should be
c a c b minimum.
Let P(a) = 1 + 0 + 0 = 1
r
P(b) = 0 + 1 + 0 = 1 In option B base 1 is minimum for
17
and P(c) = 0 + 0 + 1 = 1 0 < r < 4.
P(x) = 1 for all x R. 6 . (d)
2 . (c) a2 + b2 = 81
x2 + y2 = 121
ax + by = 99
Now, (a2 + b2) (x2 + y2) = (81) (121) … (i)
and (ax + by)2 = (99)2 … (ii)
Subtracting equation (ii) from (i) we get
Now applying angle bisector theorem
(ay – bx)2 = 0
ay – bx = 0 r p
6 q
1 1
3 . (a) x + = a and x2 + 3 = b qr 6p … (i)
x x
2 1
1 1
x a2 x2 2 a2 … (i) Now Area of ADC = (DC) (AB)
x x2 2
3
1 1
and x a3 10 = (q) (r)
x 2
Now cubing both side. q r = 20
1 1 1 Now from equation (i)
x3 x a3
3x … (ii)
x 3 x 20 = 6 p
x
Now adding equations (i) and (ii) we get 20 10
p=
1 1 1 6 3
x2 x3 2 3 x a2 a3
x 3
x 2 x 7 . (b)

1
b x3 2 3a a2 a3
x2
1
x3 a3 a2 3a 2 b
2
x
4 . (b) a b 2 a b 2 Slant height = 13
S
and b c 3 b c 3 =
r
and c d 4 c d 4 S=r
Possible values of a – d are ±9, ±5, ±3, ±1 2 (5) = 13
a d 9, 5, 3, 1 10
=
Sum = 18. 13
8 . (a) 13. (b)

3 steps.
14. (a) The clock will show the incorrect time
From figure, (between 1-2, 10-11, 11-12 , 12-1 day and night
12 20 x both)
x 8 In correct time 8 × 60 = 480 (each minute
96 = 20x – x2 it will display 1)
x2 = 20x + 96 = 0 Remaining 20 hours it will show the incorrect
time 16 × 15 = 240
x = 8, 12
9 . (a) Total incorrect time = 240 + 480 = 720
Correct time = 1 – incorrect time
720 1
=1–
24 60 2
15. (c)
Right Wrong Unattempted
(4 marks) (0 marks) (1 marks)
15 15 0
14 4
AD 40 13 8
sin 60
AB 2 12 12
2 3 11 16
AD = 2 sin 60º = 3
2 10 20
d = 1 + AD + 1 Total 6 possible values of x
d=2+ 3 16. (a)
11. (b) Top layer has (13 × 13) balls similarly layer
below top layer will have (14 × 14) balls
We have 18 layer
So total number of balls According to law of conservation of
N = (13)2 + (14)2 + ........ + (30)2 mechanical energy
30 31 61 12 13 25 Ki + Ui = K f + U f
N=
6 6 0 + Ui = 0 + Uf
N = 8805. hi = hf
12. (c) Let distance 6x Point D is on line AB
mud : tar : stream 17. (a) Fext. 0
distance x : 3x : 2x
asystem 0
speed 3v : 5v : 4v
x 3x 2x
time : :
3v 5v 4v
10 : 18 : 15
18. (c)
24. (d)

Only half part of the lens will be used so its


intensity will be decreased
25. (c) We get applying volume conservation
principle,
A × L = A’ × 2L
1 2 A
H h gt A’ =
2 2
1 2 L
h ut gt R=
2 A
H = ut … (1) 2L 4L
2
R'
u A' A
H … (2)
2g R' 4R
u
t= [From (1) and (2)] 26. (b)
2g

u 1 u2 27. (b) mgh = 300 × 10 × 6


h u g
2g 2 4 g2 mgh 300 6 6
Pi = = 300 W
3u2 3H t 60
8g 4 P0 = 750 W
19. (d) 300
= × 750 = 40%
750

28. (d)
external force does not work on system
So according to concept of mass
36x = 9 × (20 – x)
x=4m
20. (a) All the three object will be in thermal
equilibrium then T1 = T2 = T3. 214 210
29. (a) Pb 82 82 X
21. (b) Pressure of gas is same everywhere in the 2e 2He
4

vessel
82 Proton
22. (c) 210 – 82 = 128 Neutron
30. (c) PV = N × K × T
where K is Boltzmann constant
105 × 100 = N × 1.38 × 10–23 × 273
In beach velocity is higher
N 3 × 1027
beach is rarer and sea is denser medium
So when it go from rarer to denser medium 31. (d) Pressure will not change.
it bend toward normal to reach in minimum 32. (c) +R groups increases reactivity
time.
SO – OCH3 group will be most and –NO2 group
23. (c) i = 45º C
will be least.
For minimum refractive index C = 45º
µsin 45º = 1 33. (d) For Hydrogen atom, n cannot be 2

µ = 2 = 1.42 34. (d) K. E. Depends on Temperature only.


35. (a) For ideal gas, H = 0 and entropy will increase 48. (c) Fibrin Cloting factors Fibrinogen
on mixing of gases.
49. (b) Gall bladder stores and concentrate bile juice,
36. (d) Heating of (NH4)2 Cr2O7 will be as:- and it secretes bile juice when needed.
50. (c) Because green part of spectrum is reflected
NH 4 CH 2 O7 N2 Cr2 O3 4H 2 O by green plants.
2

51. (d) Tidal volume, its about 500 ml


37. (a) As per graph, 100 ml. of water has = 200 g
KNO3 52. (a) Only one sex chromosome is inherited from
father either X or Y.
As per graph, 50 ml. of water will have = 100 g KNO3
53. (c) Vit C is present in fresh vegetables (green)
38. (a) Iodoform test is shown by
54. (c) Centriole is present in animal cells only. It
helps in cell division.
O
|| 55. (a) Neuron function as receptors in this case.
CH 3 C Containing groups and
56. (b) Glucose is a reducing sugar, and reducing
sugar give the test with benedict solution.
H 57. (a) Thyroxine contain iodine.
|
CH 3 CH Containing groups. 58. (b) Lactometer measure density of milk.
59. (b) When a cell is placed in hypotonic solution
2.303 a endosmosis takes place.
39. (b) K = log
t a x 60. (d) Cancer epithelial cells is called carcinoma.

0.693 2.303 a
log
t1 2 60 a 61. (c) f(x) = ax2 + bx + c
2 16 f(1) = 0 (given)
a+b+c=0
So t 1 30 min. and 40 < f(6) < 50
2
40 < 36a + 6b + c < 50
40. (c) ZnS get converted to ZnO on heating in 40 < 35a + 5b < 50
presence of oxygen. 8 < 7a + b < 10
7a + b = integer = 9 … (i)
O O O and 60 < f(7) < 70
P P P 60 < 49a + 7b + c < 70
41. (d) OH , H, OH
H HO HO 60 < 48a + 6b < 70
H OH OH
10 < 8a + b < 11.6
42. (d) Following reaction will take place:- 8a + b = integer = 11 … (ii)
Now solving equations (i) and (ii) we get
KBr + Cl2 KCl + Br2 (Brown Colour)
a = 2, b = –5, c = 3
43. (i) is Aromatic and Heterocyclic f(x) = 2x2 – 5x + 3
f(50) = 4753
(ii) is Non-Aromatic
1000 t < f(50) < 1000(t + 1)
(iii) is Aromatic (1000 × 4) < 4753 < 1000(4 + 1)
(iv) is Aromatic and Heterocyclic t=4

45. (c) SN2 reaction takes place 22 1 32 1 42 1 (2011)2 1


62. (c)
22 1 32 1 42 1 (2011)2 1
47. (d) DNA replication S-phase of cell cycle
2011 2 2011
r 1 2
It occurs once and division will occur twice. 1
2 (r 1)(r 1)
r 2 r 1 r 2
2011 1 1 9
1 1
1 =
r 2 r 1 r 1 10 20 60
th
1 1 1 1 1 38
2010 1 So the rest part of tank will take 5
3 2 4 3 5 60
hours more.
1 1
So the tank will be filled in 8th hour.
2010 2012
66. (b)
1 1 1
2011
2 2011 2012
1
Expression lies between 2011, 2011
2
63. (b) Let initially bases have radii 5 cm and
r cm.
Finally base have radii (1.21 × 5) and r
F.B.D.
V2
Now Ratios of volumes = = 1.21
V1

h
Now, V2 = (6.05)2 (6.05)r r2
3
h 2 kx1 +
and V1 = 5 5r r2 1Vg = Vg … (1)
3 kx2 + 2Vg = Vg … (2)
V2 from (1) and (2)
1.21
V1
2 x1 1 x2 1 x2 2 x1
=
(6.05)2 (6.05)r r2 x1 x2 x2 x1
2 2
1.21 67. (d) According to work-energy principle
5 5r r
WC + Wnc + Wext = KE
2 6.3525
r 8
1 1 1
Fdx mv2f mvi2
11 4
2 2
r cm = 55 mm.
2 1 1 1 1 2
3 8 1.5 4 vf (3.16)2
65. (a) A 10 hr 2 2 2 2
B 20 hr vf 6.8 m/s
C 30 hr
68. (b)
Exactly one pair of taps is open during each
hour and every pair of taps is open at least
for one hour.
First A and B are open for 1 hour then B and
C and then C and A
1 1 1 1 1 1 22
After opening of at equilibrium temperature
10 20 20 30 30 10 60 and pressure of whole gas is T1 and P1
first hr. second hr. third hr.
th l v 0.5 V 4
22 n1 , n2
In three hours the tank will be filled RT RT
60
part. n1 + n2 = n
Now, minimum time required for rest of the V V 4 5VP1
tank to fille with A and B taps is RT 2 RT RT1
3V 5VP1 5
i
RT RT1 2
P1 0.6 2
5 25
Pi i2 R 1
T1 T 2 4
Q=0, W=0 2
10 100
U=0 Pf 9 9
12 12 12
n1CVT + n2CVT = (n1 + n2)CVT1
Pf Pi
T1 = T
P1 0.6
Salt
T T 71. (d) P H P Ka log
Acid
P1 0.6 atm
69. (c) 10
PH 4.76 log
10

So PH = 4.76
72. (c)

CH
||
LiAlH4
73. (b) CH3 C CH3 Peroxy
Benzoic Acid
–OH

1 sin i sin r
74. (a) As NH3 is strong field ligand so pairs-up
4 unpaired electrons and hence diamagnetic
sin 90 sin r
3
0.4 0.4
x 4 2 75. (a) QC 8
tan r 0.1 0.2
2h 7 2 7
2 Now, QC < KC, so reaction moves forward to
sin r form HI.
53
4 2 8 76. (b)
cos
3 53 3 53 77. (d) Vestigial organ can explain ancestory.
70. (a)
78. (c) During F1 generation only dominant allele
express itself.
79. (a) Sons will receive only Y-chromosome from
father hence they will be normal.
80. (c) Human and Horse both are mammals.

10 = 4i
(a) 72 (b) 36
1. A student notices that the roots of the equation rs rs
(c) (d)
x2 + bx + a = 0 are each 1 less than the roots of 2 7
the equation x2 + ax + b = 0. Then a + b is. 7. A cow is tied to a corner (vertex) of a regular
(a) Possibly any real number hexagonal fenced area of side a metres by a rope
(b) – 2 5a
of length metres in a grass field. (The cow
(c) – 4 2
cannot graze inside the fenced area.) What is the
(d) – 5
maximum possible area of the grass field to which
x x the cow has access to graze?
1 1
y y
2. If x, y are real numbers such that 3 3
5 2
( x y) (a) 5 a2 (b) a
= 24, then the value of is 2
( x y)
(c) 6 a2 (d) 3 a2
(a) 0 (b) 1 8. A closed conical vessel is filled with water fully
(c) 2 (d) 3 and is placed with its vertex down. The water is
3. The number of positive integers n in the set flow out at a constant speed. After 21 minutes, it
{1, 2, 3,…….100} for which the number was found that the height of the water column is
half of the original height. How much more time
12 22 32 n2 in minutes does it require to empty the vessel?
is an integer is
1 2 3 n (a) 21 (b) 14
(a) 33 (b) 34 (c) 7 (d) 3
(c) 50 (d) 100 9. I carried 1000 kg of watermelon in summer by
4. The three different face diagonals of a cuboid train. In the beginning, the water content was
(rectangular parallelopiped) have lengths 39, 40, 99%. By the time I reached the destination, the
41. The length of the main diagonal of the cuboid water content had dropped to 98%. The reduction
which joins a pair of opposite corners is - in the weight of the watermelon was-

(a) 49 (b) 49 2 (a) 10 kg (b) 50 kg


(c) 100 kg (d) 500 kg
(c) 60 (d) 60 2
10. A rectangle is divided into 16 sub-rectangles as in
5. The sides of a triangle ABC are positive integers.
the figure, the number in each sub rectangle
The smallest side has length l. Which of the
represents the are of that sub-rectangle. What is
following statement is true ?
the area of the rectangle KLMN ?
(a) The area of ABC is always a rational number
(b) The area of ABC is always an irrational
number
(c) The perimeter of ABC is an even integer
(d) The information provided is not sufficient
to conclude any of the statements A, B or C
above
6. Consider a square ABCD of side 12 and let M, N
be the midpoints of AB, CD respectively. Take a (a) 20 (b) 30
point P on MN and let AP = r, PC = s. Then the (c) 40 (d) 50
area of the triangle whose sides are r, s, 12 is-
13. A soldier with a machine gun, falling from an
airplane gets detached from his parachute. He is
11. A hollow pendulum bob filled with water has a able to resist the downward acceleration if the
small hole at the bottom through which water shoots 40 bullets a second at the speed of 500 m/
escapes at a constant rate. Which of the following s. If the weight of a bullet is 49 gm, what is the
statements describes the variation of the time weight of the man with the gun ?
period (T) of the pendulum as the water flows
Ignore resistance due to air and assume the
out -
acceleration due to gravity g = 9.8 ms–2 -
(a) T decreases first and then increases
(a) 50 kg (b) 75 kg
(b) T increases first and then decreases
(c) 100 kg (d) 125 kg
(c) T increases throughout
14. A planet of mass m is moving around a star of
(d) T does not change mass M and radius R in a circular orbit of radius
12. A block of mass M rests on a rough horizontal r. The star abruptly shrinks to half its radius
table. A steadily increasing horizontal force is without any loss of mass. What change will be
applied such that the block starts to slide on the there in the orbit of the planet?
table without toppling. The force is contained even (a) The planet will escape from the star
after sliding has started. Assume the coefficients
(b) The radius of the orbit will increase
of static and kinetic friction between the table
and block to be equal. The correct representation (c) The radius of the orbit will decrease
of the variation of the frictional force , f exerted (d) The radius of the orbit will not change
by the table on the block with time t is given by - 15. Figure (a) below shows a wheat stone bridge in
which P, Q, R, S are fixed resistances, G is a
galvanometer and B is a battery. For this
particular case the galvanometer shows zero
deflection. Now, only the positions of B and G
(a) are interchanged, as shown in figure (b). The new
deflection of the galvanometer -

(b)

(a)

(c)

(b)
(a) is to the left
(d) (b) is to the right
(c) is zero
(d) depends on the values of P, Q, R, S
16. 12 positive charges of magnitude q are placed on 20. A new temperature scale uses X as a unit of
a circle of radius R in a manner that they are temperature, where the numerical value of the
equally spaced. A charge Q is placed at the entre. temperature tx in this scale is related to the
If one of the charges q is removed, then the force absolute temperature T by tx = 3T + 300. If the
on Q is - specific heat of a material using this unit is 1400
(a) zero J kg–1X–1 its specific heat in the S.I. system of
units is-
qQ
(b) 2
away from the position of the (a) 4200 J kg–1 K–1
4 0R
removed charge (b) 1400 J kg–1 K–1
11qQ (c) 466.7 J kg–1 K–1
(c) away from the position of the (d) impossible to determine from the information
4 0 R2
removed charge provided

qQ
(d) towards the position of the removed
4 0 R2
21. The boiling points of 0.01 M aqueous solution of
charge
sucrose, NaCl and CaCl2 would be -
17. An electric heater consists of a nichrome coil and
(a) the same
runs under 220 V, consuming 1 kW power. Part
of its coil burned out and it was reconnected after (b) highest for sucrose solution
cutting off the burn portion. The power it will (c) highest for NaCl solution
consume now is - (d) highest for CaCl2 solution
(a) more than 1 kW 22. The correct electronic configuration for the
(b) less than 1 kW, but not zero ground state of silicon (atomic number 14) is -
(c) 1 kW (a) 1s2 2s2 2p6 3s2 3p2
(d) 0 kW (b) 1s2 2s2 2p4 3s2 3p4
18. White light is split into a spectrum by a prism (c) 1s2 2s2 2p6 3s2 3p4
and it is seen on a screen. If we put another (d) 1s2 2s2 2p6 3s1 3p2
indentical inverted prism behind it in contact, 23. The molar mass of CaCO3 is 100 g. The maximum
what will be seen on the screen ? amount of carbon dioxide that can be liberated
(a) Violet will appear where red was on heating 25 g of CaCO3 is -
(b) The spectrum will remain the same (a) 11 g (b) 55 g
(c) There will be no spectrum, but only the (c) 22 g (d) 2.2 g
original light with no deviation 24. The atomic radii of the elements across the second
period of the periodic table -
(d) There will be no spectrum, but the original
light will be larcrally displaced (a) decrease due to increase in atomic number
19. Two identical blocks of metal are at 20ºC and 80ºC (b) decrease due to increase in effective nuclear
respectively. The specific heat of the material of charge
the two blocks increases with temperature. Which (c) decrease due to increase in atomic weights
of the following is true about the final (d) increase due to increase in effective nuclear
temperature Tf when the two blocks are brought charge
into contact (assuming that no heat is lost to the 25. Among NH3, BCl3, Cl2 and N2 the compound that
surroundings) - does not satisfy the octet rule is -
(a) Tf will be 50ºC (a) NH3 (b) BCl3
(b) Tf will be more than 50ºC (c) Cl2 (d) N 2
(c) Tf will be less than 50ºC 26. The gas produced on heating MnO2 with conc.
(d) Tf can be either more than or less than 50ºC HCl is -
depending on the precise variation of the (a) Cl2 (b) H 2
specific heat with temperature (c) O2 (d) O3
27. The number of covalent bonds in C4H7Br2 is - 34. In frogs, body proportions do not change with their
(a) 12 (b) 10 growth. A frog that is twice as long as another
will be heavier by approximately
(c) 13 (d) 11
(a) Two-fold (b) Six-fold
28. An aqueous solution of HCl has a pH of 2.0 When
water is added to increase the pH to 5.0 the (c) Four-fold (d) Eight-fold
hydrogen ion concentration - 35. Which of the following has the widest angle of
(a) remains the same binocular vision?
(b) decreases three-fold (a) Rat (b) Duck
(c) increases three-fold (c) Eagle (d) Owl
(d) decreases thousand-fold 36. The two alleles of a locus which an offspring
receives from the male and female gametes are
29. Consider two sealed jars of equal volume. One
situated on
contains 2 g of hydrogen at 200 K and the other
contains 28 g of nitrogen at 400 K. The gases in (a) Two different homologs of the same
the two jars will have - chromosome
(a) the same pressure (b) Two different chromosomes
(b) the same average kinetic energy (c) Sex chromosomes
(c) the same number of molecules (d) A single chromosome
(d) the same average molecular speed 37. Ants locate sucrose by
30. Identify the stereoisomer pair from the following (a) Using a strong sense of smell
choices - (b) Using a keen sense of vision
(a) CH3CH2CH2OH and CH3CH2OCH3 (c) Physical contact with sucrose
(b) CH3CH2CH2Cl and CH3CHClCH3 (d) Sensing the particular wave length of light
H emitted reflected by sucrose
| 38. The interior of a cow-dung piece kept for a few
(c) CH3 C C CH3 and CH3 C C CH3
| | | days is quite warm. This is mostly because
H H H (a) Cellulose present in the dung is a good
insulator
(b) Bacterial metabolism inside the dung releases
(d) and heat
(c) Undigested material releases heat due to
oxidation by air
(d) Dung is dark and absorbs a lot of heat
31. Which of the following is a water-borne disease? 39. Which one of these is the correct path for a reflex
(a) Tuberculosis (b) Chickenpox action?
(c) Malaria (d) Cholera (a) Receptor-Motor Neuron-Spinal Cord-Sensory
32. In his seminal work on genetics, Gregor Mendel Neuron-Effector
described the physical traits in the pea plant as (b) Effector-Sensory Neuron-Spinal Cord-Motor
being controlled by two ‘factors’. What term is Neuron-Receptor
used to define these factors today ? (c) Receptor-Sensory Neuron-Spinal Cord-Motor
(a) Chromosomes (b) Alleles Neuron-Effector
(c) Genes (d) Hybrids (d) Sensory Neuron-Receptor-Motor-Neuron-
33. A majority of the tree species of penensular Indian Spinal Cord-Effector
origin fruit in the months of 40. Insectivorous plants digest insects to get an
(a) April – May essential nutrient. Other plants generally get this
nutrient from the soil. What is this nutrient?
(b) December – January
(a) Oxygen (b) Carbon dioxide
(c) August – September
(c) Nitrogen (d) Phosphates
(d) All months of the year
In each case, the resistance is estimated by using
Ohm’s law Rest = V / I, Where V and I are the
1. In a triangle ABC, D and E are points on AB, AC readings of the voltmeter and the ammeter
respectively such that DE is parallel to BC. respectively. The meter resistances, Rv and RA
Suppose BE, CD intersect at O. If the areas of are such that R A << R << R v. The internal
the triangles ADE and ODE are 3 and 1 resistance of the battery may be ignored. The
respectively, find the area of the triangle ABC, absolute error in the estimate of the resistance
with justification is denoted by R = |R = Rest|.
2. Leela and Madan pooled their music CD’s and Express Rp in terms of the given resistance
sold them. They got as many rupees for each CD values
as the total number of CD’s they sold. They share Express RQ in terms of the given resistance
the money as follows Leela first takes 10 rupees, values
then Madan takes 10 rupees and they continue
taking 10 rupees alternately till Madan is left out For what value of R will RP RQ
with less than 10 rupees to take. Find the amount 5. A point source is placed 20 cm to the left of a
that is left out for Madan at the end, with concave lens of focal length 10 cm.
justification. (a) Where is the image formed ?
3. (a) Show that for every natural number n (b) Where to the right of the lens would you place
relatively prime to 10, there is another natural a concave mirror of focal length 5 cm so that
number m all of whose digits are 1’s such that the final image is coincident with the source ?
n divides m. (c) Where would the final image be formed if the
(b) Hence or otherwise show that every positive concave mirror is replaced by a plane mirror
rational number can be expressed in the at the same position ?
a 6. A block of mass m is sliding on a fixed frictionless
form for some natural numbers concave surface of radius R. It is released from
10 b 10 c 1
rest at point P which is at a height of H << R
a, b, c. from the lowest point Q.

4. Consider the two circuits P and Q, shown below,


which are used 20 measure the unknown
resistance R.

(a) What is the potential energy as a function of


, taking the lowest point Q as the reference
level for potential energy ?
(b) What is the kinetic energy as a function of ?
(c) What is the time taken for the particle to
reach from point P to the lowest point Q?
(d) How much force is exerted by the block on
the concave surface at the point Q?
P

7. Copper in an alloy is estimated by dissolving in


conc. nitric acid. In this process copper is
converted to cupric nitrate with the evolution of
nitric oxide (NO). The misture when treated with
potassium iodide forms cupric iodide. Which is
unstable and decomposes to cuprous iodide and
iodine. The amount of copper in the alloy is
Q
estimated by litrating the libereated iodine with (iv) The compound in the bottle D did not dissolved
sodium thiosulphate. The reactions are in 1N NaOH but dissolved in 1 N HCl.
a Cu + b HNO3 c Cu(NO3)2 + d NO + e H2O (Fill up the blanks)
f Cul2 g Cu2I2 + h I2 (a) Indicate the compounds in:
bottle A : _____, bottle B : _____, bottle C =
i Na2S2O3 + j I2 k Na2S4O6 + l NaI
______ and bottle D = _____.
(Fill up the blanks) (b) The compound with the highest solubility in
(a) The coefficients are: distilled water is _____.
a = _______, b = ________ , c = ________, d = 9. Assume that a human body requires 2500 kcal of
________ and e = ________ . energy each day for metabolic activity and sucrose
is the only source of energy, as per the equation
(b) The coefficients are:
C12H22O11(s) + 12 O2(g) 12 CO2(g) + 11 H2O (l);
f = ________, g = ________ and h = ________.
H = – 5.6 × 106 J.
(c) The coefficients are:
(Fill up the blanks)
i = ________, j = ________, k = ________ and l
(a) The energy requirement of the human body
= ________.
per day is _______ kJ.
(d) If 2.54 g of I2 is evolved from a 2.0 g sample of
(b) The mass of sucrose required to provide this
the alloy, what is the percentage of copper in energy is ______ g and the volume of CO2 (at
the alloy? STP) produced is _______ litres.
(atomic, weight of iodine and coper are 127
and 63.5, respectively)
8. You have been given four bottles marked A, B, C 10. Mohini, a resident of Chandigarh went to Shimla
and D each containing one of the organic with her parents. There she found the same
compounds given below plant that they have in their backyard, at home.
However, she observed that while the plants in
their backyard bore white flowers, those in
Shimla had pink flowers. She brought home
some seeds of the plant from Shimla and planted
them in Chandigarh. Upon performing self
breeding for several generations she found that
the plant from Shimla produced only white
flowers.
(a) According to you what might be the reason
The following observation were made. for this observation-genetic or environmental
(i) The compound in the bottle A did not dissolve factors ?
in either 1 N NaOH or 1 N HCl. (b) Suggest a simple experiment to determine
(ii) The compound in the bottle B dissolved in 1 whether this variation is genetic in nature
N NaOH but not in 1 N HCl. (c) Suggest another experiment to check whether
(iii)The compound in the bottle C dissolved in both this variation in flower color is due to
1 N NaOH and 1 N HCl. environmental factors.

11. The break-down of glucose in a cell occurs in any of the following pathways:

Pyruvic
Glucose Acid
Three experiments (A, B, C) have been set up. In (a) Question : Identify which of the reactions is
each experiment, a flask contains the organism the pathways depicted above is taking place
in growth medium, glucose and a brown dye that in each experiment. Give reasons for your
changes its colour to yellow when the pH answer.
decreases. The mouth of the flask is attached to
(b) Question : Identify which of the reactions in
a test tube containing lime water (Calcium
the pathways depicted above is expected to
hydroxide ; as shown in the figure). In C, but not
occur in Red Blood Cells (RBCs)
in A and B, air is removed from the flask before
beginning the experiment. 12. A scientist has a house just beside a busy
After a period of growth, the following observation highway. He collects leaves from some plants
were made: growing in his garden to do radio-carbon dating
(to estimate the age of the plant by estimating
A : Lime water turns milky; the dye colour
remains the same the amount of a radioisotope of carbon in its
tissues). Surprisingly the radio-carbon dating
B : The dye colour changes ; lime water does not
shows that the plant is a few thousand years
turn milky
old.
C : Lime water turns milky ; the dye colour
remains the same (a) Was the result of the radio-carbon dating
wrong or can you propose a reason for such
an observation?
(b) What simple experiment can be done to test
the reason that you have proposed?

1. (c) 2. (d) 3. (d) 4. (a) 5. (b) 6. (b) 7. (a) 8. (d) 9. (d) 10. (d)

11. (b) 12. (a) 13. (c) 14. (d) 15. (c) 16. (d) 17. (a) 18. (c) 19. (d) 20. (a)

21. (d) 22. (a) 23. (a) 24. (b) 25. (b) 26. (a) 27. (a) 28. (d) 29. (c) 30. (c)

31. (d) 32. (c) 33. (a) 34. (a) 35. (c) 36. (a) 37. (c) 38. (b) 39. (c) 40. (b)
5. (b)
1. (c) Here, + = – b and =a
Now, +1+ +1=–a
+ +2=–a
1+b>c c–b<1 –1 < b – c
–b+2=–a ( b) 1+c>b b–c<1
b–a=2 … (1) –1 < b – c < 1
Now, ( + 1) ( + 1) = b b, c are integers so b – c = 0 b=c
+ + +1=b 2b 1 1
Now, Semi perimeter (S) = b
a + (– b) + 1 = b 2 2
( a and b) Now, area A = 2b 1

2b – a = 1 … (2) 1 1 1 b 1
= b b b b c 1
solving equation (1) and (2) we get 2 2 2 2 2
b = – 1, a = – 3
1 2 1
a + b = –4. b = Irrational
2 4
Option (c) is correct.
x x
6. (b)
1 1
2. (d) 3 y y = 24
3
x
x
y
y 3
3 3 24
3
8 x/ y
3 24
3
PA = r, PC = s
x
2 So PB = r
y
Triangle with sides r, s & 12 is PCB
x
1
x y y 2 1
3 S
x
x y 1 2 1
y
Option (d) is correct.
4. (a) According to question,
1 1
Now, Area of PBC = base × height =
a 2
b 2
39 … (i) 2 2
× 12 × 6 = 36
b2 c2 40 … (ii) 2 2
120 5a 60 3a
7. (a) Area 2
and c2 a2 41 … (iii) 360 2 360 2
2
a2 b2 c2 ? 60 a
+
Now solving equations (i), (ii) and (iii) we get 360 2

2 a2 b2 c2 392 402 412 2 25a2 9 a2 a2


2 4 8 8
a2 b2 c2 2401
30 a2
a2 b2 c2 2401 49 5 a2
3 4
(Mm + Mg) × 9.8 = 40 × 500 × 49 × 10–3
(Mm + Mg) = 100 kg

Bullets fired

14. (d) If radius of star is decreasing without any


9. (d) Initially 1000 kg [990 kg water + 10 kg rest] change in mass of star then it will not affect
the force exerted by star on planet which
98 x 2x
Now x kg melon water rest require centripetal force. So radius of the orbit
100 100 of planet with remain categorically
Now weight of solid part should remain same unaffected.
2x 15. (c)
10 x 500
100
Reduction in weight
= 1000 – 500 = 500 kg
10. (d) Area = 10 × 5 = 50
11. (b) Because initially the distance between of
centre of gravity from point of suspension
increases & then finally decreases hence For null deflection
time period will initially increase & then P S P Q
decrease. or
Q R S R
12. (a) f

Kinetic friction

(0, 0) t
Initially when the block does not move the P S
friction is static in nature and it will be equal still valid
Q R
(& opposite) to the magnitude of applied force
deflection is zero.
so initially friction will increase. But once the
body starts the motion the kinetic friction will 16. (d)
come and it does not change with applied force
13. (c) Mm = mass of soldier
Mg = mass of gun
mb = mass of Bullet
To nullify the downward acceleration
Force = Rate of momentum
N
(Mm + Mg) × g = V mb
t If one charge is removed then net force on Q
N q Q
= No. of Bullets fired per second is 2
t 4 0R
Towards the position of removed charge 26. (a) Chlorine gas is produced.
L
R = 27. (a) Total 12 bonds
A
28. (d) PH = 2 SO[H+] = 10–PH = 10–2
if length is decreased, resistance also
decreases. [H+] ions before adding water = 10–2 M
V 2t [H+] ions after adding water = 10–5M
P = So [H+] ions decreases 1000 times.
R
R decreases, P increases. 29. (c) PV = nRt
17. (a) Part of coil turned then resistance decreases 2
Power consumption will be more than Moles of H2 = 1
2
1 kW
28
18. (c) Moles of N2 = 1,
28
So number of molecules will be same.
H
|
30. (c) CH3 C C CH3 and CH3 C C CH3
This system will behave as slab. | | |
H H H
No dispersion
31. (d) Contaminated water spread the cholera.
No deviation
32. (c) Now factors are called genes.
19. (d) If specific heat is constant, then
A
Total heat gain or lost is zero A
Alleles and chromosomes show
ms(Tf – 20) + ms(Tf – 80) = 0
36. (a) Similar pattern
Tf – 20 + Tf – 80 = 0
Tf = 50ºC
Now Tf can be more than 50° or less than 38. (b) Cow dung contain methanogens (methane
50ºC, depending on specific heat variation producing bacteria)
with temperature. 40. (c) Insectivorous plants are deficient in nitrogen.
20. (a) tx = 3T + 300
If in SI system, the temperature has to be
changed by one unit, the temperature has to 1. We denote the area of triangle PQR by [PQR].
We see that [BOD] and [COE] are equal. Let the
be changed by three units.
common value be x, and let [BOC] = t. Using the
so specific heat in SI scale = 3(1400) fact that the ratio of areas of two triangles having
C = 4200 J/(kg – K) equal altitudes is the same as the ratio of their
respective bases, we obtain.
21. (d) Boiling point directly depends on number of ions
x BO t
in aqueous solution. .
1 OE x
22. (a) Si – 1S2 2S2 2P6 3S2 3P2
23. (a) CaCO3 CaO + CO2
Now, 100 g CaCO3 evolves = 44g CO2

44 25
25 g CaCO3 will evolve =
100
= 11 g CO2
24. (b) As left to right along a periodic table, effective
nuclear charge goes on increasing.
25. (b) BCl3 follows sub-octet.
This gives t = x2. Now ADE and ABC are similar where c is the number of digits in m. Hence
so that we can find d such that qd = 10b(10c – 1)
(multiply by a suitable power of 2 if s > r and
ADE DE 2 ODE
, by a suitable power of 5 if r > s). Then
ABC 2 OBC
BC p pd a
since ODE and OCB are also similar. This implies q qd 10 b 10 c 1
that
where a = pd.
3 1
, V V
4 2x t t 4. For P : I = IR + IV =
R Rv
which simplifies to t = 2 + x, using t = x2 we get a
quadratic in x : x2 – x – 2 = 0. Its solution are V RV
R
x = 2 and x = – 1. Since x cannot be negative, x = 2 I V
RV
and t = 4. Thus [ABC] = 4 + 2x + t = 4 + 4 + 4 = 12 I
2. Let t be the total number of CD’s that Leela and R
Rest
Madan together sold. Then they obtain t2 rupees Rest
1
together. R
Since Leela is the first one to take 10 rupees and Rest
also the last one to take 10 rupees, we must have Rest 1
Rv
t2 = 10(an old number) + (a number less than 10).
Rest
Suppose t = 10q + r, where r is the remainder (neglecting higher order terms in )
RV
when t is divided by 10.
2
Then t2 = 100q2 + 20qr + r2. Rest R2
RP Rest R
Comparing, we conclude that RV RV

r2 = 10(an odd number) + (a number less than 10). Alternatively,


But we know that 0 r < 10. Taking r = 0, 1, 2, … V RV R
Rest =
9, we see that r = 4 or 6 (for other value of r, tens I RV R
place in r2 is even). But then r2 = 16 or 36. Hence RV R2
the amount left for Madan at the end is 6 rupees. RP Rest R R 1
RV R RV
3. (a) Divide the n + 1 numbers 1, 11, 111, ...., 111,
For Q : V = I (R + RA)
....1 (all having only 1 as digits) by n. Among
the n + 1 remainders so obtained, two must V
R= – RA = Rest – RA
be equal as the possibilities for remainders I
are 0, 1, 2,...., n – 1 which are n in number.
RQ Rest R RA
Thus there must be two numbers x = 11..... 1
and y = 11.....1 having say j digits and k digits If R = RA RV ,
respectively which leave the same remainders 2 2
after division by n. We may take j < k. Now RP Rest Rest
then 1
we see that y – x is divisible by n. But y – x = RQ RA RV R2
11 .... 100 ... 0 where there are k – j number 5. (a) Object is at 2f, so the image is formed at the
of 1’s and remaining zeros. Since n is coprime same distance from the lens (20 cm) to the
to 10, we see that n divides m = 11 ... 1, a right.
number having only 1’s as its digits.
(b) Since light has to retrace its path, the mirror
p should be placed so that the previous image
(b) If is any rational number (p > 0, q > 0), is at its centre of curvature. Thus the mirror
q
then we may write q = 2r 5s t, where t is must be placed 30 cm to the right of the lens.
coprime to 10. (c) For the plane mirror, reflection forms an
Choose a number m having only 1’s as its image 40 cm to the right of the lens. Using
digits and is divisible by t. Consider 9m, Which the lens formula, we see that the final image
has only 9 as its digits and is still divisible by 40
t. Let k = 9 m/t. We see that; is formed at a distance of cm to the left of
3
qk = 9m 2r5s = (10º –1) 2r5s, the lens.
6. (a) V( ) = mgR (1 – cos ), 9. (a) Energy requirement = 2500 × 4.184 = 10460
(b) mgH – mg R(1 – cos ), kJ
(c) for H << R the body executes SHM with a (b) Now 5600 kJ energy produced by = 342 g sucrose
1 R 342
time period of – the time taken for it 10460 kJ energy = 10460
2 g 5600
to travel from P to Q will be a quarter of this, = 638 g

1 R 638
Moles of sucrose = 1.865
i.e., . 342
8 g
Volume of CO2 produced = 12 × 1.865 × 22.4
(d) At the lowest point, the speed is given by
= 501 L
1 mv2 2mgH
mv2 = mgH. So, T – mg = , 10. (a) Environmental factors will affect flower
2 R R
colours.
2H
and thus T = mg 1 . (b) Cross breeding between the plants from
R
Chandigarh and Shimla can help in
7. (a) Balanced reaction is:- identifying if the variation was genetic or
3Cu + 8HNO3 3Cu(NO3)2 + 2NO + 4H2O not.
(b) 2CuI2 Cu2I2 + I2 (c) Plant from Chandigarh can be tested in
(c) 2Na2S2O3 + I2 Na2 S4O6 + 2NaI another environment say Shimla to check
they still produce white flower or pink
2.54 flower.
(d) Moles of I2 = .01
254 11. (a)
So moles of Cu = .03 Exp A:
m (a) CO 2 is produced during fermentation of
.03 = ethanol so lime water turns milky.
MCu
Acid is not produced No change in dye
or Mass of copper = .03 × 65 g colour.
.03 65 Exp B:
% of copper = 100
2 Lactic acid is produced but not CO2 hence dye
= 97.5% colour change to yellow.
Exp C:
CH2CH3
Lime water turn milky, CO2 is being produced
8. (a) Bottle A contains from ethanol fermentation.
O
Removal of air have no effect on the reaction
hence the fermentation is anaerobic. Hence
CH2COOH yeast should be there in the flask.
Bottle B contains (b) Lactic acid fermentation occur in RBC
12. (a) Result was correct.
Since plants on highway pick C from CO2
COOH emitted by vehicles, which consume fossil
CH2 CH fuels and fossil fuel is produced by fossil
Bottle C contains NH2 organisms (plant and animals). So the carbon
in these plant will be much much older than
the age of the plant.
CH2 NH2 (b) It can grow some seeds in a farm away from
highway/where such CO 2 is not present
Bottle D contains
(produced from burning of fossil fuels) such
plants will show correct age through carbon
Also (IV) will have highest solubility. dating.
(a) 0 and 0.45 (b) 0.45 and 0.9
1. Let BC be a fixed line segment in the plane. The (c) 0.9 and 1.35 (d) 1.35 and 1.8
locus of a point A such that the triangle ABC is 6. The number of real solutions of the equation
isosceles, is (with finitely many possible 2sin 3x + sin 7x – 3 = 0 which lie in the interval
exceptional points) [–2 , 2 ] is
(a) a line (a) 1 (b) 2
(b) a circle (c) 3 (d) 4
(c) the union of a circle and a line 7. Suppose p, q, r are real numbers such that
(d) the union of two circles and a line q = p (4 – p), r = q (4 – q), p = r (4 – r).
2. The number of solution pairs (x, y) of the The maximum possible value of p + q + r is
simultaneous equations (a) 0 (b) 3
log1/3 (x + y) + log3 (x – y) = 2 and 2y = 512
2 x+1
is (c) 9 (d) 27
(a) 0 (b) 1 8. The parabola y = 4x + 1 divides the disc
2

(c) 2 (d) 3 x2 + y2 1 into two regions with areas A1 and A2.


Then | A1 – A2 | equals
3. The value of the xlim 4x 2 x 2x is
1 2
(a) (b)
(a) – 3 3

1
(b) (c) (d)
4 4 3
(c) 0 9. A shooter can hit a given target with probability

1 1
(d) . She keeps firing a bullet at the target until
4 4
she hits it successfully three times and then she
4. Let R be a relation on the set of all natural stops firing. The probability that she fires exactly
numbers given by a R b a divides b2. six bullets lies in the interval
Which of the following properties does R satisfy? (a) (0.5272, 0.5274)
I. Reflexivity (b) (0.2636, 0.2638)
II. Symmetry (c) (0.1317, 0.1319)
III. Transitivity (d) (0.0658, 0.0660)
(a) I only 10. Consider the following events :
(b) III only E1 : Six fair dice are rolled and at least one die
(c) I and III only shows six.
(d) I and II only E2 : Twelve fair dice are rolled and at least two
5. The fractional part of a real number x is x –[x], dice show six.
where [x] is the greatest integer less than or equal Let p1 be the probability of E 1 and p 2 be the
to x. Let F1 and F2 be the fractional parts of (44 probability of E2. Which of the following is true ?
– 2017 )
2017
and (44 + 2017 )2017 (a) p1 > p2 (b) p1 = p2 = 0.6651
respectively. Then F 1 + F 2 lies between the (c) p1 < p2 (d) p1 = p2 = 0.3349
numbers
11. For how many different values of a does the 12 13
following system have at least two distinct (a) (b)
25 25
solutions ?
ax + y = 0 14 15
(c) (d)
x + (a + 10) y = 0 25 25
(a) 0 (b) 1 17. Consider the following parametric equation of a
(c) 2 (d) Infinitely many curve :
12. Let R be the set of real numbers and f : R R be x ( ) = | cos 4 | cos
y( ) = | cos 4 | sin
x
defined by f(x) = 2 , where [x] is the for 0 2
1 x
Which one of the following graphs represents the
greatest integer less than or equal to x, and {x} = curve ?
x – [x]. Which of the following statements are
true ?
I. The range of f is a closed interval
II. f is continuous on R.
III. f is one-one on R. (a) (b)
(a) I only
(b) II only
(c) III only
(d) None of I, II and III
13. Let xn = (2n + 3n)1/2n for all natural numbers n.
Then

(a) lim xn (c) (d)


n

(b) lim
n
xn 3

(c) lim
n
xn 3 2 18. Let A = (a1, a2) and B = (b1, b2) be two points in the
plane with integer coordinates. Which one of the
(d) lim xn 5 following is not a possible value of the distance
n
between A and B ?
14. One of the solutions of the equation
(a) 65 (b) 74
8 sin3 – 7 sin + 3 cos = 0 lies in the interval
(c) 83 (d) 97
(a) (0, 10°] (b) 10°, 20°]
(c) (20°, 30°] (d) (30°, 40°] 1 1
2
15. Let a, b, c, d, e, be real numbers such that a 19. Let f(x) = max 3, x , for x 2. Then the
x2 2
+ b < c + d, b + c < d + e, c + d < e + a, d + e < a
+ b. Then 2

value of the integral f x dx is


(a) The largest is a and the smallest is b 1/ 2

(b) The largest is a and the smallest is c


(c) The largest is c and the smallest is e 11 13
(a) (b)
3 3
(d) The largest is c and the smallest is b
16. If a fair coin is tossed 5 times, the probability 14 16
that heads does not occur two or more times in a (c) (d)
3 3
row is
1 1 24. Using dimensional analysis the resistivity in
20. Let ai = i + for i = 1,2, ......., 20. Put p = (a1 terms of fundamental constants h, me, c, e, 0 can
i 20
+ a2 + ....... + a20) and be expressed as

h m e ce 2
1 1 1 1 (a) (b) 0

q = 20 a ..... 0 m e ce
2

1 a2 a 20 . Then h

h2 me 0
22 p (c) (d)
(a) q 0, m e ce 2 ce 2
21
25. Consider a bowl filled with water on which some
22 p 2 22 p black paper powder have been sprinkled
(b) q 21
,
21 uniformly. Now a drop of liquid soap is added at
the centre of the surface of water. The picture
of the surface immediately after this will look
2 22 p 22 p like
(c) q 21
,
7

22 p 4 22 p
(d) q , (a) (b)
7 21

21. The magnitude of acceleration of the electron in (c) (d)


the nth orbit of hydrogen atom is aH and that of
singly ionized helium atom is aHe. The ratio aH :
26. It was found that the refractive index of material
aHe is
of a certain prism varied as 1.5 + 0.004/ 2, where
(a) 1 : 8 (b) 1 : 4 is the wavelength of light used to measure the
(c) 1 : 2 (d) dependent on n refractive index. The same material was then
22. A carrot looks orange in colour because of the used to construct a thin prism of apex angle 10°.
carotene molecule in it. This means that the Angles of minimum deviation ( m) of the prism
carotene molecule absorbs light of wavelengths were recorded for the sources with wavelengths
and 2 respectively. Then
(a) longer than 550 nm. 1

(a) ( 1) < ( 2) if < .


(b) shorter than 550 nm. m m 1 2

(b) ( 1) > ( 2) if > .


(c) longer than 700 nm m m 1 2

(c) ( 1) > ( 2) if < .


(d) shorter than 700 nm. m m 1 2

(d) is the same in both the cases.


23. If some charge is given to a solid metallic sphere, m

the field inside remains zero and by Gauss’s law 27. Two circularly shaped linear polarisers are placed
all the charge resides on the surface. Suppose coaxially. The transmission axis of the first
now that Colomb’s force between two charges polarizer is at 30° from the vertical while the
varies as 1/r3. Then, for a charged solid metallic second one is at 60°, both in the clockwise sense.
sphere If an unpolarised beam of light of intensity I = 20
W/m2 is incident on this pair of polarisers, then
(a) field inside will be zero and charge density
the intensities I1 and I2 transmitted by the first
inside will be zero.
and the second polarisers, respectively, will be
(b) field inside will not be zero and charge density close to
inside will not be zero.
(a) I1 = 10.0 W/m2 and I2 = 7.5 W/m2
(c) field inside will not be zero and charge density
(b) I1 = 20.0 W/m2 and I2 = 15 W/m2
inside will be zero.
(c) I1 = 10.0 W/m2 and I2 = 8.6 W/m2
(d) field inside will be zero and charge density
inside will not be zero. (d) I1 = 15.0 W/m2 and I2 = 0.0 W/m2
28. An electron in an electron microscope with initial 31. A transverse wave of frequency 500 Hz and speed
velocity 100 m/s is traveling in the positive x direction on a
0 î enters a region of a stray transverse
long string. At time t = 0 s the displacements at x
electric field E0 ˆj . The time taken for the change = 0.0 m and at x = 0.25 m are 0.0 m and 0.02 m,
in its de-Broglie wavelength from the initial value respectively. The displacement at x = 0.2 m at t =
of to /3 is proportional to 5 × 10–4 s is
(a) –0.04 m (b) –0.02 m
1
(a) E0 (b) E (c) 0.04 m (d) 0.02 m
0
32. A thin piece of thermal conductor of constant
thermal conductivity insulated on the lateral
1
sides connects two reservoirs which are
(c) (d) E0
E0 maintained at temperatures T1 and T2 as shown.
Assuming that the system is in steady state,
29. A bird sitting on a single high tension wire does which of the following plots best represents the
not get electrocuted because dependence of the rate of change of entropy of
(a) the circuit is not complete. the ratio of temperatures T1/T2
(b) the bird feet has an insulating covering.
(c) capacitance of the bird is too small and the
line frequency is too small.
(d) resistance of the bird is too high
30. A positive charge q is placed at the center of a
neutral hollow cylindrical conducting shell with
its cross section as shown in the figure below.

(a)

Which one of the following figures correctly


indicates the induced charge distribution on the (b)
conductor (ignore edge effects).

(a) (b)
(c)

(c) (d)
(d)
33. Which of the following plots represents 35. Two satellites S1 and S2 are revolving around a
schematically the dependence of the time period planet in the opposite sense in coplanar circular
of a pendulum if measured and plotted as a concentric orbits. At time t = 0, the satellites are
function of its oscillations? (Note : amplitude need farthest apart. The periods of revolution of S1
not be small) and S2 are 3 h and 24 h respectively. The radius
of the orbit of S1 is 3 × 104 km. Then the orbital
speed of S2 as observed from
(a) the planet is 4 × 104 km h–1 when S2 is closest
from S1.
(a) (b) (b) the planet is 2 × 104 km h–1 when S2 is closest
from S1.
(c) S1 is × 104 km h–1 when S2 is closest from S1
(d) S1 is 3 × 104 km h–1 when S2 is closest from S1
36. A rectangular region of dimensions w × l (w l)
has a constant magnetic field into the plane of
the paper as shown. On one side the region is
bounded by a screen. On the other side positive
(c) (d) ions of mass m and charge q are accelerated from
rest and towards the screen by a parallel plate
capacitor at constant potential difference V < 0,
and come out through a small hole in the upper
34. On a pulley of mass M hangs a rope with two
plate. Which one of the following statements is
masses m1 and m2 (m1 > m2) tied at the ends as
correct regarding the charge on the ions that hit
shown in the figure. The pulley rotates without
the screen ?
any friction, whereas the friction between the
rope and the pulley is large enough to prevent
any slipping. Which of the following plots best
represents the difference between the tensions
in the rope on the two sides of the pulley as a
function of the mass of the pulley ?

2|v|m
m2 (a) Ions with q > will hit the screen.
B2 w 2
m1
2|v|m
(b) Ions with q < will hit the screen.
B2 w 2
(c) All ions will hit the screen.

2|v|m
(d) Only ions with q = will hit the screen.
B2 w 2
(a) (b)
37. Force F applied on a body is written as

F = ˆ ˆ nˆ
n.F G , where nˆ is a unit vector .

The vector G is equal to

(a) n̂ F (b) nˆ nˆ F
(c) (d)
(c) n̂ F F/ |F| (d) nˆ F nˆ
38. A particle of mass m moves around the origin in
1
a potential m 2r2, where r is the distance from
2 (d)
the origin. Applying the Bohr model in this case,
the radius of the particle in its nth orbit in terms
42. Among the -amino acid-threonine, tyrosine,
of a = h / 2 m is
methionine, arginine and tryptophan, those
which contain an aromatic group in their side
(a) a n (b) an
chain are
(c) an2 (d) an n (a) threonine and arginine
39. Two bottles A and B have radii RA and RB and (b) tyrosine and tryptophan
heights hA and hB respectively with RB = 2RA and (c) methionine and tyrosine
hB = 2hA. These are filled with hot water at 60°C. (d) arginine and tryptophan
Consider that heat loss for the bottles takes place
43. The number of stereoisomers possible for the
only from side surfaces. If the time the water
following compound is
to cool down to 50°C is tA and tB for the
bottles A and B, respectively, then tA and tB are CH3-CH=CH-CH(OH)-CH3
best related as (a) 1 (b) 2
(a) tA = tB (b) tB = 2tA (c) 3 (d) 4
(c) tB = 4tA (d) tA = tA/2 44. In electrophilic aromatic substitution reactions
40. The number of gas molecules striking per second of chlorobenzene, the ortho/para-directing ability
per square meter of the top surface of a table of chlorine is due to its
placed in a room at 20°C and 1 atmospheric (a) positive inductive effects (+I)
pressure is of the order of (kB = 1.4 × 10–23 J/K, (b) negative inductive effect (–I)
and the average mass of an air molecules is 5 × (c) positive resonance effect (+R)
10–27 kg)
(d) negative resonance effect (–R)
(a) 1027 (b) 1023
45. Among the following,
(c) 1025 (d) 1029

41. The major product formed in the following


reaction is

the antiaromatic compounds are


(a) I and IV (b) III and V
(a) (c) II and V (d) I and III
46. Upon reaction with CH 3 MgBr followed by
protonation, the compound that produces ethanol
is
(a) CH3CHO (b) HCOOH
(b)
(c) HCHO (d) (CHO)2
47. Which of the following is NOT an oxidation-
reduction reaction ?
(a) H2 + Br2 2HBr
(c) (b) NaCl + AgNO3 NaNO3 + AgCl
(c) 2 Na2S2O3 + I2 Na2S4O6 + 2NaI
(d) Cl2 + H2O HCl + HOCl
48. The thermal stability of alkaline earth metal and solid lines, respectively. Identify the correct
carbonates–MgCO3, CaCO3, SrCO3 and BaCO3, plot for the reaction.
follows the order
(a) BaCO3 > SrCO3 > CaCO3 > MgCO3
(b) CaCO3 > SrCO3 > BaCO3 > MgCO3
(c) MgCO3 > CaCO3 > SrCO3 > BaCO3
(a) (b)
(d) SrCO3 > CaCO3 > MgCO3 > BaCO3
49. When a mixture of diborane and ammonia is
heated, the final product is
(a) BH3 (b) NH4BH4
(c) NH2NH2 (d) B3N3H6
50. Among the following metals, the strongest
reducing agent is
(c) (d)
(a) Ni (b) Cu
(c) Zn (d) Fe
51. The molecule which is NOT hydrolysed by water
at 25°C is 56. Mg(OH)2 is precipitated when NaOH is added to
(a) AlCl3 (b) SiCl4 a solution of Mg2+. If the final concentration of
Mg2+ is 10–10 M, the concentration of OH–(M) in
(c) BF3 (d) SF6
the solution is
52. Among the following compounds, the one which
[Solubility product for Mg (OH)2 = 5.6 × 10–12]
does NOT produce nitrogen gas upon heating is
(a) 0.056 (b) 0.12
(a) (NH4)2 Cr2O7 (b) NaN3 (c) 0.24 (d) 0.025
(c) NH4NO2 (d) (NH4)2 (C2O4) 57. A constant current (0.5 amp) is passed for 1 hour
through (i) aqueous AgNO3, (ii) aqueous CuSO4
53. Chlorine has two naturally occurring isotopes,
and (iii) molten AlF3, separately. The ratio of the
35
Cl and 37Cl. If the atomic mass of Cl is 35.45, the
mass of the metals deposited on the cathode is
ratio of natural abundance of 35Cl and 37Cl is
closest to [MAg, MCu, MAl are molar masses of the respective
metals]
(a) 3.5 : 1 (b) 3 :1
(a) MAg : 2 MCu : 3 MA1
(c) 2.5 : 1 (d) 4 : 1
(b) MAg : MCu : MA1
54. The reaction C2H6 (g) C2H4 (g) + H2 (g) is at
(c) 6 MAg : 3 MCu : 2 MA1
equilibrium in a closed vessel at 1000 K. The
enthalpy change ( H) for the reaction is 137.0 kJ (d) 3 MAg : 2 MCu : MA1
mol–l. Which one of the following actions would 58. A reaction has an activation energy of 209 kJ
shift the equilibrium to the right ? mol –1. The rate increases 10–fold when the
(a) Decreasing the volume of the closed reaction temperature is increased from 27°C to X °C.
vessel The temperature X is closest to
(b) Decreasing the temperature at which the [Gas constant, R = 8.314 J mol–1 K–1]
reaction is performed (a) 35 (b) 40
(c) Adding an inert gas to the closed reaction (c) 30 (d) 45
vessel 59. A mineral consists of a cubic close-packed
(d) Increasing the volume of the closed reaction structure formed by O 2– ions where half the
vessel octahedral voids are occupied by Al3+ and one-
55. The enthalpy (H) of an elementary exothermic eighth of the tetrahedral voids are occupied by
reaction A B is schematically plotted against Mn2+. The chemical formula of the mineral is
the reaction coordinate. The plots in the presence (a) Mn3Al2O6 (b) MnAl2O4
and absence of a catalyst are shown in dashed (c) MnAl4O7 (d) Mn2Al2O5
60. For a 4p orbital, the number of radial and angular (c) free diffusion of K+ ions and pumping of Na+
nodes, respectively, are ions across the membrane.
(a) 3, 2 (b) 1, 2 (d) water channels formed by lipids in the
(c) 2, 4 (d) 2, 1 membrane.
68. In a chemical reaction, enzymes catalyze the
reaction by
61. Interferons combat viral infection by (a) lowering the activation energy.
(a) inhibiting viral packaging directly. (b) increasing the activation energy.
(b) increasing the binding of antibodies to viruses. (c) decreasing the free energy change between
(c) binding to the virus and agglutinating them. reactants and products.
(d) restricting viral spread to the neighboring (d) increasing the free energy change between
cells. reactants and products.
62. Leydig cells synthesize 69. The rigidity of cellulose is due to
(a) insulin (a) coiled structure of glucose polymer
(b) growth hormone (b) (1 4) glycosidic linkage
(c) testosterone (c) hydrogen bonding with adjacent glucose
polymer
(d) estrogen
(d) cross-linking between glucose and peptides
63. Glucagon increases the blood glucose
70. Antigen-angtibody reactions
concentration by
(a) always result in precipitation of the complex
(a) promoting glycogenolysis. (b) depend only on covalent interactions.
(b) increasing the concentration of fructose (c) are irreversible.
2,–6-bisphosphate.
(d) depend on ionic and hydrophobic interactions.
(c) increasing the concentration of pyruvate
71. Which ONE of the following combinations of
kinase.
molecular masses of polypeptides are obtained
(d) inhibiting gluconeogenesis. from purified human IgM when analysed on
64. Which ONE of the following is NOT essential for sodium dodecyl sulphate polyacrylamide gel
Polymerase Chain Reaction (PCR) ? electrophoresis (SDS-PAGE) under reducing
(a) Restriction enzyme conditions?
(b) Denaturation of DNA (a) 55 kDa, 15 kDa
(c) Primers (b) 70 kDa, 25 kDa, 15 kDa
(c) 55 kDa, 25 kDa
(d) DNA polymerase
(d) 155 kDa
65. CO2 acts as a greenhouse gas because
72. For a particular gene that determines the coat
(a) it is transparent to heat but traps sunlight.
color in a diploid organism, there are three
(b) it is transparent to sunlight but traps heat. different alleles that are codominant. How many
(c) it is transparent to both sunlight and heat. different skin colors are possible in such an
(d) it traps both sunlight and heat. organism?
66. A graph of species richness s area on log-log (a) 9 (b) 6
axes is (c) 4 (d) 3
(a) linear (c) oscillatory 73. Two genetic loci controlling two different traits
are linked. During the inheritance of these traits,
(b) sigmoidal (d) parabolic
the Mendelian laws that would be affected is/are
67. Concentration of Na+ ions outside a nerve cell is
(a) Law of dominance, law of segregation and law
100 times more than inside. The concentration
of independent assortment
of K+ ions is more inside the cells. The levels of
Na+ ions and K+ ions are maintained by (b) Law of segregation and Law of independent
assortment
(a) free diffusion of Na+ ions and pumping of K+
ions across the membrane. (c) Only Law of independent assortment
(d) Only Law of segregation
(b) Na+ and K+ pumps in the membrane.
74. Which ONE of the following statements is (a) more pO2 and less pCO2
INCORRECT? (b) less pO2 and pCO2
(a) Alleles are different forms of the same gene. (c) more pO2 and more pCO2
(b) Alleles are present at the same locus. (d) less pO2 and less pCO2
(c) Alleles code for different isoforms of a protein.
(d) Alleles are non-heritable.
75. Which ONE of the following statements is
INCORRECT about restriction endonucleases? 81. Let x,y,z be positive integers such that HCF
(a) They serve as primitive form of immune (x,y,z) = 1 and x2 + y2 = 2z2. Which of the following
system in bacteria. statements are true?
(b) They digest the DNA non-randomly. I. 4 divides x or 4 divides y.
(c) They digest the DNA at specific location. II. 3 divides x + y or 3 divides x – y.
(d) They digest the DNA from free ends. III. 5 divides z(x2 – y2)
76. The number of net ATP molecules produced from (a) I and II only
1 glucose molecule during glycolysis is (b) II and III only
(a) 1
(c) II only
(b) 2
(d) III only
(c) 3
82. How many different (mutually noncongruent)
(d) 4 trapeziums can be constructed using four distinct
77. Which ONE of the following coenzymes is required side lengths from the set {1,3,4,5,6}?
for the conversion of L-alanine to a racemic (a) 5 (b) 11
mixture of D-and L-alanine ?
(c) 15 (d) 30
(a) Pyridoxal-6-phosphate
83. A solid hemisphere is mounted on a solid cylinder,
(b) Thiamine pyrophosphate
both having equal radii. If the whole solid is to
(c) Coenzyme A have a fixed surface area and the maximum
(d) Flavin adenine dinucleotide possible volume, then the ratio of the height of
78. The cyclic electron flow during photosynthesis the cylinder to the common radius is
generates (a) 1:1 (b) 1:2
(a) NADPH alone.
(c) 2:1 (d) 2 :1
(b) ATP and NADPH.
84. Let ABC be an acute scalene triangle, and O and
(c) ATP alone.
H be its circumcentre and orthocenter
(d) ATP, NADPH and O2. respectively. Further let N be the midpoint of
79. Match the type of cells given in Column I with OH. The value of the vector sum NA NB NC
organisms given in Column II. Choose the
is
appropriate combination from the options below.
Column I Column II (a) 0 (zero vector) (b) HO
(P) Flame cells (i) Sponges
1
(Q) Collar cells (ii) Hydra (c) 1 HO (d) OH
2 2
(R) Stinging cells (iii) Planaria
85. The quotient when 1 x2 x 4 x6 ...... x34 is
(a) P-iii, Q-i, R-ii
divided by 1 x x 2 x 3 ...... x17 is
(b) P-iii, Q-ii, R-i
(a) x17 – x15 x13 – x11...... x
(c) P-i, Q-ii, R-iii
(b) x17 x15 x13 x11 ...... x
(d) P-ii, Q-iii, R-i
(c) x17 x16 x15 x14 ...... 1
80. Compared to the atmospheric air, the alveolar
(d) x – x
17 16
x –x
15 14
...... – 1
air has
86. Let R be the region of the disc x2+y2 1 in the first
quadrant. Then the area of the largest possible
circle contained in R is 91. One end of a rod of length L=1 m is fixed to a
point on the circumference of a wheel of radius
(a) 3 2 2
R 1 / 3 m. The other end is sliding freely
along a straight channel passing through the
(b) 4 3 2 center O of the wheel as shown in the figure
below. The wheel is rotating with a constant
angular velocity about O.
(c)
6

(d) 2 2 2

87. Let R be the set of real numbers and f : R R be


given by f (x) |x| log(1 |x| ) . We now
make the following assertions:
I. There exists a real number A such that f(x)
The speed of the sliding end P when = 60° is
A for all x.
II. There exists a real number B such that f(x) 2
(a) (b)
B for all x. 3 3
(a) I is true and II is false
2
(b) I is false and II is true (c) (d)
3 3
(c) I and II both are true
(d) I and II both are false 92. One mole of an ideal monatomic gas undergoes
the following four reversible processes :
3
Step1 : It is first compressed adiabatically from
88. Define g(x) f x y f (y)dy, for all real x, where
volume V1 to 1m3.
3
Step 2 : then expanded isothermally to volume
1, 0 t 1, 10 m3.
f (t) then
0, elsewhere. Step 3 : then expanded adiabatically to volume
V3.
(a) g(x) is not continuous everywhere
Step 4 : then compressed isothermally to volume
(b) g(x) is continuous everywhere but
V1.
differentiable nowhere
If the efficiency of the above cycle is 3/4 then V1
(c) g(x) is continuous everywhere and
is,
differentiable everywhere except at x = 0,1
(a) 2m3 (b) 4m3
(d) g(x) is continuous everywhere and
(c) 6 m3 (d) 8 m3
differentiable everywhere except at x
= 0,1,2 93. A neutron star with magnetic moment of
magnitude m is spinning with angular velocity
89. The integer part of the number
about its magnetic axis. The electromagnetic
44
1 power P radiated by it is given by xmy zcu
is
k 0 cos k cos k 1 where 0 and c are the permeability and speed of
(a) 50 (b) 52 light in free space, respectively. Then
(c) 57 (d) 59 (a) x = 1, y = 2, z = 4 and u = –3
90. The number of continuous functions f:[0,1] R (b) x = 1, y = 2, z = 4 and u = 3
that satisfy (c) x = –1, y = 2, z = 4 and u = –3
(a) 0 (b) 1 (d) x = –1, y = 2, z = 4 and u = 3
(c) 2 (d) infinity
94. A solid cube of wood of side 2a and mass M is
a uniform magnetic field B exists pointing into
resting on a horizontal surface as shown in the
the plane of the paper (see figure). As the loop
figure. The cube is free to rotate about a fixed
passes through this region, the plot correctly
axis AB. A bullet of mass m (<<M) and speed v is
depicting its speed ( ) as a function of x is
shot horizontally at the face opposite to ABCD at
a height 4a/3 from the surface to impart the cube
an angular speed . It strike the face and embeds
in the cube. Then c is close to (note : the
moment of inertia of the cube about an axis
perpendicular to the face and passing through
the center of mass is 2Ma2/3)

(a)

(a) Mv/ma (b) Mv/2ma


(b)
(c) mv/Ma (d) mv/2Ma
95. A gas obeying the equation of state PV = RT
undergoes a hypothetical reversible process

PV
described by the equation, PV5/3 exp E0 c1

where c1 and E0 are dimensioned constants. Then, (c)


for this process, the thermal compressibility at
high temperature
(a) approaches a constant value.
(b) is proportional to T.
(c) is proportional to T1/2
(d) is proportional to T2. (d)
96. To calculate the size of a hydrogen anion using
the Bohr model, we assume that its two electrons
move in an orbit such that they are always on 98. The figure of a centimeter scale below shows a
diametrically opposite sides of the nucleus. With particular position of the vernier calipers. In this
each electron having the angular momentum position the value of x shown in the figure is
= h/2 , and taking electron interaction into (figure is not to scale)
account the radius of the orbit in terms of the
2
0h
Bohr radius of hydrogen atom a B is
me 2
4
(a) aB (b) aB
3

2 3
(c) aB (d) aB (a) 0.02 cm
3 2
(b) 3.65 cm
97. A square-shaped conducting wire loop of
(c) 4.15 cm
dimension a moving parallel to the x-axis
approaches a square region of size b (a < b) where (d) 0.03 cm
99. A parallel beam of light is incident on a tank filled The radius of the planet’s orbit is closest to (1 A.
with water up to a height of 61.5 mm as shown U. = Earth-Sun distance)
in the figure below. Ultrasonic waves of (a) 0.004 A. U. (b) 0.008 A.U.
frequency 0.5 MHz are sent along the length of
(c) 0.004 A.U. (d) 0.12 A.U.
the water column using a transducer placed at
the top, and they form longitudinal standing
waves in the water. Which of the schematic plots
below best describes the intensity distribution 101. In the following reaction sequence
of the light as seen on the screen ? Take the
speed of sound in water to be 1,500 m/s.

X and Y are

(a)

(b)

(a) (b) (c)

(d)
102. In the following reactions

(c) (d)

100. A star of mass M (equal to the solar mass) with a


planet (much smaller than the star) revolves
around the star in a circular orbit. The velocity X and Y are
of the star with respect to the center of mass of
the star-planet system is shown below :
(a)

(b)

(c)

(d)
103. Which of the following alkenes can generate
optically active compounds upon hydrogenation?

(a) I, III and IV


(a) 1.0 (b) 0.0
(b) II and III
(c) 1.5 (d) 0.5
(c) I and III
109. A mixture of toluene and benzene boils at 100°C.
(d) II and IV
Assuming ideal behaviour, the mole fraction of
104. When heated in air, brown copper powder turns toluene in the mixture is closest to [Vapour
black. This black powder would turn brown again pressures of pure toluene and pure benzene at
when heated with 100°C are 0.742 and 1.800 bar respectively. 1 atm
(a) CO (b) O2 = 1.013 bar]
(c) H2 (d) NH3 (a) 0.824 (b) 0.744
105. The geometry and magnetic property of (c) 0.544 (d) 0.624
[NiCl4] , respectively, are
2–
110. A two-dimensional solid pattern formed by two
(a) tetrahedral, paramagnetic different atoms X and Y is shown below. The black
and white squares represent atoms X and Y,
(b) tetrahedral, diamagnetic
respectively. The simplest formula for the
(c) square planar, paramagnetic compound based on the unit cell from the pattern
(d) square planar, diamagnetic is
106. Among (i) [Cr(en)3]3+, (ii) trans-[Cr(en)2 Cl2]+ ,
(iii) Cis-[Cr(en)2 Cl2]+
(iv) [Co(NH3)4Cl2]+ the optically active complexes
are
(a) i and ii (b) i and iii
(c) ii and iii (d) ii and iv
107. Ac has a half-life of 22 years with respect to
227

radioactive decay . The decay follows two parallel


paths : 227Ac 227
Th and 227Ac 223
Fr. If the
percentage of the two daughter nuclides are 2.0
and 98.0, respectively, the decay constant (in
year–1) for 227Ac 227Th path is closest to (a) XY8 (b) X4Y9
(a) 6.3 × 10 –2
(c) XY2 (d) XY4
(b) 6.3 × 10–3
(c) 6.3 × 10–1
111. The genetic distance between genes A and B is
(d) 6.3 × 10–4
10 cm. An organism with Ab combination of the
108. A system consisting of 1 mol of an ideal gas alleles is crossed with the organism with aB
undergoes a reversible process, A B C A combination of alleles. What will be the
(schematically indicated in the figure below). If percentage of the gametes with AB allele
the temperature at the starting point A is 300 combination by an F1 individual ?
K and the work done in the process B C is 1 L
(a) 1 (b) 5
atm, the heat exchanged in the entire process in
L atm is (c) 10 (d) 50
112. Proteins P, Q, and R are associated with intact ii. Cysteine can be linked to another cysteine
organellar membrane in a cell. If the intact by S-S bond.
organellel is treated with a high ionic strength iii. Cysteine can complex with Zn2+.
buffer, only protein R remained associated with
iv. Cysteine can be linked to methionine by S-S
the membrane fraction. Based on this, one could
bond
conclude that
(a) i and ii (b) ii and iii
(a) P and Q are peripheral membrane proteins.
(c) iii and iv (d) i and iv
(b) R is a peripheral membrane protein.
118. The minimum number of plants to be screened
(c) P and Q are integral membrane bound
to obtain a plant of the genotype AabbCcDd from
proteins.
a cross beteen plants of genotypes AaBbCcDd and
(d) P is peripheral and Q is integral membrane AABbCCDd is
protein.
(a) 8 (b) 16
113. In photosynthesis, oxygen is produced by
(c) 32 (d) 64
(a) photosystem I from carbon dioxide.
119. When a pure bred, red flower-producing plant of
(b) photosystem II from carbon dioxide. genotype RR is crossed with a pure bred, white
(c) photosystem I from water. flower-producing plant of genotype rr, all the F1
(d) photosystem II from water. plants produced pink flowers If all the plants in
each generation from F1 to F6 are selfed, what
114. How many different proteins consisting of 100
will be the percentage of plants with red and
amino acids can be formed from 20 different amino
white flowers in the final population consisting
acids ?
of a large number of individuals ? (Consider that
(a) 20100 (b) 10020 flower colour has no effect on reproduction and
(c) 220
(d) 20 ×100 survival.)
115. Molecular weight of E.Coli DNA is 3.1 × 109 g/ (a) 3 – 4 (b) 12 –13
mol. Average molecular weight of nucleotide pair (c) 49 – 51 (d) 97 – 100
is 660 g/mol and each nucleotide pair contributes
120. The schematic below describes the status of lac
to 0.34 nm to the length of DNA. The length of
operon in the absence of lactose. Which ONE
E. coli DNA molecule will be approximately
of the following happens when lactose is
(a) 0.8 nm present in the cell ?
(b) 1.6 nm
(c) 1.6 m
(d) 1.6 mm
116. Which ONE of the following options is TRUE
with respect to Emigration ?
(a) It is the difference between the births and
deaths in a population.
(b) It is the difference between individuals who
have come to a habitat and who have left the
habitat.
(c) It involves individuals of different species (a) Lactose binds to P i and stops the transcription
coming to a habitat from elsewhere during of i.
the period under consideration
(b) Lactose is converted to allolactose, which binds
(d) It involves individuals of a population leaving
a habitat during the time period under to P lac and results in the displacements of the
consideration. repressor from O.
117. Choose the CORRECT combination of statements (c) Lactose is converted to allolactose, which
given below related to cysteine residue in binds to the repressor protein and prevents
proteins. its interaction with O.
i. Cysteine can be linked to tyrosine by S-O (d) Lactose has no effect on the status of the lac
bond. operon.
1. (d) 2. (b) 3. (d) 4. (a) 5. (c) 6. (b) 7. (c) 8. (b) 9. (d) 10. (a)

11. (c) 12. (d) 13. (b) 14. (b) 15. (a) 16. (b) 17. (a) 18. (c) 19. (c) 20. (a)

21. (a) 22. (b) 23. (d) 24. (c) 25. (c) 26. (c) 27. (a) 28. (b) 29. (c) 30. (a)

31. (d) 32. (b) 33. (a) 34. (c) 35. (d) 36. (b) 37. (d) 38. (a) 39. (b) 40. (a)

41. (a) 42. (b) 43. (d) 44. (c) 45. (b) 46. (c) 47. (b) 48. (a) 49. (d) 50. (c)

51. (d) 52. (d) 53. (b) 54. (d) 55. (a) 56. (c) 57. (c) 58. (a) 59. (b) 60. (d)

61. (d) 62. (c) 63. (a) 64. (a) 65. (b) 66. (a) 67. (b) 68. (a) 69. (c) 70. (d)

71. (b) 72. (b) 73. (c) 74. (d) 75. (d) 76. (b) 77. (a) 78. (c) 79. (a) 80. (b)

81. (b) 82. (b) 83. (a) 84. (c) 85. (d) 86. (a) 87. (b) 88. (d) 89. (c) 90. (b)

91. (a) 92. (d) 93. (a) 94. (d) 95. (a) 96. (b) 97. (b) 98. (d) 99. (a) 100. (c)

101. (b) 102. (a) 103. (c) 104. (c) 105. (a) 106. (b) 107. (d) 108. (c) 109. (b) 110. (a)

111. (b) 112. (b) 113. (b) 114. (a) 115. (c) 116. (d) 117. (b) 118. (c) 119. (d) 120. (c)

Circle
1. (d) Case (i) : Case (iii) :
A = B
AC = BC
2
h2 k a 2a 2

x2 + (y – a)2 = 2a2
also a circle
If B= C So union of two circle and a line.
locus of A is bisector of BC 3. (d)Rationalize
So it is a straight line
4x 2 x 2x
Case (ii) : lim 4x 2 x 2x
x 2
4x x 2x

x
lim
x
1 at x |x| = –x
|x| 4 2x
x

If A= C
x 1 1
BC fixed B(a, 0), C(0, a) lim
x
1 2 2 4
BC = AB x 4 2x
x
So, (x – a) + y = 2a
2 2 2
4. (a) (I) This relation is reflexive relation because 0 1
every natural number divides square of 8. (b) A1 = 2 4x 1dx 2 1 x 2 dx
itself a R a a divides a2 1/ 4 0

(II) Not symmetric eg. 5 R 10 5 Divide 100


But 10 R 5 10 Divide 25
(III)Not transitivity for example if 8 R 4 & 4 R
2 8 R 2 only (I) Option
2017
5. (c) I + F2 = 2017 44

2017
F2 = 2017 44 ; 0 < F2 < 1
1
2016 Solve A1 =
2017
C1 2017 44 .... 3 2
I + F – F2 = 2
1 1
F2 = F2 A2 = (1)2 – A1 = –
3 2 2 3
F2 = (0.911)2017
2
|A1 – A2| =
2017
3
Now, F1 = 44 2017 = – (0.911)2017
10. (a) p1 = 1 – (no die shows six)
Fractional part can not –ve. 6
5
So, F1 = 1 – (0.911)2017 P, I 1 0.6651
6
So, F1 + F2 = 1
p2 = 1 – (no die shown two + one die shown
1 lie Between 0.9 & 1.35 two)
6. (b) only possible when sin 3x = 1 & sin 7x = 1 12 11 1
5 12 5 1
for sin 3x = 1, p2 = 1 – C1 = 0.61866
6 6 6
sin 3x = sin (4n + 1) ,n I p1 > p 2
2
a 1
11. (c) 1 a 10
3x = (4n + 1) x = (4n + 1)
2 6
a2 + 10a – 1 = 0
are get the two values of 'a'
Similarly for sin 7x = sin(4m + 1) ,m I
2 15. (a) (i) a + b < c + d
(ii) b + c < d + e
x = (4m + 1) (iii) c + d < e + a
14
(iv) d + e < a + b
Now, From (i) & (iii), we get
For common solution a+b<e+a
b<e
(4n + 1) = (4m + 1) From (ii) & (iv), we get
6 14
Solving are get 1 = 3m – 7n b+c<a+b
First solution is m = 5, n = 2 c<a
Subtracting equation (i) and (ii), we get
Second solution is m = 12, n = 5
a–c<c–e
Hence two solutions are possible
c>e
p2 q2 r2 Subtracting equation (i) and (iv), we get
7. (c) p + q + r =
3 (a – e) + (b – d) < (c – a) + (d – b)
Now, for maximum value, p = 3, q = 3, r = 3 d>b
p+q+r=3+3+3=9 Thus we can conclude that 'a' is greatest and
'b' is least
5
1 1 1
16. (b) Case (1) : All tail 1 8 1
2 2 20
4 5
1 1 5 1 1
Case (2) : 4T, 1H5C4 .
2 2 25 2 2
Case (3) : T × T × T ×
22
3 2
1 4 1 1 6 21
C2 6
2 2 25 25 1
21. (a) r &v z
Case (4) : × T × T × z
2 2
1 1 1 v2 1
according z2 z3
2 2 25 r 1/z

13 3 3
overall = aH ZH 1 1
25
a He ZHe 2 8
2 2
18. (c) AB = a1 b1 a2 b2 22. (b) VIBG | YOR
400 nm | 700 nm
Square + Square = 65 possible when Absorbed Reflected therefore seen
= 64 + 1
1
23. (d) If coloumb’s force gauss’s law is not valid
74 = 49 +25 r3
97 = 81 + 16 q en
But 83 is possible 0

19. (c) Given Integral can be distributed into For static condition E = 0 in both of conductor
1/ 3 3 2 through a Gaussian surface just under
1 14
.dx 3 dx x 2 dx the surface of conductor = 0 but as
1/ 2
x2 1/ 3 3
3
q en is not valid
22 p =
20. (a) q > 0, try to the contra prove that q < 0
21
So qen = 0 is not correct statement. Some
p 22 charge will present insider bulk of conductor.
q
21 21
J
p 1 20
1 1 20 24. (c) Conductivity
q ai E
21 20 i 1 ai 21 i 1
1
20
1 i 1 20 1
2
i
20 i 1 i 1 21 i 1 i Then resistivity

20 20 E
1 1 i 1 =
J
2 20 i 1 i2 1 i 1 21i
E = [M L T–3 I–1]
1 1 1 20
i 20
1 J = [I L–2]
2 20 2 i 2 i 2
1 i 1 21i
h = M L2 T–1
me = M
1 1 1 2 20 1 20 c = LT–1
1 1
2 20 2 5i 2 21 i 1 –1 –3 4 2
=M L T I
1 1 1 2 1 h2
19 20 =
2 20 2 5 21 m e ce 2
25. (c) due to soap bubble surface tension is reduced
therefore in that area. Black paper powder 28. (b)
will sink.
26. (c) According to snell's law
The index of prism canbe determine by
knowing the Apex angle(A) and minimum
angle of deviation ( m )

A
sin m

sin i 2
h 1
sin r A =
sin mv v
2
v v 2y v 20
A v y = u y + a yt
sin m

0.004 2
1.5 2
A qE0
sin vy = 0 + t
2 m

1 (3v0) = v 2y v 02
m

v2y 8v20
m
( 1) > m
( ) if 1
< 2

qE0
t 2 2 v0
27. (a) m
T1

T2 2 2m 1
t v0 t
30° qE0 E0
60°

1
29. (c) X c = is very large because of frequency
C
and capaitance is too small therefore bird
does very high capacitive reactance in the
path of A.C. current.
vertical
30. (a) Option ‘A’ is correct option. According charge
conservation & Gauss’s law.
I0 = 20 W/m2
31. (d) y = A sin(kx – t)
I0 20 at x = 0.025, y = 0.02
I1 = = 10 W/m2
2 2 v=
I2 = I1 cos 30º 2
100 1
m
2 500 5
3 3
10 10.
2 4 2 1
y = 0.02 = A sin .
4
= 7.5 w/m2
5
y = 0.002 = A sin
2
A = 0.02m
y = 0.02 sin (kx – t)
= 0.02 sin(10 × 0.2 – 1000 × 5 × 10–4]
I m1 m2
= 0.02 sin[2 – 0.5 ] g
R 2 m1 m2
3
= 0.02 sin = –0.02m
2 I m1 m2 g
T1 T2
Qdt Qdt I m1 m2 R2
32. (b) ds =
T1 T2
1
MR 2 m1 m2 g
ds 1 1 2
Q 1
dt T1 T2 MR 2 m1 m2 R2
2

T1 T2 T2 T1 Hence
R T1 T2
M m1 m2 g
T1 T2
T12 T22 1 M 2 m1 m2
T1 T2 R
35. (d) We know that relation between time period
and radius
ds T1 T2 1 T2 r3
dt T2 T1 R

1 1
x
x R
33. (a) Time period will increase as the amplitude is
increases.

34. (c) Given Ts1 3hTs2 24h

rs1 3 404

3
9 3 104
24 2 r3

m1g – T1 = m1a ...(1) 3


3 3 3 10 4
T2 – m2g = m2a ...(2)
24 24 r
(T1 – T2) × R = I [ a=r ]

Ia 1 3 104
R 4 r
From the equation (1) and (2) r = 12 ×104
(m1 – m2)g + (T2 – T1) = (m1 + m2)a orbital speed of S2 seen from planet = 2
r

m1 m2 g T2 T1 2
I = × 12 × 104
T1 T2 24
R m1 m2
= × 104 km h–1

I 2
T1 T2 1 V1 = r = × 3 × 104
R 2 m1 m2
1 1
24
2 × 104 km h–1
2mV
w2 B2
q

2mV
q
w 2 B2

37. (d) Force applied an a body F (n.F)n G


2 2 then
t
3 24
nˆ F nˆ nˆ nˆ F
9t 1
24 2 nˆ n.F
ˆ ˆ ˆ
F n.n F nˆ n.F
ˆ G

12 Bohr's model of an atom


t hr
9
h
Angle rotate by both satellite 38. (a) mvr = n
2
2 12 8 v=r
1
3 9 9
h
mr2 = n
2 12 2
2
24 9 9
nh 2
velocity of S2 seen from S1 = V1 + V2 r
2 m
= 3 × 104 km h–1
h2
36. (b) r n
2 m

r n.a
39. (b) Two Bottle

Ions will hit if r > w.


w=q V

1
mv2 = qV
2

2qV
v= A B
m
hB
mv 2 RB = 2RA hA
qvB 2
r
m
mv m 2qV t
r A
qB qB m
V
t = k. = k. h
1 2mV A
r
B q t h

1 2mV tA hA h0
w tB 2t A
B q tB hB 2h0
M
3RT kT 57. (c) Moles of Ag+ deposited =
40. (a) Vrms 1
M M0
M
P = N × 2mVrms Moles of Cu+2 deposited =
2
1.01 × 105 = N × 2 × 5 × 10–27 × Vrms
M
1.01 105 5 10 27 Moles of Al+3 deposited =
N 3
27 23
2 5 10 3 1.4 10 293
Hence ratio of mass of Ag+ : Cu+2 : Al+3 is
= 6.43×1027 6 : 3 : 2.

209000 1 1
NHCOCH3 NHCOCH3 58. (a) log10 = 2.303 8.314 300 T2
Conc. HNO3
41. (a)
Conc. H 2SO4 T2 308.4 K 35 C

NO2 1 1
59. (b) O 8 6 4
8 2
43. (d) No. of stereoisomers = 2n = 22 =4
44. (c) It shows –I and +R effect. 1
Al 4 2
45. (b) III and V have 4 electrons and so are 2
antiaromatic.
1
(i)CH3 MgBr Mn = 8 × =1
46. (c) HCHO (ii)H2 O / H
CH3 CH 2 OH 8

48. (a) Thermal stability increases down the group So formula is MnAl2O4.
in Periodic table. 60. (d) Radial nodes = n – – 1 = 4 – 1 – 1 = 2
49. (d) B2 H6 NH3 B3 N3 H6 (Inorganic Benzene) Angular nodes = = 1
61. (d) Virus infected cells produces interferons and
51. (d) SF6 H2O No reaction at Room temp. interferons protect non-infected cells from the
virus.
52. (d) NH 4 C2 O 4 2NH3 COOH
2 62. (c) Leydig cells are present in testis.
| 63. (a) Glucagon is a hyperglycenic hormone, It
COOH promote glycogenolysis in liver.
64. (a) Restriction enzymes are required for cutting
M1 n1 M2 n 2 a DNA fragement not in PCR.
53. (b) Mav
n1 n2 65. (b) Green house gases trap heat.
66. (a) Graph will be linear because
35n1 37n 2
35.45 logs S = log C + 2 log A
n1 n2
67. (b) Na+-K+ pumps are proteins which maintain
n1 3 the concentration gradients across the
or membrane. They use enegy (ATP)
n2 1
68. (a) Enzyme provide an alternative path for the
55. (a) In exothermic reaction, H < 0 reaction to occur. Enzyme lower the activation
EB – Ea < 0 OR Eb < Ea energy.
2 69. (c) Hydrogen bonding between different cellulose
56. (c) K SP Mg 2 OH fibers make the cellulose rigid.

12 10
2 70. (d) Antigen-Antibody interaction are elecrostatic
5.6 10 10 OH interaction and hydrophobic interactions.

5.6
OH 0.24
100
72. (b) n(n 1) 3(3 1)
6 (n = number of
2 2
alleles)
73. (c) Only law of independent assortment would be
a b c a b c
affected. O ;
3 2
74. (d) Alleles are alternate form of the same gene.
75. (d) Endonuclease do not cut the DNA from free
1
ends. They cut from somewhere in the middle N a b c
4
depending on their restriction site.
76. (b) In glycolysis 85. (d)

Total ATP produces = 4


ATP consumed = 2
Net gain 4–2=2
78. (c) Only ATP is produced during cylic
phosphorylation.

81. (b) Take combination such as x = 1, y = 7, z = 5


Now, checking from options
(ii) and (iii) statement are correct.
option (d) is correct
83. (a) S = 2 R2 + 2 Rh + R2
86. (a) Required equation of circle is
where R is radius of hemisphere and cylinder
(x – h)2 + (y – h)2 = h2
2 Both circle touch internally
V= R3 + R2h
3 C1C2 = |r1 – r2|

2 5 3 R2 h2 h2 = |h – 1|
V= R3 R2
3 2 R
h= 2 1
Now differentiating w.r.t 'r' we get
Area of largest possible circle = ( 2 1 )2
dV 5 9
2 R2 R2 = (3 – 2 2 )
dR 2 2
87. (b) Graph of given function actually look like this
dV
for maximum & minimum 0
dR
5 R2 = 3 R2 + 2 Rh
2R = 2h
h:R=1:1
84. (c) Circumcenter (origin O)
Clear from graph option (B) is right.
1 1 1 dx d
89. (c) ... x R cos Rx sin
cos0 cos1 cos1 cos 2 cos 2 cos3 dt dt

1 dx d
.... v &
cos 44 cos 45 dt dt
Now multiplying and dividing by sin; we get
Rx sin
v
1 sin1 sin1 x R cos
sin1 cos0 cos1 cos 0 cos 2
1 23
.
,
sin1 3 3 2 3 2
....
cos 44 cos 45 2 1 1 1 3 3
.
3 3 2 32
1 sin 1 0 sin 2 1
sin1 cos0 cos1 cos 0 cos 2 2
v
3
sin 45 44 92. (d) Four reversible process shown in graph
....
cos 44 cos 45

1
[tan1º – tan0º + tan2º – tan1º
sin1
+ ... tan45º – tan44º]

1
tan 45
sin1

1
57.2987
0.0174524
Integral part = 57

1
91. (a) Given, L = 1 m, R m 2/ 3
3 T1 V2
1 1
T2 V1

2/ 3
1 1
V1 8m 3
4 V1

L 1 93. (a) P = [M L2 T–3]


tan 60 3 tan
R 1/ 3 = [M L T–2 I–2] m = [I L2]
0

= [T–1]
1 2
x 1 C = [L T–1]
3 3
[M L2 T–3] = [M L T–2 I–2]x [I L2]y [T–1]z [LT–1]u
R2 x 2 L2 x = 1, y = 2, z = 4, u = –3
cos
2Rx P z
0Xmy c
R + x – L = 2Rx cos
2 2 2
94. (d) Side = 2a
dx dx Mass = M
2x = 2R x sin cos
dt 4a
n
3
mv 2 ke 2 ke2
&
r r2 2r
2

mv 2 3 ke 2
r 4 r2

3 2
mv 2 r ke ...(2)
4
8
Ma 2 Solving (1) + (2)
3
mvr = I 4 0 h2 4
r
me 2 3
4a
mv
3 mv 4
r aB
8 2Ma 3
Ma 2
3 97. (b)
PV

95. (a) PV 5/ 3
c1e E0

5 PV
ln P + ln V = ln c1 + E
3 0

dP 5 dV PdV VdP inside B speed will be constant therefore B


0 option is correct, representation of speed
P 3 V E0
98. (d) x = 0.3 cm – 3 × scale division of vernier
calipers
1 V 5
dP dV P
P E0 3V 9
= 0.3 cm – 3 ×
100
1 V
30 27
dV P E0
100
dP 5
P
3V 3
100
1 1 = 0.03 cm
1 dV E0 PV 99. (a) v =
c
V dP 5
P 1500
3V = = 3000 × 10–6
0.5 106
1 = 3mm
At very high temperature c = E
0

96. (b)

intensity vary according to graph ‘A’


h
mvr = n (Angular momentum) ...(1)
2
2 4 2 2 104. (c) Air oxidises the substance and Hydrogen will
100. (c) T a
GM act as reducing agent.
105. (a) Chlorine is weak field ligand.
4 2
1 Ni 2 3d 8 4s0
GM
T = in year So hybridisation is sp3 and Paramagnetic.
a = radius in A.U. K1 Th
3 107. (d) Ac
T = 3 days = year
365 K2 Ac
2/ 3
3 Now, KT = K1 + K2
a=
365 0.693 98
a = 0.04 A.U. K1 K1
22 2
CH2 CH2OH COOH K1 6.3 10 4

(i) BH3 CHO3 108. (c) E=q+w


101. (b)
(ii) H2O2/OH O=q+w
(X) (Y) or w = –q = –1 lt. atm.
For A B
O OD w = –1(1.5 – 1) = –0.5 lit. atm.
(i) NaBH4 q = –w = 0.5 lt. atm.
102. (a) C CH3 – C – CH3 Total Heat = 1 + 0.5 = 1.5
+
(ii) D3O
CH3 CH3 H 109. (b) At 100°C, P = 1 atm = 1.013 bar
1.013 = 0.742XTol. + 1.8Xben.
OH 1.013 = 0.742(1 – Xben.) + 1.8Xben.
(i) NaBD4 Xben. = 0.256
+
CH3 – C – CH3
(ii) H3O Xtol. = 0.744
D 110. (a) White squares = 8
Black squares = 1

CH2 Formula XY8


112. (b) P & Q are peripheral proteins and r is an
103. (c) (i) CH3CH2 – C – CH2CH2CH3 integral protein depending upon ease of
extraction.
CH3 114. (a) Type of protein 20100.
115. (c) Number of base pair
C2H5 – C – C3H7
Molar mass of DNA 3.1 109
H = Molar mass of one nucleotide
(Optically 660
active) Length of DNA = Number of base pairs × 0.34
nm
3.1 109
CH3 = 0.34 nm
660
116. (d) Emigration = leaving a habitat
(ii) CH2 = CH – CH2 – CH – C2H5
118. (c) AaBbCcDd × Aa Bb CC Dd

CH3
Aa bb Cc Dd
C2H5 – C – C3H7 1 1 1 1 1
2 4 2 2 32
H
(Optically 120. (c) Lac operon uses lactose as inducer hence its
active) an inducible operon.
5. Suppose the tangent to the parabola y = x2 + px +
q at (0, 3) has slope –1. Then p + q equals
1 . The number of triples (x, y, z) of real numbers
satisfying the equation (a) 0 (b) 1

x4 + y4 + z4 + 1 = 4 xyz is (c) 2 (d) 3

(a) 0 (b) 4 6. Let O = (0, 0) ; let A and B be points respectively


on x-axis and y-axis such that OBA = 60º. Let D
(c) 8 (d) more than 8 be a point in the first quadrant such that OAD is
an equilateral triangle. Then the slope of DB is -
2. If P(x) be a polynomial with real coefficients such
that P(sin 2x) = P(cos 2x), for all x [0, /2]. (a) (b)
3 2
Consider the following statements :
I. P(x) is an even function. 1 1
(c) (d)
II. P(x) can be expressed as a polynomial in (2x – 2 3
1)2
7. Suppose the parabola (y – k)2 = 4 (x – h), with
I. P(x) is a polynomial of even degree vertex A, passes through O = (0, 0) and L = (0, 2).
Let D be an end point of the latus rectum. Let
Then. the y-axis intersect the axis of the parabola at P.
(a) all are false Then PDA is equal to

(b) only I and II are true 1 2


(a) tan–1 (b) tan–1
19 19
(c) only II and III are true
(d) all are true 8
4
(c) tan–1 (d) tan–1
3. For any real number r, let Ar = {e : n is a natural
i rn
19 19
number}be a set of complex numbers. Then -
8. In a circle with centre O, suppose A, P, B are
(a) A1, A 1 , A0.3 are all infinite sets three points on its circumference such that P is
the mid-point of minor arc AB. Suppose when
AOB = ,
(b) A1 is a finite set and A 1 , A0.3 are infinite sets
area AOB
5 2
area APB
(c) A1, A 1 , A0.3 are all finite sets
If AOB is doubled to 2 , then the ratio

(d) A1, A0.3 are finite sets and A 1 , is an infinite area AOB
area APB is -
sets
4. Number of integers k for which the equation x3 – 1
27x + k = 0 has at least two distinct integer roots (a) (b) 5 2
5
is -
(a) 1 (b) 2 5 1
(c) 2 3 3 (d)
(c) 3 (d) 4 2
9. X = {x R : cos (sin x) = sin (cos x)}. The number II. There are infinitely many x [0, 1] for
of elements in X is - 1
which lim fn(x) =
(a) 0 (b) 2
n
2

(c) 4 (d) not finite III. There are infinitely many x [0, 1] for which
lim f (x)+ 1
10. A sphere with centre O sits atop a pole as shown n n

in the figure. An observer on the ground is at a


distance 50m from the foot of the pole. She notes IV. There are infinitely many x [0, 1] for which
that the angles of elevation from the observer to lim f (x) = 1 does not exist
n n
points P and Q on the sphere are 30º and 60º,
respectively. Then, the radius of the sphere in (a) I and III only
meters is -
(b) II only
(c) I, II, III only
(d) I, II, III and IV

x 3
x3
12. Limit lim x 2 et dt equals
x 0

1
(a) (b) 2
3

2
(c) (d)
3
1 50 6 13. The polynomial equation x3 – 3ax2 + (27a2 + 9)x +
(a) 100 1 (b)
3 3 2016 = 0 has -
(a) exactly one real root for any real a
1 100 6
(c) 50 1 (d) (b) three real roots for any real a
3 3
(c) three real roots for any a 0, and exactly one
1 real root for any a < 0
11. The graph of the function f(x) = x + sin (2 x), 0
8 (d) three real roots for any a 0, and exactly one
x 1 is shown below. Define f1(x) = f(x), fn+1(x) = real root for any a > 0
f(fn(x)), for n 1
14. The area of the region bounded by the curve y =
| x3 – 4x2 + 3x | and the x-axis, 0 x 3, is -

37 9
(a) (b)
6 4

37
(c) (d) 0
12

15. The number of continuous function f : [0, 1]


1
1
[0, 1] such that f(x) < x for all x and f x dx
2
0
3
is:
Which of the following statements are true ?
(a) 0 (b) 1
I. There are infinitely many x [0, 1] for which
(c) 2 (d) infinite
lim f (x) = 0
n n
16. On the real line R, we define two functions f and
g as follows :
21. Physical processes are sometimes described
f (x) = min{x – [x], 1 – x + [x]} visually by lines. Only the following can cross -
g (x) = max{x – [x], 1 – x + [x]} (a) Streamlines in fluid flow
Where [x] denotes the largest integer not (b) Lines of forces in electrostatics
exceeding x. The positive integer n for which
1 (c) Rays in geometrical optics
g x f x dx 100 is
0
(d) Lines of force in magnetism
22. Uniform ring of radius R is moving on a horizontal
(a) 100 (b) 198
surface with speed v and then climbs up a ramp
(c) 200 (d) 202 of inclination 30º to a height h. There is no slipping
in the entire motion. Then h is
17. Let v be a vector in the plane such that
(a) v2/2g (b) v2/g
|v i | |v 2i| |v j|. Then |v| lies in the
(c) 3v2/2g (d) v2/g
interval -
23. A gas at initial temperature T undergoes sudden
(a) (0, 1] (b) (1, 2]
expansion from volume V to 2V. Then -
(c) (2, 3] (d) (3, 4]
(a) The process is adiabatic
18. A box contains b blue balls and r red balls. A ball
(b) The process is isothermal
is drawn randomly from the box and is returned
to the box with another ball of the same colour. (c) The work done in this process is nRT ne(2)
The probability that the second ball drawn from where n is the number of moles of the gas.
the box is blue is -
(d) The entropy in the process does not change
2
b b 24. Photons of wavelength are incident on a metal.
(a) (b) r b
2
The most energetic electrons ejected from the
r b
metal are bent into a circular arc of radius R by a
perpendicular magnetic field having a magnitude
b 1 b b 1 B. The work function of the metal is (Where
(c) (d) r b r b 1 symbols have their usual meanings) -
r b 1
2
19. The number of noncongruent integer-sided hc e 2 B2 R 2 hc eBR
triangles whose sides belong to the set {10, 11, (a) me (b) 2m e
2m e 2m e
12, …., 22} is-
(a) 283 (b) 446 2
hc e 2 B2 R 2 hc eBR
(c) 448 (d) 449 (c) m e C2 (d) 2m e
2m e 2m e
20. Suppose we have to cover the xy-plane with
identical tiles such that no two tiles overlap and 25. A container is divided into two equal part I and II
no gap is left between the tiles. Suppose that we by a partition with a small hole of diameter d.
can choose tiles of the following shapes; The two partitions are filled with same ideal gas,
equilateral triangle, square, regular pentagon, but held at temperature TI = 150 K and TII = 300
regular hexagon. Then the tiling can be done K by connecting to heat reservoirs. Let I and II
with tiles of - be the mean free paths of the gas particles in the
two parts such that d >> I and d >> II. Then I/
(a) all four shapes is close to -
II

(b) exactly three of the four shapes (a) 0.25 (b) 0.5
(c) exactly two of the four shapes (c) 0.7 (d) 1.0
(d) exactly one of the four shapes
26. A conducting bar of mass m and length moves 29. A particle at a distance of 1 m from the origin
on two frictionless parallel rails in the presence starts moving such that dr/dq = r, where (r, q)
of a constant uniform magnetic field of magnitude are polar coordinates. Then the angle between
B directed into the page as shown in the figure . resultant velocity and tangential velocity
The bar is given aninitial velocity v0 towards the component is
right at t = 0. Then the
(a) 30 degrees
(b) 45 degrees
(c) 60 degrees
(d) Dependent on where the particle is
30. Electrons accelerated from rest by an
electrostatic potential are collimated and sent
through a Young’s double slit setup. The fringe
width is w. If the accelerating potential is doubled
then the width is now close to -
(a) 0.5 w (b) 0.7 w
(c) 1.0 w (d) 2.0 w
(a) Induced current in the circuit is in the
31. A metallic sphere is kept in between two
clockwise direction
oppositely charged plates. The most appropriate
(b) Velocity of the bar decreases linearly with representation of the field lines is -
time
(c) Distance the bar travels before it comes to a (a) (b)
complete stop is proportional to R
(d) Power generated across the resistance is
proportional to l
27. A particle with total mechanical energy, which
is small and negative, is under the influence of a
one dimensional potential U(x) = x4/4 – x2/2 J
Where x is in meters. At time t = 0s, it is at x =
– 0.5 m. Then at a later time it can be found (c) (d)
(a) Anywhere on the x axis
(b) Between x = – 1.0 m to x = 1.0 m
(c) Between x = – 1.0 m to x = 0.0 m
(d) Between x = 0.0 m to x = 1.0 m
28. A nurse measures the blood pressure of a seated
patient to be 190 mm of Hg - 32. An electron with kinetic energy E collides with a
hydrogen atom in the ground state. The collision
(a) The blood pressure at the patient’s feet is less
will be elastic
than 190 mm of Hg
(a) For all values of E
(b) The actual pressure is about 0.25 times the
atmospheric pressure (b) For E < 10.2 eV
(c) The blood pressure at the patient’s neck is (c) For 10.2 eV < E < 13.6 eV only
more than 190 mm of Hg
(d) For 0 < E < 3.4 eV only
(d) The actual pressure is about 1.25 times the
atmospheric pressure
33. The continuous part of X-ray spectrum is a result 38. A nuclear fuel rod generates energy at a rate of 5
of the × 108 Watt/m3. It is in the shape of a cylinder of
radius 4.0 mm and length 0.20 m. A coolant of
(a) Photoelectric effect specific heat 4 × 103 J/(kg-K) flows past it at a
(b) Raman effect rate of 0.2 kg/s. The temperature rise in this
coolant is approximately -
(c) Compton effect
(a) 2ºC (b) 6 ºC
(d) Inverse photoelectric effect
(c) 12 ºC (d) 30 ºC
34. Thermal expansion of a solid is due to the
39. A hearing test is conducted on an aged person. It
(a) symmetric characteristic of the inter atomic is found that her threshold of hearing is 20
potential energy curve of the solid decibels at 1 kHz and it rises linearly with
(b) asymmetric characteristic of the inter atomic frequency to 60 decibels at 9 kHz. The minimum
potential energy curve of the solid intensity of sound that the person can hear at 5
kHz is-
(c) double well nature of the inter-atomic
potential energy curve of the solid (a) 10 times than that at 1 kHz

(d) Rotational motion of the atoms of the solid (b) 100 times than that at 1 kHz

35. An electron and a photon have same wavelength (c) 0.5 times than that at 9 kHz
of 10–9 m. If E is the energy of the photon and p is (d) 0.05 times than that at 9 kHz
the momentum of the electron, the magnitude
of E/p in SI units is 40. Two infinitely long parallel wires carry currents
of magnitude I1 and I2 and are at a distance 4 cm
(a) 1.00 × 10–9 (b) 1.50 × 108 apart. The magnitude of the net magnetic field
(c) 3.00 × 108 (d) 1.20 × 107 is found to reach a non-zero minimum values
between the two wires and 1 cm away from the
36. If one takes into account finite mass of the first wire. The ratio of the two currents and their
proton, the correction to the binding energy of mutual direction is
the hydrogen atom is approximately (mass of
proton = 1.60 × 10–27 kg, mass of electron = 9.10 × I2
(a) 9, antiparallel
10–31 kg)- I1
(a) (b)
I2
(c) (d) (b) 9, parallel
I1
37. A monochromatic light source S of wavelength
440 nm is placed slightly above a plane mirror M I2
as shown . Image of S in M can be used as a virtual (c) 3, antiparallel
I1
source to produce interference fringes on the
screen. The distance of source S from O is 20.0
cm, and the distance of screen from O is 100.0 I2
(d) 3, parallel
cm (figure is not to scale). If the angle = 0.50 × I1
10–3 radians, the width of the interference fringes
observed on the screen is –

41. The shape of SCl4 is best described as a


(a) square
(b) tetrahedron
(c) square pyramid
(a) 2.20 mm (b) 2.64 mm (d) see-saw
(c) 1.10 mm (d) 0.55 mm
42. Among the following atomic orbital overlap, the 47. The molecule with the highest dipole moment
non-bonding overlap is among the following is

(a) (a) NH3 (b) F3


(c) CO (d) HF
48. The most stable Lewis acid-base adduct among
the following is
(a) H2O BCl3 (b) H2S BCl3
(b)
(c) H3N BCl3 (d) H3P BCl3
49. The reaction of D-glucose with ammoniacal
AgNO3 produces

(c)
(a) (b)

(d)

(c) (d)

43. Among the following complexes, the one that can


exhibit optical activity is
(a) [CoCl6]3–
(b) [Co(en)Cl4]–
(c) cis-[Co(en)2Cl2]3+
(d) trans-[Co(en)2Cl2]+ 50. The reagent (s) used for the conversion of benzene
diazonium hydrogensulfate to benzene is / are -
44. The pKa of oxoacids of chlorine in water follows
the order (a) H2O
(a) HClO < HClO3 < HClO2 < HClO4 (b) H3PO2 + H2O
(b) HClO4 < HClO3 < HClO2 < HClO (c) H2SO4 + H2O
(c) HClO4 < HClO2 < HClO3 < HClO (d) CuCl / HCl
(d) HClO2 < HClO < HClO3 < HClO4 51. The major product obtained in the reaction of
toluene with 1-bromo-2-methyl propane in the
45. The packing efficiency of the face centered cubic presence of anhydrous AlCl3 is
(fcc), body centered cubic (bcc) and simple /
primitive cubic (pc) lattices follows the order
(a) (b)
(a) fcc > bcc > pc (b) bcc > fcc > pc
(c) pc > bcc > fcc (d) bcc > pc > fcc
46. The ratio of root mean square velocity of hydrogen
at 50 K to that of nitrogen at 500 K is closest to
(c) (d)
(a) 1.18 (b) 0.85
(c) 0.59 (d) 1.40
52. The major product in the following reaction is 55. It takes 1h for a first order reaction to go to 50%
completion. The total time required for the same
reaction to reach 87.5% completion will be

(a) 1.75 h

(b) 6.00 h

(a) (c) 3.50 h

(d) 3.00 h

56. A unit cell of calcium fluoride has four calcium


ions. The number of fluoride ions in the unit cell
is
(b)
(a)

(b) 4

(c) 6

(d) 8

57. The equilibrium constant of a 2 electron redox


(c) reaction at 298 K is 3.8 × 10–3. The cell potential
Eº (in V) and the free energy change Gº (in kJ
mol–1) for this equilibrium respectively, are

(a) – 0.071, –13.8

(d) (b) – 0.071, 13.8

(c) 0.71, –13.8

(d) 0.071, –13.8

58. The number of stereoisomer possible for the


following compound is CH3–CH=CH–CH(Br)–
CH2–CH3
53. The compounds contaning sp hybridized carbon
atom are (a) 2

(b) 3
(i) (ii) (c) 4

(d) 8
(iii)H3C–CN (iv) H2C=C=CHCH3
59. In the radioactive disintegration
(a) (i) and (ii) (b) (iii) and (iv)
208
series 232
90 Th 82 Pb, involving and decay,
(c) (ii) and (iii) (d) (i) and (iv)
the total number of and particles emitted are
54. Upon heating with acidic KMnO 4 an organic
compound produces hexan-1,6-dioic acid as the (a) 6 and 6
major product the starting compound is
(b) 6 and 4
(a) benzene
(c) 6 and 5
(b) cyclohexene
(d) 5 and 6
(c) 1-methylcyclohexene
(d) 2-methylcyclohexene
60. In the following compressibility factor (Z) vs (c) Galactose + galactose
pressure graph at 300 K, the compressibility of
CH4 at pressure < 200 bar deviates from ideal (d) Galactose + fructose
behaviour because 64. Which of the following statements is incorrect
regarding biological membrane ?
(a) It is composed of lipids and proteins
(b) Peripheral proteins are loosely associate with
the membrane
(c) Integral proteins span the lipid bilayer
(d) Lipids and membrane proteins do not provide
structural and functional asymmetry
65. The percentage of sunlight captured by plants is
(a) 2-10% (b) 10-20%
(c) 60-80% (d) 100%
66. The hard outer layer of pollens, named exine, is
Pressure (bar) made of
(a) The molar volume of CH4 is less than its molar (a) cellulose (b) tapetum
volume in the ideal state
(c) sporopollenin (d) pectin
(b) The molar volume of CH4 is same as that in
its ideal state 67. Insectivorous plants such as Venus fly trap catch
and digest insects in order to supplement the
(c) Intermolecular interactions between CH 4 deficiency of
molecules decresases
(a) Sulphur
(d) The molar volume of CH4 is more than its molar
volume in the ideal state (b) Nitrogen
(c) Potassium
(d) Phosphorus
61. Which of the following molecules is a primary
acceptor of CO2 in photosynthesis ? 68. Which of the following statements about
nucleosome is true ?
(a) Pyruvate
(a) It consists of only DNA
(b) 3-phosphoglycerate
(b) It is a nucleus-like structure found in
(c) Phosphoenol pyruvate prokaryotes
(d) Oxaloacetate (c) It consists of DNA and proteins
62. Which one of the following pairs of molecules (d) It consists of only histone proteins
never forms a hydrogen bond between them ?
69. Epithelial cells in animals are held by specialized
(a) Water and water junctions, one of them being “Gap junction”.
(b) Water and glucose Function of a “Gap junction” is to

(c) Water and ethanol (a) Facilitate cell-cell communication by rapid


transfer of small molecules
(d) Water and octane
(b) Cement the neighbouring cells
63. Lactase hydrolyses lactose into
(c) Stop substances from leaking
(a) Glucose + glucose
(d) Provide gaps in the tissue to facilitate uptake
(b) Glucose + galactose of small molecules across tissues
70. Which of the following statements is true about 76. In the presence of glucose and lactose,
glandular epithelium in salivary gland ? Escherichia coli utilizes glucose. However, lactose
also enters the cells because
(a) It consists of isolated single cells
(a) low level of permease is always present in
(b) It consists of mutlicellular cluster of cells
the cell
(c) Its secretions are endocrine
(b) it uses the same transporter as glucose
(d) It consists of squamous epithelial cells
(c) if diffuses through the bacterial cell membrane
71. Which one of the following ion pairs is involved
(d) it is transported through porins
in nerve impulses ?
77. Passive immunization is achieved by
(a) Na+, K+
administering
(b) Na+, Cl–
(a) Heat killed vaccines
(c) K , Cl
+ –
(b) Toxoids
(d) K , Ca
+ 2+
(c) Live attenuated vaccines
72. Which of the following hormones that controls
(d) Antibodies
blood pressure is secreted by human heart ?
78. Which of the following anions neutralize the
(a) Erythropoietin
acidic pH of the chyme that enters into the
(b) Atrial natriuretic factor duodenum from the stomach ?

(c) ACTH (a) H 2 PO 4


(d) Glucocorticoid
(b) HSO 4
73. Oxytocin and vasopressin are synthesized in

(a) Hypothalamus (c) HCO3

(b) Adrenal gland


(d) CH 3 COO
(c) Pituitary gland
79. If 14 CO 2 is added to a suspension of
(d) Ovary photosynthesizing chloroplasts, which of the
following will be the first compound to be
74. If you exhale multiple times into a conical flask
radioactive ?
containing lime water through a single inlet fixed
through a stop cork, lime water will ? (a) ATP
(a) Become cooler (b) NADPH
(b) Turn milky (c) NADH
(c) Remain unchanged (d) 3-phosphoglycerate
(d) Turn yellow 80. Which of the following species makes the largest
true flower in the world ?
75. The path of passage of stimulus when you
accidentally touch a hotplate is (a) Amorphophallus titanium
(a) Receptor Brain Muscles (b) Rafflesia arnoldii
(b) Muscles Spinal cord Receptor (c) Nelumbo nucifera
(c) Muscles Brain Receptor (d) Helianthus annuus
(d) Receptor Spinal cord Muscles
85. Let AB be a sector of a circle with centre O and

81. The remainder when the polynomial 1 + x2 + x4 + radius d, AOB = , and D be a point on
x6 + …. + x22 is divided by 1 + x + x2 + x3 + …. + x11 2
is - OA such that BD is perpendicular OA. Let E be
the midpoint of BD and F be a point on the arc
(a) 0
AB such that EF is parallel to OA. Then the ratio
(b) 2 of length of the arc AF to the length of the arc
AB is -
(c) 1 + x2 + x4 + …. + x10
(d) 2(1 + x2 + x4 + …. + x10) 1
(a)
2
82. The range of the polynomial p(x) = 4x3 – 3x as x
1 1
varies over the interval , is (b)
2 2 2

(a) [–1, 1 ] 1
(c) sin
(b) (–1, 1] 2

(c) (–1, 1)
1 1
sin sin
(d) 1 1 (d) 2
,
2 2

86. Let f(x) be a non-negative differentiable function


83. Ten ants are on the real line. At time t = 0, the
on [0, ) such that f(0) = 0 and f’(x) 2f(x) for all x
k-th ant starts at the point k2 and travelling at
> 0. Then, on [0, )
uniform speed, reaches the point (11 – k)2 at time
t = 1. The number of distinct times at which at (a) f(x) is always a constant function
least two ants are at the same location is
(b) f(x) is strictly increasing
(a) 45 (b) 11
(c) f(x) is strictly decreasing
(c) 17 (d) 9
(d) f’(x) changes sign
84. A wall is inclined to the floor at an angle of 135º.
A ladder of length is resting on the wall. As the 87. For each positive real number , let A be the set
ladder slides down, its mid-point traces an arc of of all natural numbers n such that
an ellipse. Then the area of the ellipse is |sin n 1 sin n | . Let A c be the
complement of A in the set of all natural
numbers. Then -

(a) A 1 , A 1 , A 2 are all finite sets


2 3 5

(b) A 1 is a finite set but A 1 , A 2 are infinite sets


3 2 5

(c) A c1 , A c1 , A c2 are all finite sets


2 3 5
l2
(a) (b)
4
(d) A 1 , A 2 are finite sets and A 1 is an infinite
(c) 4 (d) 2 2 3 5 2

set
88. Let f be a continuous function defined on [0, 1] (c) The ballon undergoes simple harmonic
1 1 2
such that f 2 x dx f 2 x dx . Then the He l
0 0 motion with period 2
air He g
range of
(a) has exactly two points (d) The ballon undergoes conical oscillations with

(c) is the interval [0, 1] l


period 2 air He

(b) has more than two points air He g

(d) is a singleton 92. Consider a cube of uniform charge density . The


89. Three schools send 2, 4 and 6 students, ratio of electrostatic potential at the centre of
respectively, to a summer camp. The 12 students the cube to that at one of the corners of the cube
must be accommodated in 6 rooms numbered is
1,2,3,4,5,6 in such a way that each room has
exactly 2 students and both from the same 3
(a) 2 (b)
school. The number of ways, the students can be 2
accommodated in the rooms is -
(c) 2 (d) 1
(a) 60 (b) 45
(c) 32400 (d) 2700 93. Two infinitely long wires each carrying current I
along the same direction are made into the
90. Let a be a fixed non-zero complex number geometry as shown in the figure. The magnetic
z a field at the point P is
with|a| 1 and w . Where z is a
1 az
complex number.
Then
(a) there exists a complex number z with |z| < 1
such that |w| > 1
(b) |w| > 1 for all z such that |z| < 1
(c) |w| < 1 for all z such that |z| < 1
(d) there exists z such with |z| < 1 and |w| = 1

0I 0 I 1 1
91. A light balloon filled with helium of density He (a) (b)
r r 4
is tied to a long light string of length and the
string is attached to the ground. If the balloon is
0I
displaced slightly in the horizontal direction from (c) Zero (d)
the equilibrium and released then . 2 r

The ballon undergoes simple harmonic 94. A photon of wavelength is absorbed by an


electron confined to a box of length
air l 35h / 8mc . As a result, the electron makes a
motion with period 2
air He g transition from state k = 1 to the state n.
Subsequently the electron transits from the state
The ballon undergoes simple harmonic n to the state m by emitting a photon of
wavelength ’ = 1.85 . Then
air He l
motion with period 2 (a) n = 4; m = 2 (b) n = 5; m = 3
air g
(c) n = 6; m = 4 (d) n = 3; m = 1
95. Consider two masses with m1 > m2 connected by
a light inextensible string that passes over a
pulley of radius R and moment of inertia I about
its axis of rotation. The string does not slip on
(b)
the pulley and the pulley turns without friction.
The two masses are released from rest separated
by a vertical distance 2h. When the two masses
pass each other, the speed of the masses is
proportional to

m1 m2
(a) 1
m1 m2 (c)
R2

m1 m 2 m1 m2
(b) 1
m1 m2
R2

1
m1 m2 (d)
(c) R2
m1 m2

1 97. The heat capacity of one mole an ideal is found


R2 to be CV = 3R (1 + aRT)/2 where a is a constant.
(d)
m1 m 2 The equation obeyed by this gas during a
reversible adiabatic expansion is -
96. An ideal gas is taken reversibly around the cycle (a) TV3/2e aRT constant
a-b-c-d-a as shown on the T (temperature) – S
(entrophy) diagram (b) TV3/2e3aRT/2 constant
(c) TV3/2 constant
(d) TV3/2e 2aRT/3 constant
98. If the input voltage Vi to the circuit below is given
by Vi(t) = A cos(2 ft), the output voltage is given
by Vo(t) = B cos(2 ft + ) -

The most appropriate representation of above


cycle on a U (internal energy) – V (volume) Which one of the following four graphs best depict
diagrame is the variation of vs f ?

(a) (b)
(a)
(c)

(c) (d)
(d)

102.The reagent required for the following two step


99. A glass prism has a right-triangular cross section
transformation are
ABC, with A = 90º. A ray of light parallel to the
hypotenuse BC and incident on the side AB
emerges grazing the side AC. Another ray, again
parallel to the hypotenuse BC, incident on the
side AC suffers total internal reflection at the
side AB. Which one of the following must be true
about the refractive index of the material of
the prism ?
(a) (i) HBr, benzoyl peroxide ; (ii) CH3CN
3
(a) 2 (b) 3 (b) (i) HBr, (ii) NaCN
2

3 (c) (i) Br2, (ii) NaCN


(c) (d) 2 3
2
(d) (i) NaBr, (ii) NaCN
100.A smaller cube with side b (depicted by dashed
103.In the reaction sequence
lines) is excised from a bigger uniform cube with
side a as shown below such that both cubes have
a common vertex P. Let X = a/b. If the centre of
mass of the remaining solid is at the vertex O of
smaller cube then X satisfies.

The major product X and Y, respectively, are

(a)

(b)

(a) X3 – X 2 –X – 1 = 0 (b) X 2 –X – 1 = 0
(c) X3 + X 2 –X – 1 = 0 (d) X3 – X 2 –X + 1 = 0

(c)
101.X, Y and Z in the following reaction sequence are

(d)

(a)

(b)
104.In the following reactions 108.The hybridization of the central atom and the
shape of [IO2F5]2– ion respectively, are -

(a)

X and Y, respectively, are


(a)

(b)

(b)

(c)
(c)

(d)

105.Copper (atomic mass = 63.5) crystallizers in a


FCC lattice and has density 8.93 g.cm–3. (d)
The radius of copper atom is closest to
(a) 361.6 pm (b) 511.4 pm
(c) 127.8 pm (d) 102.8 pm
106.Given the standard potentials Eº(Cu2+/Cu) and
Eº(Cu+/Cu) as 0.340 V and 0.522 V respectively,
the value of Eº(Cu2+/Cu+) is
(a) 0.364 V (b) 0.158 V
109.2.33 g of compound X (empirical formula
(c) – 0.182 V (d) – 0.316 V CoH12N4Cl3) upon treatment with excess AgNO3
solution produces 1.435 g of a white precipitate.
107. For electroplating, 1.5 amp current is passed for
The primary and secondary valences of cobalt in
250 s through 250 mL of 0.15 M solution of MSO4.
compound X, respectively, are
Only 85% of the current was utilized for
electrolysis. The molarity of MSO4 solution after [Given : Atomic mass : Co = 59, Cl = 35.5,
electrolysis is closest to [Assume that the volume Ag = 108]
of the solution remained constant]
(a) 3, 6 (b) 3, 4
(a) 0.14 (b) 0.014
(c) 2, 4 (d) 4, 3
(c) 0.07 (d) 0.035
110.The specific conductance ( ) of 0.02 M aqueous
acetic acid solution at 298 K is 1.65 × 10–4 S cm–1. (b)
The degree of dissociation of acetic acid is
[Given : equivalent conductance at infinite
dilution of H+ = 349.1 S cm2 mol–1 and CH3COO–
= 40.9 S cm2 mol–1]
(a) 0.021 (b) 0.21
(c) 0.012 (d) 0.12

111.Match the following organelles in Group I with (c)


the structures in Group II. Choose the correct
combination.
Group I Group II
P . Mitochondrion i. Cisternae
Q. Golgi ii. Cristae
R. Chloroplast iii. Thylakoids
S. Centrosome iv. Radial spokes
(d)
(a) P-ii , Q-i, R-iii, S-iv
(b) P-iii, Q-i, R-ii, S-iv
(c) P-iv, Q-i, R-ii, S-iii
(d) P-iv, Q-ii, R-i, S-iii

112.A human population containing 200 individuals


has two alleles at the ‘T’ locus, named T and t.
T, which produces tall individuals, is dominant 114. Which of the following best describes the DNA
over t, which produces short individuals. If the content and the number of chromosomes at the
population has 90 TT, 40 Tt and 70 tt genotypes, end of S and M phases of the cell cycle in mitosis,
what will be the frequencies of these two alleles if the DNA content of the cell in the beginning of
in this population ? cell cycle (G1 phase) is considered as C and the
number of chromosomes 2N ?
(a) T, 0.50 ; t, 0.50
(a) 2C and 2N for S phase ; 2C and 2N for M
(b) T, 0.55 ; t, 0.45 phase
(c) T, 0.45 ; t, 0.35 (b) 2C and N for S phase ; 2C and N for M
(d) T, 0.90 ; t, 0.10 phase

113.Which of the following graphs best describes the (c) 2C and 2N for S phase ; C and 2N for M
oxygen dissociation curve where pO2 is the partial phase
pressure of oxygen ? (d) C and N for S phase ; C and 2N for M phase
(a)
115. Study the following graph of metabolic rate of 117. An mRNA is transcribed from a DNA segment
various terrestrial mammals as a function of having the base sequence 3'-TACATGGGTCCG-
their body mass and choose the correct option 5'. What will be the correct order of binding of
below. the four amino acyl-tRNA complexes given below
during translation of this mRNA ?

(a) Animals are distributed throughout the curve


with the smaller animals towards the left and (a) a, b, c, d
progressively bigger animals towards the right
(b) b, a, c, d
(b) The smaller animals below a certain critical
mass cluster at the left end of the curve and (c) c, d, a, b
the larger animals above the critical mass (d) b, a, d, c
cluster on the right end
118. If the initial number of template DNA molecules
(c) Animals are distributed throughout the curve
in a PCR reaction is 1000, the number of product
with the larger animals towards the left and
DNA molecules at the end of 20 cycles will be
progressively smaller animals towards the
right closest to

(d) The larger animals above a certain critical (a) 103


mass cluster at the left end of the curve and
(b) 106
the smaller animals below the critical mass
cluster on the right end (c) 109
116. Match the human disorders shown in Group I (d) 1012
with the biochemical processes in Group II.
Choose the correct combination 119. The allele for black hair (b) is dominant over
brown hair (b) and the allele for brown eye (E) is
Group I Group II
dominant over blue eye (e). Out of the offsprings
P . Phenylketonuria i. Melanin synthesis obtained upon mating a black-haired and brown-
eyed individual (BbEe) with a brown-haired and
Q. Albinism ii. Conversion of
brown-eyed individual (bbEE), the ratio of brown-
Phenylalanine to haired and brown-eyed individuals to black- haired
Tyrosine and brown-eyed individuals is

R. Homocystinuria iii. Tyrosine (a) 2 : 1


degradation (b) 3 : 1
S. Argininemia iv. Methionine (c) 1 : 1
metabolism (d) 1 : 2
v. Urea Synthesis
(a) P-ii, Q-i, R-iv, S-v
(b) P-i, Q-iv, R-ii, S-v
(c) P-ii, Q-i, R-v, S-iii
(d) P-v, Q-iii, R-i, S-ii
120. In an experiment represented in the schematic below, a plant species was grown in different day and night
cycles and its photoperiodic flowering behaviour was noted. This species is a

(a) short day plant and actually measures day length to flower
(b) short day plant and actually measures night length to flower
(c) long day plant and actually measures night length to flower
(d) long day plant and actually measures day length to flower

1. (b) 2. (c) 3. (d) 4. (b) 5. (c) 6. (d) 7. (b) 8. (a) 9. (a) 10. (c)
11. (b) 12. (a) 13. (a) 14. (c) 15. (a) 16. (c) 17. (c) 18. (a) 19. (c) 20. (b)
21. (c) 22. (b) 23. (a) 24. (d) 25. (c) 26. (c) 27. (c) 28. (d) 29. (b) 30. (b)
31. (b) 32. (b) 33. (d) 34. (b) 35. (c) 36. (a) 37. (b) 38. (b) 39. (b) 40. (a)
41. (b) 42. (a) 43. (c) 44. (b) 45. (a) 46. (a) 47. (d) 48. (c) 49. (c) 50. (b)
51. (c) 52. (b) 53. (b) 54. (b) 55. (d) 56. (d) 57. (b) 58. (c) 59. (b) 60. (a)
61. (c) 62. (d) 63. (b) 64. (d) 65. (a) 66. (c) 67. (b) 68. (c) 69. (a) 70. (b)
71. (a) 72. (b) 73. (a) 74. (b) 75. (d) 76. (a) 77. (d) 78. (c) 79. (d) 80. (b)
81. (d) 82. (c) 83. (c) 84. (a) 85. (d) 86. (a) 87. (c) 88. (d) 89. (c) 90. (c)
91. (c) 92. (a) 93. (d) 94. (c) 95. (c) 96. (a) 97. (a) 98. (c) 99. (a) 100. (a)
101. (d) 102. (b) 103. (a) 104. (a) 105. (c) 106. (b) 107. (a) 108. (d) 109. (a) 110. (a)
111. (a) 112. (b) 113. (d) 114. (c) 115. (a) 116. (a) 117. (d) 118. (c) 119. (c) 120. (b)
54 cos 3 = – k
1. (b) (x, y, z) are real and x , y , z are positive real
4 4 4 Now for 3 = 0, 2 we get integral solution
numbers So we get two values of ‘k’.
5. (c) (0, 3) lies on the curve
x4 y 4 z4 1
xyz So q = 3
4
(xyz) |xyz| dy
Now 2x p
i.e., xyz > 0 dx
So it holds equality
dy
x4 = y4 = z4 = 1 ; But xyz > 0 Now at (0, 3)
dx
(x, y, z) {(1, 1, 1), (1, –1, –1), (–1, 1, –1),
(–1, –1, 1)} dy
So no. of triplets (x, y, z) is 4. p 1
dx 0, 3
2. (c) P(sin2x) = P(cos2x)
P(sin2x) = P(1 – sin2x) p + q = –1 + 3 = 2
P(x) = P(1 – x) x [0, 1] 6. (d)
differentiating both sides w.r.t. 'x', we get
P'(x) = – P'(1 – x)

So P'(x) is symmetric about point x = 1


2

So P'(x) has highest odd degree


P(x) has highest even degree
3. (d) e i rn
is always a finite set when r is a rational
1
and is infinite when r = .

4. (b) Let f(x) = x3 – 27x


f '(x) = 3x2 – 27 = 3(x2 – 9)

1
MBD =
3
7. (b)

As sum of the roots is zero, so if two roots are


integer then 3rd root has to be integer Now
putting x = 6t, we get
216 t3 – 27 × 6t + k = 0
54 (4t3 – 3t) + k = 0
Put t = cos Curve, S : (y – k)2 = 4(x – h)
LLR = 4 ; Clearly k = 1 ; A(h, 1) & ‘M’ is
h r
focus (h + 1, 1) Now, tan 60° =
50 r
So D (h +1, 3)
S(0, 0) = 0 k2 = – 4h 3 50 r h r

1 3
h= D ,3 50 50
4 4 3 50 r r h
3 3

8
2 100 = 3 3 r
m1 m 2 3 2
Now ; tan =
1 m1 m 2 8 19
1 2 100
3 r
3 3

3 1 2 8 100 3 3
m1 = r
3 3 3 3 3 3 3
0
4
100 3 3
r
3 1 6
and m2 = 2
1
1
9. (a) cos (sin x) = sin (cos x) r 50 1
3

sin ( sin x ) = sin (cos x) 11. (b) lim


n
fn (x) f (f (f (......... times (x))
2
1
cos x = n + (–1)n ( sin x ), n I Now for x1 0,
2 2

n f(x1) > x1 as f(x) is concave - downward


cos x sin x n 1 ,n I
2
1
As LHS [ Thus fn as n
2, 2 , and it does not satisfies 2
RHS 1
Similarly for x1 ,1
So no solution possible. 2

10. (c) f(x1) < x1 as f(x) is concave upward


1
Thus fn as n
2
x 3
x3
x2 et dt
12. (a) lim 0
3
x
ex
Apply L Hospital
x 3 3
x 2 ex
2x 2 et
0
= lim 3
x
3x 2 e x
Let DE = BC = r x 3
xe x
3

2 et
0
h lim 3

tan 30° =
x
3xe x
50
3 3 3
2e x ex 3x 3 e x
lim
50 x
3e x
3
9x 3 e x
3

h
3
3 3x 3 1
lim
x 3 9x 3 3
13. (a) f '(x) = 3x2 – 6ax + 27a2 + 9 21. (c) A If stream lines intersect then there will
= 3[x2 – 2ax + 9a2 + 3] = 3((x – a)2 + 8a2 + 3) be two direction of fluid flow at a point, which
f '(x) is + ve for x R f(x) is monotonicaly is absurd.
increase B Lines of forces in electrostatic never
for x R. intersect
1 3 1 D Line of force in magnetism never
14. (c) A = f x dx f x dx x3 4x 2 3x dx intersect each other.
0 1 0
22. (b) radius = R, speed = V, = 30°
3
3 2
x 4x 3x dx 2
1 1 V
1 mV 2 mR 2 = mgh
2 2 R
1
x 4 4x3 3x 2 x 4 4x 3 3x 2 37
{Using conservation of energy)
4 3 2 0 4 3 2 12
15. (a) f(x) is always positive for x [0, 1] V2 V2
m mgh
1 1 2 2
f(x) < x2 f x dx x
0 0
V2
1 height (h)
I g
3
23. (a) In sudden expansion gas from volume V to
1
But it is given that I (which is not 2V do not get enough time for exchange of
3 heat.
possible)
17. (c) V is the circumcentre of ABC Process is adiabatic.

A (1, 0), B (0, 1), C(2, 0) mv


24. (d) Radius of particle R =
Let V(x, y) qB
VA = VB = VC
(x – 1)2 + y2 = x2 + (y – 1)2 = (x – 2)2 + y2 qBR eBR
Velocity of particye V
m me
3 3
(x, y) = ,
2 2 hc
=KEmax
3i 3 j
V
2 (Einstein photo electric equation)
3
|v| 2, 3 hc
2 = – KEmax

b2 b2
18. (a) P(b2) = P(b1) P + P(R1) P R 2
b1 1
hc 1 eBR
me
b b 1 r b 2 me
b r b r 1 b r b r 1
2
hc eBR
b(b 1) br b b 1 r b 2m e
2m e
(b r)(b r 1) (b r)(b r 1) b r
25. (c) As dimension of hole is very small than mean
20. (b) We can cover the entire xy-plane plane using
path, then at equilibrium effusion rate of gas
squares definitely using equilateral triangle.
in both direction must be equal.
Also regular hexagon is made of equilateral
triangles. While pentagon cannot cover the
plane because of its shape.
27. (c) Potential

x4 x2
U
4 2
at t = 0, x = 0.5
TI = 150 K, TII = 300 K 1 1 1 1
P1 P2 4 16 4 2
For this
T1 T2 1
T 4
Mean free path
P
du 4x 3 2x
T1 P2 x3 x
1 dx 4 2
2 T2 P1
du
x x2 1
T1 T2 dx
T2 T1
du
0 at point of maxima & minima
dx
T1 150
1
0.7
2 T2 300 x=0;x=±1

26. (c) F = IBLsin d2u


= – 1 point of maxima
dx 2
iBl x 0
a ( f = ma)
m
d2u
= 2 point of minima
dx 2
d dx x 1
B.l.
dt dt
E V
BlV
i E/ R
R

BlV Bl
a
R m
particle will found between (–1, 0)
B2l2
a .V 28. (d) Blood pressure is gauge pressure = 190 mm
Rm
Hg Atmospheric pressure = 760 mm Hg
dv Actual pressure = 190 + 760 mm Hg = 950
a V.
dx mm Hg = 1.25 × 760 mm Hg

dV B2l2
V. .V 29. (b)
dx Rm

B2 l 2
dv . dx
Rm

B2 l 2 vRM
v .' X ' X
Rm B' l2
h
Momentum of photon = P =

E
E = PC = C = 3 × 108 m/s
P

37. (b)

AB = Direction of resultant velocity

AD = Direction of tangential velocity

dr r
tan =
rd r
tan =1 S and SI are source of YDSE
= 45º = 0.5 × 10–3 radian (very small)
30. (b) In young's double silt experiment D = SO cos + 100
D = 20 × 1 + 100
= = 120 cm
d
d = 2 × SO sin
h h 2 × 20 ×
mV 2mq V 40 × 0.5 × 10–3 cm
2 × 10–2 cm
1
V D 440 10 6 120 102
=
as V is double d 2 10 2 102
264 × 10–2
1
is times of 2.64 mm
2 old

38. (b) Radius of cylinder


= 0.7
new
r = 4 mm = 0.20 m
= 0.7w
31. (b) Electric field lines should be perpendicular
to surface of metal.
32. (b) When e– collide with atom which is massive in
comparison to e–. Max possible loss of KE = KE
of e– (initial KE) = E if this E is less than min dm d
excitation energy then collision is elastic S T (S = specific heat)
dt dt
E < 10.2 eV (Minimum excitation energy)
33. (d) Continuous X Ray is inverse of photoelectric d
= 5 × 108 × volume of rod
effect dt
34. (b) Thermal expansion of a solid is due to
0.2
asymmetric characteristic of inter atomic = 5 × 108 × × (4)2 × 10–6 ×
potential energy curve of the solid. 10

35. (c) Wavelength of electron and proton is , then = 5 × 10 × × 16 × 2


= 1600
hc
Energy of photon = E = 0.2 × 4 × 103 T = 1600
8 × 102 T = 16 × 102 Solve these equation
T = 3.14 × 2
I 1 I 1
6.28ºC
0 1 0 2
0
2 x2 2 4 x
2

39. (b) Intensity of sound


I1 I2
x2 4 x
2

2
I1 x
I2 4 x
2
I1 1
I2 4 1
Ratio of intensities
at 5 KHz Hearing capacity = 40 dB I1 9
Intensity at 1 KHz I2 1

I
= 10 log I
0 Cl – S – Cl
41. (b)
Cl Cl
10
I I0 10
43. (c) Cis – shows optical activity

I I0 10
20/10
I0 10
2 44. (b) Acidic strength increases with increase in
1KHZ
oxidation number of central atom.
40/10 4
I 5KHZ
I0 10 I0 10 HClO4 HClO3 HClO2 HOCl
45. (a) F.C.C. > B.C.C. > S.C.
I 1
1KHZ
No. of atoms 4>2>1
I5KHZ 100

40. (a) Two long wire carrying current I1 and I2 3 R 50


u r.m.s. H 2 2 1.18
46. (a) u r.m.s. O2 3 R 500
28

48. (c) NH3 is strong base and BCl3 is strong Acid.


49. (c) It produces D-Gluconic Acid

+ –
N2HSO4

50. (b) H3PO2 + H2O


+ N2 + H2SO4

CH3 CH3
I I AlCl3
Bp 0 1 0 2
51. (c) + CH3 – CH – CH2Br
2 x 2 4 x
CH3
C
dBp
= 0 for minima of B CH3 CH3
dx CH3
62. (d) Hydrogen bond is a weak bond between two
molecules resulting from an electrostatic
52. (b) CH3 OH
(CH3CO)2O attraction between a proton in one molecule
in CH3COOH and an electronegative atom in another
COOH molecule. This is not possible in case of water
O and octane

CH3 O – C – CH3 63. (b) Lactose Lactase Glucose + Galactose


64. (d) Proteins provide asymmetry to plasma
COOH membrane, as they are of 2 types i.e.
peripheral and integral.
65. (a) Plants capture 2-10 % of PAR
53. (b) CH3 C N and CH2 C CH CH3 66. (c) Sporopollenin forms the exine of pollen grain
SP SP which is resistant to acids, high temperature
and radiations.
Acid COOH 67. (b) Insectivorous plants grow in Nitrogen
54. (b) deficient soils and in order to compensate the
KMnO4 COOH
deficiency they catch and digest insects and
obtain N from chitin (NAG).
2.303 a 0.693
55. (d) t 87.5 log , Also K 68. (c) Nucleosome is the smallest unit of DNA
K a x 1
packaging containing 200 nitrogen bases and
four types of histone proteins i.e. H2A, H2B,
2.303 a
t 87.5 log H3 and H 4. H 1 type of histone is used in
0.693 a 87.5% of a
plugging.
69. (a) Gap junctions are cytoplasmic
2.303 a communications between two epithelial cells
log
0.693 875 for rapid transfer of some ions and small
a a
1000 molecule
70. (b) Salivary gland is multicellular exocrine
2.303 2.303 gland, made up cuboidal epithelium.
log 8 3 0.3010 3
0.693 0.693 71. (a) Na + - K + pump is essential for impulse
56. (d) For CaF2 Coordination no. is 8 : 4 conduction
72. (b) ANF [Anti-Natriuretic Factor] is antagonist
57. (b) G0 = –2.303RTlogK
of Renin Hormone and act as Vasodilator to
= –2.303 × 8.314 × 298log(3.8 × 10–3) reduce blood pressure. ANF is secreted from
= +13809.3876 J = +13.8 kJ heart.
Now G0 = –nFE0 73. (a) Oxytocin and Vasopressin synthesised in
hypothalamus then comes into
+ 13809.3876 J = –2 × 96500 × E0
neurohypophysis of pituitary gland to release
E0Cell = –0.071 in blood
58. (c) No. of isomers = 22 = 4 [Given answer by KVPY is C]

59. (b) 232 208 74. (b) Exhaled air has CO2 in it.
90 Th 82 Pb x 42 He y 01
Ca(OH)2 + CO2 CaCO3 + H2 O
232 = 208 + 4x
x=6 Lime water Exhaled air Milky white ppt

and 90 = 82 + 2x – y So lime water become milky white.


y=4 75. (d) Reflex arch
61. (c) In C4 plants the primary acceptor of CO2 is Receptor Sensory Neuron Spinal cord
phosphoenol pyruvate, a 3C compound. e.g. Motor neuron Muscle [Effector]
In Maize, Sugarcane etc. 76. (a) The preferred molecule is glucose first, then
lactose is used by E.coli because ultimately Now at any time distance travelled by any
lactose is also broken down into galactose ant will be
and glucose. Permease is a carrier protein S = S0 + ut
which helps in facilitated diffusion. Lac
Where S0 is the initial position
operon is always operational at low levels.
Now two ants will be at same position
77. (d) Administration of already prepared
antibodies is called passive immunisation If Si = Sj
78. (c) Duodenum receives Bile and Pancreatic juice k 2i 22 k t 121 t
alkaline in nature due to high amount of
HCO – anion which helps in neutralization = k 2j 22 k
of acidity of chyme.
79. (d) During Calvin cycle, the 14CO2 is incorporated k 2j k i2 kj ki
t ; t (as ki kj)
with a 5C compound Ribulose 1-5 22 k j ki 22
Biphosphate and forms 2 molecules of 3C
compound 3-phosphoglycerate. Now for i = 1
80. (b) Rafflesia arnoldii is the total root parasite
3 4 5 11
and forms the largest flower in angiosperms Values of t will be , , ,.........
while Amorphophallus titanium is the largest 22 22 22 22
inflorescence in the angiosperms (9 values)
for, i = 2

81. (d) P(x) = 1 + x2 + x4 + x6 + …… + x22 4 5 11 12


values of t will be , ,........ ,
22 22 22 22
= (1 + x2) (1 + x4) (1 + x4 + x8) (1 – x4 + x8)
and Q(x) = 1 + x + x2 + x3 + …… + x11 We can see that there is only 1 distinct value
= (1 + x) (1 + x2) (1 + x4 + x8) Similarly for i = 3,4,5,6,7,8,9 we get only
1 distinct value each.
P x 1 x4 1 x4 x8 So in all there 17 distinct values of ‘t’
Now, 84. (a)
Q x 1 x

1 x4 x8 x4 x8 x 12
1 x

1 x12
1 x
When (1 + x12) is divided by (1 + x) is = 2
The remainder is 2.
x x 1 x1
Now remainder P(x) divided by Q(x) Mid point (h, k) = ,
2 2
= 2(1 + x2) (1 + x4 + x8)
Now (x + x1)2 + x 2 = 2
= 2(1 + x2 + ……. + x10)
As 2h + 4k = x + x1, 2k = x,
82. (c) P'(x) = 12x2 – 3 = 3 (4x2 – 1)
So required locus is
1 1 4(h + 2k)2 + 4k2 = 2
In , P'(x) < 0
2 2 2

P(x) is decreasing h2 + 5k2 + 4hk =


4
Range (P(–1), P(1)) 2

Range (–1, 1) x2 + 5y2 + 4xy =


4
83. (c) Velocity of any ant U = (11 – k)2 – k2 2
= 121 – 22 k Whose area is
4
For small angular displacement ( )
85. (d)
0
=V( Air
– He
)g
I =V[ Air
– He
]g
He
V 2
=V[ Air
– He
] g

Air He g
He l

Air He g
He l
CF = r(1 – cos sec )
EC = r(sin – cos tan )
CD = r cos tan l He
T 2
EC + CD = ED g Air He

r sin 92. (a) Let consider a cube with charge density


r sin =
2

sin
= sin–1
2
88. (d) By Cauchy Schwarz inequality

2
a
b b 2 b 2
f 2 x g x dx f x dx g x dx
a a a
Let at the corner of cube potential = V0
Here g(x) = 1
Q
Potential
f x Side of cube
and equality holds only when g x
Q= × a3

So, f(x) is constant a3


So potential
a
4 6
89. (c) Required no. of ways = 6
2
2
2 2
3
3 Potential a2

= 32400
91. (c)

Big cube consist of 8 cube


At centre of big cube of side 2a, potential is
8V0
Potential at corner of big cube = V 0 × (2)2
= 4V0

0
=V( Air
– He
)g sin 8V0
Required ratio = =2:1
4V0
93. (d) From the sign we can draw FBD
m0 35
2 2 2 2

m0 35 = 5.8 6
i.e. e– get excite to state 6.
n=6
95. (c) Two masses M1 and M2

(B)0 = (B)wire AB + (B)BC Arc


+ B CD wire + (B)PQ wire
+ B QR (Arc) + B RS wire

BPQ = BRS = 0
B BC = – B QR
(B)wire AB = B CD
The total mechanical energy of system
Bnet= (B) AB + (B)wire CD = conserved
wire
Hence
0I 0I 2 0I KEi + PEi = KEf + PEf
4 r 4 r 4 r
1 1
0 – m2g × 2h = m 2v + m 1v + 2
2 2
0I
Bnet =
2 r – m1gh – m2gh
94. (c) Box of length L v
Also =
R
2
1 1 v
(m1 – m2)gh = (m1 + m2)v2 + I
2 2 R

m1 m2
v
1
Wavelength ' = 1.85
m1 m2
R2
hc 96. (a)
KE of e– =

h h
de broglie
2mKE hc
2m

h
2mc bc Isothermal process so U remain
constant
m0 debroglie
l cd Isentropic process so S remain constant
2
m0 h 35h
2 2mc 8mc
bc should be straight line parallel to V & cd
B lag behind
graph should be
At higher frequency become – ve.
99. (a)

97. (a) Adiabatic process TV – 1


=C

2
=1+
f
2
TV f C

fR 3R 1 aRT
Cv
2 2

fR 3 Re aRT
2 2

2 2
f 3e aRT
2
aRT
TV 3e C

3eaRT
TV 2
C
3
Ans. given is TV 2 eaRT
So no option is matching may be due to
printing mistake. r+ = 90º …..(1)
C

98. (c) 1 × sin (90º – ) = sin r


cos = sin r …..(2)
90º – e > C
…..(3)
sin e = 1 × sin …..(4)
(3) & (4)
90º – C
>e
cos C
> sin e

sin
cos C
>
Resultant of VC, VR & B give Vi and angle
between Vi & B is . When f is very high XC
O then VC O Resultant of VC, VR & B 1
1 – sin2 C
> 2 [1 – 2
sin2 r]
lie between B and VR.
We will consider removed mass as a negative
1 1 2 2 mass
1 2 2
1 sin 90 c

a4 a4
1 1 b 2 2
1 2 2
cos2 c a3 b3

1 1 a4 b4
1 1 a3b b4
2 2 2 2
2a3b – 2b4 = a4 – b4
3 put a = bx 2b4x3b – 2b4 = b4x4 – b4
2 2
2x3 – 1 = = x4
2x3 – 2 + 1 = x4
3
2[x3 – 1] = (x2 – 1) (x2 + 1)
2
2[x – 1][x2 + 1 + x] = [x – 1][x + 1][x2 + 1]
(1) & (2)
2x2 + 2 + 2x = x3 + x + x2 + 1
cos = sin (90º – C
)
cos = cos x3 – x 2 – x – 1 = 0
C

cos <1
cos C
<1 CH3 CH3
CH3
1 1 CH CH3 – C – OOH
1 2
+
O2 H2O/H
101. (d) Air Aln.
1 1
1 2 2 (X)

OH
2
1 2

+ CH3COCH3
2
(Y) (Z)

3
2
2 Br CN
100. (a) Centre of mass of remaining cube x coordinate CH = CH2 CH – CH3 CH – CH3
=bx
HBr NaCN
102. (b)

CHO COO COOH


+
OH H
103. (a)
CHO CH2OH CH2OH

O
–H2O
a b
a3 b3
X CM 2 2
a3 b3
3 d Na 8.93 6.023 10 23 1000 K 1000 1.65 10 4

105. (c) a 11 a) m
Z M 4 63.5 M 0.2

24
1/ 3 = 8.25
a 47.2 10
m 349.1 40.9 390
10
a 3.61 10 m 361Pm
8.25
Now, a 2 2r 0.0211
390
361
r 127.8
2 2

106. (b) Cu 2 e Cu E10 ?


0
Cu e Cu E 2 0.522 V
Cu 2 2e Cu E 0
3 0.340 V
220
400
0 n1 E10 n 2 E02 2 0.34 0.522
Now E1 180
n3 1
400
= 0.158 V
10 (a) Remainng moles
= initial moles – moles deposited

250 0.15
initial moles = 0.0375
1000
moles deposited,

w It 1.5 250 0.85


M 96500n 96500 2
= 0.00165 GGU
Remaining moles = 0.03585

0.03585
Molarity = = 0.143
0.25
×
108. (d) Pentagonal bipyramidal sp3d3

109. (a) X AgNO3 AgX

2.33
Moles of X = 0.01
233.5

So, complex is Co NH 3 4
Cl 2 Cl

Oxi. No. = 3, Coordination No. of Co = 6


7. The number of real numbers for which the
equality
1. The number of ordered pairs (x, y) of real numbers
sin cos
that satisfy the simultaneous equations x + y2 = = – 1,
x2 + y = 12 is sin cos
holds for all real which are not integral multiples
(a) 0 (b) 1
of /2 is -
(c) 2 (d) 4
(a) 1 (b) 2
2. If z is a complex number satisfying
(c) 3 (d) Infinite
|z3 + z–3| 2, then the maximum possible value
of |z + z–1| is - 8. Suppose ABCDEF is a hexagon such that AB =
BC = CD = 1 and DE = EF = FA = 2. If the vertices
(a) 2 (b) 3
2 A, B, C, D, E, F are concylic, the radius of the
circle passing through them is -
(c) 2 2 (d) 1
3. The largest perfect square that divides 5 7
(a) (b)
20143 – 20133 + 20123 – 20113 + .... + 23 – 13 is - 2 3
(a) 12 (b) 22 11
(c) (d) 2
(c) 1007 2
(d) 2014 2
5
4. Suppose OABC is a rectangle in the xy-plane 9. Let p(x) be a polynomial such that p(x) – p’(x) = xn,
where O is the origin and A, B lie on the parabola where n is a positive integer. Then p(0) equals -
y = x2. Then C must lie on the curve - (a) n! (b) (n – 1)!
(a) y = x + 22
(b) y = 2x + 1 2
1
1
(c) y = – x + 2 2
(d) y = – 2x2 + 1 (c) (d) n 1!
n!
5. Circles C1 and C2, of radii r and R respectively, 6 / x2
touch each other as shown in the figure. The x
10. The value of the lim is -
line , which is parallel to the line joining x 0 sin x
the centres of C1 and C2, is tangent to C1 at P (a) e (b) e–1
and intersects C2 at A, B. If R2 = 2r2, then AOB
(c) e–1/6 (d) e6
equals -
11. Among all sectors of a fixed perimeter, choose
the one with maximum area. Then the angle at
1 the center of this sector (i.e., the angle between
(a) 22
2 the bounding radii) is -

(b) 45º 3
(a) (b)
3 2
(c) 60º
(c) 3 (d) 2
1 12. Define a function f : R R by f(x) = max {|x|,
(d) 67
2 |x – 1|, ... , |x – 2n|} where n is a fixed natural
6. The shortest distance from the origin to a variable 2n

point on the sphere (x – 2)2 + (y – 3)2 + (z – 6)2 = 1 number. Then f x dx is -


0
is -
(a) n (b) n2
(a) 5 (b) 6
(c) 3n (d) 3n2
(c) 7 (d) 8
13. If p(x) is a cubic polynomial with p(1) = 3, p(0) = 2 18. If log(3x – 1)(x – 2) = log (9x2 –6x 1)
(2x2 – 10x – 2), then
x equals -
1
and p(–1) = 4, then p x dx is - (a) 9 15 (b) 3 15
1

(a) 2 (b) 3 (c) 2 5 (d) 6 5


(c) 4 (d) 5 19. Suppose a, b, c are positive integers such that
14. Let x > 0 be a fixed real number. Then the integral a 2b 3c
2a + 4b + 8c = 328. Then is equal to -
abc
e 1
|x – t| dt is equal to -
1
0 (a)
2
(a) x + 2e–x – 1 (b) x – 2e–x + 1
5
(c) x + 2e–x + 1 (d) – x – 2e–x + 1 (b)
8
15. An urn contains marbles of four colours : red,
white, blue and green. When four marbles are 17
(c)
drawn without replacement, the following events 24
are equally likely :
5
(1) the selection of four red marbles (d)
6
(2) the selection of one white and three red 20. The sides of a right-angled triangle are integers.
marbles The length of one of the sides is 12. The largest
(3) the selection of one white, one blue and two possible radius of the incircle of such a triangle
red marbles is -
(4) the selection of one marble of each colour (a) 2 (b) 3
The smallest total number of marbles satisfying (c) 4 (d) 5
the given condition is
(a) 19 (b) 21
(c) 46 (d) 69 21. A small box resting on one edge of the table is
16. There are 6 boxes labelled B1, B2, ..... , B6. In each struck in such a way that it slides off the other
trial, two fair dice D1, D2 are thrown. If D1 shows edge, 1 m away, after 2 seconds. The coefficient
j and D2 shows k, then j balls are put into the box of kinetic friction between the box and the table-
Bk. After n trials, what is the probability that B1 (a) must be less than 0.05
contains at most one ball ? (b) must be exactly zero
5n 1
5n 1 (c) must be more than 0.05
(a) (d) must be exactly 0.05
6n 1
6n 6
22. Carbon-II decays to boron-II according to the
5n 5n 1
1 following formula.
(b)
6n 6n 1
6 12
6
C 11
5
B + e+ + e
+ 0.96 MeV
n n 1
5 5 1 Assume that positrons (e+) produced in the decay
(c) n n
6n 6 1
6 combine with free electrons in the atmosphere
and annihilate each other almost immediately.
5n 5n 1
1 Also assume that the neutrinos ( e) are massless
(d) n
6n 6n 1
62 and do not interact with the environment. At t =
0 we have 1 g of 126C . If the half-life of the decay
17. Let a 6i 3 j 6k and d i j k. Suppose process is t0, the net energy produced between
time t = 0 and t = 2t0 will be nearly -
that a b c where b is parallel to d and c is
(a) 8 × 1018 MeV
perpendicular to d . Then c is -
(b) 8 × 1016 MeV
(a) 5i 4j k (b) 7i 2 j 5k (c) 4 × 1018 MeV
(d) 4 × 1016 MeV
(c) 7i 5j k (d) 3i 6 j 9k
23. Two uniform plates of the same thickness and 26. A boy is standing on top of a tower of height 85 m
area but of different materials, one shaped like and throws a ball in the vertically upward
an isosceles triangle and the other shaped like a direction with a certain speed. If 5.25 seconds later
rectangle are joined together to form a composite he hears the ball hitting the ground, then the
body as shown in the figure. If the center of mass speed with which the boy threw the ball is (take
of the composite body is located at the midpoint g = 10 m/s2, speed of sound in air = 340 m/s)
of their common side, the ratio between masses (a) 6 m/s (b) 8 m/s
of the triangle to that of the rectangle is -
(c) 10 m/s (d) 12 m/s
27. For a diode connected in parallel with a resistor,
which is the most likely current (I) – voltage (V)
characteristic ?

(a) 1 : 1 (b) 4 : 3
(c) 3 : 4 (d) 2 : 1
24. Two spherical objects each of radii R and masses
m1 and m2 are suspended using two strings of
equal length L as shown in the figure (R << L).
The angle, which mass m 2 makes with the
vertical is approximately -

(a) (b)

(c) (d)
m1 R 2m1 R
(a) m m2 L (b) m1 m2 L
1
28. A beam of monoenergetic electrons, which have
2m 2 R m2R been accelerated from rest by a potential U, is
(c) m m2 L (d) m1 m2 L used to form an interference pattern in a Young’s
1
Double slit experiment. The electrons are now
25. A horizontal disk of moment of inertia 4.25 kg-m2
accelerated by potential 4U. The fringe width -
with respect to its axis of symmetry is spinning
counter clockwise at 15 revolutions per second (a) remains the same
about its axis, as viewed from above. A second (c) is twice the original fringe width
disk of moment of inertia 1.80 kg-m2 with respect (b) is half the original fringe width
to its axis of symmetry is spinning clockwise at
(d) is one-fourth the original fringe width
25 revolutions per second as viewed from above
about the same axis and is dropped on top of the 29. A point charge Q(= 3 × 10–12C) rotates uniformly
first disk. The two disks stick together and rotate in a vertical circle of radius R =1 mm. The axis
as one about their axis of symmetry. The new of the circle is aligned along the magnetic axis of
angular velocity of the system as viewed from the earth. At what value of the angular speed ,
above is close to - the effective magnetic field at the center of the
circle will be reduced to zero ? (Horizontal
(a) 18 revolutions/second and clockwise
component of Earth’s magnetic field is 30 micro
(b) 18 revolutions/second and counter clockwise Tesla)
(c) 3 revolutions/second and clockwise (a) 1011 rad/s (b) 109 rad/s
(d) 3 revolutions/second and counter clockwise (c) 1013 rad/s (d) 107 rad/s
30. A closed bottle containing water at 30ºC is open 34. The state of an ideal gas was changed isobarically.
on the surface of the moon. Then - The graph depicts three such isobaric lines. Which
(a) the water will boil of the following is true about the pressures of the
gas ?
(b) the water will come as a spherical ball
(c) the water will freeze
(d) the water will decompose into hydrogen and
oxygen
31. A simple pendulum of length is made to oscillate
with an amplitude of 45 degrees. The acceleration
due to gravity is g. Let T0 = 2 l / g. The time
(a) P1=P2=P3 (b) P1>P2>P3
period of oscillation of this pendulum will be -
(c) P1<P2<P3 (d) P1/P2=P3/P1
(a) T0 irrespective of the amplitude
35. A metallic ring of radius a and resistance R is
(b) slightly less than T0 held fixed with its axis along a spatially uniform
(c) slightly more than T0 magnetic field whose magnitude is B0 sin ( t).
(d) dependent on whether it swings in a plane Neglect gravity. Then,
aligned with the north-south or east-west (a) the current in the ring oscillates with a
directions frequency of 2 .
32. An ac voltmeter connected between points A and (b) the joule hearting loss in the ring is
B in the circuit below reads 36 V. If it is connected proportional to a2,
between A and C, the reading is 39 V. The reading (c) the force per unit length on the ring will be
when it is connected between B and D is 25 V. proportional to B02 .
What will the voltmeter read when it is connected
between A and D ? (Assume that the voltmeter (d) the net force on the ring is non-zero
reads true rms voltage values and that the source 36. The dimensions of the area A of a black hole can
generates a pure ac) be written in terms of the universal gravitational
constant G, its mass M and the speed of light c
as A = G M c . Here -
(a) = –2, = –2, and = 4
(b) = 2, = 2, and = –4
(c) = 3, = 3, and = –2
(a) (b) 31V (d) = –3, = –3, and = 2
481 V
37. A 160 watt infrared source is radiating light of
(c) 61 V (d) 3361 V wavelength 50000Å uniformly in all directions.
33. A donor atom in a semiconductor has a loosely The photon flux at a distance of 1.l8 m is of the
bound electron. The orbit of this electron is order of -
considerably affected by the semiconductor (a) 10 m–2s–1 (b) 1010m–2s–1
material but behaves in many ways like an (c) 10 m s
15 –2 –1
(d) 1020m–2s–1
electron orbiting a hydrogen nucleus. Given that 38. A wire bent in the shape of a regular n-polygonal
the electron has an effective mass of 0.07 me, loop carries a steady current I. Let be the
(where me is mass of the free electron) and the perpendicular distance of a given segment and R
space in which it moves has a permittivity 13 0, be the distance of vertex both from the centre of
then the radius of the electron’s lowermost energy the loop. The magnitude of the magnetic field at
orbit will be close to (The Bohr radius of the the centre of the loop is given by -
hydrogen atom is 0.53 Å)
n 0I n 0I
(a) 0.53 Å (a) sin /n (b) sin /n
2 l 2 R
(b) 243 Å
(c) 10 Å n 0I n 0I
(c) cos /n (d) cos /n
(d) 100 Å 2 l 2 R
39. The intensity of sound during the festival season 46. The standard reduction potentials (in V) of a few
increased by 100 times. This could imply a decibel metal ion/metal electrodes are given below. Cr3+/
level rise from - Cr = –0.74; Cu2+/Cu = +0.34; Pb2+/Pb = –0.13; Ag+/
(a) 20 to 120 dB (b) 70 to 72 dB Ag = +0.8. The reducing strength of the metals
follows the order
(c) 100 to 10000 dB (d) 80 to 100 dB
(a) Ag > Cu > Pb > Cr
40. One end of a slack wire (Young’s modulus Y,
length L and cross-section area A) is clamped to (b) Cr > Pb > Cu > Ag
rigid wall and the other end to a block (mass m) (c) Pb > Cr > Ag > Cu
which rests on a smooth horizontal plane. The (d) Cr > Ag > Cu > Pb
block is set in motion with a speed . What is the
47. Which of the following molecules can exhibit
maximum distance the block will travel after the
optical activity?
wire becomes taut ?
(a) 1-bromopropane (b) 2-bromobutane
mL 2mL (c) 3-bromopentane (d) bromocyclohexane
(a) v (b) v
AY AY
48. The structure of the polymer obtained by the
mL mv following reaction is
(c) v (d) L
2AY AY

41. The Lewis acid strength of BBr3, BCl3 and BF3 is


in the order
(a) BBr3<BCl3<BF3 (b) BCl3<BF3<BBr3
(c) BF3<BCl3<BBr3 (d) BBr3<BF3<BCl3
42. O2– is isoelectronic with
(a) Zn2+ (b) Mg2+
(c) K+ (d) Ni2+
43. The H-C-H, H-N-H, and H-O-H bond angles (in
degrees) in methane, ammonia and water are
respectively, closest to
(a) 109.5, 104.5, 107.1
(a) I (b) II
(b) 109.5, 107.1, 104.5
(c) III (d) IV
(c) 104.5, 107.1, 109.5
49. The major product of the reaction between
(d) 107.1, 104.5, 109.5
CH3CH2ONa and (CH3)3CCl in ethanol is
44. In alkaline medium, the reaction of hydrogen
(a) CH3CH2OC(CH3)3 (b) CH2 = C(CH3)2
peroxide with potassium permanganate produces
a compound in which the oxidation state of Mn is (c) CH3CH2C(CH3)3 (d) CH3CH=CHCH3
(a) 0 50. When H2S gas is passed through a hot acidic
aqueous solution containing Al3+, Cu2+, Pb2+ and
(b) +2
Ni2+, a precipitate is formed which consists of
(c) +3
(a) CuS and Al2S3 (b) PbS and NiS
(d) +4
(c) CuS and NiS (d) PbS and CuS
45. The rate constant of a chemical reaction at a very
51. The electronic configuration of an element with
high temperature will approach
the largest difference between the 1st and 2nd
(a) Arrhenius frequency factor divided by the ideal ionization energies is
gas constant
(a) 1s2 2s2 2p6
(b) activation energy
(b) 1s2 2s2 2p6 3s1
(c) Arrhenius frequency factor
(c) 1s2 2s2 2p6 3s2
(d) activation energy divided by the ideal gas
(d) 1s2 2s2 2p1
constant
52. The order of electronegativity of carbon in sp, sp2 59. Reaction of aniline with NaNO2 + dil. HCl at 0 ºC
and sp3 hybridized states follows followed by reaction with CuCN yields
(a) sp > sp2 > sp3 (b) sp3 > sp2 > sp
(c) sp > sp3 > sp2 (d) sp2 > sp > sp3
53. The most abundant transition metal in human
body is
(a) copper (b) iron
(c) zinc (d) manganese
54. The molar conductivities of HCl, NaCl,
CH3COOH, and CH3COONa at infinite dilution
follow the order
(a) HCl > CH3COOH > NaCl > CH3COONa
(b) CH3COONa > HCl > NaCl > CH3COOH
(c) HCl > NaCl > CH3COOH > CH3COONa (a) I (b) II
(d) CH3COOH > CH3COONa > HCl > NaCl (c) III (d) IV
55. The spin only magnetic moment of [ZCl4]2– is 3.87 60. Schottky defect in a crystal arises due to
BM where Z is (a) creation of equal number of cation and anion
(a) Mn (b) Ni vacancies
(c) Co (d) Cu (b) creation of unequal number of cation and anion
56. If –D–glucose is dissolved in water and kept for vacancies
a few hours, the major constituent(s) present in (c) migration of cations to interstitial voids
the solution is (are) (d) migration of anions to interstitial voids
(a) –D-glucose
(b) mixture of –D-glucose and open chain
D-glucose 61. Immunosuppressive drugs like cyclosporin delay
(c) open chain D-glucose the rejection of graft post organ transplantation
(d) mixture of –D-glucose and –D-glucose by

57. The pH of 1N aqueous solutions of HCl, CH3COOH (a) inhibiting T cell infiltration
and HCOOH follows the order (b) killing B cells
(a) HCl > HCOOH > CH3COOH (c) killing macrophages
(b) HCl = HCOOH > CH3COOH (d) killing dendrite cells
(c) CH3COOH > HCOOH > HCl 62. Which one of these substances will repress the
(d) CH3COOH = HCOOH > HCl lac operon?

58. The major product of the reaction (a) Arabinose (b) Glucose
(c) Lactose (d) Tryptophan
63. Assume a spherical mammalian cell has a
Products is
diameter of 27 microns. If a polypeptide chain with
alpha helical conformation has to stretch across
the cell, how many amino acids should it be
comprised of?
(a) 18000 (b) 1800
(c) 27000 (d) 12000
64. Which one of the following has phosphoric acid
anhydride bonds?
(a) Deoxy ribonucleic acid
(a) I (b) II (b) Ribonucleic acid
(c) III (d) IV (c) dNTPs
(d) Phospholipids
65. The two components of autonomous nervous 72. If molecular weight of a polypeptide is 15.3 kDa,
system have antagonistic actions. But in certain what would be the minimum number of
cases their effects are mutually helpful. Which of nucleotides in the mRNA that codes for this
the following statement is correct? polypeptide? Assume that molecular weight of
each amino acid is 90 Da.
(a) At rest, the control of heart beat is not by the
vagus nerve (a) 510 (b) 663
(b) During exercise the sympathetic control (c) 123 (d) 170
decreases 73. Melting temperature for double stranded DNA is
(c) During exercise the parasympathetic control the temperature at which 50% of the double
decreases stranded molecules are converted into single
stranded molecules. Which one of the following
(d) Stimulation of sympathetic system results in DNA will have the highest melting temperature?
constriction of the pupil
(a) DNA with 15% guanine
66. In a random DNA sequence, what is the lowest
(b) DNA with 30% cytosine
frequency of encountering a stop codon?
(c) DNA with 40% thymine
(a) 1 in 20 (b) 1 in 3
(d) DNA with 50% adenine
(c) 1 in 64 (D ) 1 in 10
74. Following are the types of immunoglobulin and
67. The two alleles that determine the blood group their functions. Which one of the following is
AB of an individual are located on INCORRECTLY paired?
(a) two different autosomes (a) IgD : viral pathogen
(b) the same autosome (b) IgG : phagocytosis
(c) two different sex chromosomes (c) IgE : allergic reaction
(d) one on sex chromosome and the other on an (d) IgM : complement fixation
autosome 75. Which one of the following can be used to detect
68. In biotechnology applications, a selectable marker amino acids?
is incorporated in a plasmid (a) Iodine vapour (b) Ninhydrin
(a) to increase its copy number (c) Ethidium bromide (d) Bromophenol blue
(b) to increase the transformation efficiency 76. Mutation in a single gene can lead to changes in
(c) to eliminate the non-transformants multiple traits. This is an example of
(d) to increase the expression of the gene of (a) Heterotrophy (b) Co-dominance
interest (c) Penetrance (d) Pleiotropy
69. Spermatids are formed after the second meiotic 77. Which one of the following is used to treat
division from secondary spermatocytes. The ploidy cancers?
of the secondary spermatocytes is (a) Albumin (b) Cyclosporin A
(a) n (b) 2n (c) Antibodies (d) Growth hormone
(c) 3n (d) 4n 78. Which of the following processes leads to DNA
70. Phospholipids are formed by the esterification of ladder formation?
(a) three ethanol molecules with three fatty acid (a) Necrosis (b) Plasmolysis
molecules (c) Apoptosis (d) Mitosis
(b) one glycerol and two fatty acid molecules 79. Co-enzymes are components of an enzyme
complex which are necessary for its function.
(c) one glycerol and three fatty acid molecules
Which of these is a known co-enzyme?
(d) one ethylene glycol and two fatty acids
(a) Zinc (b) Vitamin B12
molecules
(c) Chlorophyll (d) Heme
71. Given the fact that histone binds DNA, it should
be rich in 80. The peptidoglycans of bacteria consist of
(a) arginine, lysine (a) sugars, D-amino acids and L-amino acids
(b) sugars and only D-amino acids
(b) cysteine, methionine
(c) sugars and only L-amino acids
(c) glutamate, aspartate
(d) sugars and glycine
(d) isoleucine, leucine
1
86. Let f(x) = x2 – 2 + where is a real constant.
x
81. Let x = ( 50 7)
1/3
– ( 50 7 ) . Then -
1/3 The smallest for which f(x) 0 for all x > 0 is -

(a) x = 2 22 23
(a) (b)
(b) x = 3 33 33

(c) x is a rational number, but not an integer 24 25


(c) (d)
(d) x is an irrational number 33 33
87. Let f : R R be a continuous function satisfying
82. Let
(1 + x + x2)2014 = a0 + a1x + a2x2 + a3x3 + ..... x
f(x) + tf t dt x 2 0 . for all x R. Then -
+ a4028x4028, and let 0
A = a0 – a3 + a6 – .... + a4026, (a) limx f(x) = 2
B = a1– a4 + a7 – ....... – a4027, (b) limx –
f(x) = – 2
C = a2 – a5 + a8 – .... + a4028, (c) f(x) has more than one point in common with
the x-axis
Then -
(d) f(x) is an odd functions
(a) |A| = |B| > |C|
88. The figure shows a portion of the graph y = 2x –
(b) |A| = |B| < |C|
4x3. The line y = c is such that the areas of the
(c) |A| = |C| > |B| regions marked I and II are equal. If a, b are the
(d) |A| = |C| < |B| x-coordinates of A, B respectively, then a + b
83. A mirror in the first quadrant is in the shape of a equals -
hyperbola whose equation is xy = 1. A light source
in the second quadrant emits a beam of light that
hits the mirror at the point (2, 1/2). If the reflected
ray is parallel to the y-axis, the slope of the
incident beam is -
13 7
(a) (b)
8 4
15
(c) (d) 2 2 3
8 (a) (b)
84. Let 7 7

cos n 4 5
C (c) (d)
n! 7 7
n 0

Which of the following statements is FALSE ? 89. Let Xn = {1, 2, 3, ..., n} and let a subset A of Xn be
chosen so that every pair of elements of A differ
(a) C(0).C( ) = 1 by at least 3. (For example, if n = 5, A can be ,
(b) C(0) + C( ) > 2 {2} or {1,5} among others). When n = 10, let the
(c) C( ) > 0 for all R probability that 1 A be p and let the probability
(d) C’( ) 0 for all R that 2 A be q. Then -
85. Let a > 0 be a real number. Then the limit 1
(a) p > q and p – q =
6
ax a3 x
a2 a
lim 3 x x/2
is - 1
x 2 a a (b) p < q and q – p =
6
4
(a) 2 log a (b) a 1
3 (c) p > q and p – q =
10
a2 a 2
(c) (d) 1 a 1
2 3 (d) p < q and q – p =
10
90. The remainder when the determinant 94. Electromagnetic waves emanating from a point
A (in air) are incident on a rectangular block of
2014 2014 20152014 20162016 material M and emerge from the other side as
20172017 2018 2018 20192019 is divided by 5 is - shown. The angles i and r are angles of incidence
20202020 20212021 20222022 and refraction when the wave travels from air to
(a) 1 (b) 2 the medium. Such paths for the rays are possible
(c) 3 (d) 4

91. A cubical vessel has opaque walls. An observer


(dark circle in figure below) is located such that
she can see only the wall CD but not the bottom.
Nearly to what height should water be poured so
that she can see an object placed at the bottom at
a distance of 10 cm from the corner C ? Refractive
index of water is 1.33. (a) if the material has a refractive index very
nearly equal to zero.
(b) only with gamma rays with a wavelength
smaller than the atomic nuclei of the material
(c) if the material has a refractive index less than
zero.
(a) 10 cm (b) 16 cm (d) only if the wave travels in M with a speed
(c) 27 cm (d) 45 cm faster than the speed of light in vacuum.
92. The moments of inertia of a non-uniform circular 95. Two small metal balls of different mass m1 and
disc (of mass M and radius R) about four mutually m2 are connected by strings of equal length to a
perpendicular tangents AB, BC, CD, DA are I1, I2, fixed point. When the balls are given equal
I 3 and I 4 , respectively (the square ABCD charges, the angles that the two strings make
circumscribes the circle). The distance of the with the vertical are 30º and 60º, respectively.
center of mass of the disc from its geometrical The ratio m1/m2 is close to -
center is given by - (a) 1.7 (b) 3.0
1 2 2 (c) 0.58 (d) 2.0
(a) I1 I3 I2 I4
4MR 96. Consider the regular array of vertical identical
current carrying wires (with direction of current
1 2 2 flow as indicated in the figure below) protruding
(b) I1 I3 I2 I4
12MR through a horizontal table. If we scatter some
diamagnetic particles on the table, they are likely
1 2 2 to accumulate -
(c) I1 I2 I3 I4
3MR

1 2 2
(d) I1 I3 I2 I4
2MR
93. A horizontal steel railroad track has a length of
100 m when the temperature is 25ºC. The track
is constrained from expanding or bending. The
stress on the strack on a hot summer day, when
the temperature is 40ºC, is (Note : the linear (a) around regions such as A
coefficient of thermal expansion for steel is 1.1 ×
(b) around regions such as B
10–5/ºC and the Young’s modulus of steel is 2 ×
1011 Pa) (c) in circular regions around individual wires
such as C
(a) 6.6 × 10 Pa
7
(b) 8.8 × 10 Pa 7

(d) uniformly everywhere


(c) 3.3 × 10 Pa
7
(d) 5.5 × 107 Pa
97. The distance between the vertex and the center
of mass of a uniform solid planar circular segment
of angular size and radius R is given by - 101. For the reaction N2 + 3X2 2NX3 where X = F,
Cl (the average bond energies are F-F = 155 kJ
4 sin /2 mol–1 N-F = 272 kJ mol–1, Cl-Cl = 242 kJ mol–1, N-
(a) R
3 Cl = 200 kJ mol–1 and N N = 941 kJ mol–1), the
sin /2 heats of formation of NF3 and NCl3 in kJ mol–1,
(b) R respectively, are closest to
(a) – 226 and +467 (b) + 226 and – 467
4
(c) R cos (c) – 151 and + 311 (d) + 151 and – 311
3 2
102. The equilibrium constants for the reactions
2
(d) R cos X = 2Y and Z = P + Q are K1 and K2, respectively.
3 If the initial concentrations and the degree of
98. An object is propelled vertically to a maximum dissociation of X and Z are the same, the ratio
height of 4R from the surface of a planet of radius K1/K2 is
R and mass M. The speed of object when it
(a) 4 (b) 1
returns to the surface of the planet is -
(c) 0.5 (d) 2
2GM GM 103. The geometry and the number of unpaired
(a) 2 (b)
5R 2R electron(s) of [MnBr4]2–, respectively, are
3GM GM (a) tetrahedral and 1 (b) square planar and 1
(c) (d)
2R 5R (c) tetrahedral and 5 (d) square planar and 5
99. In the circuit shown below, all the inductors 104. The standard cell potential for Zn|Zn 2+
(assumed ideal) and resistors are identical. The ||Cu2+|Cu is 1.10 V. When the cell is completely
current through the resistance on the right is I discharged, log [Zn2+]/[Cu2+] is closest to
after the key K has been switched on for along (a) 37.3 (b) 0.026
time. The currents through the three resistors
(c) 18.7 (d) 0.052
(in order, from left to right) immediately after
the key is switched off are - 105. In the reaction

x, y and z are
(a) x = Mg, dry ether; y = CH3Cl; z = H2O
(a) 2I upwards, I downwards and I downwards (b) x = Mg, dry methanol; y = CO2; z = dil.HCl
(b) 2I downwards, I downwards and I downwards (c) x = Mg, dry ether; y = CO2; z = dil. HCl
(c) I downwards, I downwards and I downwards (d) x = Mg, dry methanol; y = CH3Cl; z = H2O
(d) 0, I downwards and I downwards 106. An organic compound having molecular formula
100. An ideal gas undergoes a circular cycle centered C2H6O undergoes oxidation with K2Cr2O7/H2SO4
at 4 atm, 4 lit as shown in the diagram. The to produce X which contains 40% carbon, 6.7%
maximum temperature attained in this process hydrogen and 53.3% oxygen. The molecular
is closed to - formula of the compound X is
(a) CH2O (b) C2H4O2
(a) 30/R
(c) C2H4O (d) C2H6O2
107. The maximum number of cyclic isomers (positional
(b) 36/R
and optical) of a compound having molecular
formula C3H2Cl2 is
(c) 24/R
(a) 2 (b) 3
(d) 16/R (c) 4 (d) 5
108. The volume vs. temperature graph of 1 mole of 112. In a mixed culture of slow and fast growing
an ideal gas is given below bacteria, penicillin will
(a) kill the fast growing bacteria more than the
slow growing
(b) kill slow growing bacteria more than the fast
growing
(c) kill both the fast and slow growing bacteria
equally
(d) will not kill bacteria at all
113. Consider the following pedigree over four
generations and mark the correct answer below
about the inheritance of haemophilia.

Temperature (K)
The pressure of the gas (in atm) at X, Y and Z,
respectively, are
(a) 0.328, 0.820, 0.820
(b) 3.28, 8.20, 3.28
(c) 0.238, 0.280, 0.280
(d) 32.8, 0.280, 82.0
109. MnO2 when fused with KOH and oxidized in air
Normal male Haemophilic male
gives a dark green compound X. In acidic solution,
X undergoes disproportionation to give an intense Normal female Haemophilic female
purple compound Y and MnO2. The compounds X
and Y, respectively, are (a) Haemophilia is X-linked dominant
(a) K2MnO4 and KMnO4 (b) Haemophilia is autosomal dominant
(b) Mn2O7 and KMnO4 (c) Haemophilia is X-linked recessive
(c) K2MnO4 and Mn2O7 (d) Haemophilia is Y-linked dominant
(d) KMnO4 and K2MnO4 114. A person has 400 million alveoli per lung with an
average radius of 0.1 mm for each alveolus.
110. A metal (X) dissolves both in dilute HCl and dilute
Considering the alveoli are spherical in shape,
NaOH to liberate H2. Addition of NH4Cl and
the total respiratory surface of that person is
excess NH4OH to an HCl solution of X produces
closest to
Y as a precipitate. Y is also produced by adding
NH4Cl to the NaOH solution of X. The species X (a) 500 mm2 (b) 200 mm2
and Y, respectively, are (c) 100 mm2 (d) 1000 mm2
(a) Zn and Zn(OH)2 115. A mixture of equal numbers of fast and slow
(b) Al and Al(OH)3 dividing cells is cultured in a medium containing
a trace amount of radioactively labeled thymidine
(c) Zn and Na2ZnO2
for one hour. The cells are then transferred to
(d) Al and NaAlO2 regular (unlabelled) medium. After 24hrs of
growth in regular media
(a) fast dividing cells will have maximum
111. How many bands are seen when immunoglobulin radioactivity
G molecules are analysed on a sodium dodecyl
(b) slow dividing cells will have maximum
sulphate polyacrylamide gel electrophoresis
radioactivity
(SDS – PAGE) under reducing conditions?
(c) both will have same amount of radioactivity
(a) 6 (b) 1
(d) there will be no radioactivity in either types
(c) 2 (d) 4
of cells
116. If a double stranded DNA has 15% cytosine, what 119. Consider the linear double stranded DNA shown
is the % of adenine in the DNA? below. The restriction enzyme sites and the
(a) 15% lengths demarcated are shown. This DNA is
completely digested with both EcoRI and BamHI
(b) 70%
restriction enzymes. If the product is analyzed
(c) 35% by gel electrophoresis, how many distinct bands
(d) 30% would be observed?
117. The mitochondrial inner membrane consists of a
number of infoldings called cristae. The increased
surface area due to cristae helps in :
(a) 5 (b) 2
(a) Increasing the volume of mitochondria
(c) 3 (d) 4
(b) Incorporating more of the protein complexes
essential for electron transport chain 120. Enzyme X catalyzes hydrolysis of GTP into GDP.
The GTP-bound form of X transmits a signal that
(c) Changing the pH
leads to cell proliferation. The GDP – bound form
(d) Increasing diffusion of ions does not transmit any such signal. Mutations in
118. The activity of a certain protein is dependent on X are found in many cancers. Which of the
its phosphorylation. A mutation in its gene following alterations of X are most likely to
changed a single amino acid which affected the contribute to cancer?
function of the molecule. Which amino acid (a) Mutations that increase the affinity of X for
change is most likely to account for this GDP
observation?
(b) Mutations that decrease the affinity of X for
(a) Tyrosine to Tryptophan GTP
(b) Lysine to valine (c) Mutations that decrease the rate of GTP
(c) Leucine to isoleucine hydrolysis
(d) Valine to alanine (d) Mutations that prevent expression of
enzyme X.

1. (d) 2. (a) 3. (c) 4. (a) 5. (b) 6. (b) 7. (b) 8. (b) 9. (a) 10. (a)
11. (d) 12. (d) 13. (d) 14. (a) 15. (b) 16. (d) 17. (b) 18. (b) 19. (c) 20. (d)
21. (a) 22. (b) 23. (c) 24. (b) 25. (d) 26. (b) 27. (a) 28. (b) 29. (a) 30. (a)
31. (c) 32. (a) 33. (d) 34. (b) 35. (c) 36. (b) 37. (b) 38. (a) 39. (b) 40. (a)
41. (a) 42. (b) 43. (b) 44. (d) 45. (c) 46. (b) 47. (b) 48. (a) 49. (b) 50. (d)
51. (b) 52. (a) 53. (b) 54. (a) 55. (c) 56. (d) 57. (c) 58. (a) 59. (c) 60. (a)
61. (a) 62. (b) 63. (a) 64. (c) 65. (c) 66. (a) 67. (b) 68. (c) 69. (a) 70. (b)
71. (a) 72. (a) 73. (b) 74. (a) 75. (b) 76. (d) 77. (c) 78. (c) 79. (b) 80. (a)
81. (a) 82. (d) 83. (c) 84. (d) 85. (d) 86. (d) 87. (b) 88. (a) 89. (c) 90. (d)
91. (c) 92. (a) 93. (c) 94. (c) 95. (a) 96. (a) 97. (a) 98. (a) 99. (a) 100. (a)
101. (a) 102. (a) 103. (c) 104. (a) 105. (c) 106. (b) 107. (c) 108. (a) 109. (a) 110. (b)
111. (c) 112. (a) 113. (c) 114. (d) 115. (a) 116. (c) 117. (b) 118. (a) 119. (c) 120. (c)
t–2 0 t –2
1. (d) x + y = x + y = 12
2 2
t 2 t [0, 3)
curve (1)
x + y2 = 12 1
z 2
y2 = – (x – 12) z max
curve (2) 3. (c) 2{(2014)3 + (2012)3 +…+ 23} – {(2014)3
x2 + y = 12 + (2013)3 +…+ 13}
x2 = – (y – 12)
= 2 × 8 {(1007)2 + (1006)2 +…+ 13} – {(2014)3
Curve y2 = – (x – 12) intersect on x-axis at
+ (2013)2 +…+ 13}
(12, 0), while intersect on x-axis at ( 12 , 0) 2 2
1007 1008 2014 2015
Intersect of curve on y-axis at (0, ± 12 ) 2 8
2 2
intersection of curve x2 = –(y – 12) on y-axis
2 2 2 2
at (0, 12) 1007 1008 2014 2015
2 8
4 4
= (1007)2 (2016)2 – (1007)2 (2015)2
= (1007)2 {2016 – 2015} {2016 + 2015}
= (1007)2 (4031)
which is divisible by (1007)2.
4. (a)

four point of intersection


2. (a) |z3 + z–3| 2

1
z3 2
z3
1 1
z z2 1 2 OB OA
z z2
t1t2 = – 1
2
1 1
z z 3 2 h t1 t 2
z z Now
2 2
2
1 1 t1 + t2 = h …(1)
z z 3 2
z z
while t12 t22 k …(2)
2
1 1 from eq. (1) and (2) we get
z z 3 2 { |z1 z2 | ||z1 | |z2 ||}
z z
(t1 + t2)2 – 2t1t2 = k
1 h2 + 2 = k
t |t2 – 3| 2 t=z where t 0
z Hence, locus of point C is x2 + 2 = y
for, t 3 for 0 t< 3
t(t2 – 3) 2 t(3 – t2) 2
t3 – 3t – 2 0 3t – t3 2
(t – 2) (t + 1)2 0 t3 – 3t + 2 0
(t – 1)2 (t + 2) 0
5. (b) 1
cos ·cos – sin ·sin =
2

1 1 1 1 1
1 1
4r 2 r2 2r r 2

4r 2 1 r2 1 1 r2
(4r2 – 1) (r2 – 1) = (r2 + 1)2
4r4 – 5r2 + 1= r4\ + 2r2 +1
3r4 = 7r2
7
Chose AB subtend 90º at centre. r2 =
3
so that AB subtend 45º at O (circumference
of circle) 7
r
6. (b) Sphere x + y + z – 4x – 6x – 12z + 48 = 0
2 2 2
3
Centre is at (2, 3, 6) 9. (a) Let P(x) = a0xn + a1xn–1 + a2xn–2 +…+ an
and radius = 4 9 36 48 = 1 P'(x) = na0xn–1 + (n – 1)a1xn–2 +…+ an–1
Now distance between centre and origin P(x)P'(x) = a0xn + (a1 – na0) xn–1
+ (a2 – (n – 1)a1) xn–2+…+ (an – an–1)
= 4 9 36 = 7
and shortest distance = 7 – 1 = 6 (Origin lies P(x) – P'(x) = a0xn. (Given)
outside the sphere) a1
a1 – na0 = 0 n
sin cos a0
7. (b) = –1
sin cos a2
a2 – (n – 1)a1 = 0 n 1
By observation a1
sin( ) cos – cos ( ) sin = ( – 1) sin cos an
sin( – 1) = ( – 1) sin cos an – an–1 = 0 1
an 1
clearly = 1, = 3 is solution
an an 1 a1
Hence, we get two values of ' '. P 0 an ...
an 1 an 2 a0
8. (b) = 1 × 2 × 3 × …× n = n!
11. (d) Given that 2r + r = P
P
r
2
1 2 1 P
2
1 P2
Now, area = r 2 2 2
2 2 2
Now, inorder to find maximum Area
dA
0 2
d
12. (d) f(x) = max {|x|, |x – 1|, ... , |x – 2n|}
1 1
sin = & sin = For and For
2r r
x n x<n
3 × (2 ) + (2 ) × 3 = 360º
f(x) = |x| f(x) =|x – 2n|
+ = 60º
2n n 2n

1 f x dx f x dx f x dx
Now, cos( + )= 0 0 n
2
n 2n
|x 2n|dx |x| dx
0 n
n
2n x dx
2n
x. dx 17. (b) Given b ˆi ˆj kˆ
0 n

x2
n
x2
2n
and a b c
2nx
2 0
2 c a b

n2 4n 2 n2 c a ˆi ˆj kˆ b iˆ ˆj kˆ
2n 2
2 2 2
c 6iˆ 3ˆj 6kˆ ˆi ˆj kˆ
2 2
3n 3n
3n 2 c 6 ˆi 3 ˆj 6 kˆ
2 2
13. (d) Let P(x) = ax3 + bx2 + cx + d Now, c 6 3 6 0
Now P(1) = a + b + c + d = 3 3 3
Now P(0) = 2, d = 2 1
–a+b–c+d=4
c 7iˆ 2ˆj 5kˆ
2b + 2d = 7
18. (b) log(3x – 1)(x – 2) = log(3x (2x2 – 10x – 2)
2b + 4 = 7 1)2

log(3x – 1) (x – 2)2 = log(3x – 1)(2x2 – 10x – 2)


2b = 3
(x – 2)2 = 2x2 – 10x – 2
3
b= x2 – 4x + 4 = 2x2 – 10x – 2
2
1
x2 – 6x – 6 = 0
ax 3
bx 2
cx d dx x = 3 ± 15
1
x=3– 15 (is not possible)
1 3 1
x x=3 15
2 bx 2 d dx 2 b dx
0
3 0 19. (c) c = 3 is not possible
b Equation is possible if
2 d
3 a=3 b = 4 and c=2
a 2b 3c 17
1 .
2 2 5 abc 24
2
20. (d)
15. (b) Let number of Red Balls be x
number of White Balls be y
number of Blue Balls be z
number of Green Balls be w
x
x
C4 C3 . y C1 x
C2 y C1z C1 Clearly
x y z w x y z w x y z w
C4 C4 C4 x2 – y2 = 144
x
C1 y
C
1
z
C1 w
C1 (x – y) (x + y) = 144
x y z w
C4 Now, factorsize 144 into two even factors
C4 = xC3 yC1
x
x – 3 = 4y x = 4y + 3 x + y = 72 or x + y = 36 or x + y = 18
x
C3 C1 = C2 C1 C1
y x y z
x – 2 = 3z x = 3z + 2 x–y=2 and x–y=9 or x–y=8
x
C C1 C1 = C C C C1
2
y z x
1
y
1
z
1
w
x – 1 = 2w x = 2w + 1 x = 37 or x = 20 or x = 13
Cleary y = 1 is not possible y = 35 y = 16 y=5
Now for y = 2 x = 11 21. (a) Given, t = 2 second
z = 3 x = 11
w = 5 x = 11
so, minimum number of Ball
= 11 + 2 + 3 + 5 = 21
Velocity after time t v = v0 – gt
v 0 v0 gt … (i) 23. (c)
Displacement in time ‘t’
1
S = v 0t – gt2
2
1
1 = 2v0 – g(2)2
2
2v0 = (1 + 2 g)
1
v0 g
2
1 Area of rectangle and triangle are equal.
+ g > gt (t = 2 sec)
2 Then,
1 1
+ g>2 g ah = ab, h = 2b
2 2
1
g< h b
2 M1 = M2 [centre of mass of combination
3 2
1 at the mid-point of their common edge]
<
2 M1 3b
< 0.05 M2 2h
22. (b) 12
6
C 11
5
B + e+ + e
+ 0.96 MeV
M1 3 1
M0 1 g
M M M2 2 2
2n 22
2t 0 M1 3
[Half life = t0 ; n = t/t0 n= n = 2] M2 4
t
M = 0.25 g (remained) 24. (b)
Carbon used M0 – M
0.75 g
6
0.75 10
Number of moles =
12
6
0.75 10
Number of reaction = × 6.023 × 1023
12
= 0.37 × 1017 reaction
Energy from reaction = 0.376 × 1017 × 0.96 MeV
= 0.36 × 1017
= 3.6 × 1016 MeV
Using concept of COM (Center of Mass)
4 × 1016 MeV
m 1r 1 = m2r 2
Energy from annihilation
r1 + r2 = 2R
= 2 m0c2 (0.376 × 1017)
1.02 (0.376 × 1017) MeV m2
1 r2 2R
4 × 10 MeV 16 m1
Total energy = Ereaction + Eanhillation 2m1 R
r2
ET 8 × 1016 MeV m1 m 2
L sin 2
= r2 [R < < L]

2m1 R
2
= m1 m 2 L
25. (d) Both disc are horizontal and axis of rotation For ball
is common. 1 1
I1 = 4.25 kg-m2 V0 t – gt2 = – H V0 (5) – 10(5)2 = – 85
2 2
N1 = 15rps 5V0 = 40
V0 = 8 m/s
27. (a)

I2 = 1.8 kg-m2
N2 = 25rps

(I)
I

Angular momentum conservation


I1 – I = (I1 + I2) Open
1 2 2
I
2 (4.25) N1 – 2 (1.8) N2
= (4.25 + 1.80) N (2 )
– +
(4.25 × 15 – 1.8 × 25) = (6.05) N (II)
63.75 – 45 = 6.05 N When V is positive in forward bias the
N = 3 rev/s. potential drop on diode is low. When V is
negative current will pass through the
26. (b) resister as V = IR. (in diode current almost
zero because of reverse bias diode)

28. (b) Debroglie wavelength of electron is,


h
2mK
H Kinetic energy K qV
Time taken to reach sound after hit ts = V
s
1
V
85
ts sec ; ts = 0.25 sec Potential becomes four time so wavelength
340
becomes half.
For ball time of flight Tƒ D
Fringe width ( =
Tƒ + ts = 5.25 sec d
Tƒ = 5.25 – 0.25
Tƒ = 5 sec Fringe width becomes half.
29. (a) Q = 3 × 10–12 C r = 1 mm When connected through AD
2
As, Z R2 XL XC

So,
2
Vrms VL VC VR2

162 152
481 V
33. (d) From Bohr postulates
0
B
4 R kze 2 mv 2
….(i)
B = BH (at centre effective magnetic field r2 r
become zero) nh
mvr = ….(ii)
2
0Q
BH
4 R e2 z
v=
2 0h h
BH 4 R
(BH = 30 × 10–6 T; R = 1 mm; nh
0 Q r
2 mv
Q = 3 × 10–12C)
nh
= 1011 rad/s r
e2 z
30. (a) Because on earth there is no atmosphere. So 2 m
2 0h n
water will boil.
(At Boiling point vapour pressure = 0h2 n2
r
Atmospheric pressure, in open vessel) me 2 z
2
l n2
31. (c) T = 2 1 r r0
g 16 z
(This is valid when is not small) Because the medium of permittivity = 13 0

l effective massm = 0.07 me


T0 = 2 (for small )
g 13r0 n 2
r
2 0.07 z
T = T0 1 At ground state (n = 1, assuming like H atom,
16
z = 1)
T > T0
13
32. (a) r= (0.53) Å
0.07
r 100 Å
34. (b) Ideal gas equation PV = nRT
For isobaric process
Voltmeter between A & B VL = 36 V ...(1)
nR
between A & C 2 2
....(2) V= T (V T (straight line))
V L VR 39 P
nR
between B & D VL2 VR2 25 ....(3) Slope of line =
P
from equation (1) & (2) VR2 = 392 – 362
1
VR = 15 V ...(4) slope
P
From Eq. (3) & (4) VC2 = 252 – 152 slope 3 > slope 2 > slope 1
VC = 20 V ...(5) P3 < P2 < P1
35. (c) 36. (b) A = G M C
[M0L2T0] = [M–1L3T–2] [M] [LT–1]
– + =0 =
3 + =2
–2 – = 0
on solving
= 2, = 2, = –4
Induced emf (E)
37. (b) P = 160 watt, = 50000Å
d
E=
dt
BA
P
d I=
E BA 4 r2
dt
P
AdB nh =
E 4 r2
dt
P
E = – A B0 cos t n 2
4 r h
E
i= (n = no. of photons per sec per m2)
R
B0 A P
i cos t n
R 4 r 2 hc
20 –2 –1
current oscillates with “ ”. n = 10 m s
Heating loss = i2R 38. (a) Draw a regular n-Polygonal
B0 A
i cos t
H i 2
R

H B2 a
2 4

Force on d length
|F| = Bid
B0 a2
|F| = B0 sin t R cos t . d
n sides, n wires
Force per unit length
|F| B20 a 2 = =
= sin t cos t
1
n2

dl R Bnet at centre = n × B due to one side


Force per unit length B02
n 0 I
Net force on ring will be zero. Bnet = [sin + sin ]
4 l 1 2

n 0I
sin
2 l n
39. (b) Let intensity of sound is I0 and increased by
100I0
Loudness of sound in decibel dB = 10 log 10

I
I0
when intensity of sound become 100 I then
(Force cancel) 100I
new decibel level = dB’ = 10 log 10 I0
dB’ – dB = 10 log 10 100
CH3 CH3
dB’ – dB = 20 | |
C2H5O Na
decibel rise by 20 dB 49. (b) CH3 C Cl E2
CH3 C CH2
|
only one option i.e. 80 to 100 dB match with CH3
it.
40. (a) Initially wire is slack so it do not have any 50. (d) 2nd group elements forms precipitate.
deformation energy. When block is given 52. (a) sp s-character – 50%
some velocity it move due to kinetic energy, sp2 s-character – 33%
one wire get taut. Internal force get develop
sp3 s-character – 25%
in wire and KE start decreases and
deformation energy of wire increase. Till 53. (b) Iron is present in Haemoglobin.
block come at rest using energy conservation 54. (a) Molar conductivity of H+ ion is highest.
1 1
mv2 = Y × (strain)2 × A × L 55. (c) n n 2 3.87
2 2
2 n=3
1 1 x
mv2 = Y × ×A×L
2 2 L So Co 2
3d 7 4s0
mL 57. (c) As CH3COOH is weakest acid of all and HCl
x=v
AY is strongest acid.
41. (a) Acidity decreases in the order OH
BBR3 > BCl3 > BF3 +
H 3O
42. (b) O2–, No. of electrons = 10 58. (a)
Mg2+, No. of electrons = 10
+ –
43. (b) CH 4 109.5 NH2 N2Cl CN
diazotisation CuCN
NH3 107.1 59. (c)

H2O 104.5
60. (a) In Schottky defect, equal number of cations
44. (d) KMnO4 H 2 O2 OH
MnO2 H2 O and anions miss from their normal lattice
sites.
Oxi. state of Mn in MnO2 = +4
61. (a) Immunosuppressive Drug inhibits T-cell
Ea
infiltration
45. (c) K = Ae RT
62. (b) Presence of glucose inhibit the lac operon
If T is high, K = A
63. (a) No. of amino acid in one term in helix = 3.6
46. (b) Acc. to Reducing Power. Pitch length for Helix = 5.4 Aº
The No. of Amino acid in polypeptide
Br
3.6
47. (b) CH3 – C – CH2CH3 27 10 6
5.4 10 10
H = 1.8 × 104 Amino acid
Chiral Carbon (Optically active) = 18000 amino acid
64. (c)

CH = CH2 CH = CH2

48. (a) n Poly. n

Polystyrene
65. (c) Parasympathetic control decreased during
exercise.
66. (a) Total no. of codon = 64 81. (a) x = ( 50 7 )1/3 – ( 50 7 )1/3
Now cubing both the sides,
Functional codon = 61
x3 50 7 50 7 3 50 7 50 7
Stop codon = 3
1/3 1/3
frequency of encountering stop codon 50 7 50 7
3 1 x3 = 14 – 3 (1) (x)
=
61 20
x3 = 14 – 3x
67. (b) Allele of the same gene are present at same
x3 + 3x – 14 = 0
gene locus on homologous chromosome
x = 2 satisfies the above equation.
68. (c) Selectable marker is used to eliminate the
non transformant from the transformants 83. (c)
69. (a) Secondary spermatocyte formed by meiosis-I
.
70. (b) Phospholipid : two fatty acid and one
phosphorylated nitrogenous organic
compound attached to glycerol

1
Curve y =
x
dy 1
dx x2
1 1
slope of tangent = at 2,
71. (a) Histone is basic protein and it is rich in lysine 4 2
and arginine and slope of normal = 4
72. (a) Molecular weight of polypeptide = 15.3 kda
Molecular weight of amino acid = 90 Da where foot of normal is 2, 1
2
No. of Amino acid in polypeptide
Now let the slope at incident beam is ‘m’
15.3 103
= = 170 4 m 4
90
One amino acid is coded by = 3 Nitrogen base 1 4m 1 4.
170 amino acid would be coded by 4 m 1
= 170 × 3 = 510 Nitrogen base 1 4m 4
73. (b) Melting temperature of DNA GC content 15
m
74. (a) IgD activates , B- lymphocyte 8
75. (b) Ninhydrin stain is used to detect primary or cos n
84. (d) C
secondary amino acid and ammonia. n 0 n!
76. (d) Pleiotrophy : A single gene affects more than cos cos 2 cos3 cos n
one phenotype. lim 1 ...
n 1! 2! 3! n!
77. (c) Monoclonal antibodies are used for treatment
1 1 1
of cancer. C 0 1 ... up to term = e
1! 2! 3!
78. (c) In apoptosis or programmed cell death DNA
degradation occur which form DNA ladder. 1 1 1
C 1 ... up to term = e–1
79. (b) Co-enzymes are the organic compound which 1! 2! 3!
are necessary for function of enzyme. Clearly C(0) . C( ) = 1
1 f' x
C(0) . C( ) = e + > 2 x
e f x 2
sin sin 2 3sin 3
cos" 2 Now integrating on both side wrt 'x', we get
1! 2! 3!
... up to term f' x
dx xdx
And that value is equal to zero at =0 f x 2

ax a3 x
a2 a x2
85. (d) lim n (f(x) + 2) = c
x 2 a3 x ax / 2 2
Applying L hospital rule f(x) + 2= e x2 / 2
c
x 3 x
a ln a a ln a x2 / 2
lim f(x) = k e 2
x 2 1 x/2
a 3 x ln a a ln a Now, when x = 0 k=2
2
a 2 ln a al n a a 2 a 2 f(x) = 2 e x2 / 2
1
1 a
a 3a 3
a ln a ln a clearly lim f(x) = – 2
2 2 x
1 88. (a)
86. (d) f(x) = x2 – 2 +
x
x 3 2x 1
f(x) =
x
(x) = x3 – 2x + 1 should be positive
(x) = x3 – 2x + 1
Now differentiating (x) wrt 'x', we get
(x) = 3 x2 – 2 = 0 b
2x 4x 3 dx 2 b a c
a
2
x (x 2 x 4 )b = 2 (b – a) c
3
(a + b) (1 – (a2 + b2)) = 2c
2 (a + b) (1 – (a + b)2 + 2ab) = 2c …(i)
Clearly x is a point of minima
3 Also 2x – 4x = c 3

2
0 4x3 – 2x + c = 0
3

2 2 a+b+ =0 clearly a + b = – …(ii)


2 1 0
3 3 1 1
ab + (a + b) = – ab = 2
– …(iii)
2 2
2 4 c
1 0 ab = –
3 3 4
1
2 4 c=–4
2
…(iv)
1 2
3 3
Putting the value of (a + b), ab, c from eq. (ii),
32 (iii), (iv) in equation (i), we get
27 1 1
2 2
1 2 8 2
x 2 2
87. (b) f(x) + tf t dt x2 0
1– 2
+2 2
–1=8 2
–4
0

Now differentiating wrt 'x', we get


2
=8 2
–4
f '(x) + xf(x) + 2x = 0 2
7
89. (c) At n = 10 From diagram r = 45°, = 1.33
Let Ar be no. of ways of selecting r numbers. using snell law
Number of selection of A is 4
sin i = sin r
= n (A0) + n(A1) + n (A2) + n(A3) + n(A4) 3
= 1 + 10 + (7 + 6 + 5 +…+ 1) + (4 + 3 + 2 + 1) 3
sin i = sin 45
+ (3 + 2+ 1) + (2 + 1) + 1 4

7.8 3
= 11 + + 10 + 6 + 3 + 1 + 1 = 60 sin i
2 4 2
N(p) = n(no. of ways to detect '1')
= 1 + 7 + 4 + 3 + 2 + 1 + 1 = 19
N(q) = n(no. of ways to detect '2')
= 1 + 6 + 3 + 2 + 1 = 13
19 13
p= and q=
60 60
1 3
p–q= tan i =
10 23
90. (d) Determinant
h 10
2014 2015 2016 tan i =
2015 1 2015 2015 1 h
2017 2018 2019
= 2015 2 2020 2 2020 1 h tan i = h – 10
2020
2020
2020 1
2021
2020 2
2022
10 = h [1 – tan i]

Determinant of remainder 10
h=
2014 1 tan i
1 0 1
2017 2018 2019 27 approx. = 27 cm
= 2 2 1
0 12021 22022
92. (a)
= 1(24040 + 1) + 1 (22017)
= ((4)2020 + 1) + 2 · 22016
(5 – 1)2020 + 1 + 2 · 41008
= (5 – 1)2020 + 1 + 2 · (5 – 1)1008
remainder, (– 1)2020 + 1 + 2 · (– 1)1008
=1+1+2=4

91. (c)

Icm + m (R + y)2 = I3 ...(1)


Icm + m (R– y) = I1 2
...(2)
from (1) & (2)
I1 – I3 = m [(R – y)2 – (R + y)2]
I1 – I3 = m (2R) (– 2y) ...(3)
Icm' + m (R + x)2 = I4 ...(4)
Icm' + m (R – x) = I2 2
...(5)
from (4) & (5)
(I2 – I4) = m [(R – x)2 – (R + x)2]
I2 – I4 = m [(2R) (–2x)] ...(6) using Lami theorem on m2
(3) + (6)
2 2
F m2g
(I1 – I3)2 + (I2 – I4)2 = (m2 × 4R2 × 4(x2 + y2) sin 2
sin
distance of CM from O
using (1) & (2)
= x2 y2 m1 sin 1
= m2 sin 2

m1 × sin 30º = m2 sin 60º


1 2 2
I1 I3 I2 I4 m1
4mR 3 1.7
m2
93. (c) When some body is constrained from
expanding or bending then on heating
96. (a) Diamagnetic material move from high
thermal stress get develop in the body.
magnetic field to low magnetic field so this
T1 = 25°C T2 = 40°C material are likely to accumulate over the
= 1.1 × 10–3 /°C region such as A as here magnetic field is
minimum.
Stress = Y T
97. (a) Planar circular segment can be seen as it
= 2 × 1011 × 1.1 × 10–5 × (40 – 25)
consist of Arc element
= 3.3 × 107 N/m2
= 3.3 × 107 Pa
94. (c)

Mass of element = dm = × r × dr

For five refractive index incident wave move r sin


toward normal line but Meta materials are 2
the material for which refractive index is centre of mass of Arc element is at
negative for them. Refraction diagram is 2
shown, here. In question same type of Centre of mass location of segment
diagram is given.
r sin / 2
dm
95. (a) = /2
dm
R
r dr r sin /2
0
/2
R

r dr
0

sin
2 R3
2
= 30º, = 60º R2
1 2 3
2
using Lami theorem on m1
4 sin /2
F m1 g R
3
sin 1
sin

F m1 g
...(1)
sin 1
sin
98. (a) Using energy conservation law
24
GMM 1 GMM temp. is more than on the given process
mV 2 R
R 2 5R
24 36
2GM So Tmax lie between and
V=2 R R
5R
V is the velocity by which object is projected. only one option is present
When object return to earth its speed will be V.
99. (a) (2 )2
2
K1 1 4
102. (a) 2 2
4
K2
1

2
(when switch is closed) 103. (c) MnBr4 , Oxi. No. of Mn = +2
Initially circuit is in steady state current
2
through each resistor as all are identical & Mn 3d 5 4s0
are in parallel combination. When switch is As Br is weak field so hybridisation is sp3.
off current through L 1 & L 2 just after
unchange. So flow of current remain some. 2
0.0591 Zn
104. (a) Ecell E0cell log
2 Cu 2

2
0.0591 Zn
1.1 log
2 Cu 2

2
Zn
In right & middle wire current is I downward log 2
37.3
and in left wire current is 2I upward
Cu

PV Br MgBr COOH
100. (a) T =
R
105. (c) Mg, (i) CO2
T will be maximum when PV is maximum
dry ether (ii) dil. HCl
PV 4 2sin 4 2 cos
T=
R R 106. (b) CH3CH2OH
Acidified
CH3COOH
K 2Cr2O7
As sin and cos both can not be equal to 1
for same value of
107. (c) Cl Cl Cl
36
T can not be and and
R
Cl Cl Cl
36
Tmax should be less than (Optically
R
active carbon)

nRT 1 0.0821 200


108. (a) Pressure at X =
V 50
= 0.328

1 0.0821 500
Pressure at Y = 0.821
50

curve is above isothermal curve


115. (a) Radioactivity is mostly incorporated in
109. (a) MnO2 KOH K 2 MnO4
(X) rapidly dividing cell during S phase. Thus
after 24 hour it will be mainly present in few
disproportionation
KMnO4 MnO2 rapidly dividing cell
116. (c) Cytosine = 15%
According to chargaff principle
110. (b) (X) AlCl3 + H2
Al A=T&G=C
A + T + G + C = 100
NaAlO2 + H2
Thus A will be 35%
118. (a) Serine, tyrosine, threonine and histidine are
Al(OH)3 most common target for phosphorylation in
(Y) White ppt. eukaryotes.
If Tyrosine get mutated in tryptophan than
phosphorylation of gene does not occur and
111. (c) On SDS – PAGE of IgG two principal Bands gene can not be activated
are visible
119. (c)
112. (a) Penicillin inhibit the cell wall formation in
bacteria thus it kill rapidly growing bacteria
more than the slow growing bacteria.
113. (c)

There are 4 fragment but 2 fragment of 3 Kb


will appear as one band so three band will
appear

Female parent is carrier and male and female 120. (c) GTP –X Complex Active cell division
offspring is affected. GDP –X Complex No cell division
114. (d) Radius of spherical alveoli = 0.1 mm and GTP
Hydrolysis
Xenzyme GDP
Total No. of Alveoli = 400 × 2 = 800 million Mutation cause the decrease of rate of
Surface area of sphere = 4 R2 hydrolysis of GTP. Thus GTP –X complex
Total respiratory surface Area remain present for long time and cell division
= 4 R2 × 800 million become uncontrolled.
= 1000 mm2 (approx)
6. In an ellipse, its foci and the ends of its major
axis are equally spaced. If the length of its semi-
1. Let C0 be a circle of radius 1. For n 1, let Cn be
minor axis is 2 2, then the length of its semi-
a circle whose area equals the area of a square
major axis is
inscribed in Cn – 1. Then i 0
Area (Ci) equals
(a) 4 (b) 2 3
2
(a) 2
(b) 2
(c) 10 (d) 3
1 2 7. Let ABC be a triangle such that AB = BC. Let F
(c) 2 (d) be the midpoint of AB and X be a point on BC
2
such that FX is perpendicular to AB. If BX = 3XC
2. For a real number r we denote by [r] the largest
then the ratio BC/AC equals
integer less than or equal to r. If x, y are real
numbers with x, y 1 then which of the following (a) 3 (b) 2
statements is always true?
(a) [x + y] . [x] + [y] (b) [xy] [x] [y] 3
(c) (d) 1
2
x x
(c) [2x] 2[x] (d) y y 8. The number of solutions to the equation

3. For each positive integer n, let 1 1


cos4x + 2 = sin4 x + in the interval
cos x sin 2 x
n
An = max |0 r n . Then the number of [0, 2 ] is
r
(a) 6 (b) 4
An
elements n in {1,2,....,20} for which 1.9 2 (c) 2 (d) 0
An 1
is
x 5
(a) 9 (b) 10 if x 2
9. Consider the function f(x) = x 2 if x 2 . Then
(c) 11 (d) 12 1
4. Let b, d > 0. The locus of all points P(r, ) for
f (f (x)) is discontinuous
which the line OP (where O is the origin) cuts
the line r sin = b in Q such that PQ = d is (a) at all real numbers
(a) (r – d) sin =b (b) (r ± d) sin =b (b) at exactly two values of x
(c) (r – d) cos =b (d) (r ± d) cos =b (c) at exactly one value of x
5. Let C be the circle x2 + y2 = 1 in the xy-plane. For (d) at exactly three values of x
each t 0, let Lt be the line passing through (0, 1) 10. For a real number x let [x] denote the largest
and (t, 0). Note that Lt intersects C in two points, number less than or equal to x. For x R let
one of which is (0, 1). Let Qt be the other point. f (x) = [x] sin x.
As t varies between 1 and 1 + 2, the collection Then
of points Qt sweeps out an arc on C. The angle (a) f is differentiable on R.
subtended by this arc at (0, 0) is (b) f is symmetric about the line x = 0.
3
(a) (b) (c) f (x)dx 0.
8 4 3

(d) For each real the equation f (x) – = 0 has


3
(c) (d) infinitely many roots.
3 8
11. Let f : [0, ] R be defined as 16. Let n 3. A list of numbers x1, x2,....., xn has mean
and standard deviation . A new list of numbers
sin x, if x is irrational and x 0,
f (x) = 2 x1 x2
tan x, if x is rational and x 0, y1, y2, ....., yn is made as follows: y1 = ,
2
The number of points in [0, ] at which the function x1 x2
y2 = and yj = xj for j = 3, 4, ...., n. The mean
f is continuous is 2
(a) 6 (b) 4 and the standard deviation of the new list are ˆ .
(c) 2 (d) 0 and ˆ . Then which of the following is necessarily
12. Let f : [0, 1] [0, ] be a continuous function true?
1 (a) = ˆ and ˆ (b) = ˆ and ˆ
such that f x dx 10. Which of the following
0
(c) = ˆ (d) ˆ
statements is NOT necessarily true? 17. What is the angle subtended by an edge of a
1 regular tetrahedron at its center?
(a) e x f x dx 10
1 1 1
0
(a) cos (b) cos 1
2 2
1
f x
(b) dx 10
1 x
2
1 1 1 1
0 (c) cos (d) cos
3 3
1

(c) 10 sin 100x f x dx 10 18. Let S = {(a, b) : a, b Z, 0 a, b 18}. The


0 number of elements (x, y) in S such that 3x + 4y
1 + 5 is divisible by 19 is
2
(d) f x dx 100 (a) 38 (b) 19
0

13. A continuous function f : R R satisfies the (c) 18 (d) 1


x 19. For a real number r let [r] denote the largest
equation f (x) = x + f t dt. Which of the integer less than or equal to r. Let a > 1 be a real
following options is true?
0
number which is not an integer and let k be the
smallest positive integer such that [ak] > [a]k. Then
(a) f (x + y) = f (x) + f (y)
which of the following statements is always true?
(b) f (x + y) = f (x) f (y)
(a) k 2 ([a] + 1)2 (b) k ([a] + 1)4
(c) f (x + y) = f (x) + f (y) + f (x) f (y)
1
(d) f (x + y) = f (xy) (c) k 2[a]+1 (d) k 1
a a
14. For a real number x let [x] denote the largest integer
less than or equal to x and {x} = x – [x]. Let n be a 20. Let X be a set of 5 elements. The number d of
n ordered pairs (A, B) of subsets of X such that A
positive integer. Then cos 2 x x dx is equal ,B ,A B = satisfies
0
(a) 50 d 100 (b) 101 d 150
to
(a) 0 (b) 1 (c) 151 d 200 (d) 201 d
(c) n (d) 2n – 1
15. Two persons A and B throw a (fair) die (six-faced
21. A uniform thin rod of length 2L and mass m lies
cube with faces numbered from 1 to 6) alternately,
on a horizontal table. A horizontal impulse J is
starting with A. The first person to get an outcome
given to the rod at one end. There is no friction.
different from the previous one by the opponent
The total kinetic energy of the rod just after the
wins. The probability that B wins is
impulse will be
5 6 J2 J2
(a) (b) (a) (b)
6 7 2m m

7 8 2J 2 6J 2
(c) (d)
(c) (d) m m
8 9
22. A solid cylinder P rolls without slipping from rest 26. A solid expands upon heating because-
down an inclined plane attaining a speed vP at (a) the potential energy of interaction between
the bottom. Another smooth solid cylinder Q of atoms in the solid is asymmetric about the
same mass and dimensions slides without friction equilibrium positions of atoms.
from rest down the inclined plane attaining a (b) the frequency of vibration of the atoms increases.
speed vQ at the bottom. The ratio of the speeds
(c) the heating generates a thermal gradient
vQ between opposite sides.
v p is - (d) a fluid called the caloric flows into the
interatomic spacing of the solid during heating
3 3 thereby expanding it.
(a) (b)
4 2 27. Consider two thermometers T1 and T2 of equal
2 4 length which can be used to measure
(c) (d) temperature over the range 1 to 2. T1 contains
3 3
mercury as the thermometric liquid while T 2
23. A body moves in a circular orbit of radius R under contains bromine. The volumes of the two liquids
the action of a central force. Potential due to the are the same at the temperature 1 . The
central force is given by V(r) = kr (k is a positive volumetric coefficients of expansion of mercury
constant). Period of revolution of the body is and bromine are 18 × 10–5 K–1 and 108 × 10–5 K–1,
proportional to- respectively . The increase in length of each liquid
(a) R1/2 (b) R–1/2 is the same for the same increase in temperature.
(c) R 3/2
(d) R–5/2 If the diameters of the capillary tubes of the two
thermometers are d1 and d2 respectively, then the
24. A simple pendulum is attached to the block which
ratio d1 : d2 would be closest to
slides without friction down an inclined plane
(ABC) having an angle of inclination as shown. (a) 6.0 (b) 2.5
(c) 0.6 (d) 0.4
28. An ideal gas follows a process described by
PV2 = C from (P1, V1, T1) to (P2, V2, T2) (C is a
constant). Then
(a) if P1 > P2 then T2 > T1
(b) if V2 > V1 then T2 < T1
(c) if V2 > V1 then T2 > T1
While the block is sliding down the pendulum
oscillates in such a way that at its mean position (d) if P1 > P2 then V1 > V2
the direction of the string is- 29. A whistle emitting a loud sound of frequency 540
Hz is whirled in a horizontal circle of radius 2 m
(a) at angle to the perpendicular to the inclined
and at a constant angular speed of 15 rad/s. The
plane AC .
speed of sound is 330 m/s. The ratio of the highest
(b) parallel to the inclined plane AC. to the lowest frequency heard by a listener
(c) vertically downwards standing at rest at a large distance from the center
(d) perpendicular to the inclined plane AC. of the circle is
25. Water containing air bubbles flows without (a) 1.0 (b) 1.1
turbulence through a horizontal pipe which has (c) 1.2 (d) 1.4
a region of narrow cross-section. In this region 30. Monochromatic light passes through a prism.
the bubbles Compared to that in air, inside the prism the light's
(a) move with greater speed and are smaller than (a) speed and wavelength are different but
in the rest of the pipe frequency remains same.
(b) move with greater speed and are larger in size (b) speed and frequency are different but
than in the rest of the pipe wavelength remains same.
(c) move with lesser speed and are smaller than (c) wavelength and frequency are different, but
in the rest of the pipe speed remains same.
(d) move with lesser speed and are of the same (d) speed, wavelength and frequency are all
size as in the rest of the pipe different.
31. The flat face of a plano-convex lens of focal length k 1 k 1
10 cm is silvered. A point source placed 30 cm in (a) 2 k 1 CE (b) 2 k 1 CE
front of the curved surface will produce a
(a) real image 15 cm away from the lens k 2 k 2
(c) CE (d) CE
k 2 k 2
(b) real image 6 cm away from the lens
36. A certain p-n junction, having a depletion region
(c) virtual image 15 cm away from the lens of width 20 m, was found to have a breakdown
(d) virtual image 6 cm away from the lens voltage of 100 V. If the width of the depletion
32. Two identical metallic square loops L1 and L2 are region is reduced to 1 m during its production,
placed next to each other with their sides parallel then it can be used as a Zener diode for voltage
on a smooth horizontal table. Loop L1 is fixed and regulation of -
a current which increases as a function of time is (a) 5 V (b) 10 V
passed through it. Then loop L2 (c) 7.5 V (d) 2000 V
(a) rotates about its center of mass. 37. The half life of a particle of mass 1.6 × 10–26 kg is
(b) moves towards L1. 6.9 s and a stream of such particles is travelling
(c) remains stationary. with the kinetic energy of a particle being 0.05
(d) moves away from L1. eV. The fraction of particles which will decay when
they travel a distance of 1 m is -
33. An electron enters a parallel plate capacitor with
horizontal speed u and is found to deflect by angle (a) 0.1 (b) 0.01
on leaving the capacitor as shown. It is found (c) 0.001 (d) 0.0001
that tan = 0.4 and gravity is negligible 38. A 160 watt light source is radiating light of
wavelength 6200 A uniformly in all directions.
The photon flux at a distance of 1.8 m is of the
order of (Planck's constant 6.63 × 10–34 J-s)
(a) 102 m–2 s–1 (b) 1012 m–2 s–1
(c) 1019 m–2 s–1 (d) 1025 m–2 s–1
If the initial horizontal speed is doubled, then tan 39. The wavelength of the first Balmer line caused
will be by a transition from the n = 3 level to the n = 2
(a) 0.1 (b) 0.2 level in hydrogen is 1. The wavelength of the
(c) 0.8 (d) 1.6 line caused by an electronic transition from n = 5
34. Consider a spherical shell of radius R with a total to n = 3 is -
charge +Q uniformly spread on its surface (center 375 125
(a) 1 (b) 1
of the shell lies at the origin x= 0). Two point 128 64
charge, +q and – q are brought, one after the 64 128
other, from far away and placed at x = – a/2 and (c) 1 (d) 1
125 375
x = + a/2 (a < R), respectively. Magnitude of the 40. The binding energy per nucleon of 5B10 is 8.0 MeV
work done in this process is and that of 5B11 is 7.5 MeV. The energy required
(a) (Q + q)2 /4 a (b) zero to remove a neutron from 5B11 is (mass of electron
(c) q /4
2
a (d) Qq /4 a and proton are 9.11 × 1031 kg and 1.67 × 1027 kg,
35. Two identical parallel plate capacitors of respectively) -
capacitance C each are connected in series with (a) 2.5 MeV (b) 8.0 MeV
a battery of emf, E as shown. If one of the (c) 0.5 MeV (d) 7.5 MeV
capacitors is now filled with a dielectric of
dielectric constant k, the amount of charge which
will flow through the battery is (neglect internal 41. When 1.88 g of AgBr(s) is added to a 10 –3 M
resistance of the battery) aqueous solution of KBr, the concentration of Ag
is 5 × 10–10 M. If the same amount of AgBr(s) is
added to a 10–2 M aqueous solution of AgNO3, the
concentration of Br– is
(a) 9.4 × 10–9 M (b) 5 × 10–10 M
(c) 1 × 10–11 M (d) 5 × 10–11 M
42. Aniline reacts with excess Br2/H2O to give the 48. In the following reaction -
major product

the major product X is-


(a) (b)

(a) (b)

(c) (d) (c) (d)

49. Given the structure of D-(+)-glucose as


43. The metal with the highest oxidation state present
in K2CrO4, NbCl5 and MnO2 is -
(a) Nb (b) Mn
(c) K (d) Cr
44. The number of geometrical isomers of
[CrCl2(en)(NH3)2], where en = ethylenediamine,
is -
(a) 2 (b) 3 The structure of L-(–)-glucose is
(c) 4 (d) 1 (a) (b)

45. The element that combines with oxygen to give


an amphoteric oxide is -
(a) N (b) P
(c) Al (d) Na
46. The Arrhenius plots of two reactions, I and II are
shown graphically -
(c) (d)

50. In a cubic close packed structure, fractional


The graph suggests that - contributions of an atom at the corner and at the
face in the unit cell are, respectively -
(a) EI > EII and AI > AII
(a) 1/8 and 1/2 (b) 1/2 and 1/4
(b) EII > EI and AII > AI
(c) 1/4 and 1/2 (d) 1/4 and 1/8
(c) EI > EII and AII > AI
51. The equilibrium constant Kc of the reaction, 2A
(d) EII > EI and AI > AII
B + C is 0.5 at 25°C and 1 atm. The reaction
47. Ni(CO)4 is will proceed in the backward direction when
(a) tetrahedral and paramagnetic concentrations [A], [B] and [C] are, respectively -
(b) square planar and diamagnetic (a) 10–3, 10–2 and 10–2 M
(c) tetrahedral and diamagnetic (b) 10–1, 10–2 and 10–2 M
(d) square planar and paramagnetic (c) 10–2, 10–2 and 10–3 M
(d) 10–2, 10–3 and 10–3 M
52. Major products formed in the reaction of t-butyl 58. The latent heat of melting of ice at 0 °C is
methyl ether with HI are - 6 kJ mol–1. The entropy change during the melting
in J K–1 mol–1 is closest to -
(a) H3C –– I and OH
(a) 22 (b) 11
(c) –11 (d) –22
(b) and H3C –– OH
59. The major product of the following reaction
(c) H3C –– OH and I
is
(d) and H3C –– OH
(a)
53. If the molar conductivities (in S cm mol ) of
2 –1

NaCl, KCl and NaOH at infinite dilution are 126,


150 and 250 respectively, the molar conductivity
of KOH (in S cm2 mol–1) is – (b)
(a) 526 (b) 226
(c) 26 (d) 274
54. 4-Formylbenzoic acid on treatment with one
equivalent of hydrazine followed by heating with (c)
alcoholic KOH gives the major product -
(a) (b)

(d)

2
60. The energies of dxy and d z orbitals in octahedral
(c) (d) and tetrahedral transition metal complexes are
such that -
(a) E (d xy) > E ( d z2 ) in both tetrahedral and
octahedral complexes
(b) E (d xy) < E ( d z2 ) in both tetrahedral and
octahedral complexes
(c) E (dxy) > E ( d z2 ) in tetrahedral but E (dxy) < E
( d z2 ) in octahedral complexes
55. Two elements, X and Y, have atomic numbers 33 (d) E (dxy) < E ( d z2 ) in tetrahedral but E (dxy) > E
and 17, respectively. The molecular formula of a ( d z2 ) in octahedral complexes
stable compound formed between them is -
(a) XY (b) XY2
(c) XY3 (d) XY4 61. In which of the following types of glands is the
56. The number of moles of KMnO4 required to secretion collected inside the cell and discharged
oxidize one equivalent of KI in the presence of by disintegration of the entire gland ?
sulfuric acid is - (a) Apocrine (b) Merocrine
(a) 5 (b) 2 (c) Holocrine (d) Epicrine
(c) 1/2 (d) 1/5 62. Which one of the following interactions does NOT
57. Three successive measurements in an experiment promote coevolution ?
gave the values 10.9, 11.4042 and 11.42. The (a) Commensalism
correct way of reporting the average value is - (b) Mutualism
(a) 11.2080 (b) 11.21 (c) Parasitism
(c) 11.2 (d) 11 (d) Interspecific competition
63. Stratification is more common in which of the 73. Rhinoviruses are the causative agents of
following ? (a) Diarrhoea (b) AIDS
(a) Deciduous forest (b) Tropical rain forest (c) Dengue (d) Common cold
(c) Temperate forest (d) Tropical savannah 74. What is the genetic material of Ebola virus ?
64. Where is the third ventricle of the brain located ? (a) Single-stranded DNA
(a) Cerebrum (b) Cerebellum (b) Double-stranded RNA
(c) Pons varoli (d) Diencephalon (c) Single-stranded RNA
65. Which of the following is the final product of a (d) Double-stranded DNA
gene ?
75. Name the terminal acceptor of electrons in the
(a) a polypeptide only mitochondrial electron transport chain
(b) an RNA only (a) Nitrate (b) Fumarate
(c) either polypeptide or RNA (c) Succinate (d) Oxygen
(d) a nucleotide only 76. Two tubes labelled 'P' and 'Q' contain food stuff.
66. Forelimbs of whales, bats, humans and cheetah Tube 'P' gave positive test with Benedict's solution
are examples of which of the following processes ? while tube 'Q' gave positive test with Nitric acid.
(a) Divergent evolution Which of the following is correct ?
(b) Convergent evolution (a) Tube 'P' contains sugar; tube 'Q' contains
protein
(c) Adaptation
(b) Tube 'P' contains protein; tube 'Q' contains
(d) Saltation
sugar
67. Which of the following results from conjugation
(c) Both, tube 'P' and tube 'Q' contain sugar
in Paramecium ?
(d) Both, tube 'P' and tube 'Q' contain protein
(a) Cell death (b) Cell division
77. How many linear DNA fragments will be produced
(c) Budding (d) Recombination
when a circular plasmid is digested with a
68. In an experiment investigating photoperiodic restriction enzyme having 3 sites ?
response, the leaves of a plant are removed. What
(a) 4 (b) 5
is the most likely outcome ?
(c) 3 (d) 2
(a) Photoperiodism is not affected
78. If the humidity of the atmosphere suddenly
(b) Photoperiodic response does not occur
increases substantially, the water flow in the
(c) The plant starts flowering xylem will -
(d) The plant starts to grow taller (a) increase
69. Testosterone is secreted by which endocrine part (b) decrease
of testis ?
(c) remain unaltered
(a) Leydig cells (b) Seminiferous tubules
(d) increase sharply and then reduce slowly to
(c) Tunica albugenia (d) Sertoli cells the preexisting level
70. The mutation of a purine to a pyrimidine is known 79. Which one of the following is the complementary
as sequence for the DNA with 5 -CGTACTA-3
(a) transition (b) frame shift (a) 5 -TAGTACG-3
(c) nonsense (d) transversion (b) 5 -ATCATGC-3
71. Which of the following is secreted at the ends of (c) 5 -UTCUTGC-3
an axon ?
(d) 5 -GCUAGCA-3
(a) Ascorbic acid (b) Acetic acid
80. A diploid plant has 14 chromosomes, but its egg
(c) Acetyl choline (d) Acetyl CoA cell has 6 chromosomes. which one of the following
72. A bacterial colony is produced from is the most likely explanation of this ?
(a) a single bacterium by its repetitive division (a) Non-disjunction in meiosis I and II
(b) multiple bacterium without replication (b) Non-disjunction in meiosis I
(c) clumping of two to three bacteria (c) Non-disjunction in mitosis
(d) a single bacterium without cell division (d) Normal meiosis
86. An ellipse inscribed in a semi-circle touches the
circular arc at two distinct points and also touches
81. Let n 3 be an integer. For a permutation the bounding diameter. Its major axis is parallel
= (a 1 , a 2, ....., a n ) of (1, 2, ....., n) we let to the bounding diameter. When the ellipse has
f (x) = anxn–1 + an–1xn–2 + ......+ a2x + a1. Let S be the maximum possible area, its eccentricity is -
the sum of the roots of f (x) = 0 and let S denote
the sum over all permutations of (1, 2, ....., n) 1 1
(a) (b)
of the numbers S . Then - 2 2
(a) S < . n! (b) – n! < S < 0 1 2
(c) 0 < S < n! (d) n! < S (c) (d)
3 3
82. If n is a positive integer and 1 is a cube root of /2
unity, the number of possible values of x n cos x dx , where n is a non-negative
87. Let In =
0
n
n k
integer.
ek 0 k
(a) 2 (b) 3
In In 2
Then n! n 2 ! equals -
n 2
(c) 4 (d) 6
83. Suppose a parabola y = ax2 + bx + c has two x
intercepts, one positive and one negative, and its (a) e 2 1 (b) e 2 1
2
vertex is (2, –2). Then which of the following is
true?
(c) e 2 (d) e 2
(a) ab > 0 2
(b) bc > 0 88. For a real number x let [x] denote the largest
integer less than or equal to x. The smallest
(c) ca > 0
positive integer n for which the integral
(d) a + b + c > 0 n

84. Let n 3 and let C1, C2, ...., Cn, be circles with x x dx exceeds 60 is -
radii r1, r2, ...., rn, respectively. Assume that Ci 1

and Ci+1 touch externally for 1 i n – 1. It is also (a) 8 (b) 9


given that the x-axis and the line y = 2 2x + 10 (c) 10 (d) [602/3]
are tangential to each of the circles. Then r1, r2, 89. Choose a number n uniformly at random from
....., rn are in - the set {1, 2, ..., 100}. Choose one of the first seven
days of the year 2014 at random and consider n
(a) an arithmetic progression with common
consecutive days starting from the chosen day.
difference 3 2 What is the probability that among the chosen n
(b) a geometric progression with common ratio days, the number of Sundays is different from
the number of Mondays?
3 2
(c) an arithmetic progression with common 1 2
(a) (b)
difference 2 2 7
3
12 43
(d) a geometric progression with common ratio (c) (d)
49 175
2 3
90. Let S = {(a, b)|a, b Z, 0 a, b 18}. The number
85. The number of integers n for which 3x3 – 25x + n of lines in R2 passing through (0, 0) and exactly
= 0 has three real roots is - one other point in S is -
(a) 1 (a) 16
(b) 25 (b) 22
(c) 55 (c) 28
(d) infinite (d) 32
96. The angles of incidence and refraction of a
monochromatic ray of light of wavelength at an
91. A solid sphere spinning about a horizontal axis air-glass interface are i and r, respectively. A
with an angular velocity is placed on a parallel beam of light with a small spread in
horizontal surface. Subsequently it rolls without wavelength about a mean wavelength is
slipping with an angular velocity of - refracted at the same air-glass interface. The
(a) 2 /5 (b) 7 /5 refractive index of glass depends on the
wavelength as ( ) = a + b/ 2 where a and b are
(c) 2 /7 (d)
constants. Then the angular speed in the angle
92. Consider the system shown below. of refraction of the beam is
sin i
(a) 3
corr
2b
(b) 3

A horizontal force F is applied to a block X of 2b tan r


mass 8 kg such that the block Y of mass 2 kg (c)
a 3 b
adjacent to it does not slip downwards under 2
gravity. There is no friction between the 2b a b/ sin r
(d)
3
horizontal plane and the base of the block X. The
coefficient of friction between the surfaces of
blocks X and Y is 0.5. Take acceleration due to 97. What are the charges stored in the 1 F and 2 F
gravity to be 10 ms–2. The minimum value of F is capacitors in the circuit below, once the currents
become steady ?
(a) 200 N (b) 160 N
(c) 40 N (d) 240 N
93. The maximum value attained by the tension in
the string of a swinging pendulum is four times
the minimum value it attains. There is no slack
in the string. The angular amplitude of the
pendulum is
(a) 90° (b) 60°
(c) 45° (d) 30° (a) 8 C and 4 C respectively
94. One mole of a monoatomic ideal gas is expanded (b) 4 C and 8 C respectively
by a process described by PV3 = C where C is a (c) 3 C and 6 C respectively
constant. (d) 6 C and 3 C respectively
The heat capacity of the gas during the process is 98. A 1.5 kW (kilo-watt) laser beam of wavelength
given by (R is the gas constant). 6400 Å is used to levitate a thin aluminium disc
5 of same area as the cross section of the beam.
(a) 2 R (b) R
2 The laser light is reflected by the aluminium disc
3 without any absorption. The mass of the foil is
(c) R (d) R close to
2
95. A concave mirror of radius of curvature R has a (a) 10–9 kg (b) 10–3 kg
circular outline of radius r. A circular disc is to (c) 10–4 kg (d) 10–6 kg
be placed normal to the axis at the focus so that
99. When ultraviolet radiation of a certain frequency
it collects all the light that is reflected from the
falls on a potassium target, the photoelectrons
mirror from a beam parallel to the axis. For r <<
released can be stopped completely by a retarding
R, the area of this disc has to be at least
potential of 0.6 V. If the frequency of the radiation
r6 r4 is increased by 10%, this stopping potential rises
(a) (b)
4R 4 4R 2 to 0.9 V. The work function of potassium is
r5 r4 (a) 2.0 eV (b) 2.4 eV
(c) (d)
4R 3 R2 (c) 3.0 eV (d) 2.8 eV
100. The dimensions of Stefan-Boltzmann constant 103. The molar enthalpy change for H2O(l) H2O(g)
can be written in terms of Planck's constant h, at 373 K and 1 atm is 41 kJ/mol. Assuming ideal
Boltzmann constant kB and the speed of light c as behaviour, the internal energy change for
= h kB c . Here vaporization of 1 mol of water at 373 K and 1 atm
(a) = 3, = 4 and = –3 in kJ mol–1 is :
(b) = 3, = . 4 and = 2 (a) 30.2 (b) 41.0
(c) = .3, = 4 and = –2 (c) 48.1 (d) 37.9
(d) = 2, = .3 and = –1 104. The equilibrium constant (Kc) of two reactions H2
+ I2 2HI and N2 + 3H2 2NH3 are 50 and
1000, respectively. The equilibrium constant of
101. In the reaction sequence the reaction N2 + 6HI 2NH3 + 3I2 is closest to:
(a) 50000 (b) 20
(c) 0.008 (d) 0.005
105. Given that the bond energies of : N N is 946 kJ
mol–1, H–H is 435 kJ mol–1, N–N is 159 kJ mol–1,
X and Y are, respectively,
and N–H is 389 kJ mol–1, the heat of formation of
(a) hydrazine in the gas phase in kJ mol–1 is :
(a) 833 (b) 101
(c) 334 (d) 1268
106. The radius of K is 133 pm and that of Cl is 181
+

pm. The volume of the unit cell of KCl expressed


in 10–22 cm3 is :
(a) 0.31 (b) 1.21
(b)
(c) 2.48 (d) 6.28
107. The reaction, K2Cr2O7 + m FeSO4 + n H2SO4
Cr2(SO4)3 + p Fe2(SO4)3 + K2SO4 + q H2O when
balanced, m, n, p and q are, respectively :
(a) 6, 14, 3, 14 (b) 6, 7, 3, 7
(c) 3, 7, 2, 7 (d) 4, 14, 2, 14
108. The standard free energy change (in J) for the
(c) reaction 3Fe2+ (aq) + 2Cr(s) = 2Cr3+(aq) + 3Fe(s)
0 0
given EFe2 / Fe = –0.44 V and ECr3 / Cr = –0.74 V is
(F = 96500 C)
(a) 57,900
(b) –57,900
(d) (c) –173,700
(d) 173, 700
109. Calcium butanoate on heating followed by
treatment with 1,2-ethanediol in the presence of
catalytic amount of an acid, produces a major
product which is :
102. The density of acetic acid vapour at 300 K and 1
atm is 5 mg cm–3. The number of acetic acid (a)
molecules in the cluster that is formed in the gas
phase is closest to
(a) 5 (b) 2 (b)
(c) 3 (d) 4
(c) (a) The mutation reverses spontaneously in the
third generation
(b) The mutation occurs on the X chromosome
(d) and is both recessive and lethal
(c) The mutation occurs on the X chromosome
and is both recessive and dominant
(d) The mutation occurs on an autosome and is
dominant
115. A circular plasmid of 10,000 base pairs (bp) is
digested with two restriction enzymes, A and B,
to produce a 3000 bp and a 2000 bp bands when
110. XeF 6 on complete hydrolysis yields ‘X’. The visualized on an agarose gel. When digested with
molecular formula of X and its geometry, one enzyme at a time, only one band is visible at
respectively, are : 5000 bp. If the first site for enzyme A (A1) is
present at the 100th base, the order in which the
(a) XeO2 and linear
remaining sites (A2, B1 and B2) are present is -
(b) XeO3 and trigonal planar
(a) 3100, 5100, 8100 (b) 8100, 3100, 5100
(c) XeO3 and pyramidal
(c) 5100, 3100, 8100 (d) 8100, 5100,. 3100
(d) XeO4 and tetrahedral
116. After meiosis-II, daughter cells differ from the
parent cells and each other in their genotypes.
This can occur because of which one of the
111. Following the cell cycle scheme given below, what following mechanism(s) ?
is the probability that a cell would be in M-phase
(a) Only synaptic crossing over
at any given time ?
(b) Only crossing over and independent
assortment of chromosomes
(c) Only crossing over and chromosomal
segregation
(d) Crossing over, independent assortment and
segregation of chromosomes
117. A desert lizard (an ectotherm) and a mouse (an
(a) 1/24 (b) 1/12 endotherm) are placed inside a chamber at 15 ºC
and their body temperature [T(L) for the lizard
(c) 1/6 (d) 1/2 and T(M) for the mouse] and metabolic rates [M(L)
112. A flower with Tt genotype is cross-pollinated by TT for the lizard and M(M) for the mouse] are
pollens. What will the genotypes of the resulting monitored. Which one of the following is correct -
endosperm and embryo, respectively, be ? (a) T(L) and M(L) will fall while T(M) and M(M)
(a) TTT, (TT + Tt) (b) (TTT + TTt), TT will increase
(c) TTt, Tt (d) TTt, (TT + Tt) (b) T(L) and M(L) will increase while T(M) and
113. A new life form discovered on a distant planet M(M) will fall
has a genetic code consisting of five unique (c) T(L) and M(L) will fall, T(M) will remain same
nucleotides and only one stop codon. If each codon and M(M) will increase
has four bases, what is the maximum number of (d) T(L) and M(L) will remain same and T(M) and
unique amino acids this life form can use ? M(M) will decrease
(a) 624 (b) 20 118. In Griffith's experiments mice died when injected
(c) 124 (d) 3124 with -
114. A spontaneous mutation results in a couple having (a) heat killed S-strain
only female progeny. When the daughter marries (b) heat killed S-strain combined with R-strain
and has children, none of them are males.
However, in the third generation there are few (c) heat killed R-strain
male offspring. What is the most likely (d) live R-strain
explanation of this observation -
119. Human height is a multigenic character. If the (a) P
heights of all the individuals living in a metropolis (b) Q
are measured and the percentages of the
(c) R
population belonging to a specific heat are plotted
as shown below, which of the plots would (d) S
represent the most realistic distribution - 120. If mitochondria isolated from a cell are first placed
without carbon source in a buffer at pH 8.0 and
then transferred to a buffer at pH4 , it will lead
to -
(a) an increase in intra-mitochondrial acidity
(b) a decrease in intra-mitochondrial acidity
(c) blockage of ATP synthesis
(d) synthesis of ATP

1. (d) 2. (d) 3. (c) 4. (b) 5. (b) 6. (d) 7. (c) 8. (b) 9. (b) 10. (d)
11. (b) 12. (d) 13. (c) 14. (b) 15. (b) 16. (b) 17. (c) 18. (b) 19. (b) 20. (c)
21. (c) 22. (b) 23. (a) 24. (d) 25. (b) 26. (a) 27. (d) 28. (b) 29. (c) 30. (a)
31. (b) 32. (d) 33. (a) 34. (c) 35. (b) 36. (a) 37. (d) 38. (c) 39. (b) 40. (a)
41. (d) 42. (a) 43. (d) 44. (b) 45. (c) 46. (a) 47. (c) 48. (a) 49. (b) 50. (a)
51. (a) 52. (c) 53. (d) 54. (b) 55. (c) 56. (d) 57. (c) 58. (a) 59. (a) 60. (c)
61. (c) 62. (d) 63. (b) 64. (d) 65. (c) 66. (a) 67. (d) 68. (b) 69. (a) 70. (d)
71. (a) 72. (a) 73. (d) 74. (c) 75. (d) 76. (a) 77. (c) 78. (b) 79. (a) 80. (b)
81. (b) 82. (c) 83. (b) 84. (d) 85. (c) 86. (d) 87. (a) 88. (b) 89. (*) 90. (*)
91. (c) 92. (a) 93. (b) 94. (d) 95. (a) 96. (c) 97. (b) 98. (d) 99. (b) 100. (c)
101. (a) 102. (b) 103. (d) 104. (c) 105. (b) 106. (c) 107. (b) 108. (c) 109. (a) 110. (c)
111. (b) 112. (a) 113. (a) 114. (b) 115. (c) 116. (d) 117. (c) 118. (b) 119. (a) 120. (d)
x x
if x < y =0 0 true
y y
1. (d) Area(Ci ) = r02 + r12 + r22 + r32 +...
i 0 x x
if x y y y always true
3. (c) If n is even
n
An Cn / 2
n 1
2
An 1 Cn 1 1
2

so for all even n, given relation is true.


If n is odd
n
Cn 1
Area of Cn = rn2 = ( 2 rn–1)2 An 2 2n
n 1
2 2 An 1 Cn 1
n 1
rn2 = r n–1 2

2 2 which satisfies only for n = 19


so r1 2 = 2 r0 2, r22 = 2 r12 r0 2
4. (b)
2 2 2 2 22 2
r3 r2 r0
3

Area Ci
i 0

2 2 2 2
= r0 2 r0 2 . r0 ....

r0 2 2
r0 2
r0 1 equation of OP
2 2
1 y = x tan
2 point Q is (b cot , b)
2 point P is y = b d sin
2. (d) (A) [x + y] . [x] + [y] r sin =b d sin
let x = 0.1 (r d) sin =b
y = 0.9 6. (d)
[0.1 + 0.9] [0.1] + [0.9]
1 0+0 (which is not possible)
(B) [xy] [x ] [y]
1 A'S' = SS' = SA
x = 2; y =
2 2ae = a – ae
1 1 3ae = a
2. 2
2 2
e = 1/3
1 0 wrong
b2 1 b2 8
(C) [2x] 2[x] Now, 1
a2 9 a2 9
x = 0.99 [20.99] 2[0.99]
8 8
[20.99] 2º = 1 (which is not possible) a 3
a2 9
x x
(D)
y y
given x, y 1
7. (c) 9. (b) discontinuous at x = 2
x 5
5
x 5 x 2 6x 5
f(f(x)) = f x 5
x 2 9 x
2
x 2
At x = 9 it is discontinuous
10. (d) f (x) = [x] sin x

2
cos B
3
16y 2 16y 2 AC2
Also cos B =
2.4y.4y
2 32y 2 AC2 Not differentiable for x R
3 32y 2 Not symmetry about x = 0.
3
64y +16y – 3AC
2 2 2
f x dx 0
3AC2 = 32y2 3
f (x) = will have infinite soln
32 2
AC2 y
3 12. (d) Q f x 0
1
4 2 f x
2
dx 100 not necessarily true.
AC y
3 0

because (f(x))2 can take very high values


BC 4y 3
Now, area bounded by (f(x))2, x-axis and x = 0 to
AC 4 2y 2
1 may cross 100.
3
x
1 1 13. (c) ƒ(x) = x + f t dt
8. (b) cos4x – sin4x =
sin 2 x cos 2 x 0

cos2 x sin 2 x Now differentiating wrt, 'x' we get


(cos2x – sin2x) =
sin 2 x cos2 x f' x 1 f x f' x f x 1
4 cos 2x
cos 2x e xf ' x f x e x
e x

sin 2 2x
cos 2x (1 – 4 cosec22x) = 0 d x x
f x e e
1 dx
cos 2x = 0 or cosec22x =
4 e x
x
f x e c
1
2x = 2n ±
2 f(x) = – 1 + cex
f(0) = 0 = – 1 + ce0 c=1
x=n ±
4 f(x) = e – 1 x

At n = 0, x = Now f(x) + f(y) + f(x) f(y)


4 = ex – 1 + ey – 1 + (ex – 1) (ey – 1)
5 3 = ex – 1 + ey – 1 + ex.ey – ey – ex + 1
and at n = 1; x = ,
4 4
= ex.ey – 1 = ex + y – 1
7 = f(x + y)
and at n = 2, x =
4
n
x12 x 22
14. (b) cos 2 x x x1 x 2 x 32 ... x 2n
0 ˆ 2 2 2 ....(2)
1 2 n
= cos 0 dx cos 2 x 1 dx x 12 x 22 x 12 x 22 2x1 x 2
0 0
2
ˆ2
n 2n
3

+ cos 4 x 2 dx x 2
1 x 2
2 2x 1 x 2
2
2n
n
2
+..... cos 2 n 1 x n 1 dx x1 x2
n 1 0
2n
2 3
1 0 cos 2 x dx cos 4 x dx ....... ˆ & ˆ
1 2 17. (c)
n

cos 2 n 1 x dx
n 1

2 3
sin 2 x sin 4 x
1
2 1 4 2

n
sin 2 n 1 x
.......
2 n 1 n 1

=1+0=1
a, b, c are unit vectors
6 5 6 1 1 5 6 1 1 1 1 5
15. (b) P . . . . . . . . . ....
a^b b^c c^a
6 6 6 6 6 6 6 6 6 6 6 6
3
5 5 5
......... o a b c
6 63 65 centre p 4
5/6 30 6
1 35 7 Now angle between AP & BP
1
36 AP.BP
cos
x1 x2 .... x n |AP||BP|
16. (b)
n
a b c a b c
y1 y2 .... y n a . b
ˆ 4 4
n
x1 x2 x1 x2 a b c a b c
x3 ... x n a b
2 2 4 4
n
b c 3a . a c 3b
x1 x2 .... x n
ˆ ˆ
n b c 3a a c 3b

2
x 2i 2 a.b b . c 3b 2 a.c c2 3b.c 3a 2 3a.c 9a.b
n b 2
c 2
9a 2
2b.c 6a.c 6a.b
2 x12 x 22 .... x 2n 2
....(1) 1 1 1 3 3 9
n 31 3
2 2 2 2 2 2
y 12 y 22 .... y 2n 1 1 9 1 3 3
ˆ2 2

n
2 2
5 3 1
x1 x2 x1 x2 2 2 6 3
x 3 ....x n
2 2 2
1 1
n cos
3
18. (b) 3x + 4y + 5 = 19 I
5 (3x + 4y + 5) 131
5 19 I 131
Case (i) 3x + 4y + 5 = 19 Case (ii) 3x + 4y + 5 = 38 Case (iii) 3x + 4y = 52
3x + 4y = 14 3x + 4y = 33 52 3x
x=
14 3x 4y = 33 – 3x 4
y=
4 33 3x x = 0, 4, 8, 12, 16
y=
x=2 4
x = 3, 7, 11
Case (iv) 3x + 4y = 71 Case (v) 3x + 4y = 90 Case (vi) 3x + 4y = 109
4y = 71 – 3x 90 3x 109 3x
y= y=
71 3x 4 4
y=
4 x = 6, 10, 14, 18 x = 15 is only possibility.
x = 1, 5, 9, 13, 17
Total Solution = 19

19. (b) By taking different values of a & k. J 2 36J 2


option (B) is possible. 2m 24m
5 5 3! 48J 2 2J 2
20. (c) C2 .2! C3 2!
1! 2! 24m m
22. (b) If perfect rolling (solid cylinder P)
5 4! 4! 2!
C4 2! According to energy conservation law
1!3! 2!2! 2! 2
1 1 vp
5 5! 5! mgh = mv 2p I
C5 2! 2! 2 2 R
1!4! 2!3!
Here,
= 10(2) + 10(6) + 5(8 + 6) + (10 + 20)
I moment of inertia, R Radius
= 20 + 60 + 70 + 30 = 180 2
Vp
mR
21. (c) mass = m, length = 2L fri = 0 I= ,
2 R
2
1 2 1 mR 2 Vp
mgh = mv p+
2 2 2 R
1 1 1 3
mgh = mv 2p 1 mv 2p
J = mv .... (1) 2 2 2 2
where v is the velocity of centre of mass. 3
mgh = mv2p
After impulse rod get angular velocity 4
4
Angular impulse = I v2p gh
2 3
m 2L If sliding without friction (solid cylinder Q)
J L ....(2)
12 According to energy conservation law
mL 1
J mgh = mv Q
2
3 2
3J
v2Q 2gh
mL
J from equation (1) and (2)
from equation (1); v = v 2Q
m 2gh 3
1 1 v 2P 4 2
Total Kinetic energy = KE = mv
2
I 2
gh
2 2 3
1 J 2 1 m 4L2 9J 2 vQ2 3
m 2
2 m 2 12 m 2 L2 v 2
2
P
23. (a) V(r) = kr, R = radius 25. (b) According to Bernoulli theorem
dU d In the region of narrow cross section of pipe,
F=– qV q constant
dr dr KE of fluid will be greater and pressure
v kr energy will be lesser.
dV dV
F=–q ; F = – qk, k less pressure results into larger in size of
dr dr air bubble and greater KE results its greater
speed.
m 2R = – qk
2
27. (d) Increase in length of each liquid is same
2 l= l
m R = – qk
T
VHg VBro min e
2
m 4 R d12 d 22
= –qk
T2 V Hg V Bro min e
T R d12 d2
2
2
T R d1 Hg 18 10 5
24. (d) Block slides downward along the inclined d2 Bro min e 108 10 5
plane with acceleration g sin .
d1 1
0.4
d2 6
28. (b) PV2 = C
nRT
V2 C
V
TV = C
T1 V 1 = T 2 V 2
If temperature increases, volume decreases
and vice versa
V2 > V1 then T2 < T1.
29. (c) f = 540 Hz, = 15 rad/s

v1 and v2 are speed of whistle


mg[1 – sin2 ] mg cos2
|v1| = |v2| = r = 15 × 2 = 30 m/s
mg cos2
tan = Maximum frequency heard
mg sin cos
tan = cot Here,

tan = tan (90 – ) f original frequency (540 Hz)


v speed of sound
= (90 – )
string is perpendicular to inclined plane. vs speed of whistle
v
fmax = f
v vs
Minimum frequency heard
v
f min f
v vs
f max v vs 330 30 360 When current through L1 increases then flux
f min v vs linked through L2 will increase.
330 30 300
According to lenz law L2 will move away.
f max 6
1.2 33. (a) tan = 0.4, speed = u
f min 5
30. (a) On refraction of light, frequency remain
unchanged. However speed and wavelength
get change.
31. (b)

Horizontal displacement =
f = 10 cm
t=
After silvering of flat face lens behave as mirror u
of focal length feq. eE l
vy = uy + at 0
m u
eE l
vy
m u
vx remain same and it is equal to u
eE l l vy
eEl
tan =
vx m u u mu 2
1 1 1 1
1
f eq f1 f2 f3 tan
u2
1 2 1 When speed u is doubled then tan will
f eq f1 f2 1
become th.
4
1 2 1
0.4
f eq 10 tan = = 0.1
4
feq = 5
a
34. (c) Radius = R, x = ± a<R
2

1 1 1
mirror formula
f u v
1 1 1
5 30 v
Q2
v = –6 cm PEi = Initial energy of system = (self
8 0 R
Image is real and 6 cm away from silvered energy of shell)
lens.
PEf = Final energy of system
32. (d)
= Q2 q q kQ q kQ q
8 0 R 4 0 R R R
Q2 q2
8 0 R 4 0 a
(self energy of shell) (Interaction energy 1
between various charges) 0.05 × 1.6 × 10–19 = × 1.6 × 10–26 × v2
2
q2 0.05 × 2 × 107 = v2
Work done = PEf – PEi 106 = v2
4 0 a
v = 1000 m/sec
q2
Magnitude of work done = 1
4 a time taken to travel a distance of 1 m is
0
v
35. (b) 1
0.001 sec
1000
Half life of radioactive material = 6.9 sec
T1/2 = 0.693

0.693
0.1
6.9
Initial charge on both capacitor = CE
2 fraction of particle decay in 0.001 sec or
1
sec = 1 – e– t
1000
1
0.1
1 e 1000

1
1 e 10000
0.0001
38. (c) P = 160 watt, = 6200 Å, d = 1.8 m
New charge on each capacitor =
kC
E P
k 1 Intensity of light at 1.8 m = 2
Change in charge on C is supplied by battery 4 1.8
kC CE 160
Charge supply by battery = E I 2
k 1 2 4 1.8
k 1 Photon flux = Number of photon per unit area.
CE
k 1 2 I
hc
k 1
CE
2 k 1
I
Charge passes through battery is change hc
supply by battery 10
160 6200 10
k 1 4 1.8
2
6.63 10 34
3 108
Ans. CE
2 k 1
1.22 1019
36. (a) Break down voltage width of depletion region. 39. (b) n = 3 to n = 2
When width reduce to 1 m thus become
1 1 1 1
times then break down voltage also R
20 1 n12 n 22
1
become times thus it become 5 volt. So 1 1 1
20 R
Zener diode can used for voltage regulation 1 22 32
of 5 volt.
1 1 1
37. (d) t 1 = 6.9, m = 1.6 × 10–26 kg R
1
4 9
2

1 1 5
mv 2 R ....(1)
KE = 36
2 1
1 1 1 49. (b) Mirror image of D-glucose
R
2
9 25 1 1
1 16 50. (a) Contribution at Corner =
R no. of corners 8
9 25 ....(2)
2
1
From (1) & (2) Contribution of face =
2
2 5 16
2 2
36 9 25 10 10
1 51. (a) Q 2
100
3
10
5 9 25 125
2 1
36 16 64 KC = 0.5
40. (a)
As Q > KC, reaction moves backward.

CH3 CH3
HI
52. (c) CH3 – C – OCH3 CH3 – C – I – CH3OH
formation of 5B10 will release energy E2 CH3 CH3
E1 = Binding energy of 5B 11

53. (d) KOH KCl NaOH NaCl


7.5 × 11 MeV = 82.5 MeV m m m m

E2 = Binding energy of 5B10 = 8.0 × 10 = 80 MeV = 150 + 250 – 126 = 274


Energy given = E1 – E2 = 82.5 – 80 = 2.5 MeV CHO CH3
41. (d) K SP Ag Br NH2–NH2
54. (b)
alc. KOH
10 3 13
K SP 5 10 10 5 10
COOH COOH
Now, 5 10 13 2
10 [Br ] +3 –1
55. (c) X Y
11
Br 5 10
NH2 NH2
XY3
Br Br
Br2/H 2O
42. (a) 56. (d) MnO 4 I Mn 2 I2
n 5 n 1
Br
Moles of MnO4– = 1, moles of I– = 5
43. (d) K 2CrO4 Oxi. No. of Cr 6
57. (c)
44. (b) 2-Trans isomers
58. (a) At equilibrium, G = 0
+
H=T S
1-Cis isomer
H 6KJ 6000J
S= 22 J/K
3 Tfreezing 273K 273K
45. (c) 2Al O2 Al 2 O3
2

47. (c) CO is strong field ligand so pairs 4p unpaired CH3


electrons of Ni and hence hybridisation is sp3. CH3
Cu,
O 59. (a)
(i) O3 OH Ulmann
48. (a) O
Reaction
(ii) H2O, Zn O Intra-
molecular I
Aldol
CH3 CH3
63. (b) Stratification = Vertical zonations. As we know that, A.M. H.M.
66. (a) Homologous organs represent divergent 1 1 1
a1 a2 ...... a n .... n2
evolution. a1 a2 an
67. (d) Conjugation results in recombination in S n n 1
paramecium.
n
68. (b) Leaves are the site for perception of 82. (c) n
Ck k
= nC0 + nC1 + .....+ nCn n

photoperiods. k 0

= (1 + )n = (– 2)n = (–1)n 2
n
69. (a) testosterone is secreted by leydig cells of
2 n
testis. e
1
n 2n
e
70. (d) Transversion
n
Purine Pyrimidine cos
4
3
isin
4
3
e
(A, G) (C, U, T)
71. (a) Acetylcholine is a neuro transmitter. n n
cos ison
72. (a) A single bacterium divides repeatedly to e 3 3

produce a colony.
n
73. (d) Rhinovirus causes common cold. e
cos
3

74. (c) Ebola virus has single stranded RNA.


75. (d) Terminal electron accepter in mitochondrial. = {e1, e1/2, e–1/2, e–1}
Electron transport system is oxygen. Four values.
76. (a) Benedict test – For presene of reducing sugar 83. (b) The graph according to the question is
(monosaccharides)
Nitric acid test – Proteins.

1 2
77. (c) Restriction sites
3

3-Fragments will be produced.

1 Clearly it can be observed


78. (b) Transpiration Humidity c<0
a>0
Hence in humidity will the transpiration.
b
79. (a) 5’ – GTACTA-3’ >0 –b>0 b<0
a
3’– CATGAT -5’ 5’-TAGTAC-3’ f(1) < 0 a+b+c<0
ab < 0
80. (b) Non-disjunction of chromosomes during
ac < 0
meosis may result is aneuploidy.
bc > 0
84. (d)

an an a1
81. (b) S 1
......
an an 1 a1
a1 a2 ....... a n
1 1 1
S a1 a2 ...... an .... n
a1 a2 an
1 1 1
S n a1 a2 ...... a n ....
a1 a2 an
250 250
2 tan n ,
tan 2 9 9
2
1 tan But n I
2
Total integer = 55
2 tan 86. (d)
2 2 2
2
1 tan
2

2 tan 2 tan 2 0
2
1 1 8 1
tan or 2
2 2 2 2
1
tan
2 2

x2 y2
r1 Let ellipse 1
In OMB sin /2 = a2 b2
ON
and circle x2 + (y + b)2 = r2 {let radius = r}
1
sin /2 = 2
a y 2
3 put x2 = a2 –
b2
ON = 3r1
r2 a2y 2
In OPQ sin /2 = in circle a2 – + (y + b)2 = r2
ON r1 r2 b2
1 r2 a2 2
1 y 2by a2 b2 r2 0
3 3r1 r1 r2 b2

3r1 r1 r2 3r2 a4
D=0 r2
a2 b2
r1 3 1 r2 3 1
a2
2 b a 1
r2 3 1 3 1 r2
r1 2 3
3 1 2 a2
85. (c) x(3x – 25) = – n
2 Area = = ab = a2 1
r2
25 d 2r 2 2
3x x 2 n 0 a2 a r
3 da 3 3
25 25 a 2
3x x x n b a 1 r
3 3 3 3 3
n
87. (a) I n x cos x dx
I II

/2 /2
x n sin x 0
nx n 1 sin xdx
0

n
/2
0 nx n 1
cos x
2 0

/2
n n 1 xn 2
cos x dx
0

n
.2
0 n n 1 x n 2 cos xdx
2 0
n
2 2
In n n 1 In MR 2 MR2 MR 2 '
2
2 5 5
2
In In 2 '
7
n 2 n! n 2 !
92. (a) Draw the free body diagram
n

n n 1 In 2
2 In 2
n 2 n! n 2 !

n
1
n 2 2 n!
F – N = 8a ....(1)
2 3 4
1 1 1 N=2a ....(2)
......
2 2! 2 3! 2 4! f = 2g = 20 ....(3)

e /2
1 N = 20
2
20 20
n N= 40 ....(4)
0.5
88. (b) Let I = x x dx
1 N = 2a

1 x<4 [ x ] =1 N 40
a 20m / s2 ....(5)
2 2
4 x<9 [ x]= 2 F = N + 8a = 10 a [from equation (1)]
9 x < 16 [ x]= 3 F = 10 × 20 = 200 newton
Now, 93. (b)
2 3 4 5 6 7
I dx 2dx 3dx 8dx 10dx 12dx
1 2 3 4 5 6

8 9 10 11
14dx 16dx 27dx 30dx....
7 8 9 10

I = 1 + 2 + 3 + 8 + 10 + 12 + 14 + 16 = 66
n=9
91. (c) Initial sphere is slipping and finally it start
rolling. During its motion Ą about point of Centripetal force at point A :
contact is zero. mv 2
T1 – mg = ....(1)
Angular momentum of sphere about point
of contact remain conserved. At point B :
T2 = mg cos ....(2)
According to question
T1 = 4T2 ....(3)

mv 2
mg + = 4 mg cos

[from equation (1) & (2)]


mv 2
mg (4 cos – 1) = ....(4)

According to conservation of energy between


I = (I + MR2) ' point A and B
1 r 2 cos 1
Also mv2 + 0 = 0 + mgl (1 – cos )
2 d cos 1
mv2 = 2 mgl (1 – cos ) cos 1
d .r
mv 2
2cos 1
= 2 mg (1 – cos ) ....(5)
r
sin
From equation (4) & (5) R
mg (4 cos – 1) = 2 mg (1 – cos ) R2 r2
4 cos – 1 = 2 – 2 cos cos
R
6 cos =3 1/ 2
r2
1 cos 1
cos = R2
2
= 60° 1 r2
cos 1
2 R2
94. (d) Monoatomic gas PV3 = C
r2
5 1 cos
2R 2
3
n=1 r2 r
d
PV3 = Con comparing with PV = C r2
2R 2 2 1 1
Here =3 2R 2

Heat capacity r3 r3
d
r2 2R 2
R R 2R 2 1
C 2R 2
1 1
r6
A d2
R R 4R 4
C
2 2 96. (c) Snell law
3
sin i = sin r

3 1 b
C R sin i = a 2
sin r
2 2
Differentiating with respect to
C=R
95. (a) Concave mirror with curvature R b b
0 = cos r dr a 2
+ sin r 3
2 d

2
a b 2b
0 = cos r dr 2
sin r 3
d

2bd tan r
dr 2
a b

2b tan r
r 3
a b

d = radius of disc 97. (b)


A= d 2

From similar triangle


R
R
r 2 cos
d R R
R
2 2cos
At steady state current does not flow in the hf = KEmax +
branch of capacitor. Only current flow through E = 0.6 + ....(1)
2 k resister.
1.1 E = 0.9 + ....(2)
we can replace all resistor connected in
branch of capacitor with wire new circuit is 0.9
1.1 =
0.6
1.1 + 0.66 = 0.9 +
0.1 = 0.24
= 2.4 eV
100. (c) = h kB c
Steffan boltzmann constant
h Planck fs constant
kB Boltzmann constant
6 c speed of sight
i
2 1 103
According to stefan's law
2 mA
Q
T4
Potential drop across 2k is same as At
potential drop across 1 F & 2 F.
Q 1
Potential drop across 2 k = i × 2 × 103 = 2 × AT T4
103 × 2 × 103 = 4 volt.
M1 L2 T 2
Charge on 1 F = Q = 1 × 4 × 10–6 = 4 C M1 T 3 K 4

Charge on 2 F = q = 2 × 4 × 10–6 = 8 C L2 T K4

98. (d) Power of light = P & E = hv

E M1 L2 T 2

h h
v T 1

[h] = [M1L2T–1]
[c] = [LT–1]
3
E= k T
2 B
M1 L2 T 2
kB
K
[kB] = [M1L2T–2K–1]
3
2P 2 1.5 10 According to homogeneity principle of
Force acting on Al disc =
C 3.0 108 dimension
= 10–5 [M1T–3K–4] = [M1L2T–1] [M1L2T–2K–1] [L1T–1]
Force acting on Al disc = mg on comparing powers of M, L, T, K on both
10–5 = m × 10 sides
m = 10–6 kg + =1 ....(1)
99. (b) Vretarding = 0.6 V 2 +2 + =0 ....(2)
KEmax = e × Vretarding = e × 0.6 = 0.6 eV – .2 – =.3 ....(3)
Photon energy = hf = E – = –4 ....(4)
When frequency increase by 10% energy of =4
photon also increases by 10% =–3
New energy = E' = 1.1 E Putting values is equation (2)
New KEmax. = e × Vretarding = e × 0.9 = 0.9 eV 2 (–3) + 2 (4) + = 0
Einstein photoelectric equation =–2
Ans : 107. (b) Balanced reaction is:
= –3
K 2 Cr2O7 6FeSO4 7H2 SO4 Cr2 SO4
=4 3

= –2
3Fe2 SO4 3
K 2SO4 7H2 O
NH2 NHCOCH3 NHCOCH3
(CH3CO)2O Br2 108. (c) E0 0.44 0.74 0.3 V
101. (a)
Acetanilide G0 nFE0 6 96500 0.3
NH2 Br
(X)
= –173700 J

aq. NaOH
109. (a) CH3CH2CH2 COO 2 Ca
Br (Y)
O CH2OH
dRT 5 0.0821 300 |
102. (b) M 123.15
CH3CH 2CH2 C CH2 CH2 CH3 CH2OH
P 1

123.15
No. of acetic acid molecule = 2 CH3 – CH2 – CH2 CH2CH2CH3
60
C
103. (d) w nRT (1 8.314 103 373)
O O
= –3.1 KJ
q = H = 41 KJ
E = q + w = (41 – 3.1) = 37.9 kJ 110. (c) XeF6 H2O XeO3 HF

104. (c) H2 I2 2HI 3, KC 50


Xe
N2 3H2 2NH3 K 'C 1000
N2 6HI 2NH3 3I 2 K ? O O
O
sp3 & Pyramidal geometry.
K 'C 1000 111. (b) In human one cell cycle loots for 24 hours and
K 0.008
KC
3
50
3 M-phase around 2 hours.

2 1
105. (b) N2 2H 2 N 2 H4 So probability for M-phase
24 12
112. (a) Female – Tt × TT (Male/pollen)
N N 2H H H N N H
| | Endosperm = 2 Polar nuclei + 1 male nuclei
H H

H = (1 × 946) + (2 × 435) – (4 × 389) – (1 × 159) TT & T TTT


Or
tt & T ttT
= 101 KJ/mol

a Embryo Egg cell + 1 male nuclei


106. (c) rK r
Cl 2 T and T TT
Or t and T Tt
a = 2(133 + 181) = 628 Pm = 628 × 10–10 m
113. (a) Total number of combination (5)4 = 625
Volume = a3 = (628 × 10–10)3
Number of stop codens - 1
= 2.48 × 10–22 cm3 So maximum number of unique amino acids
will be 625 – 1 = 624.
114. (b) XXm (Female) Survives (Carrier) 117. (c) Desert lizard Temp will change with
XmY (Male) lethal (dies) environment. (will fall in the given case)
115. (c) A2 5100 In mouse (homeotherm) temperature will
remain same and metabolic rate in mouse will
B1 3100
increase.
B2 8100
118. (b) S

A1 injected
Start 3100 bp Heat killed S + Link R mice died
inmice
–3000 bp
DNA transferming principle
B2 119. (a) Human height is a polygenic character and
8100
bp shows bell shape curve.
Maximum percentage of people will have
B1 medium height.
A2 120. (d) ATP synthesis in mitochondria is based on
116. (d) During crossing over segements of DNA are concentration gradient of protons. (i.e. pH)
exchanged between two chromosomes
resulting in independent assortment and
segregation of chromosomes.
(a) 3 (b) 3.5
(c) 4 (d) 4.5
1. The sum of non-real roots of the polynomial
7. Let ABC be an acute-angled triangle and let D be
equation x3 + 3x2 + 3x + 3 = 0.
(a) equals 0 tan(B)
the midpoint of BC. If AB = AD, then
(b) lies between 0 and 1 tan(C)
(c) lies between –1 and 0 equals
(d) has absolute value bigger than 1 (a) 2 (b) 3
2. Let n be a positive integer such that (c) 2 (d) 3
log 2 log 2 log 2 log 2 log 2 (n) 0 log 2 log 2 log 2 log 2 (n) 8. The angles , , of a triangle satisfy the
Let be the number of digits in the binary equations 2sin + 3cos = 3 2 and 3sin + 2cos
expansion of n. Then the minimum and the = 1. Then angle equals
maximum possible values of are
(a) 150° (b) 120°
(a) 5 and 16 (b) 5 and 17
(c) 60° (d) 30°
(c) 4 and 16 (d) 4 and 17
9. Let f : R R be a function such that
3. Let be a cube root of unity not equal to 1. Then
lim f (x) M 0 . Then which of the following is
x
the maximum possible value of a bw cw2
where a, b, c {+1, –1} is false?

(a) 0 (b) 2 1
(a) lim x sin f (x) M
(c) 3 (d) 1+ 3 x x

4. If a, b are positive real numbers such that the (b) lim sin f (x) sin M
lines ax + 9y = 5 and 4x + by = 3 are parallel, then x

the least possible value of a + b is


x
(a) 13 (b) 12 (c) lim x sin e f (x) M
x
(c) 8 (d) 6
sin x
5. Two line segments AB and CD are constrained to (d) lim f (x) 0
move along the x and y axes, respectively, in such x x
a way that the points A, B, C, D are concyclic. If
AB = a and CD = b, then the locus of the centre of 10. For x, t R let p t x (sin t)x 2 (2 cos t)x sin t
the circle passing through A, B, C, D in polar be a family of quadratic polynomials in x with
coordinates is
1

2 a2 b2 2 a2 b2 variable coefficients. Let A(t) = p t (x)dx . Which


(a) r (b) r cos 2 0
4 4
of the following statements are true?
(c) r 2 4 a2 b2 (d) r 2 cos2 4 a2 b2 I. A(t) < 0 for all t.
6. Consider a triangle ABC in the xy -plane with II. A(t) has infinitely many critical points.
vertices A = (0, 0), B = (1, 1) and C = (9, 1). If the III. A(t) = 0 for infinitely many t.
line x = a divides the triangle into two parts of IV. A(t) < 0 for all t.
equal area, then a equals (a) I and II only (b) II and III only
(c) III and IV only (d) IV and I only
11. Let f(x) = 2 x x 2 and g(x) = cosx. Which of 1 1 n
n
(a) (b) 2
the following statements are true? 2 n k 1 k
I. Domain of f((g(x))2) = Domain of f(g(x))
n n
II. Domain of f(g(x)) + g(f(x)) = Domain of g(f(x)) 1 1 n 1 2 n
(c) 2 (d) 2
III. Domain of f(g(x)) = Domain of f(g(x)) n n k 1 k n n k 1 k
IV. Domain of g((f(x))3) = Domain of f(g(x)) 16. Let n 3. A list of numbers 0 < x1 < x2 < ... < xn
(a) Only I (b) Only I and II has mean and standard deviatiion . A new list
(c) Only III and IV (d) Only I and IV of numbers is made as follows : y1 = 0, y2 = x2,
...., yn–1 = xn–1, yn = x1 + xn. The mean and the
x
2[t] dt , standard deviation of the new list are ˆ and ˆ .
12. For real x with –10 x 10 define f(x) =
10 Which of the following is necessarily true?

where for a real number r we denote by [r] the (a) ˆ, ˆ


largest integer less than or equal to r. The
(b) ˆ, ˆ
number of points of discontinutiy of f in the
interval (–10, 10) is (c) ˆ
(a) 0 (b) 10
(d) may or may not be equal to ˆ
(c) 18 (d) 19
13. For a real number x let [x] denote the largest 17. Let 1, 2, 3, 4 be unit vectors in the xy -plane,
integer less than or equal to x and {x} = x – [x].
one each in the interior of the four quadrants.
The smallest possible integer value of n for which
Which of the following statements is necessarily
n true?
x x dx exceeds 2013 is
(a) 1 2 3 4 =0
1

(a) 63 (b) 64 (b) There exist i, j with 1 i<j 4 such i j


(c) 90 (d) 91 is in the first quadrant
14. The area bounded by the curve y = cos x, the line (c) There exist i, j with 1 i< j 4 such that

joining , cos and (0, 2) and the line i j


4 4
(d) There exist i, j with 1 i< j 4 such that

, cos i j >0
joining and (0, 2) is
4 4
18. The number of integers n with 100 n 999 and
containing at most two distinct digits is
4 2 4 2
(a) 2 (b) 2 (a) 252
8 8
(b) 280
4 2 4 2 (c) 324
(c) 2 (d) 2
4 4 (d) 360
19. For an integer n let Sn = {n + 1, n + 2, ....., n +
15. A box contains coupons labeled 1, 2, 3....n. A
18}. Which of the following is true for all n 10?
coupon is picked at random and the number x is
noted. The coupon is put back into the box and a (a) Sn has a multiple of 19
new coupon is picked at random. The new number (b) Sn has a prime
is y. Then the probability that one of the numbers (c) Sn has at least four multiples of 5
x, y divides the other is (in the options below [r]
(d) Sn has at most six primes
denotes the largest integer less than or equal
to r)
20. Let P be a closed polygon with 10 sides 22. Consider the circuit shown in the figure below :
and 10 vertices (assume that the sides do not
intersect except at the vertices). Let k be
the number of interior angles of P that are
greater than 180°. The maximum possible value
of k is
(a) 3
(b) 5
(c) 7
All the resistors are identical. The charge stored
(d) 9 in the capacitor, once it is fully charged, is

5
(a) 0 (b) CV
21. Consider an initially neutral hollow conducting 13
spherical shell with inner radius r and outer
radius 2r. A point charge +Q is now placed inside 2 5
(c) CV (d) CV
the shell at a distance r/2 from the centre. The 3 8
shell is then grounded by connecting the outer 23. A nuclear decay is possible if the mass of the
surface to the earth. P is an external point at a parent nucleus exceeds the total mass of the
distance 2r from the point charge +Q on the line decay particles. If M(A, Z) denotes the mass of a
passing through the centre and the point charge single neutral atom of an element with mass
+Q as shown in the figure. number A and atomic number Z, then the minimal
condition that the decay

X ZA YZA 1 ve
will occur is (me denotes the mass of the particle
and the neutrino mass mv can be neglected):
(a) M(A, Z) > M(A, Z + 1 ) + me
(b) M(A, Z) > M(A, Z + 1)
(c) M(A, Z) > M(A, Z + 1) + Zme
(d) M(A, Z) > M(A, Z + 1) – me
24. The equation of state of n moles of a non-ideal
gas can be approximated by the equation

n2a
The magnitude of the force on a test charge +q P V nb nRT
V2
placed at P will be
where a and b are constants characteristic of the
1 qQ gas. Which of the following can represent the
(a)
4 0 4r 2 equation of a quasistatic adiabat for this gas
(Assume that CV, the molar heat capacity at
1 9qQ constant volume, is independent of temperature)?
(b)
4 0 100r 2 (a) T V nb
R /Cv
constant
Cv / R
1 4qQ (b) T V nb constant
(c)
4 0 25r 2
ab R /C v
(c) T V nb constant
(d) 0 2
V R

n 2ab Cv / R
(d) T V nb constant
V 2R
25. A blackbox (BB) which may contain a combination 27. An engine moving away from a vertical cliff blows
of electrical circuit elements (resistor, capacitor a born at a frequency f. Its speed is 0.5% of the
or inductor) is connected with other external speed of sound in air. The frequency of the
circuit elements as shown below in the figure (a). reflected sound received at the engine is
After the switch (S) is closed at time t = 0, the (a) 0.990 f (b) 0.995 f
current (I) as a function of time (t) is shown in
the figure (b). (c) 1.005 f (d) 1.010 f
28. An arrangement with a pair of quarter circular
coils of radii r and R with a common centre C and
carrying a current I is shown.

From this we can infer that the blackbox contains


(a) A resistor and a capacitor in series The permeability of free space is 0. The magnetic
(b) A resistor and a capacitor in parallel field at C is
(c) A resistor and an inductor in series
1 1
(d) A resistor and an inductor in parallel 0I
r R
(a) into the page
26. In a photocell circuit the stopping potential, V0, 8
is a measure of the maximum kinetic energy of
the photoelectrons. The following graph shows 1 1
experimentally measured values of stopping 0I
r R
potential versus frequency v of incident light. (b) out of the page
8

1 1
0I
r R
(c) out of the page
8

1 1
0I
r R
(d) into the page
8
29. The circuit shown has been connected for a long
time. The voltage across the capacitor is

The values of Plank's constant and the work


function as determined from the graph are
(taking the magnitude of electronic charge to be
e = 1.6 × 10–19 C)
(a) 6.4 × 10–34 Js, 2.0 eV
(b) 6.0 × 10–34 Js, 2.0 eV
(c) 6.4 × 10–34 Js, 3.2 eV
(d) 6.0 × 10–34 Js, 3.2 eV (a) 1.2 V (b) 2.0 V
(c) 2.4 V (d) 4.0 V
30. A wheel of radius R with an axle of radius R/2 is (a) 0.5 (b) 1.4
shown in the figure and is free to rotate about a (c) 4.0 (d) 2.0
frictionless axis through its centre and
34. An electron enters a chamber in which a uniform
perpendicular to the page. Three forces (F, F, 2F)
magnetic field is present as shown. Ignore gravity.
are exerted tangentially to the respective rims
as shown in the figure.

During its motion inside the chamber


(a) the force on the electron remains constant
(b) the kinetic energy of the electron remains
constant
The magnitude of the net torque acting on the
(c) the momentum of the electron remains
system is nearly
constant
(a) 3.5FR (b) 3.2 FR
(d) the speed of the electron increases at a uniform
(c) 2.5 FR (d) 1.5 FR
rate
31. Two species of radioactive atoms are mixed in
35. A ray of light incident on a glass sphere (refractive
equal number. The disintegration of the first
index 3 ) suffers total internal reflection before
species is and of the second is . After a long emerging out exactly parallel to the incident ray.
3
The angle of incidence was
time the mixture will behave as a species with
(a) 75° (b) 30°
mean life of approximately
(c) 45° (d) 60°
0.70 2.10
(a) (b) 36. Young-Laplace law states that the excess pressure
inside a soap bubble of radius R is given by

1.00 0.52 4
(c) (d) P where is the coefficient of surface
R
tension of the soap. The number E0 is a
VdP
32. The bulk modulus of a gas is defined as B = . dimensionless number that is used to describe
dV
the shape of bubbles rising through a surrounding
For an adiabatic process the variation of B is
fluid. It is a combination of g, the acceleration
proportional to Pn. For an idea gas, n is
due to gravity, , the density of the surrounding
(a) 0 (b) 1 fluid, and a characteristic length scale L which
5 could be the radius of the bubble. A possible
(c) (d) 2 expression for E0 is
2
33. Photons of energy 7 eV are incident on two metals g L2
A and B with work functions 6 eV and 3 eV (a) (b)
L3 g
respectively. The minimum de Broglie
wavelengths of the emitted photoelectrons with
maximum energies are A and B, respectively gL2 gL2
(c) (d)
A
where is nearly
B
37. A plank is resting on a horizontal ground in the
northern hemisphere of the Earth at a 45°
latitude. Let the angular speed of the Earth be
and its radius re. The magnitude of the frictional
force on the plank will be

2
2
mre
(a) mre (b)
2

2
mre
(c) (d) Zero
2
38. The average distance between molecules of an
ideal gas at STP is approximately of the order of
(a) 1 nm Which of the following statements is true?
(b) 100 nm (a) Wave 1 has lower frequency and smaller
amplitude compared to wave 2
(c) 100 cm
(b) Wave 1 has higher frequency and greater
(d) 1 m
amplitude compared to wave 2
39. A point particle of mass 0.5 kg is moving along
(c) Wave 1 has shorter wavelength and greater
the x-axis under a force described by the potential
amplitude compared to wave 2
energy V shown below. It is projected towards
the right from the origin with a speed v. (d) Wave 1 has shorter wavelength and smaller
amplitude compared to wave 2
What is the minimum value of v for which the
particle will escape infinitely far away from the
origin? 41. Among the following, the set of isoelectronic ions
is
(a) Na+, Mg2+, F–, Cl– (b) Na+, Ca2+, F–, O2–
(c) Na+, Mg2+, F–, O2– (d) Na+, K+, S2–, Cl–
42. For a zero-order reaction with rate constant k,
the slope of the plot of reactant concentration
against time is

k
(a) (b) k
2.303

k
(c) (d) –k
2.303
43. The compound which reacts with excess bromine
to produce 2, 4, 6-tribromophenol, is
(a) 1, 3-cyclohexadiene
1 (b) 1, 3-cyclohexanedione
(a) 2 2 ms
(c) salicylic acid
(b) 2 ms–1
(d) cyclohexanone
(c) 4 ms–1
44. Ethyl acetate reacts with NH2NHCONH2 to form
(d) The particle will never escape
(a) CH3CONHCONHNH2
40. The figure below shows pressure variation in two
different sound waves in air with time at a given (b) CH3CON(NH2)CONH2
position. Both the figures are drawn to the same (c) CH3CONHNHCONH2
scale. (d) CH3CH2NHNHCONH2
45. The variation of solubility of four different gases 51. The entropy change in the isothermal reversible
(G1, G2, etc.) in a given solvent with pressure at expansion of 2 moles of an ideal gas from 10 to
a constant temperature is shown in the plot. 100 L at 300 K is
(a) 42.3 JK–1 (b) 35.8 J K–1
(c) 38.3 JK–1 (d) 32.3 J K–1
52. D-Glucose upon treatment with bromine-water
gives

The gas with the highest value of Henry’s law


constant is (a) (b)

(a) G4 (b) G2
(c) G3 (d) G1
46. For the reaction, A nB the concentration of
A decreases from 0.06 to 0.03 mol L–1 and that of
B rises from 0 to 0.06 mol L–1 at equilibrium.
The values of n and the equilibrium constant for
the reaction, respectively, are
(a) 2 and 0.12 (b) 2 and 1.2
(c) 3 and 0.12 (d) 3 and 1.2 (c) (d)
47. The reaction of ethyl methyl ketone with Cl2/
excess OH–. gives the following major product
(a) ClCH2CH2COCH3
(b) CH3CH2COCCl3 53. In the structure of borax, the numbers of boron
atoms and B-O-B units, respectively, are
(c) ClCH2CH2COCH2Cl
(a) 4 and 5 (b) 4 and 3
(d) CH3CCl2COCH2Cl
(c) 5 and 4 (d) 5 and 3
48. The compound that readily tautomerizes is
54. The number of peptide bonds in the compound
(a) CH3COCH2CO2C2H5
(b) CH3COCH2CH2CH3
(c) CH3COCH2CH2CH3
(d) (CH3)3CCOC(CH3)3
49. Hydrolysis of BCl3 gives X which on treatment
with sodium carbonate produces Y, X and Y,
respectively, are
(a) H3BO3 and NaBO2
(b) H3BO3 and Na2B4O7 is
(c) B2O3 and NaBO2 (a) 1 (b) 2
(d) B2O3 and Na2B4O7 (c) 3 (d) 4
50. The numbers of lone pair(s) on Xe in XeF2 and 55. For the isothermal reversible expansion of an
XeF4 are, respectively, ideal gas
(a) 2 and 3 (b) 4 and 1 (a) H > 0 and U = 0 (b) H > 0 and U < 0
(c) 3 and 2 (d) 4 and 2 (c) H = 0 and U = 0 (d) H = 0 and U > 0
56. If the angle of incidence of X-ray of wavelength 62. Nucleolus is an organelle responsible for the
3Å which produces a second order diffracted beam production of
from the (100) planes in a simple cubic lattice with (a) carbohydrates
interlayer spacing a = 6 Å is 30°, the angle of
(b) messenger RNA
incidence that produces a first-order diffracted
beam from the (200) planes is (c) lipids
(a) 15° (b) 45° (d) ribosomal RNA
(c) 30° (d) 60° 63. The sequences of four DNA molecules are given
below:
57. The number of ions produced in water by
dissolution of the complex having the empirical i. TATATATATATATA
formula, COCl34NH3, is ATATATATATATAT
(a) 1 (b) 2 ii. TTTCCCGGGAAA
(c) 4 (d) 3 AAAGGGCCCTTT
58. The spin-only magnetic moments of [Fe(NH3)6]3+ iii. TTGCGTTGCCC
and [FeF6]3– in BM are, respectively,
AACGCAACGGG
(a) 1.73 and 1.73
iv. GCCGGATCCGGC
(b) 5.92 and 1.73
CGGCCTAGGCCG
(c) 1.73 and 5.92
Which one of these DNA molecules will have the
(d) 5.92 and 5.92 highest melting temperature (Tm)?
59. The order of SN1 reactivity in aqueous acetic acid
(a) i (b) ii
solution for the compounds
(c) iii (d) iv
64. If DNA codons are ATG GAA, insertion of thymine
1. after the first codon results in,
(a) non-sense mutation
2.
(b) mis-sense mutation
3. (c) frameshift mutation
is (d) silent mutation
(a) 1 > 2 > 3 65. Genetic content of a cell reduces to half during
(b) 1 > 3 > 2 (a) meiotic prophase I
(c) 3 > 2 > 1 (b) mitotic prophase
(d) 3 > 1 > 2 (c) meiotic prophase II
60. An ionic compound is formed between a metal M (d) meiotic telophase
and a non-metal Y. If M occupies half the 66. Which one of the following techniques is used for
octahedral voids in the cubic close-packed the detection of proteins?
arrangement formed by Y, the chemical formula
(a) Northern blotting
of the ionic compound is
(b) Western blotting
(a) MY (b) MY2
(c) M2Y (d) MY3 (c) Southern blotting
(d) In-situ hybridization
67. Fission yeasts are
61. Human fetal haemoglobin differs from the adult (a) Archaebacteria
haemoglobin in that it has (b) Eubacteria
(a) higher affinity for oxygen (c) Prokaryotes
(b) lower affinity for oxygen (d) Eukaryotes
(c) two subunits only
(d) is glycosylated
68. In green leaves, the light and dark reactions occur 75. In orange and lemon, the edible part of the fruit is
in (a) placenta
(a) stroma and grana respectively (b) thalamus
(b) grana and stroma respectively (c) hairs of the ovary wall
(c) cristae and matrix respectively (d) succulent Mesocap
(d) both occur in cytoplasm 76. Which one of the following statements about
69. According to Mendel, ..................................... nitrogenase is correct?
segregate and .................... assort independently. (a) It is sensitive to CO2 and therefore present in
(a) alleles of a gene; alleles of different genes isolated nodules.
(b) alleles of different genes; alleles of a gene (b) It requires O2 and therefore functional during
(c) dominant traits; recessive traits the day
(d) recessive traits; recessive traits (c) It is sensitive to O2 and therefore is functional
in anaerobic environments
70. The two enzymatic activities associated with
RUBISCO are (d) It is sensitive to light and therefore functions
only in dark.
(a) oxidase and oxygenase
77. Part of epidermis that keeps out unwanted
(b) oxygenase and carboxylase
particles is called
(c) oxidase and carboxylase
(a) columnar epithelium
(d) oxygenase and carbamylation
(b) squamous epithelium
71. Chlorofluorocarbons (CFCs) are belived to be
(c) ciliated epithelium
associated with cancers because,
(d) cuboidal epithelium
(a) CFCs react with DNA and cause mutations
78. Species that are most effective at colonising new
(b) CFCs react with proteins involved in DNA
habitats show
repair
(a) low reproductive ability
(c) CFCs destroy the ozone layer and permit
harmful UV rays to reach the earth (b) high dispersal ability
(d) CFCs react with DNA polymerase and reduce (c) slow growth and maturation
fidelity of DNA replication (d) high competitive ability
72. Morphogenetic movements take place 79. In a large isolated population, alleles p and q at a
predominantly during the following embryonic locus are at Hardy Weinberg equilibrium. The
stage frequencies are p = 0.6 and q = 0.4. The proportion
(a) blastula of the heterozygous genotype in the population
is
(b) Morula
(a) 0.24
(c) Gastrula
(b) 1
(d) Fertilized eggs
(c) 0.48
73. The only organ which is capable of producing
Fructose in humans is (d) 0.12
(a) liver 80. In vertebrates ‘glycogen’ is stored chiefly in
(b) pancreas (a) heart and blood
(c) seminal vesicles (b) spleen and stomach
(d) muscle (c) bones and lymph
74. Stroke could be prevented/treated with (d) liver and muscles
(a) balanced diet
(b) clotting factors
(c) insulin
(d) blood thinners
81. Let f(x) be a non-constant polynomial with real 85. Let XY be the diameter of a semicircle with centre
O. Let A be a variable point on the semicircle
1
coefficients such that f 100 and f (x) 100 and B another point on the semicircle such that
2 AB is parallel to XY. The value of BOY for which
for all real x. Which of the following statements the inradius of triangle AOB is maximum, is
is NOT necessarily true? 5 1
1
(a) The coefficient of the highest degree term in (a) cos
2
f(x) is negative
(b) f(x) has at least two real roots 1 5 1
(b) sin
2
1
(c) If x then f(x) < 100
2
(c)
(d) At least one of the coefficients of f(x) is bigger 3
than 50.
82. Let a, b, c, d be real numbers such that (d)
5
n
ak 3 bk 2 ck d n4 x x2 x3 x4
k 1 86. Let f(x) = 1 + . The number of
1! 2! 3! 4!
for every natural number n. Then real roots of f(x) = 0 is
a b c d is equal to (a) 0
(a) 15 (b) 16 (b) 1
(c) 31 (d) 32 (c) 2
83. The vertices of the base of an isosceles triangle (d) 4
lie on a parabola y2 = 4x and the base is a part of 87. Suppose that the earth is a sphere of radius 6400
the line y = 2x – 4. If the third vertex of the kilometers. The height from the earth’s surface
triangle lies on the x-axis, its coordinates are from where exactly a fourth of the earth’s surface
is visible, is
5 7
(a) , 0 (b) , 0 (a) 3200 km
2 2
(b) 3200 2 km
9 11
(c) , 0 (d) , 0 (c) 3200 3 km
2 2
(d) 6400 km
84. In a triangle ABC, let G denote its centroid and
let M, N be points in the interiors of the segments 88. Let n be a positive integer. For a real number x,
AB, AC, respectively, such that M, G, N are let [x] denote the largest integer not exceeding x
collinear. If r denotes the ratio of the area of
n 1 x
triangle AMN to the area of ABC then x
and {x} = x – [x]. Then dx is equal to
1
x
1 1
(a) r = (b) r >
2 2
1
(a) loge(n) (b)
4 1 4 n 1
(c) r< (d) <r
9 2 9
n 1 1
(c) (d) 1
n 1 2 n
89. A box contains coupons labelled 1, 2, ..., 100. Five 92. A small boy is throwing a ball towards a wall 6 m
coupons are picked at random one after another in front of him. He releases the ball at a height of
without replacement. Let the numbers on the 1.4 m from the ground. The ball bounces from
coupons be x1, x2, …, x5. What is the probability the wall at a height of 3 m, rebounds from the
that x1 > x2 > x3 and x3 < x4 < x5? ground and reaches the boy’s hand exactly at the
point of release. Assuming the two bounces (one
1 1
(a) (b) from the wall and the other from the ground) to
120 60
be perfectly elastic, how far ahead of the boy did
1 1 the ball bounce from the ground?
(c) (d) (a) 1.5 m
20 10
90. In a tournament with five teams, each team plays (b) 2.5 m
against every other team exactly once. Each game (c) 3.5 m
is won by one of the playing teams and the winning (d) 4.5 m
team scores one point, while the losing team
93. In the P-V diagram below the dashed curved line
scores zero. Which of the following is NOT
is an adiabat.
necessarily true?
(a) There are at least two teams which have at
most two points each.
(b) There are at least two teams which have at
least two points each.
(c) There are at most three teams which have at
least three points each
(d) There are at most four teams which have at
most two points each

For a process that is described by a straight line


91. A bullet of mass m is fired horizontally into a joining two points X and Y on the adiabat (solid
large sphere of mass M and radius R resting on a line in the diagram) heat is : (hint : Consider the
smooth horizontal table.
variations in temperature from X to Y along the
straight line)
(a) absorbed throughout from X to Y
(b) released throughout from X to Y
(c) absorbed from X up to an intermediate point
Z (not shown in the figure) and then released
from Z to Y
The bullet hits the sphere at a height h from the
(d) released from X up to an intermediate point Z
table and sticks to its surface. If the sphere starts
(not shown in the figure) and then absorbed
rolling without slipping immediately on impact,
from Z to Y
then
94. A singly ionized helium atom in an excited state
h 4m 3M (n = 4) emits a photon of energy 2.6 eV. Given
(a)
R 2 m M that the ground state energy of hydrogen atom is
.13.6 eV, the energy (Et) and quantum number
h m 3M (n) of the resulting state are respectively,
(b)
R m 2M (a) Et = –13.6 eV, n = 1
h 10m 7M (b) Et = –6.0 eV, n = 3
(c)
R 5 m M (c) Et = –6.0 eV, n = 2
(d) Et = –13.6 eV, n = 2
h 4m 3M
(d)
R m M
95. The figure below shows a circuit and its input 96. A ball is rolling without slipping in a spherical
voltage vi as function of time t. shallow bowl (radius R) as shown in the figure
and is executing simple harmonic motion. If the
radius of the ball is doubled, the period of
oscillation

(a) increases slightly

1
Assuming the diodes to be ideal, which of the (b) is reduced by a factor of
following graphs depicts the output voltage v0 as 2
function of time t? (c) is increased by a factor of 2
(d) decreases slightly
97. A solid sphere rolls without slipping, first
horizontal and then up to a point X at height h on
an inclined plane before rolling down, as shown.

(a)

The initial horizontal speed of the sphere is

10gh 7gh
(b) (a) (b)
7 5

5gh
(c) (d) 2gh
7

98. The three processes in a thermodynamic cycle


shown in the figure are : Process 1 2 is
isothermal; Process 2 3 is isochoric (volume
remains constant); Process 3 1 is adiabatic.
The total work done by the ideal gas in this cycle
(c) is 10 J. The internal energy decreases by 20 J in
the isochoric process. The work done by the gas
in the adiabatic process is –20 J. The heat added
to the system in the isothermal process is

(a) 0 J

(b) 10 J

(c) 20 J
(d)
(d) 30 J
99. A block of mass m slides from rest at a height H
on a frictionless inclined plane as shown in the
figure. It travels a distance d across a rough (a)
horizontal surface with coefficient of kinetic
friction , and compresses a spring of spring k by
a distance x before coming to rest momentarily.
Then the spring extends and the block travels
back attaining a final height of h. Then (b)

(c)

(d)
(a) h = H – 2 (d + x)
(b) h = H + 2 (d + x)
102. The maximum number of isomers that can result
2 from monobromination of 2-methyl-2-pentene
H 2 d kx
(c) h with N-bromosuccinimide in boiling CCl4 is
mg
(a) 1 (b) 2
2 (c) 3 (d) 4
H 2 (d x) kx
(d) h
2mg 103. The compound X (C 7 H 9 N) reacts with
benzensulfonyl chloride to give Y (C13H13NO2S)
100. A metallic prong consists of 4 rods made of the
which is insoluble in alkali. The compound X is
same material, cross-section and same lengths
as shown.

(a) (b)

(c) (d)

104. In 108 g of water, 18 g of a non-volatile compound


The three forked ends are kept at 100° C and the is dissolved. At 100°C the vapor pressure of the
handle end is at 0°C. The temperature of the solution is 750 mm Hg. Assuming that the
junction is compound does not undergo association or
(a) 25°C (b) 50°C dissociation, the molar mass of the compound in
(c) 60°C (d) 75°C g mol–1 is
(a) 128 (b) 182
(c) 152 (d) 228
101. The major product obtained in the reaction of
aniline with acetic anhydride is
105. The standard electrode potential of Zn2+/Zn is 110. In aqueous solution, [Co(H2O)6]2+ (X) reacts with
–0.76 V and that of Ca2+/Cu is 0.34 V. The emf (V) molecular oxygen in the presence of excess liquor
and the free energy change (kJ mol –1 ), NH3 to give a new complex Y. The number of
respectively, for a Daniel cell will be unpaired electrons in X and Y are, respectively
(a) –0.42 and 81 (a) 3, 1 (b) 3, 0
(b) 1.1 and –213 (c) 3, 3 (d) 7, 0

(c) –1.1 and 213


(d) 0.42 and –81 111. 10 9 bacteria were spread on an agar plate
106. Consider the equilibria (1) and (2) with equilibrium containing penicillin. After incubation overnight
at 37°C, 10 bacterial colonies were observed on
constants K1 and K2, respectively
the plate. That the colonies are likely to be
1 resistant to penicillin can be tested by
SO 2 (g) O2 (g) SO3 (g) … (1)
2 (a) measuring their growth rate
(b) observing the colour of the colonies
2SO3 (g) 2SO2 (g) O2 (g) … (2)
(c) checking their ability to grow on another plate
K1 and K2 are related as containing penicillin.
(a) 2K1 = K22 (d) checking their ability to cause disease
1 112. Watson and Crick model of DNA is
(b) K12 = (a) B-form DNA with a spiral length of 34 Å and a
K2
diameter of 20 Å
2 1 (b) A-form DNA with a spiral length of 15 Å and a
(c) K 2 diameter of 20 Å
K1
(c) Z-form DNA with a spiral length of 34 Å and a
2 diameter of 20 Å
(d) K 2 (d) B-form DNA with a spiral length of 28 Å and a
K 12
diameter of 14 Å
107. Aqueous solution of metallic nitrate X reacts with 113. Eco RI and Rsa I restriction endonucleases require
NH4OH to form Y which dissolves in excess 6 bp and 4 bp sequences respectively for cleavage.
NH4OH. The resulting complex is reduced by In a 10 kb DNA fragment how many probable
acetaldehyde to deposit the metal. X and Y, cleavage sites are present for these enzymes
respectively, are
(a) 0 Eco RI and 10 Rsa I
(a) Cs(NO3) and CsOH
(b) 1 Eco RI and 29 Rsa I
(b) Zn(NO3)2 and ZnO
(c) 4 Eco RI and 69 Rsa I
(c) AgNO3 and Ag2O
(d) 2 Eco RI and 39 Rsa I
(d) Mg(NO3)2 and Mg(OH)2
114. From an early amphibian embryo the cells that
108. The density of eq. wt of a metal are 10.5 g cm–3 would give rise to skin in adults were transplanted
and 100, respectively. The time required for a into the developing brain region of another
current of 3 amp to deposit a 0.005 mm thick layer embryo. The transplanted cells developed into
of the same metal on an area of 80 cm2 is closest brain tissue in the recipient embryo. What do you
to infer from this experiment?
(a) 120 s (b) 135 s (a) Cell fate is permanently determined during
(c) 67.5 s (d) 270 s early embryonic development.
109. The amount of Na 2 S 2 O 3 ·5H 2 O required to (b) Developmental fate of donor cells is influenced
completely reduce 100 mL of 0.25 N iodine by the surrounding cells.
solution, is (c) Developmental fate of donor cells is not
(a) 6.20 g (b) 9.30 g influenced by recipient cells.
(c) 3.10 g (d) 7.75 g (d) Any cell which is transplanted into another
embryo always develops into a brain.
115. Presence of plastids in Plasmodium suggests Which one of the following sets of DNA fragments
(a) it is a plant species generated by digestion with both Eco RI and Bam
HI as shown in (iii) is from the gene?
(b) it is a parasite with a cynobacterium as an
endosymbiont (a) 1 Kb and 4 Kb (b) 1 Kb and 2.5 Kb
(c) it is a parasite with a archebacterium as an (c) 1 Kb and 3 Kb (d) 1 Kb and 3.5 Kb
endosymbiont 118. Brown fat is a specialised adipose tissue with
(d) it is a plant species with a archebacterium as abundant mitochondria and rich blood supply.
an endosymbiont Brown fat
116. The figure below demonstrates the growth curves (a) insulates animals that are acclimatised to
of two organisms A and B growing in the same cold.
area. What kind of relation exists between A and (b) is the major source of heat production of birds.
B? (c) provides energy to muscles.
(d) produces heat without producing ATP.
119. In some species, individuals forego reproduction
and help bring up another individual's offspring.
Such altruistic behaviour CANNOT be explained
by which of the following?
(a) An individual helps relatives only and gets
indirect genetic benefits.
(b) The individual benefits because it can later
inherit the breeding position.
(c) The individual benefits because it gets access
(a) Competition
to resources, such as food and security from
(b) Symbiosis predators, in return.
(c) Commensalisms (d) The species benefits from a reduction in
(d) Mutualism competition among offspring.
117. A scientist has cloned an 8 Kb fragment of a 120. Lions in India are currently restricted to Gir,
mouse gene into the Eco RI site of a vector of 6 Gujarat. Efforts are being made to move them to
Kb size. The cloned DNA has no other Eco RI other parts of the country. This is because they
site within. Digestions of the cloned DNA is shown are MOST susceptible to extinction due to
below. infectious diseases under the following conditions
when present as
(a) several small, isolated populations
(b) one large population
(c) several large, connected populations
(d) several large, isolated populations
1. (c) 2. (a) 3. (b) 4. (b) 5. (b) 6. (a) 7. (d) 8. (d) 9. (c) 10. (b)
11. (b) 12. (a) 13. (d) 14. (a) 15. (*) 16. (a) 17. (a) 18. (a) 19. (d) 20. (c)
21. (d) 22. (d) 23. (a) 24. (a) 25. (c) 26. (b) 27. (a) 28. (b) 29. (d) 30. (a)
31. (b) 32. (b) 33. (d) 34. (b) 35. (d) 36. (*) 37. (c) 38. (a) 39. (b) 40. (d)
41. (c) 42. (d) 43. (c) 44. (c) 45. (d) 46. (a) 47. (b) 48. (a) 49. (b) 50. (c)
51. (c) 52. (a) 53. (a) 54. (a) 55. (c) 56. (c) 57. (b) 58. (c) 59. (c) 60. (b)
61. (a) 62. (d) 63. (d) 64. (a) 65. (d) 66. (b) 67. (d) 68. (b) 69. (a) 70. (b)
71. (c) 72. (c) 73. (c) 74. (d) 75. (c) 76. (c) 77. (c) 78. (d) 79. (c) 80. (d)
81. (c) 82. (a) 83. (c) 84. (c) 85. (a) 86. (*) 87. (d) 88. (c) 89. (a) 90. (d)
91. (c) 92. (a) 93. (c) 94. (b) 95. (a) 96. (d) 97. (a) 98. (d) 99. (a) 100. (d)
101. (a) 102. (d) 103. (a) 104. (d) 105. (b) 106. (b) 107. (*) 108. (b) 109. (a) 110. (b)
111. (c) 112. (a) 113. (d) 114. (b) 115. (b) 116. (a) 117. (a,c) 118. (d) 119. (c) 120. (b)

a b
ab a b 12
1. (c) f(x) = x3 + 3x2 + 3x + 3 = 0 2
Now differenciating wrt 'x' we get
2
2 2
5. (b) 2 g c a
f '(x) 3x 6x 3 3(x 1) 0
f(x) is Increasing function and 2 f 2 c b
Now f(–3) = –6 < 0 Let the polar coordinates of centre of circle
f(–2) = 1 > 0 be (rcos , rsin )
real root lies between –3 and –2 g = – r cos and g2 – f2
+ + = –3; –3 < < –2
a2 b2
Now, =
4
+ –3< + + < + –2
+ –3<–3< + –2 7. (d)
–1< + <0

3. (b) a bw cw2
|a – c + (b – c)w|, for maximum value taking
a = 1 , c = –1, b = 1
|a + bw + cw2| = |2 + 2w| = 2|w2| = 2 Using M - N Rule
(1 + 1)cot ( – B) = 1·cot B – cot C
a 9
4. (b) ( parallel) 3cotB = cotC
4 b
ab = 36 tan B
3
tan C
Now, as we know thatAM GM
8. (d) 2sin + 3cos = 3 2 … (i) Domain of g f 3 x is Domain of g(f(x))
3sin + 2cos = 1 … (ii) i.e., [–2, 1]
Now, sum of squares of equation (i) and (ii), 12. (a) Let r be an integer in (–10, 10)
we get
4 + 9 + 12 sin ( + ) = 19 x
Now, LHL = lim 2[t] dt
1 x r 10
sin( + ) = = 150° or 30°
2 9 8 r h
If + = 30° = 30 – lim 2[t] dt 2[t] dt 2[t] dt
h 0 10 9 r 1
Putting the value of ' ' from in eq (i) and (ii)
10 9
7 sin + 3 3 cos =6 2 lim 2 2 2r 1 (1 h)
h 0
7 cos – 3 3 sin =2
2 10
2 9
2r 1 … (i)
7 9 6 3 x
cos = 0.8
37 2 lim 2[t] dt
x r 10
cos cos 30 30
30 9 8 r h
lim 2[t] dt 2[t] dt 2[t] dt
h 0
sin e x
x 102 9 r
x
9. (c) lim x sin e f (x) lim
x
f (x)
x x e ex = 2–10 + 2–9 + … + 2r–1 … (ii)
= 1 × (0) × M = 0 r

1
f (r) 2[t] dt = 2–10 + 2–9 + … + 2r–1
10. (b) A(t) = (sin t)x 2 (2cos t)x sin t dx 10

0 From eq (i), (ii) & (iii)


f(x) is continous at all integers.
sin t 4
A(t) = cos t sin t sin t cos t n
3 3 n 1r 1
13. (d) r r dx r(x r)dx
Now differentiating A(t) wrt (t) we get 1 r 1 r

4 cos t
A '(t) sin t r 1
3
n 1
x2
r rx
11. (b) Domain of f(g(x)) is R r 1 2 r

2 cos x cos2 x 0 n 1
(r 1)2 r2
(cos x + 2)(cosx – 1) 0 r r 1
r 1 2
– 2 cos x 1
x R n 1
1 1 n(n 1)
r
Domain of g(f(x)) is [–2, 1] 2 2 2
r 1

cos 2 x x2 n(n 1)
Now, 2013
2 4
2 x x 0
Domain of f(g(x)2) is R n(n 1) 4 2013
2
2 cos2 x cos4 x 0 1 2013 16 1
n
2 4
cos2 x 2 cos2 x 1 0
32209 1
1 cos x 1 n
2 2
x R least n = 91
14. (a) 1 2 2 2
0 x 22 x 32 xn 1 x1 xn 2
n

2x1 x n
ˆ2 2
0
n
so ˆ

17. (a)
Required area

1 1 /4
= 2 2 cos xdx
2 2 4 0

4 2
= 2
8
15. (*) Let x = 1
Required favourable outcomes (1, 1), (1, 2)
.......(1, n) Number of favourable out comes In this case B, C, D are not possible.
18. (a) Total Integers = 999 – 99 = 900
n
when x = 1 = Total Integers in which all distinct digits
1
Number of favourable outcomes when 9 9 8 648

n 900 – 648 = 252


x = 1 or y = 1 = 2 –1 19. (d) n + 1, n + 2, ......, n + 18
1
(a) False, if n = 19
no. of favourable outcomes when x = 2 or
y = 2 but x 1, y 1 (c) False if n = 15
16 to 33
n 20, 25, 30 only three multiples of 5
2 –1
2 (d) no. of odd integers in Sn = 9
Similarly every third odd integer is multiple of 3
Number of favourable outcomes when x = k so maximum prime no. = 6.
or y = k but x, y {1, 2, … k – 1}
21. (d) As charge on outer most surface is zero.
n Therefore, force on q is also ‘0’
2 1
k
22. (d)
So required probability =
n
n
(1 1 n times)
k 1 k
n2

n
2 n 1
R
n 2 k n 5
k 1 3 5
VAB V V
5R 8
R
0 x2 x3 xn 1 x1 xn 3
16. (a) ˆ
n Now, charge stored in capacitor is given by,

1 5
ˆ2 y i2 ˆ2 Q CV
n 8
25. (c) I - t graph is for L-R series circuit. 32. (b) For adiabatic process
PV = C (Constant)
h
26. (b) V
e 1 dp
PV v 0
From graph = 2eV dv

h 6 dp
slope P V
e 15 dv
1.6 10

19
Therefore, n = 1
6 1.6 10 34
h 6.0 10 34. (b) Kinetic energy of the electron remains
1.6 1015 constant because work done is zero.

V 35. (d)
27. (a) f1 f0
V Vs

Now,

v vs v vs 2vs
fR f1 f0 1 f
v v vs v

= 0.990f
Here,
0i
28. (b) Magnetic field due to Arc = i 2 r
4 r
sin i
0i 1 1 3
Hence, out of the page sin r
8 r R
29. (d) In stready state i through capacitor is zero. 2cos r 3
Hence V across 2k = V across capacitor r 30 & i 60

2 2 Note: But for r = 30°, Total Internal Reflection


Vacross 2k = 6= 6 4 cannot take place at B.
2 1 3
30. (a) Net torque acting on the system 37. (c)

R
Net F FR 2FR 3.5FR
2

tdN1 tdN2
31. (b) Average life =
2N0

t
where dN1 = N0 e dt

t Here, Frictional force on the plank,


dN2 No e 3 dt
3 2
Fr m r cos 45

t 3
t where r = Rcos 45°
t No e dt No e dt
3
Average life = o 2 1 1
2N0 Fr m R
2 2
By integrating we get
2
m R
2 2.10 Fr
Average life = 2
38. (a) PV = nRT O O

PV 105 1 48. (a) CH3 C CH 2 C OC2 H5


n 40
RT 25
300
3 Active Methylene Group

Now, N 40 6.023 1023 24 10 24


50. (c) F – Xe – F and
1/3
1
Avreage sep. = 1 nm
N
F F
39. (b) Here, by equating
Xe
1 F F
mv 2 0 0 1
2
V2
2 51. (c) S 2.303nR log
v 4 or v 2 m/s V1
41. (c) Na+ = 10 electrons
100
Mg2+ = 10 electrons 2.303 2 8.314 log
10
F– = 10 electrons
O2– = 10 electrons = 38.3
42. (d) (a – x) = a – Kt, slope = –K 52. (a) D-Glucose Bro min e
D-Gluconic Acid
Water

OH OH
COOH Br Br
43. (c) + Br2 (excess)
53. (a)
Br

44. (c) Condensation Reaction


45. (d) Pgas = KHXgas

1 55. (c) U = 0 and H = 0


Xgas = Pgas ×
KH
57. (b) CO NH 3 4
Cl2 Cl so total ions 2
1
Slope = , hence d is correct. 58. (c) As NH3 is strong field ligand so pairs 4p
KH unpaired electrons.
46. (a) A decreases from 0.06 to 0.03. Fe3+ = 3d5
B increases from 0 to 0.06 So unpaired electron = 1

[A] 0.03, B 0.06 1(1 2) 3 1.732 B.M.

A nB While F– is weak field ligand.


n=2 5(5 2) 35 5.92 B.M.

47. (b) CH3CH2 C CH3 59. (c) Because S N 1 depends on stability of


carbocation.
O
Cl2 (excess) 1
60. (b) No. of M atoms = ×4=2
O 2
CH3CH2 C CCl3 No. of Y atoms = 4
So, formula is MY2
61. (a) Fetal Hb has more affinity for oxygen.
62. (d) Nucleolus is the site of rRNA synthesis.
63. (d) As maximum number of GC pairs are present
in (iv).
64. (a) Insertion of thiamine causes formation of stop
codon.
65. (d) During meotic telophase.
66. (b) Western blotting is used. Now, let the highest coefficients, it can have
67. (d) Yeast is an eukaryote.
1 49 49
68. (b) Grana Thylacoids Photosystems present is 49 then, f 49
2 2 22
hence light reaction.
Stroma Contain enzymes for dark reaction. But the sum cannot be equal to 100.
69. (a) During meosis, two alleles of a gene separate n
and alleles of different genes assort ak 3 bk 2 ck d n4
82. (a)
independently. k 1
70. (b) RBISCO is anbifunctional enzyme. It has
oxygenase and carboxylase activity. n n n n
a k3 b k2 c k d n2
71. (c) CFC destroy the ozone layer and permit UV
k 1 k 1 k 1 k 1
rays to reach the earth. UV rays demage DNA
and cause cancer.
n 4 12 3a n 3 4b 6a n 2 6c 6b 3a
72. (c) Morphogenetic moment occur during
gastrulation resulting in formation of 3 germ n 6c 2b 12d 0
layers. 12 – 3a = 0, 4b + 6a = 0, 6c + 6b + 3a = 0,
73. (c) Fructose in humans is produced inside 6c + 2b + 12d = 0
seminal vericles.
a = 4, b = –6, c = 4, d = –1
74. (d) Blood thinners present the clotting of blood
inside the human body. Hence prevent stroke. a b c d = 15
75. (c) Hairs of ovary wall. 83. (c) (2x – 4)2 = 4x
76. (c) Molecular oxygen inhibit the activity of
(x – 2)2 =x
nitrogenase.
79. (c) 2pq 2 × 0.6 × 0.4 0.48 x2 – 5x + 4 = 0
80. (d) Liver and muscle stores glycogen. It is x = 1, 4
converted into glucose when the blood glucose
level falls.

81. (c) Coefficient of highest degree term must be


negative becuase if it is positive, then x ,
y and it is not possible, since f(x) 100.
Now, graph will be like

C(1, –2)
B(4, 4)
AB AC

2 2
at least two real roots will be there, & if x 4 16 1 4

1 9
then f(x) < 100, it is not always true, as On solving, we get
2 2
the graph can be like this also x2 – 5x + 4 = 0
84. (c) Let AB b, AC c

AM b

AN mc

1 1
OD AB R sin 2R cos
2 2
2R AB 2R 2R cos
2 2
R sin cos
rOAB
1 cos
Let G divides MN in the ratio K : 1
drOAB 1 cos cos 2 sin cos sin
0
d 2
k c b b c 1 cos
So
k 1 3
5 1
at cos
k 1 1 2
k 1 3 k 1 3
x2 x3 x4
86. (*) f(x) = 1 + x +
1 1 2 6 24
k 3
x2 x3
f '(x) 1 x
AM GM 2 6

1 1 x2
1 2
2 f '(x) 1 x 0
… (1) 2
2 3

1
b c
area of AMN 2
Now,
area of ABC 1
b c
2
=
f (x) is an increasing f "
1 1 0, 1
using 3 Ratio =
3 1 f '(x) 0 at x x0
2 x02 x 03
maximum value of ratio = attain when f ' x0 0 1 x0 0 … (1)
3 1 2 6
1 sing derivative but is not 1 because f ( 2)f '( 1) 0
M is an interior point. x0 2, 1
4 1 x 02 x 03 x 04 x 04
so ratio < f x0 1 x0 0
9 2 2 6 24 24
85. (a) OD = R sin
AB = 2Rcos

ar OAB
rOAB
semi perimeter

no solution
87. (d) n 1 x 2 x 3 x
x x x
88. (c) dx dx dx
1
x 1
x 2
x

n 1 x n r 1 x
x x
dx dx
1
x r 1 r x

AP = Rsin n r 1
(x r)r
area of ring = (2 Rsin )·Rd dx
r 1 r r
Total area required, R2
r 1
2
n
(x r)r 1
= 2 R sin d
0 r 1 r(r 1) r

1 n
1 n
1 cos
2
r 1 r(r 1) n 1

1
cos 100
C5 1 2 3
2
89. (a) 100
C5 5!
Now,
Suppose 1, 2, 3, 4, 5 are
selected coupons.

1
20
Place of 1 is fixed
Total arrangements of 5 is 2

and 1 5

arrangements of 2, 3, 4, are

2 3 1 4 5
4 2 1 3 5 3 ways
4 3 1 2 5

90. (d) Let teams be T1, T2, T3, T4 & T5


R Now, we can have 5 teams with the scores of
cos
R H 2 points each matches are
(I) T1 T2 (II) T1 T3
1 R
R H 6400 (III) T1 T4 (IV) T1 T5
2 R H
(V) T2 T3 (VI) T2 T4
(VII) T2T5 (VIII) T3 T4
(IX) T3 T5 (X) T4 T5
Now,

6 x
3 6 x tan 1
12

(6 x)(6 x) tan
3
12

x
1.4 x tan 1
12

x 12 x
1.4 tan
12

This score board contradicts, option D 30 36 x 2


D is not always necssarily true. 14 12x x2
91. (c) By applying conservation of linear Hence,
momentum, we get
mv = (m + M)v0 360 (360)2 4 36 14 6
x
32
2
mv sin R MR2 mR2 0
5 360 312 48
x 1.5 m
32 32
h R 2M 5M 2
mv R 0R 94. (b) Now, energy of photon = 2.6 eV
R 5
2 1 1
2M 5M 2.6 = 13.6z 2
m M h R 0R 0R
2
n 42
5
2.6 1 1
Hence, 22
2
13.6 4 n 42
h 10m 7M
2.6 1 1
R 5 m M 2
13.6 4 n 16
92. (a) Let the ball bounce from the ground at x m
far ahead of boy, n=3

13.6Z2
Et eV
n2
By putting values of Z and n
we get,
13.6 4
Et eV 6 eV
9
95. (a) Q1 2 = w12
wTotal = w12 + w31
10 = w12 – 20 w 30J
Hence, Q = w = 30 J
99. (a) mgH – 2 mg(d + x) – mgh = 0
By solving, we get
Here, h = H – 2 (d + x)
V0 = Vi when no current flow through 1k \
for negative values of Vi, i from D1 = 0 (always)
i from D2 = 0 upto 3V
So from 0 to –3V Vi = V0 100. (d)
from –3 to –4 V V0 = –3V
For positive values of Vi i from D1 = 0 upto
1V i from D2 = 0
(always)
Hence 0 to 1V Vi = V0
1 to 4 V V0 = 1 volt kA kA
3 100 T T 0
Correct graph is (a)
300 – 3T = T
96. (d) mg sin Fr ma
300 = 4T
2 a
Fr mr 2 300
5 r2 T 75 C
4
5 g sin
a NH2 NHCOCH3
7 R r
101. (a) + (CH3CO)2O
5g
7 R r Acetanilide

7 R r NBS
T 2 102. (d) H3C C CH2 CH3
5r |
CH3
97. (a) Here,

1 12 2 CH3 C CH CH2CH3
mgh m 2
mR 2 2 |
2 25 R CH2Br
(Z and E)
7 2
mgh m
10 Br
|
Hence, initial horizontal speed of sphere, CH3 C CH CH CH3 Chiral center
|
CH3 (d+ )
10gh
7
NH – CH3
98. (d)
103. (a)
2 2
PB0 PS nA 110. (b) Co H2 O 6
6NH3 CO NH3 6
104. (d)
PB0 nB
O2 (Oxidation) 3
CO NH3 6
760 750 18 / M
6
760 108 / 18 1
113. (d) For EcoRF number of sites = 10,000
4
10 18 / M
= 2.44
760 6
4
1
6 18 18 76 For Rsa number of sites 1000 = 39.06
or M 228 4
76 M 6
114. (b) Because early embryonic cells are yet to
differentiate.
105. (b) E 0
0.34 0.76 1.1
Cell 115. (b) Plastid present in plasmodium is not capable
of photosynthesis.
Now G nFE0Cell 116. (a) Comptition is harmful for both interacting
species.
2 96500 1.1 213 V
117. (a, c)
1
106. (b) K 2 K 12
4 kb
MIt EIt 100 3 t
108. (b) w Bam HI
96500n 96500 96500
w = density × volume Bam HI 3 kb
= 10.5 × 80 × 5 × 10–4 = 42 × 10–2
1 kb EcoRI
42 10 2 96500
t 135 sec.
100 3 EcoRI 3.5 kb 2.5 kb

w Hence, 1 kb, 3 kb, 4 kb


109. (a) N1 V1 118. (d) Brown fat helps in conservation of body heat.
E

w
100 0.25
E

w
or 100 0.25 (As n = 1)
M/n
w = 100 × 0.25 × 248 = 6.2 g
sin(x a) sin(x a)
7. Let f (x) , then–
cos(x a) cos(x a)
1. Three children, each accompanied by a guardian,
seek admission in a school. The principal want to (a) f (x 2 ) f (x) but f (x ) f (x) for any
interview all the 6 persons one after the other 0< <2
subject to the condition that no child is (b) f is a strictly increasing function
interviewed before its guardian. In how many (c) f is strictly decreasing function
ways can this be done – (d) f is a constant function
(a) 60 (b) 90 8. The value of tan81° – tan63° – tan27° + tan9° is –
(c) 120 (d) 180 (a) 0 (b) 2
2. In the real number system, the equation (c) 3 (d) 4
x 3 4 x 1 x 8 6 x 1 1 has – 9. The mid- point of the domain of the function

(a) No solution f(x) 4 2x 5 for real x is –


(b) Exactly two distinct solutions 1 3
(a) (b)
(c) Exactly four distinct solutions 4 2
(d) Infinitely may solutions 2 2
(c) (d) –
3. The maximum value M of + – 3x 5x 9x + 15x –25x, 3 5
as x varies over reals, satisfies – 10. Let n be a natural number and let ‘a’ be a real
(a) 3 < M < 5 (b) 0 < M < 2 number. The number of zeros of x2n+1 – (2n + 1)x
+ a = 0 in the interval [–1, 1] is –
(c) 9 < M < 25 (d) 5 < M < 9
(a) 2 if a > 0
4. Suppose two perpendicular tangents can be drawn
from the origin to the circle x2 + y2 – 6x –2py + (b) 2 if a < 0
17 = 0, for some real p. then |p| = (c) At most one for every value of a
(a) 0 (b) 3 (d) At least three for every value of a
(c) 5 (d) 17 11. Let f : R R be the function f(x) = (x – a 1)
5. Let a, b, c, d be numbers in the set {1, 2, 3, 4, 5, 6} (x – a2)+(x – a2)(x – a3) + (x – a3) (x – a1) with a1,
such that the curves y = 2x3 + ax + b and y = 2x3 a2, a3 R. Then f(x) > 0 if and only if –
+ cx + d have no point in common. The maximum (a) At least two of a1, a2, a3 are equal
possible value of (a – c)2 + b –d is – (b) a1 = a2 = a3
(a) 0 (b) 5 (c) a1, a2, a3 are all distinct
(c) 30 (d) 36 (d) a1, a2, a3, are all positive and distinct
6. Consider the conic ex2 + y2 –2e2x –2 2y + e3 + 3 /2
= e. Suppose P is any point on the conic and S1, (sin x) 2x 1
dx
S2 are the foci of the conic, then the maximum 0
value of (PS1 + PS2) is – 12. The value /2 is –
2 1
(a) e (sin x) dx
0
(b) e
2 1 2 1
(a) (b)
(c) 2 2 1 2 1

(d) 2 e 2 1
(c) (d) 2 2
2
2012 19. Suppose a1, a2, a3, …, a2012 are integers arranged
13. The value of sin x3 x5 1 dx is – on a circle. Each number is equal to the average
2012 of its two adjacent numbers. If the sum of all even
(a) 2012 (b) 2013 indexed numbers is 3018, what is the sum of all
numbers?
(c) 0 (d) 4024
(a) 0 (b) 1509
14. Let [x] and {x} be the integer part and fractional
part of a real number x respectively. The value (c) 3018 (d) 6036

5
20. Let S = {1, 2, 3, …..,n} and A = {(a, b) |1 a,
of the integral [x]{x}dx is – b n} = S x S. A subset B of A is said to be a good
0 subset if (x, x) B for every x S. Then the
number of good subsets of A is –
5 (a) 1 (b) 2n
(a) (b) 5
2 2
(c) 2n(n–1) (d) 2n
(c) 34.5 (d) 35.5
n
15. Let Sn = k denote the sum of the first n 21. An ideal monatomic gas expands to twice its
k 1 volume. If the process is isothermal, the
positive integers. The numbers S1, S2, S3, … S99 magnitude of work done by the gas is W1. If the
are written on 99 cards. The probability of drawing process is adiabatic, the magnitude of work done
a card with an even number written on it is – by the gas is Wa . Which of the following is true?
1 49 (a) Wi = Wa > 0 (b) Wi > Wa = 0
(a) (b) (c) Wi >Wa > 0 (d) Wi > Wa > 0
2 100
22. The capacitor of capacitance C in the circuit shown
49 48
(c) (d) in fully charged initially. Resistance is R.–
99 99
S
16. A purse contains 4 copper coins and 3 silver coins.
A second purse contains 6 copper coins and 4 silver
coins. A purse is chosen randomly and a coin is R
C
taken out of it. What is the probability that it is a
copper coin
After the switch S is closed, the time taken to
41 31 reduce the stored energy in the capacitor to half
(a) (b)
70 70 its initial value is
RC
27 1 (a) (b) 2RC in 2
(c) (d) 2
70 3
RC ln 2
17. Let H be the orthocenter of an acute - angled (c) RC ln 2 (d)
2
triangle ABC and O be its circumcenter. Then 23. A liquid drop placed on a horizontal plane has a
HA HB HC near spherical shape (slightly flattened due to
gravity). Let R be the radius of its largest
(a) is equal to HO horizontal section. A small disturbance causes the
drop to vibrate with frequency v about its
(b) is equal to 3 HO
equilibrium shape. By dimensional analysis the
(c) is equal to 2 HO v
ratio can be (Here is surface tension,
(d) is not a scalar multiple of HO in general / R3
is density, g is acceleration due to gravity, and
18. The number of ordered pairs (m, n), where m, n
k is arbitrary dimensionless constant)–
{1, 2, 3, …, 50}, such that 6m + 9n is a multiple of
5 is – k gR 2 k R2
(a) (b)
(a) 1250 (b) 2500 g
(c) 625 (d) 500 k R3 k
(c) (d)
g g
24. Seven identical coins are rigidly arranged on a 26. In a Young’s double slit experiment the intensity
flat table in the pattern shown below so that each of light at each slit is I0. Interference pattern in
coin touches its neighbours. Each coin is a thin observed along a direction parallel to the line S1
disc of mass m and radius r. Note that the moment S2, on screen S.–
of inertia of an individual coin about an axis
passing through centre and perpendicular to the
plane of the coin is mr2 / 2
S1

S2
P
S
The minimum, maximum, and the intensity
averaged over the entire screen are respectively
The moment of inertia of the system of seven (a) 0, 4I0, 2I0 (b) 0, 4I0, I0
coins about an axis that passes through the point
(c) I0, 2I0, 3I0/2 (d) 0, 20, I0
P (the centre of the coin positioned directly to
the right of the central coin) and perpendicular 27. A loop carrying current I has the shape of a regular
to the plane of the coins is– polygon of n sides. If R is the distance from the centre
to any vertex, then the magnitude of the magnetic
55 127
(a) mr2 (b) mr2 induction vector B at the centre of the loop is –
2 2
0I 0I
111 (a) n tan (b)
(c) mr2 (d) 55 mr2 2 R n 2R
2 I
0 2 0I
25. A planet orbits in an elliptical path of eccentricity (c) n tan (d) tan
2 R n R n
e around a massive star considered fixed at one
of the foci. The point in space where it is closest 28. A conducting rod of mass m and length l is free to
move without friction on two parallel long
to the star is denoted by P and the point where it
conducting rails, as shown below. There is a
is farthest is denoted by A. Let vp and va be the
resistance R across the rails. In the entire space
respective speeds at P and A. Then–
around, there is a uniform magnetic field B normal
vP to the plane of the rod and rails. The rod is given
an impulsive velocity v0 –

A B
P Star
R l v0

vA

vP 1 e 1
(a)
vA 1 e Finally, the initial energy mv02
2
(a) Will be converted fully into heat energy in
vP 1 e2
(b) the resistor
vA 1 e
(b) Will enable rod to continue to move with
vP velocity v0 since the rails are frictionless.
(c) 1 (c) Will be converted fully into magnetic energy
vA
due to induced current
vP 1 e2 (d) Will be converted into the work done against
(d) 2 the magnetic field
vA 1 e
29. A steady current I flows through a wire of radius 35. The potential energy of a point particle is given
r, length L and resistivity . The current produced x
heat in the wire. The rate of heat loss in a wire is by the expression V(x) x sin . A
proportional to its surface area. The steady dimensionless combination of the constant ,
temperature of the wire is independent of– and is–
(a) L (b) r 2

(c) I (d) (a) (b)


30. The ratio of the speed of sound to the average
speed of an air molecule at 300 K and 1 (c) (d)
atmospheric pressure is close to–
36. A ball of mass m suspended from a rigid support
1
(a) 1 (b) by an inextensible massless string is released
300 from a height h above its lowest point. At its
(c) 300 (d) 300 lowest point it collides elastically with a block of
31. In one model of the electron, the electron of mass mass 2m at rest on a frictionless surface. Neglect
me is thought to be a uniformly charged shell of the dimensions of the ball and the block. After
radius R and total charge e, whose electrostatic the collision the ball rises to a maximum height
energy E is equivalent to its mass m e via of–
Einstein’s mass energy relation E = mec2. In this
model, R is approximately (me = 9.1 × 10–31 kg,
c = 3 × 108 ms–1, 1/4 0 = 9 × 109 Farad m–1,
magnitude of the electron charge = 1.6 × 10–19 C) –
m
(a) 1.4 10–15 m (b) 5.3 × 10–11 m
(c) 2 × 10–13 m (d) 2.8 × 10–35 m h
32. A body is executing simple harmonic motion of 2m
amplitude a and period T about the equilibrium h h
position x = 0. large numbers of snapshots are (a) (b)
3 2
taken at random of this body in motion. The
probability of the body being found in a very small h h
(c) (d)
interval x to x + dx is highest at – 8 9
a 37. A particle released from rest is falling through a
(a) x = a (b) x =
2 thick fluid under gravity. The fluid exerts a
(c) x = 0 (d) x = / 2 resistive force on the particle proportional to the
33. Two identical bodies are made of a material for square of its speed. Which one of the following
which the heat capacity increases with graphs best depicts the variation of its speed v
with time t –
temperature. One of these is held at a
v
temperature of 100ºC while the other one is kept
0º C. If the two are brought into contact, then,
assuming no heat loss to the environment, the (a)
final temperature that they will reach is – t
(a) 50º C (b) Less than 50º C v
(c) More than 50º C (d) 0º C
34. A particle is acted upon by a force given by (b)
F = – x 3 – x 4 where and are positive t
constants. At the point x = 0, the particle is – v
(a) In stable equilibrium
(b) In unstable equilibrium (c)
(c) In neutral equilibrium t
(d) Not in equilibrium v

(d)
t
38. A cylindrical steel rod of length 0.10 m and thermal 44. In the reaction benzene with an electrophile E+,
conductivity 50 W.m–1 K–1 is welded end to end to the structure of the intermediate – complex can
copper rod of thermal conductivity 400 W.m–1.K–1 be represented as
and of the same area of cross section but 0.20 m
long. The free end of the steel rod is maintained
(a) (b)
at 100º C and that of the copper and at 0º C.
Assuming that the rods are perfectly insulated
from the surrounding, the temperature at the
junction of the two rods–
(c) (d)
(a) 20º C (b) 30º C
(c) 40º C (d) 50º C
39. A parent nucleus X is decaying into daughter 45. The most stable conformation of 2, 3-
nucleus Y which in turn decays to Z. The half dibromobutane is –
lives of X and Y are 40000 years and 20 years
respectively. In a certain sample, it is found that
the number of Y nuclei hardly changes with time.
If the number of X nuclei in the sample is 4 × 1020, (a) (b)
the number of Y nuclei present in its is–
(a) 2 × 1017 (b) 2 × 1020
(c) 4 × 1023 (d) 4 × 1020
40. An unpolarized beam of light of intensity I0 passes
through two linear polarizers making an angle of
30º with respect to each other. The emergent (c) (d)
beam will have an intensity –
3I0 3I0
(a) (b) 46. Typical electronic energy gaps in molecules are
4 4 about 1.0 eV. In terms of temperature, the gap is
3I0 I0 closed to –
(c) (d)
8 8 (a) 102 K (b) 104 K
(c) 103 K (d) 105 K
47. The major final product in the following reaction
41. Among the following, the species with the highest is –
bond order is –
1) CH3MgBr
(a) O2 (b) F 2 CH3CH2CN
2) H2O
(c) O2+ (d) F 2
NH
42. The molecule with non-zero dipole moment is –
(a) BCl3 (b) BeCl2 (a) H3C
(c) CCl4 (d) NCl3 CH3
43. For a one-electron atom, the set of allowed (b) H3C CH3
quantum numbers is –
N
1
(a) n = 1, l = 0, m1 = 0, ms = + (c) O
2
H3C
1
(b) n = 1, l = 1, m1 = 0, ms = + CH3
2
(d) O
1
(c) n = 1, l = 0, m1 = –1, ms = – H3C
2 CH3
1 NH
(d) n = 1, l = 1, m1 = 1, ms = –
2
48. A zero-order reaction, A Product, with an initial 52. The oxidation state of cobalt in the following
concentration [A]0 has a half-life of 0.2 s. If one molecule is –
starts with the concentration 2[A]0, then the half-
life is –
(a) 0.1 s
(b) 0.4 s
(c) 0.2 s
(d) 0.8 s
49. The isoelectronic pair of ions is –
(a) Sc2+ and V3+ (b) Mn2+ and Fe3+
(c) Mn3+ and Fe2+ (d) Ni3+ and Fe2+ (a) 3 (b) 1
50. The major product in the following reaction is – (c) 2 (d) 0
H H 53. The pKa of a weak acid is 5.85. The concentrations
of the acid and its conjugate base are equal at a
NaNH2 pH of –
(a) 6.85
H Br (b) 5.85
(a) H H (c) 4.85
(d) 7.85
H H 54. For a tetrahedral complex [MCl4]2–, the spin-only
magnetic moment is 3.83 B.M. The element M
(b) is–
(a) Co
H NH2
(b) Cu
H2C CH2 (c) Mn

(c) (d) Fe
H2N NH2 55. Among the following graphs showing variation of
rate (k) with temperature (T) for a reaction, the
one that exhibits arrhenius behavior over the
(d) NH2
H3C entire temperature range is –
51. The major product of the following reaction is –

Conc. HBr
HO

Br
(a)
(a) HO
CH3
1/T
(b) Br

(c) Br Br

Br

(d) Br
(b)
H3C

1/T
58. The equilibrium constant for the following
reactions are K1 and K2, respectively.
2P(g) + 3Cl2(g) 2PCl3(g)
PCl3(g) + Cl2(g) PCl5(g)
(c) Then the equilibrium constant for the reaction
2P(g) + 5Cl2(g) 2PCl5(g) is –
1/T (a) K1K2 (b) K 1K22
(c) K12K22 (d) K12K 2
59. The major product of the following reaction is –
CH2CH3
AlCl3
+ (CH3C)2CHCH2Cl

(d)
CH2CH3

1/T (a)
56. The reaction that gives the following molecule as (H3C)3C
the major product is –
(b) CH2CH3
CH3
H3C
CH3
O
CH2CH(CH3)2
H3C
H3C
CH2CH3
(a) H3C
Br + Ch3ONa
(c)
H3C
(H3C)2HCH2C
H3C
(d) CH2CH3
(b) H3C ONa + CH3Br
H3C
C(CH3)3
H3C 60. Doping silicon with boron produces a –
(c) H3C OH + CH3ONa (a) n-type semiconductor
(b) Metallic conductor
H3C
(c) p-type semiconductor
H3C (d) Insulator
(d) CH3 + CH3ONa

H3C 61. The disorders that arise when the immune system
destroys self cells are called autoimmune
57. The C–O bond length in CO, CO2 and CO32– disorders. Which of the following would be
follows the order – classified under this?
(a) CO < CO2 < CO32– (a) rheumatoid arthritis
(b) CO2 < CO32– < CO (b) asthma
(c) CO > CO2 > CO32– (c) rhinitis
(d) CO32– < CO2 < CO (d) eczema
62. When of the following class of immunoglobulins 71. In mammals, the hormones secreted by the
can trigger the complement cascade ? pituitary, the master gland, is itself regulated by–
(a) IgA (b) IgM (a) hypothalamus (b) median cortex
(c) IgD (d) IgE (c) pineal gland (d) cerebrum
63. Diabetes insipidus is due to – 72. Which of the following is true for TCA cycle in
(a) hypersecretion of vasopressin eukaryotes?
(b) hyposecretion of insulin (a) takes place in mitochondrion
(c) hypersecretion of insulin (b) produces no ATP
(d) hyposecretion of vasopressin (c) takes place in golgi complex
64. Fossils are most often found in which kind of (d) independent of electron transport chain
rocks?
73. A hormone molecule binds to a specific protein
(a) meteorites (b) sedimentary rocks on the plasma membrane inducing a signal. The
(c) igneous rocks (d) metamorphic rocks protein it binds to is called –
65. Peptic ulcers are caused by – (a) ligand
(a) a fungus, Candida albicans (b) antibody
(b) a virus, cytomegalovirus (c) receptor
(c) a parasite, Trypanosoma brucei (d) histone
(d) a bacterium, Helicobacter pylori 74. DNA mutations that do not cause any functional
66. Transfer RNA (tRNA) – change in the protein product are known as –
(a) is present in the ribosomes and provides (a) nonsense mutations
structural integrity (b) missense mutations
(b) usually has clover leaf-like structure
(c) deletion mutations
(c) carries genetic information form DNA to
(d) silent mutations
ribosomes
75. Plant roots are usually devoid of chlorophyll and
(d) codes for proteins
cannot perform photosynthesis. However, there
67. Some animals excrete uric acid in urine are exceptions. Which of the following plant root
(uricotelic) as it requires very little water. This can perform photosynthesis ?
is an adaptation to conserve water loss. Which
(a) Arabidopsis
animals among the following are most likely to
be uricotelic? (b) Tinospora
(a) fishes (b) amphibians (c) Rice
(c) birds (d) mammals (d) Hibiscus
68. A ripe mango, kept with unripe mangoes causes 76. Vitamin A deficiency leads to night-blindness.
their ripening. This is due to the release of a Which of the following is the reason for the
gaseous plant hormone– disease?
(a) auxin (b) gibberlin (a) rod cells are not converted to cone cells
(c) cytokinine (d) ethylene (b) rhodopsin pigment of rod cells is defective
69. Human chromosomes undergo structural changes (c) melanin pigment is not synthesized in cone
during the cell cycle. Chromosomal structure can cells
be best visualized if a chromosome is isolated from (d) cornea of eye gets dried
a cell at – 77. In Dengue virus infection, patients often develop
(a) G1 phase (b) S phase haemorrhagic fever due to internal bleeding. This
(c) G2 phase (d) M phase happens due to the reduction of –
70. By which of the following mechanisms is glucose (a) platelets
reabsorbed from the glomerular filtrate by the (b) RBCs
kidney tubule ? (c) WBCs
(a) osmosis (b) diffusion (d) lymphocytes
(c) active transport (d) passive transport
78. If the sequence of bases in sense strand of DNA 84. The sum of all x [0, ] which satisfy the equation
is 5’-GTTCATCG-3’, then the sequence of bases
1
in its RNA transcript would be – sin x cos x sin 2 x is –
2 4
(a) 5’-GTTCATCG-3’
(b) 5’-GUUCAUCG-3’ 5
(a) (b)
(c) 5’-CAAGTAGC-3’ 6 6

(d) 5’-CAAGUAGC-3’ (c) (d) 2


79. A reflex action is a quick involuntary response to 85. A polynomial P(x) with real coefficients has the
stimulus. Which of the following is an example of property that P’’(x) 0 for all x. Suppose P(0) = 1
BOTH, unconditioned and conditioned reflex ? and P’(0) = –1. What can you say about P(1)?
(a) knee Jerk reflex (a) P(1) 0 (b) P(1) 0
(b) secretion of saliva in response to the aroma (c) P(1) 0 (d) –½ < P(1) < ½
of food 86. Define a sequence (an) by a1 5, an a1a 2 … a n 1
(c) sneezing reflex
an
(d) contraction of the pupil in response to bright 4 for n > 1. Then lim
light n an 1

80. In a food chain such as grass deer lion, the 1


energy cost of respiration as a proportion of total (a) Equals
2
assimilated energy at each level would be –
(b) equals 1
(a) 60% - 30 % - 20%
2
(b) 20% - 30 % - 60% (c) equals
5
(c) 20% - 60 % - 30%
(d) does not exist
(d) 30% - 30 % - 30%
cos2 dx
87. The value of the integral dx , where a
1 ax
> 0, is –
(a) (b) a
81. Suppose a, b, c are real numbers, and each of the
equations x2 + 2ax + b2 = 0 and x2 + 2bx + c2 = 0
(c) (d) 2
has two distinct real roots. Then the equation x2 2
+ 2cx + a2 =0 has– 88. Consider
(a) Two distinct positive real roots
3 3 3
L 2012 2013 3011
(b) Two equal roots
(c) One positive and one negative root 3 3 3
R 2013 2014 3012
(d) No real roots
3012
1 x and I 3
xdx Then –
82. The coefficient of x2012 in is –
2
1 x (1 x) 2012

(a) 2010 (b) 2011 (a) L + R < 2I (b) L + R > 2I


(c) 2012 (d) 2013
(c) L + R = 2I (d) LR I
83. Let (x, y) be a variable point on the curve
89. A man tosses a coin 10 times, scoring 1 point for
4x2 + 9y2 – 8x –36y + 15 = 0. Then
each head and 2 points for each tail. Let P(K) be
min(x2 2x y2 4y 5) max(x2 2x y2 4y 5) the probability of scoring at least K points. The
is–
1
325 36 largest value of K such that P(K) > is –
(a) (b) 2
36 325
13 25
(a) 14 (b) 15
(c) (d) (c) 16 (d) 17
25 13
94. A stream of photons having energy 3 eV each
x 1
90. Let f(x) = for all x 1. impinges on a potassium surface. The work
x 1 function of potassium is 2.3 eV. The emerging
Let f1(x) f (x), f2(x) f (f (x)) and generally photo-electrons are slowed down by a copper plate
fn (x) f(fn 1(x)) for n > 1 placed 5 mm away. If the potential difference
between the two metal plates is 1 V, the maximum
Let P f 1 (2) f 2 (3) f 3 (4) f 4 (5) distance the electrons can move away from the
potassium surface before being turned back is–
Which of the following is a multiple of P –
(a) 3.5 mm (b) 1.5 mm
(a) 125
(c) 2.5 mm (d) 5.0 mm
(b) 375
95. Consider three concentric metallic spheres A, B
(c) 250
and C of radii a, b, c respectively where a < b < c,
(d) 147 A and B are connected whereas C is grounded.
The potential of the middle sphere B is raised to
V then the charge on the sphere C is–
91. The total energy of a black body radiation source bc ac
is collected for five minutes and used to heat (a) 4 0 V (b) 4 0 V
c b c a
water. The temperature of the water increases
bc
from 10.0º C to 11.0ºC. The absolute temperature (c) 4 0 V (d) zero
c b
of the black body is doubled and its surface area
halved and the experiment repeated for the same 96. On a bright sunny day a diver of height h stands
time. Which of the following statements would at the bottom of a lake of depth H. Looking
be most nearly correct? upward, he can see objects outside the lake in a
circular region of radius R. Beyond this circle he
(a) The temperature of the water would increase
sees the image of objects lying on the floor of the
from 10.0º C to a final temperature of 12º C
4
(b) The temperature of the water would increase lake. If refractive index of water is , then the
3
from 10.0º C to a final temperature of 18º C value of R is–
(c) The temperature of the water would increase 3(H h)
from 10.0º C to a final temperature of 14ºC (a)
7
(d) The temperature of the water would increases (H h)
from 10.0º C to a final temperature of 11º C (b)
7/3
92. A small asteroid is orbiting around the sun in a (c) 3h 7
circular orbit of radius r0 with speed V0. A rocket
(H h)
is launched from the asteroid with speed V = V0, (d)
where V is the speed relative to the sun. The 5/3
highest value of for which the rocket will remain 97. As shown in the figure below, a cube is formed
bound to the solar system is (ignoring gravity due with ten identical resistance R (thick lines) and
to the asteroid and effects of other planets) – two shortening wires (dotted lines) along the arms
(a) 2 (b) 2 AC and BD.
(c) 3 (d) 1
93. A radioactive nucleus A has a single decay mode
with half life A. Another radioactive nucleus B
has two decay modes 1 and 2. If decay mode 2
were absent, the half life of B would have been
A
. If decay mode 1 were absent, the half life of
2
B
B would have been 3 A, then the ratio is–
A
3 7 Resistance between point A and B is–
(a) (b) R 5R
7 2 (a) (b)
7 2 6
(c) (d) 1 3R
3
(c) (d) R
4
98. A standing wave in a pipe with a length L = 1.2 m 100. A simple pendulum is released from rest at the
is described by horizontally stretched position. When the string
makes an angle with the vertical, the angle
2 2
y (x, t) y 0 sin x sin x which the acceleration vector of the bob makes
L L 4 with the string is given by–
Based on above information, which one of the
following statements is incorrect.
(Speed of sound in air is 300 m s–1)–
(a) The pipe is closed at both ends ac
(b) The wavelength of the wave could be 1.2 m anet
(c) There could be a node at x = 0 and antinode at
L at
x=
2
(a) 0
(d) The frequency of the fundamental mode of
vibrations is 137.5 Hz 1 tan
(b) tan
99. Two blocks (1 and 2) of equal mass m are connected 2
by an ideal string (see figure shown) over a 1
(c) tan 2 tan
frictionless pulley. The blocks are attached to the
ground by springs having spring constants k1 and
k2 such that k1 > k2 (d)
2

101. The final major product obtained in the following


sequence of reactions is –

1. NaNH2, NH3
Ph
2. CH3I
3. H2, Pd/C
1 2
k1 k2 Ph H Ph CH3

Initially, both springs are unstretched. The block (a) (b)


1 is slowly pulled down a distance x and released.
H CH3 H H
Just after the release the possible values of the
magnitude of the acceleration of the blocks a1
and a2 can be– Ph H
k1 k2 x
(a) Either a1 a2 or CH3
2m (c) (d)
k1 x k2x Ph
a1 g and a 2 g H3C H
m m
k1 k2 x 102. In the DNA of E. Coli the mole ratio of adenine to
(b) a1 a2 only cytosine is 0.7. If the number of moles of adenine
2m
in the DNA is 350000, the number of moles of
k1 k2 x guanine is equal to –
(c) a1 a2 only
2m (a) 350000
k1 k2 x (b) 500000
(d) Either a1 a2 or
2m (c) 225000
k1 k 2 x (d) 700000
a1 a2 g
k1 k2 m
103. (R)-2-bromobutane upon treatment with aq. OH OH
NaOH gives –

CH
(d) and
H
(a) CH3 OH
H3C NO2
105. A metal is irradiated with light of wavelength 660
OH OH nm. Given that the work that the work function
of the metal is 1.0 eV, the de Broglie wavelength
H H of the ejected electron is close to –
(b) CH3+ H3C
H3C CH3 (a) 6.6 × 10–7 m
(1 : 1 mixture)
(b) 8.9 × 10–11 m
OH (c) 1.3 × 10–9 m
(d) 6.6 × 10–13 m
(c) H3C H
106. The inter-planar spacing between the (2 2 1)
CH3 planes of a cubic lattice of length 450 pm is –
(a) 50 pm (b) 150 pm
H3C
OH (c) 300 pm (d) 450 pm
(d) 107. The H for vaporization of a liquid is 20 kJ/mol.
CH3 Assuming ideal behaviour, the change in internal
energy for the vaporization of 1 mol of the liquid
104. Phenol on treatment with dil. HNO3 gives two
at 60°C and 1 bar is close to –
products P and Q. P is steam volatile but Q is not.
P and Q are, respectively– (a) 13.2 kJ/mol
OH OH (b) 17.2 kJ/mol
(c) 19.5 kJ/mol
NO2
(d) 20.0 kJ/mol
and 108. Among the following, the species that is both
(a) tetrahedral and diamagnetic is –
(a) [NiCl4]2– (b) [Ni(CN)4]2–
NO2 (c) Ni(CO)4 (d) [Ni(H2O)6]2+
OH OH 109. Three moles of an ideal gas expands reversibly
under isothermal condition form 2 L to 20 L at
300 K. The amount of heat-change (in kJ/mol) in
the process is –
and
(b) (a) 0 (b) 7.2
NO2 (c) 10.2 (d) 17.2
NO2 110. The following data are obtained for a reaction,
X + Y Products.
OH OH Expt. X 0 /mol Y0 /mol rate/mol L 1s 1

OH 6
NO2 1 0.25 0.25 1.0 10
6
and 2 0.50 0.25 4.0 10
6
(c) 3 0.25 0.50 8.0 10
The overall order of the reaction is
NO2 OH (a) 2 (b) 4
(c) 3 (d) 5
(c) The rate of the reaction is retarded in the
presence of an enzyme
111. Why hydrogen peroxide is applied on the wound
as a disinfectant, there is frothing at the site of (d) The rate of the reaction is independent of
injury, which is due to the presence of an enzyme substrate concentration
in the skin that used hydrogen peroxide as a 117. Vibrio cholerae causes cholera in humans. Ganga
substrate to produce– water was once used successfully to combat the
(a) Hydrogen (b) Carbon Dioxide infection. The possible reason could be–
(c) Water (d) Oxygen (a) High salt content of Ganga water
112. Persons suffering from hypertension (high blood (b) Low salt content of Ganga water
pressure) are advised a low-salt diet because– (c) Presence of bacterophases in Ganga water
(a) More salt is absorbed in the body of a patient (d) Presence of antibiotics in Ganga Water
with hypertension 118. When a person beings to fast, after some time
(b) High salt leads to water retention in the blood glycongen stored in the liver is mobilized as a
that further increases the blood pressure source of glucose. Which of the following graphs
(c) High salt increases nerve conduction and best represents the change of glucose level (y axis)
increases blood pressure in his blood, starting from the time (x -axis) when
the beings to fast ?
(d) High salt causes adrenaline release that
increases blood pressure (a)
113. Insectivorous plants that mostly grow on swampy
soil use insects as a source of– Glucose
(a) Carbon (b) Nitrogen
(c) Phosporous (d) Magnesium
114. In cattle, the coat colour red and white are two time
dominant traits, which express equally F 1 to
produce roan (red and white colour in equal (b)
proportion). If F 1 progeny are selfbred, the
resulting progency in F2 will have phenotypic Glucose
ration (red : roan: white) is –
(a) 1: 1:1 (b) 3 : 9 : 3
(c) 1 : 2 : 1 (d) 3 : 9 : 4
time
115. The restriction endonuclease EcoR-I recognizes
and cleaves DNA sequence as shown below
(c)
5’ -G A A T T C-3’
3’ -C T T A A G-5’ Glucose
What is the probable number of cleavage sites
that can occur in a 10 kb long random DNA
sequence?
time
(a) 10 (b) 2
(c) 100 (d) 50 (d)
116. Which one of the following is true about enzyme
catalysis?
(a) The enzyme changes at the end of the reaction Glucose
(b) The activation barrier of the process is lower
in the presence of an enzyme
time
119. The following sequence contains the open reading 120. Insects constitute the largest animal group on
frame of a polypeptide. How many amino acids earth. About 25-30% of the insect species are
will the polypeptide consists of ? known to be herbivores. In spite of such huge
5’ AGCATATGATCGTTTCTCTGCTTTGAACT-3’ herbivore pressure, globally, green plants have
(a) 4 (b) 2 persisted. one possible reason for this persistence
is –
(c) 10 (d) 7
(a) Food preference of insects has tended to
change with time
(b) Herbivore insects have become inefficient
feeders of green plants
(c) Herbivore population has been kept in control
by predators
(d) Decline in reproduction of herbivores with
time

1. (b) 2. (d) 3. (b) 4. (c) 5. (b) 6. (c) 7. (d) 8. (d) 9. (b) 10. (c)

11. (b) 12. (d) 13. (d) 14. (b) 15. (c) 16. (a) 17. (c) 18. (a) 19. (d) 20. (c)

21. (c) 22. (d) 23. (a) 24. (c) 25. (a) 26. (a) 27. (a) 28. (a) 29. (a) 30. (a)

31. (a) 32. (a) 33. (c) 34. (c) 35. (d) 36. (d) 37. (a) 38. (a) 39. (a) 40. (a)

41. (c) 42. (d) 43. (a) 44. (d) 45. (c) 46. (d) 47. (c) 48. (b) 49. (b) 50. (a)

51. (b) 52. (d) 53. (b) 54. (a) 55. (d) 56. (b) 57. (a) 58. (b) 59. (a) 60. (c)

61. (a) 62. (b) 63. (d) 64. (b) 65. (d) 66. (b) 67. (c) 68. (d) 69. (d) 70. (c)

71. (a) 72. (a) 73. (c) 74. (d) 75. (b) 76. (b) 77. (a) 78. (b) 79. (b) 80. (b)

81. (d) 82. (*) 83. (a) 84. (c) 85. (c) 86. (c) 87. (c) 88. (c) 89. (c) 90. (b)

91. (b) 92. (d) 93. (a) 94. (a) 95. (a) 96. (a) 97. (a) 98. (d) 99. (b) 100. (b)

101. (b) 102. (b) 103. (b) 104. (a) 105. (c) 106. (b) 107. (b) 108. (c) 109. (d) 110. (d)

111. (d) 112. (b) 113. (b) 114. (c) 115. (b) 116. (b) 117. (d) 118. (a) 119. (b) 120. (c)
6. (c) ex 2 y2 2e 2 x 2 2
y e3 3
e
6! e x2 2ex e2 y2 2 y 2
e
1. (b) 3
= 90
(2!)
(x e)2 (y )2
1
2. (d) x 3 4 x 1 x 8 6 x 1 1; x 1 e

(x 1) 2 x 14 (x 1) 6 x 1 9 1 a2 a e
PS1 PS2 2a Major axis is || to axis
x 1 1 x 1 3 1
Case-I PS1 PS2 2
x 1 2 x 1 3 0 x 10 sin(x a) sin(x a)
7. (d) f (x)
2 x 1 16 cos(x a) cos(x a)
x = 10 2sin(x) cos a
cot a
Case-II 2sin x sin a
x 1 2 x 1 3 1 5 x 10 8. (d) tan81° – tan63° – tan27° + tan9°
Case-III
tan(90° – 9°) – tan(90° – 27°) – tan27° + tan9°
x 1 2 x 1 3 1 1 x 5 cot9° – cot27° – tan27° + tan9°
2 x 1 4 =4
x=5
9. (b) f(x) 4 2x 5
3. (b) M a b a2 ab b2
4 2x 5 0 2x 5 0
a2 b2 5
Now, ab 2x 5 0 x
2 2
11
a2 b2 2ab x
2
a2 b2 2ab 5 11
x ,
M a b ab 2 2
M 1 (a 1)(b 1) 5 11
2 3 2
hence 0 < M < 2 Mid point =
2 2
4. (c) (x 3)2 (y p)2 9 17 p2
10. (c) f(x) = x2n+1 – (2n + 1)x + a
Director circle is We get,
(x 3)2 (y p)2 2(p2 8)
Passes through (0, 0)
f '(x) (2n 1)x 2n (2n 1)
9 + p2 = 2p2 – 16 (2n 1) x 2n 1 0 when x 1, 1

p2 25 p 5 p 5 f(x)is strictlydecreasing in[–1, 1]


5. (b) y 2x3 ax b y 2x3 cx d f(x) cut x axis at most one point in given
No Solution interval
11. (b) Only for a1 = a2 = a3 f(x) > 0
2x 3 ax b 2x 3 cx d In other cases f(x) will take both positive and
ax b cx d for no real negative values
(a c)x d b /2
2
12. (d) I1 (sin x) sin xdx
d b
x a=c 0
a c /2
(a – c)2 + (b – d) = 0 + 6 – 1 = 5 I2 (sin x) 2 1
dx
0
16. (a) P1 : 4 copper coins 3 silver coins
/2
2
I1 (sin x) sin xdx P2 : 6 copper coins 4 silver coins
0
E = Event of copper coin
/2
2 1 E E
2(sin x) cos x sin xdx
P(E) P P1 P P P2 P
0 P1 P2
/2 /2
cos x sin x
2
0 2 (sin x) 2 1
1 sin 2 x dx 1 4 1 6 41
0 2 7 2 10 70

I1 2 2 1 18. (a) 6m + 9n
2 2
I2 1 2 2 1 61 = 6 91 = 9
62 = 6 92 = 1
2012 2012
63 = 6 93 = 9
13. (d) sin x3 x5 1 dx sin x3 dx
2012 2012
64 = 6 94 = 1
m can be any value and n will be odd number
2012 2012 then sum is multiple of 5
x 5 dx dx 4024
50× 25 = 1250
2012 2012
19. (d) a1, a2, a3, …, a2012 = 3018 … (1)
5 5 1
[x]{x}dx [x](x [x])dx 0 dx a1 a3
14. (b) a2
0 0 0 2

2 3 4 2a 2 2a 4 2a 6 2a 2012 6036
1 (x 1)dx 2(x 2)dx 3(x 3)dx
1 2 3
a1 a3 a3 a5 a5 a7

5 a 2011 a1 6036
4(x 4)dx
4 2 a1 a3 a5 a 2011 6036

2 3 4 a1 a3 a5 a 2011 3018 … (2)


(x 1)2 (x 2)2 (x 3)2
2 3 Add (1) and (2)
2 1 2 2 2 3
Sum of all number = 3018 + 3018 = 6036
2 5
(x 4)
4 21. (c)
2 4

1 2 3 4
2 2 2 2
15. (c) 1, 3, 6, 10, 15, 21, 28, 36, 45, 55, 66, 78, 91,
105 till 98 terms
Among these 48 terms are even and 48 terms
odd

99 100 Wi > Wa > 0


Now, 99th term = = even
2
v
Total even terms = 48 + 1 = 49 23. (a) is dimensionless
/ R3
49
Required probability =
99 k gR 2
is also dimensionless
111 39. (a) In radioactive equilibrium
24. (c) By using parallel axis theorem, I = mr2 rate of decay of X = rate of decay of Y
2
25. (a) By using conservation of angular Nx Ny
momentum, x Nx y Ny ,
Tx Ty
We get
Vp rp VA rA 10 6
41. (c) B.O. of O2 = 2
2
VP rA a ae
VA rP a ae 10 5
O2 2.5
2
27. (a)
10 8
F2 1
2

10 9
F2 0.5
2

Bnet = n × B1 42. (d) N 0


Cl Cl
0 I Cl
n 2sin
4 n
R cos
n 43. (a)

n 0I E
tan
2 R N +
44. (d) +E
+
28. (a) Because of to energy conservation
29. (a) By Concept of fuse wire
45. (c) Because Anti form is most stable.
Vsound kT M
30. (a) 3
Vav m 8kT KT 1.6 10 19
46. (d)
2
33. (c)
19 2 1
T 1.6 10 23
3 1.38 10

T = 105 K
Heat capacity increases with temperature. (i)CH3 MgBr
47. (c) CH3CH2 C N C 2 H5 C NMgBr
2 |
dU dU d U CH3
34. (c) F, 0 and 0
dx dx x 0 dx 2 x 0
O
36. (d) Using energy conservation and law of
H2 O / H
restitution and momentum conservation C2 H5 C CH3

t1/ 2 1 a1 0.2 1
38. (a) 48. (b) OR
t1/ 2 2
a2 t1/ 2 2 2

50 A (100 T) 400 A (T 0)
or t1/ 2 2
0.4
0.1 0.2
T = 20°C
49. (b) Mn2+ = 25 – 2 = 23 electrons 61. (a) In rheumatoid arthritis our immune system
attacks self cells and destroy them. It may
Fe3+ = 26 – 3 = 23 electrons affect many tissues and organ. But it mainly
attacks joints.
50. (a) CH2 CH
NaNH2
CH CH NaBr NH3 62. (b) IgG or IgM can trigger the complement
| cascade. Complement system helps the
Br antibodies and phagocytic cell to clear the
pathogens.
51. (b) CH2 CH CH 2OH Conc.
HBr
63. (d) Diabetes incipidus is a condition in which a
person produce large amount of diluted urine
and also suffer from excessive thirst.
CH2 CH CH2 Br H2O
It is caused by hyposecretion of ADH.
52. (d) As all are neutral ligands. 64. (b) Fossils are found mostly in sedimentary
rocks.
65. (d) A spiral shape bacterium helicobacter pylori
H [salt] is responsible for peptic ulcers.
53. (b) P P Ka log
[Acid] 66. (b) Clover leaf model – 2D structure.
inverted L-shape model – 3D structure
PH P Ka log1 67. (c) Birds, arthopods, lizards and snakes are
uricotelic.
68. (d) Ethylene is a gaseous PGR. It helps in ripening.
OR P H P Ka 5.85
69. (d) Chromosome morphology is best visible at
54. (a) n(n 2) 3.83 M-phase of cell cycle.
70. (c) Active transport is responsible for reabsorption
so n = 3, so M2+ = 3d7 of glucose from glomerular filtrate.
72. (a) TCA cycle (Kreb’s cycle) takes place in
and hence cobalt.
mitochondria.
73. (c) Protein hormones generate secondary
Ea messenger and receptor for protein hormones
55. (d) nK nA
RT are present on cell membrane.
74. (d) Silent mutations do not change the amino
Comparing with y = mx + C, we can get graph acids. Hence they do not cause any functional
56. (b) Williamson’s synthesis reaction. change.
75. (b) Photosynthetic roots are present in tinospora.
76. (b) Vitamin A is required for the synthesis of
58. (b) 2P 3Cl2 2PCl3 K1 rhodopsin pigment of rod cells. Deficiency of
Vitamin A will cause defects in rhodopsin.
2PCl3 2Cl2 2PCl5 K2
77. (a) Platelets count is in dengue fever and
2P 5Cl2 2PCl5 K K 1 K 22
patient develop haemorrhagic fever due to
internal bleeding.
78. (b) DNA 5’ -GTTCATCG-3’
CH2CH3 mRNA 5’-GUUCAUCG-3’
Anhyd. AlCl3
80. (b) Grass Dear Lion
59. (a) + (CH3)2CHCH2Cl 20% 30% 60%
(Rearrangement)

CH2CH3
81. (d) Considering x2 + 2ax + b2 = 0 and x2 + 2bx + c2 = 0
D1 > 0 D2 > 0
4a2 + b2 > 0 4b2 – 4c2 > 0
C a2 > b 2 … (i)
CH3 CH3 b2 > c 2 … (ii)
CH3 From equ. (i) and we get (ii)
a2 b2 c2 85. (c) P(x) = e–x P(0) = 1
a2 c2 Differenting write 'x' we get
c2 a2 0 P'(x) = –e–x P(0) = –1
x2 + 2cx + a2 =0 x
P ''(x) e 0 x R
D= 4c2 – 4a2 < 0
Hence, no real roots 1
P(1)
83. (a) 4x2 + 9y2 – 8x –36y + 15 = 0 e
4(x2 – 2x) + 9(y2 – 4y) = –15
4(x2 – 2x + 1) + 9(y2 – 4y + 4) = –15 + 4 + 36
4(x – 1)2 + 9(y – 2)2 = 25

(x 1)2 (y 2)2
2 2
1 … (1)
5 5
2 3
x2 – 2xy + y2 – 4y + 5

25
min of ((x – 1)2 + (y – 2)2 =
9

25
and max of ((x – 1)2 + (y – 2)2 =
4
Sum of min. and max.

25 25 325
9 4 36

1 P(x) = –ex + 2
84. (c) sin x cos x sin 2 x
2 4
P'(x) ex
1 1 P '(0) 1
sin x cos x 1 cos 2x
2 2 2
P''(x) ex
cos 1 2sin
P(1) e 2 P(1) 0 = –0.7
1 1 89. (c) Ways to make the sum K is coefficient of xK
sin x cos x 1 sin 2x in (x + x2)10
2 2
Coefficient of xK in x10(1 + x)10
2sin x cos x 1 sin x cos x
Coefficient of xK – 10 in (1 + x)10
2sin x cos x 2sin x 1 cos x 0 Which is 10C
K–10
(1 cos x) 2sin x(1 cos x) 0 So ways to make sum minimum K is

(1 cos x)(1 2sin x) 0 10


CK 10
10
CK 9
10
CK 8
10
C10
1 cos x 0 or 1 2sin x 0 Probability
1 10 10 10
cos x 1 sin x CK 10 CK 9 C10
2 P(K) 10
2
5
x 0, x ,
6 6 210 10
C0 10
CK 11
P(K)
10
5 2
Hence sum of all x 0
6 6
10 10
97. (a) The given circuit can be simplified into two
C0 CK 11 1 wheatstone bridge in parallel
1
210 2
2 2
But K should be maximum so 98. (d) x x
L
10 10
CK 11 C5 (middle value) L 1.2 m
So that 10C +…+ 10C is max at x 0, x L, y 0
0 K–11
So K = 16 v 300
v 250 Hz
x 1 1.2
90. (b) f (x)
x 1
99. (b)
x 1
1
x 1 x 1
f 2 (x) f (f (x)) f x
x 1 x 1
1
x 1

x 1
f 3 (x) f (x)
x 1

f 4 (x) x

P f (2) f 3 (3) f 3 (4) f 4 (5) T + k1x – mg = ma1


k2x + mg – T = ma2
5
P 3 3 5 75 By constraint relation
3
a1 = a2
Multiple of P is 375
NaNH2
3 A 101.(b) Ph C CH Ph C CNa
A /2 liq. NH3
93. (a) B
A /2 3 A
CH3I H2 ,Pd/C
Ph C C CH3
B 3
B 7 Ph CH3
C=C
95. (a) H H
Cis-alkene

A
102.(b) 0.7
C

350000
Now, 0.7
C

C = 500000
103.(b) Racemisation takes place.

kq k( q) OH OH OH
VB V (Given)
b c NO2
dil. HNO3
104.(a) +
4 0 bc
q V
c b
NO2
Charge on C = –q Steam Steam Non-
Volatile Volatile
hc 6.6 10 34 3 108 Rate1 K 0.25
x
0.25
y
1 10 6
105.(c) E
660 10 9 Rate3 K 0.25
x
0.50
y
8 10 6

= 3 × 10–19 J
1 1
K.E. = 3 × 10–19 – 1.6 × 10–19 = 1.4 × 10–19 J Solving, ,y=3
2y 23
34
h 6.6 10 So x + y = 5
2mK.E. 2 9.1 10 31
1.4 10 19 113.(b) insectivorous plants use insects as a source
of nitrogen.
= 1.32 × 10–9 m Utricularia, Nepenthes and dionaea are a few
example of such plants.
114.(c) Red : Roan : White = 1 : 2 : 1
a 450
106.(b) d = 150 Pm
h 2
k 2 2
4 4 1
Represent co-dominance.
115.(b) Number of bases in Eco RI = 6
107.(b) H = 20 kJ/mol, Now E = H – ngRT
6
E = 20 – 8.314 × 10–3 × 333 1 1
will cut after 4096 bp.
4 4096
= 17.2 kJ/mA
10,000
Number of cleaving locations = = 2.44
20 4096
109.(d) w = –2.303 × 3 × 8.314 × 10–3 × 300 log
2 116.(b) Activation energy
117.(d) Presence of antibiotics in Ganga water.
= –17.2 kJ/mA 119.(b) DNA 5’
AGCATATGATCGTTTCTCTGCTTTGAACT3’
x y 6 mRNA 5’
Rate1 K 0.25 0.25 1 10
110.(d) x y 6
Rate 2 K 0.50 0.25 4 10
Stop codar

1 1 Only these two amino acids will be formed.


Solving, ,x=2
2x 22
7. In a rectangle ABCD, the coordinates of A and B
are (1, 2) and (3, 6) respectively and some diameter
1. Suppose logab + logba = c. The smallest possible of the circumscribing circle of ABCD has equation
integer value of c for all a, b > 1 is - 2x – y + 4 = 0. Then the area of the rectangle is -
(a) 4 (b) 3
(a) 16 (b) 2 10
(c) 2 (d) 1
2. Suppose n is a natural number such that (c) 2 5 (d) 20
8. In the xy-plane, three distinct lines l1, l2, l3 concur
i 2i2 3i3 nin 18 2 , where i is the at a point ( , 0). Further the lines l1, l2, l3 are
square root of – 1. Then n is - normals to the parabola y2 = 6x at the points
(a) 9 (b) 18 A = (x1, y1), B = (x2, y2), C = (x3, y3) respectively.
Then we have -
(c) 36 (d) 72
(a) <–5 (b) >3
3. Let P be an m × m matrix such that P2 = P. Then
(1 + P)n equals - (c) – 5 < < – 3 (d) 0 < < 3
9. Let f (x) = cos 5x + A cos 4x + B cos 3x + C cos 2x
(a) I + P (b) I + nP
x 2
(c) I + 2nP (d) I + (2n – 1)P + D cos x + E, and T = f (0) – f f
5 5
4. Consider the cubic equation x3 + ax2 + bx + c = 0,
where a, b, c are real numbers. Which of the 3 8 9
–f f f . Then T
following statements is correct? 5 5 5
(a) If a2 – 2b < 0, then the equation has one real (a) depends on A, B, C, D, E
and two imaginary roots (b) depends on A, C, E but independent of B and D
(b) If a2
– 2b 0, then the equation has all real (c) depends on B, D but independent of A, C, E
roots (d) is independent of A, B, C, D, E
(c) If a2 – 2b > 0, then the equation has all real 10. In triangle ABC, we are given that 3 sin A + 4 cos
and distinct roots B = 6 and 4 sin B + 3 cos A = 1. Then the measure
(d) If 4a3 – 27b2 > 0, then the equation has real of the angle C is -
and distinct roots (a) 30º (b) 150º
5. All the points (x, y) in the plane satisfying the (c) 60º (d) 75º
equation x2 + 2x sin (xy) + 1 = 0 lie on - 11. Which of the following intervals is a possible
(a) a pair of straight lines domain of the function f (x) = log{x}[x] + log[x]{x},
(b) a family of hyperbolas where [x] is the greatest integer not exceeding x
and {x} = x – [x]?
(c) a parabola
(a) (0, 1) (b) (1, 2)
(d) an ellipse
(c) (2, 3) (d) (3, 5)
6. Let A = (4, 0), B = (0, 12) be two points in the
plane. The locus of a point C such that the area 12. If f(x) = (2011 + x)n, where x is a real variable and
of triangle ABC is 18 sq. units is - n is a positive integer, then the value of

(a) (y + 3x + 12)2 = 81 f "(0) f (n 1) (0)


f (0) f '(0) is-
(b) (y + 3x + 81)2 = 12 2! (n 1)!
(c) (y + 3x – 12)2 = 81 (a) (2011)n (b) (2012)n
(d) (y + 3x – 81)2 = 12 (c) (2012)n – 1 (d) n(2011)n
13. The minimum distance between a point on the 19. The sum of 12 1 1 1! 22 2 1 2!
curve y = ex and a point on the curve y = logex is-
2
n n 1 n ! is -
1
(a) (b) 2 (a) (n + 2)! (b) (n – 1)((n + 1)!) + 1
2
(c) (n + 2)! – 1 (d) n((n + 1)!) – 1
(c) 3 (d) 2 2
20. Let X be a nonempty set and let P(X) denote the
14. Let f : (2, ) N be defined by f (x) = the largest collection of all subsets of X. Define
8
prime factor of [x]. Then f ( x)dx is equal to - f : X × P(X) R by
2 1, if x A
(a) 17 (b) 22 f ( x, A)
0, if x A
(c) 23 (d) 25 Then f (x, A B) equals -
15. Let [x] denote the largest integer not exceeding x (a) f (x, A) + f (x, B)
and {x} = x – [x].
(b) f (x, A) + f (x, B) – 1
2012 cos( x )
e (c) f (x, A) + f (x, B) – f (x, A) f (x, B)
Then dx is equal to-
cos x cos x (d) f (x, A) + | f (x, A) – f (x, B)|
0 e e
(a) 0 (b) 1006
(c) 2012 (d) 2012
21. A narrow but tall cabin is falling freely near the
16. The value of earth’s surface. Inside the cabin, two small stones
A and B are released from rest (relative to the
1 1 1
lim is - cabin). Initially A is much above the centre of
2 2 2
n
4n 1 4n 4 4n n2 mass and B much below the centre of mass of the
cabin. A close observation of the motion of A and
1
(a) (b) B will reveal that -
4 12
(a) both A and B continue to be exactly at rest
relative to the cabin
(c) (d)
4 6 (b) A moves slowly upward and B moves slowly
17. Two players play the following game : A writes 3, downward relative to the cabin
5, 6 on three different cards ; B writes 8, 9, 10 on (c) both A and B fall to the bottom of the cabin
three different cards. Both draw randomly two with constant acceleration due to gravity
cards from their collections. Then A computes (d) A and B move slightly towards each other
the product of two numbers he/she has drawn, vertically
and B computes the sum of two numbers he/she
22. Two plates each of the mass m are connected by
has drawn. The player getting the larger number
a massless spring as shown.
wins. What is the probability that A wins?
1 5
(a) (b)
3 9
4 1
(c) (d)
9 9
A weight W is put on the upper plate which
18. Let a, b, c , be three vectors in the xyz space such compresses the spring further. When W is
removed, the entire assembly jumps up. The
that a b b c c a 0 If A, B, C are points minimum weight W needed for the assembly to
with position vectors a, b, c respectively, then the jump up when the weight is removed is just more
number of possible positions of the centroid of than -
triangle ABC is - (a) mg
(a) 1 (b) 2 (b) 2 mg
(c) 3 (d) 6 (c) 3 mg
(d) 4 mg
23. If the speed (v) of the bob in a simple pendulum is (a) 12ºC (b) 50ºC
plotted against the tangential acceleration (a), the (c) 73ºC (d) 88ºC
correct graph will be represented by - 27. Jet aircrafts fly at altitudes above 30,000 ft where
the air is very cold at – 40ºC and the pressure is
0.28 atm. The cabin is maintained at 1 atm
pressure by means of a compressor which
exchanges air from outside adiabatically. In order
to have a comfortable cabin temperature of 25ºC,
we will require in addition-
(a) a heater to warm the air injected into the cabin
(b) an air-conditioner to cool the air injected into
the cabin
(c) neither a heater nor an air-conditioner ; the
compressor is sufficient
(d) alternatively heating and cooling in the two
halves of the compressor cycle
28. A speaker emits a sound wave of frequency f0.
When it moves towards a stationary observer with
(a) I (b) II
speed u, the observer measures a frequency f1. If
(c) III (d) IV the speaker is stationary, and the observer moves
24. A container with rigid walls is covered with perfectly towards it with speed u, the measured frequency
insulating material. The container is divided into is f2. Then -
two parts by a partition. One part contains a gas (a) f1 = f2 < f0 (b) f1 > f2
while the other is fully evacuated (vacuum). The
(c) f1 < f2 (d) f1 = f2 > f0
partition is suddenly removed. The gas rushes to
fill the entire volume and comes to equilibrium after 29. A plane polarized light passed through successive
a little time. If the gas is not ideal, polarizers which are rotated by 30º with respect
(a) the initial internal energy of the gas equals to each other in the clockwise direction.
its final internal energy Neglecting absorption by the polarizers and given
that the first polarizer’s axis is parallel to the plane
(b) the initial temperature of the gas equals its of polarization of the incident light, the intensity
final temperature of light at the exit of the fifth polarizer is closest
(c) the initial pressure of the gas equals its final to -
pressure
(a) same as that of the incident light
(d) the initial entropy of the gas equals its final
(b) 17.5% of the incident light
entropy
(c) 30% of the incident light
25. Two bulbs of identical volumes connected by a
small capillary are initially filled with an ideal (d) zero
gas at temperature T. Bulb 2 is heated to maintain 30. At 23ºC, a pipe open at both ends resonates at a
a temperature 2T while bulb 1 remains at frequency of 450 Hz. At what frequency does the
temperature T. Assume throughout that the heat same pipe resonate on a hot day when the speed
conduction by the capillary is negligible. Then the of sound is 4 percent higher than it would be at
ratio of final mass of the gas in bulb 2 to the initial 23ºC?
mass of the gas in the same bulb is close to - (a) 446 Hz (b) 454 Hz
1 2 (c) 468 Hz (d) 459 Hz
(a) (b)
2 3 31. In a Young’s double slit set-up, light from a laser
1 source falls on a pair of very narrow slits
(c) (d) 1
3 separated by 1.0 micrometer and bright fringes
26. Two rods, one made of copper and the other steel of separated by 1.0 millimeter are observed on a
the same length and cross sectional area are joined distant screen. If the frequency of the laser light
together. (The thermal conductivity of copper is is doubled, what will be the separation of the
385 J.s–1 .m–1.K–1 and steel is 50 J.s–1.m–1.K–1.) If bright fringes?
the copper end is held at 100ºC and the steel end is (a) 0.25 mm (b) 0.5 mm
held at 0ºC, what is the junction temperature (c) 1.0 mm (d) 2.0 mm
(assuming no other heat losses)?
32. For a domestic AC supply of 220 V at 50 cycles Which of the graphs shown below best represents
per second, the potential difference between the the voltage across the inductor, as seen on an
terminals of a two pin electric outlet in a room is oscilloscope?
given by -
(a) V(t) 220 2 cos 100 t

(b) V(t) 220 cos 50t

(c) V(t) 220 cos 100 t

(d) V(t) 220 2 cos 50t


33. In the circuit shown below the resistance are
given in ohms and the battery is assumed ideal
with emf equal to 3.0 volts. The resistor that
dissipates the most power is -

(a) I (b) II
(a) R1 (b) R2 (c) III (d) IV
(c) R3 (d) R4 36. Given below are three schematic graphs of
34. An electron collides with a free molecules initially potential energy V(r) versus distance r for three
in its ground state. The collision leaves the atomic particles: electron (e–), proton (p+) and
molecules in an excited state that is metastable neutron (n), in the presence of a nucleus at the
and does not decay to the ground state by origin O. The radius of the nucleus is r 0. The
radiation. Let K be the sum of the initial kinetic scale on the V-axis may not be the same for all
figures. The correct pairing of each graph with
energies of the electron and the molecule, and P the corresponding atomic particle is -
the sum of their initial momenta. Let K’ and P '
represent the same physical quantities after the
collision. Then -
(a) K K ', P P'

(b) K ' K, P P'


(c) K K ', P P'
(d) K ' K, P P'
35. In the circuit shown, the switch is closed at time
t = 0.

(a) (1, n), (2, p+), (3, e–)


(b) (1, p+), (2, e–), (3, n)
(c) (1, e–), (2, p+), (3, n)
(d) (1, p+), (2, n), (3, e–)
37. Due to transitions among its first three energy
levels, hydrogenic atom emits radiation at three
discrete wavelengths 1, 2, and 3 ( 1 < 2 < 3).
Then -
(a) 1= 2+ 3 (b) 1+ 2= 3

1 1 1 1 1 1
(c) (d)
1 2 3 1 2 3

38. The total radiative power emitted by spherical


blackbody with radius R and temperature T is P.
If the radius if doubled and the temperature is
halved then the radiative power will be -

P P
(a) (b)
4 2
(c) 2P (d) 4P
39. The Quantum Hall Resistance R H is a
fundamental constant with dimensions of
resistance. If h is Planck’s constant and e the
electron charge, then the dimension of RH is the
(a) I (b) II
same as -
(c) III (d) IV
2 h
e
(a) (b)
h e2
41. The hybridizations of Ni(CO)4 and Cr(H2O)62+,
h2 e respectively, are
(c) (d)
e h2 (a) sp3 and d3sp2 (b) dsp2 and d2sp3
40. Four students measure the height of a tower. (c) sp3 and d2sp3 (d) dsp2 and sp3d2
Each student uses a different method and each 42. Extraction of silver is achieved by initial
measures the height many different times. The complexation of the ore (Argentite) with X
data for each are plotted below. The measurement followed by reduction with Y. X and Y respectively
with highest precision is are
(a) CN– and Zn
(b) CN– and Cu
(c) Cl– and Zn
(d) Br– and Zn
43. Assuming ideal behaviour, the enthalpy and
volume of mixing of two liquids, respectively, are
(a) zero and zero
(b) + ve and zero
(c) –ve and zero
(d) – ve and – ve
44. At 298 K, the ratio of osmotic pressures of two
solutions of a substance with concentrations of
0.01 M and 0.001 M, respectively, is
(a) 1
(b) 100
(c) 10
(d) 1000
45. The rate of gas phase chemical reactions generally 50. The major product of the following reaction is :
increases rapidly with rise in temperature. This
is mainly because
(a) the collision frequency increases with
temperature
(b) the fraction of molecules having energy in
excess of the activation energy increases with
temperature
(c) the activation energy decreases with
temperature
(d) the average kinetic energy of molecules
(a) (b)
increases with temperature
46. Among i-iv

(c) (d)
(i) (ii)

51. For the transformation


(iii) (iv)
the compound that does not undergo
polymerization under radical initiation, is
(a) i (b) ii
(c) iii (d) iv
47. Two possible stereoisomers for the reagent used is
(a) LIAlH4 (b) H3PO2
(c) H3O+ (d) H2/Pt
52. The value of the limiting molar conductivity ( º)
for NaCl, HCl and NaOAc are 126.4, 425.9 and
are 91.0 S cm2 mol–1, respectively. For HOAc, º in
(a) enantiomers (b) diastereomers S cm2 mol–1 is
(c) conformers (d) rotamers (a) 390.5 (b) 299.5
48. For a process to occur spontaneously (c) 208.5 (d) 217.4
(a) only the entropy of the system must increase 53. To obtain a diffraction peak, for a crystalline solid
with interplane distance equal to the wavelength
(b) only the entropy of the suroundings must
of incident X-ray radiation, the angle of incidence
increase
should be
(c) either the entropy of the system or that of
(a) 90º (b) 0º
the surroundings must increase
(c) 30º (d) 60º
(d) the total entropy of the system and the
surroundings must increase 54. The standard Gibbs free energy change ( Gº in
49. When the size of a spherical nanoparticle decreases kJ mol–1), in a Daniel cell Ecell 1.1V , when 2
from 30 nm to 10nm, the ratio surface area/volume
moles of Zn(s) is oxidized at 298 K, is closest to
becomes
(a) – 212.3
(a) 1/3 of the original
(b) – 106.2
(b) 3 times the original
(c) – 424.6
(c) 1/9 of the original
(d) – 53.1
(d) 9 times the original
55. All the products formed in the oxidation of NaBH4
by I2, are
61. The major constituents of neurofilaments are-
(a) B2H6 and NaI (b) B2H6, H2 and NaI
(a) microtubules
(c) BI3 and NaH (d) NaBI4 and HI
(b) intermediate filaments
56. The spin-only magnetic moments of [Mn(CN)6]4–
and [MnBr4]2– in Bohr Magnetons, respectively, (c) actin filaments
are (d) protofilaments
(a) 5.92 and 5.92 (b) 4.89 and 1.73 62. In which phase of the cell cycle are sister
(c) 1.73 and 5.92 (d) 1.73 and 1.73 chromatids available as template for repair ?
57. In a zero-order reaction, if the initial (a) G1 phase (b) G2 phase
concentration of the reactant is doubled, the time (c) S phase (d) M phase
required for half the reactant to be consumed 63. A person has difficulty in breathing at higher
(a) increases two-fold (b) increases four-fold altitudes because-
(c) decreases by half (d) does not change (a) oxygen is likely to diffuse from lungs to blood
58. The adsorption isotherm for a gas is given by the (b) oxygen is likely to diffuse from blood to lungs
(c) partial pressure of O2 is lower than partial
ap
relation x = where x is moles of gas pressure of CO2
(1 bp)
(d) overall intake of O2 by the blood becomes low
adsorbed per gram of the adsorbent, p is the
64. In humans, the composition of a zygote that will
pressure of the gas, and a and b are constants.
develop into a female is-
Then x
(a) 44 A + XX
(a) increases with p
(b) 44 A + XY
(b) remains unchanged with p
(c) 22 + X
(c) decreases with p
(d) 23 A
(d) increases with p at low pressures and then
remains the same at high pressure 65. If you fractionate all the organelles from the
cytoplasm of a plant cell, in which one of the
59. The reaction following sets of fractions will you find nucleic
acids?
(a) nucleus, mitochondria, chloroplast, cytoplasm
(b) nucleus, mitochondria, chloroplast, glyoxysome
(c) nucleus, chloroplast, cytoplasm and peroxisome
is known as (d) nucleus, mitochondria, chloroplast, Golgi
(a) Perkin reaction bodies
(b) Sandmeyer reaction 66. A protein with 100 amino acid residues has been
(c) Reimer-Tiemann reaction translated based on triplet genetic code. Had the
genetic code been quadruplet, the gene that codes
(d) Cannizzaro reaction for the protein would have been-
60. Among i-iii (a) same in size
(b) longer in size by 25%
(c) longer in size by 100%
(d) shorter in size
67. If the sequence of bases in DNA is 5’-
ATGTATCTCAAT-3’, then the sequence of bases
in its transcript will be-
(a) 5’-TACATAGAGTTA-3’
the boiling point follows the order (b) 5’-UACAUAGAGUUA-3’
(a) ii < i < iii (b) iii < ii < i (c) 5’-AUGUAUCUCAAU-3’
(c) i < ii < iii (d) ii < iii < i (d) 5’-AUUGAGAUACAU-3’
68. The Na + /K + pump is present in the plasma 75. Soon after the three germ layers are formed
membrane of mammalian cells where it- in a developing embryo, the process of
(a) expels potassium from the cell organogenesis starts. The human brain is
formed from the-
(b) expels sodium and potassium from the cell
(a) ectoderm
(c) pumps sodium into the cell
(b) endoderm
(d) expels sodium from the cell
(c) mesoderm
69. The CO2 in the blood is mostly carried-
(d) partly endoderm and partly mesoderm
(a) by hemoglobin in RBCs
76. Puffs in the polytene chromosomes of Drosophila
(b) in the cytoplasm of WBCs
melanogaster salivary glands represent-
(c) in the plasma as bicarbonate ions
(a) transcriptionally active genes
(d) by plasma proteins
(b) transcriptionally inactive genes
70. Patients who have undergone organ transplants
(c) heterochromatin
are given anti-rejection medications to-
(d) housekeeping genes
(a) minimize infection
77. The process of cell death involving DNA cleavage
(b) stimulate B-macrophage cell interaction
in cells is known as-
(c) prevent T-lymphocyte proliferation
(a) necrosis
(d) adopt the HLA of donor
(b) apoptosis
71. Saline drip is given to a Cholera patient because-
(c) cytokinesis
(a) NaCl kills Vibrio cholera
(d) endocytosis
(b) NaCl generates ATP
78. According to the original model of DNA, as
(c) Na + ions stops nerve impulse and hence proposed by Watson & Crick in 1953, DNA is a-
sensation of pain
(a) left handed helix
(d) Na+ ions help in retention of water in body
(b) helix that makes a full turn every 70 nm
tissue
(c) helix where one turn of DNA contains 20
72. A water molecule can from a maximum of ……….
basepairs
hydrogen bonds.
(d) two stranded helix where each strand has
(a) 1 (b) 2
opposite polarity
(c) 3 (d) 4
79. At which stage of Meiosis I does crossing over
73. Circadian Rhythm is an endogenously driven cycle occur ?
for biochemical, physiological and behavioral
(a) lepoptene
processes. In humans, the approximate duration
of this ‘biological clock’ is- (b) zygotene
(a) 1 Hour (c) pachytene
(b) 6 Hours (d) diplotene
(c) 12 Hours 80. An electrode is placed in the axioplasm of a
mammalian axon and another electrode is placed
(d) 24 Hours
just outside the axon. The potential difference
74. Modern evolutionary theory consists of the measured will be-
concepts of Darwin modified by knowledge
(a) 0
concerning-
(b) –70 mV
(a) population statistics
(c) –70 µV
(b) Mendel’s laws
(d) +70 µV
(c) the idea of the survival of the fittest
(d) competition
87. Let f(x) = x12 – x9 + x4 – x + 1. Which of the
following is true?
81. Let A and B be any two n × n matrices such that (a) f is one-one
the following conditions hold : AB = BA and there
exist positive integers k and such that Ak = I (b) f has a real root
(the identity matrix) and B = 0 (the zero matrix). (c) f never vanishes
Then- (d) f takes only positive values
(a) A + B = I 88. For each positive integer n, define f n (x) =
(b) det (AB) = 0 x n (1 x)n
minimum , , for 0 x 1 . Let
(c) det (A + B) 0 n! n!
1
(d) (A + B)m = 0 for some integer m
In fn ( x)dx, n 1 Then In In is equal to-
82. The minimum value of n for which 0 n 1
22 42 62 (2n)2 (a) 2 e (b) 2 e
1.01 3 2
12 32 52 (2n 1)2
(c) 2 e 1 (d) 2 e
(a) is 101 (b) is 121
89. The maximum possible value of x 2 + y 2
(c) is 151 (d) does not exist – 4x – 6y, x, y real, subject to the condition
83. The locus of the point P = (a, b) where a, b are
x y x y 4
real numbers such that the roots of x3 + ax2 + bx
+ a = 0 are in arithmetic progression is- (a) is 12 (b) is 28
(a) an ellipse (c) is 72 (d) does not exist
(b) a circle 90. The arithmetic mean and the geometric mean of
(c) a parabola whose vertex in on the y-axis two distinct 2-digit numbers x and y are two
integers one of which can be obtained by reversing
(d) a parabola whose vertex is on the x-axis
the digits of the other (in base 10 representation).
84. The smallest possible positive slope of a line whose Then x + y equals-
y-intercept is 5 and which has a common point
(a) 82 (b) 116
with the ellipse 9x2 + 16y2 = 144 is-
(c) 130 (d) 148
3
(a) (b) 1
4
4 9
(c) (d) 91. An isolated sphere of radius R contains uniform
3 16 volume distribution of positive charge. Which of
85. Let A = { R | cos2(sin ) + sin2(cos ) = 1} and the curves shown below correctly illustrates the
B={ R | cos(sin ) sin(cos ) = 0}. Then A B dependence of the magnitude of the electric field
(a) is the empty set of the sphere as a function of the distance r from
(b) has exactly one element its centre?
(c) has more than one but finitely many elements
(d) has infinitely many elements
86. Let f(x) = x3 + ax2 + bx + c, where a, b, c are real (I) (II)
numbers. If f(x) has a local minimum at x = 1 and
1
a local maximum at x = – and f(2) = 0, then
3
1
f ( x)dx equals-
(III) (IV)
1
14 14
(a) (b)
3 3
(a) I (b) II
7 7
(c) (d) (c) III (d) IV
3 3
92. The surface of a planet is found to be uniformly 96. A material is embedded between two glass
charged. When a particle of mass m and no charge plates. Refractive index n of the material
is thrown at an angle from the surface of the varies with thickness as shown below. The
planet, it has a parabolic trajectory as in projectile maximum incident angle (in degrees) on the
motion with horizontal range L. A particle of mass material for which beam will pass through the
m and charge q, with the same initial conditions material is-
has a range L/2. The range of particle of mass m
and charge 2q with the same initial conditions is-
(a) L (b) L/2
(c) L/3 (d) L/4
93. Figure below shows a small mass connected to a
string, which is attached to a vertical post. If the
ball is released when the string is horizontal as
shown, the magnitude of the total acceleration
(including radial and tangential) of the mass as a
(a) 60.0 (b) 53.1
function of the angle is-
(c) 43.5 (d) 32.3
97. At a distance from a uniformly charged long
wire, a charged particle is thrown radially
outward with a velocity u in the direction
perpendicular to the wire. When the particle
reaches a distance 2 from the wire its speed is
found to be 2 u. The magnitude of the velocity,
when it is a distance 4 away from the wire, is
(ignore gravity)
(a) g sin (b) g 3cos2 q +1 (a) 3u (b) 2u
(c) g cos (d) g 3sin q +1
2
(c) 2 2 u (d) 4u
94. One mole of an ideal gas at initial temperature 98. A rectangular loop of wire shown below is coplanar
T, undergoes a quasi-static process during which with a long wire carrying current I.
the volume V is doubled. During the process the
internal energy U obeys the equation U = aV3,
where a is a constant. The work done during this
process is-
(a) 3RT / 2 (b) 5RT / 2
(c) 5RT / 3 (d) 7RT / 3
95. A constant amount of an ideal gas undergoes the
cyclic process ABCA in the PV diagram shown
below The loop is pulled to the right as indicated. What
are the directions of the induced current in the
loop and the magnetic forces on the left and the
right sides of the loop?
Induced Force on Force on
current left side right side
(a) Counter To the left To the right
clockwise
The path BC is an isothermal. The work done by (b) Clockwise To the left To the right
the gas during one complete cycle, beginning and (c) Counter To the right To the left
ending at A, is nearly- clockwise
(a) 600 kJ (b) 300 kJ (d) Clockwise To the right To the left
(c) –300 kJ (d) –600 kJ
99. Two batteries V1 and V2 are connected to three 105. In the following conversion,
resistors as shown below. CN
(i) MeMgBr NaOH/I 2
+ X + Y
(ii) H3 O H3 O

the major products X and Y, respectively, are-

If V1 = 2V and V2 = 0 V, the current I = 3 mA. If


(i) and
V1 = 0 V and V2 = 4V, the current I = 4mA. Now,
if V 1 = 10 V and V 2 = 10 V, the current I
will be-
(a) 7 mA (b) 15 mA
(ii) and
(c) 20 mA (d) 25 mA
100. A particle moves in a plane along an elliptic path

x2 y2
given by 1 . At point (0, b), the (iii) and
a2 b2
x-component of velocity is u. The y-component of
acceleration at this point is-
(a) –bu2 / a2 (b) – u2 / b
(iv) and
(c) – au2 / b2 (d) – u2 / a

(a) i (b) ii (c) iii (d) iv

101. XeF6 hydrolyses to give an oxide. The structure 106. In the reaction sequence,
of XeF6 and the oxide, respectively, are-
(a) octahedral and tetrahedral
(b) distorted octahedral and pyramidal
(c) octahedral and pyramidal
the major products X and Y, respectively, are-
(d) distorted octahedral and tetrahedral
102. MnO4– oxidizes (i) oxalate ion in acidic medium
at 333 K and (ii) HCl. For balanced chemical
equations, the ratios [MnO4– : C2O42–] in (i) and (i)
[MnO4– : HCl] in (ii), respectively, are-
(a) 1 : 5 and 2 : 5 (b) 2 : 5 and 1 : 8
(c) 2 : 5 and 1 : 5 (d) 5 : 2 and 1 : 8

103. If E 0.440 V and


Fe2 / Fe

E 0.770 V , then EFe3 is-


Fe3 / Fe2 / Fe
(a) 0.330 V (b) –0.037 V
(c) –0.330 V (d) –1.210 V
(ii)
104. The electron in hydrogen atom is in the first Bohr
orbit (n = 1). The ratio of transition energies,
E(n = 1 n = 3) to E(n = 1 n = 2), is-
32 16 (iii)
(a) (b)
27 27
32 8
(c) (d)
9 9
111. Which sequence of events gives rise to flaccid
(iv) guard cells and stomatal closure at night ?
(a) low [Glucose] low osmotic pressure low
(a) i (b) ii pH high pCO2
(c) iii (d) iv (b) low pH high pCO2 low [Glucose] low
107. Optically active (S)- -methoxyacetaldehyde on osmotic pressure
reaction with MeMgX gave a mixture of alcohols. (c) low osmotic pressure high pCO2 low pH
The major diastereomer ‘P’ on treatment with low [Glucose]
MeI/K2CO3 gave an optically inactive compound. (d) high pCO2 low pH low [Glucose] low
P is- osmotic pressure
112. Rice has a diploid genome with 2n = 24. If crossing-
over is stopped in a rice plant and then selfed
seeds are collected, will all the offsprings be
(i) (ii)
genetically identical to the parent plant ?
(a) yes, because crossing-over is the only source
of genetic variation
(b) no, because stopping of crossing-over
(iii) (iv) automatically increases rate of point mutation
(c) yes, only if the parent plant was a completely
inbred line
(a) i (b) ii
(d) yes, only if the parent plant was a hybrid
(c) iii (d) iv
between two pure-bred lines
108. At 300 K the vapour pressure of two pure liquids,
113. Rodents can distinguish between many different
A and B are 100 and 500 mm Hg, respectively. If
types of odours. The basis for odour discrimination
in a mixture of A and B, the vapour pressure is
is that-
300 mm Hg, the mole fractions of A in the liquid
and in the vapour phase, respectively, are- (a) they have a small number of odorant receptors
that bind to many different odorant molecules
(a) 1/2 and 1/10
(b) the mechanoreceptors in the nasal cavity are
(b) 1/4 and 1/6
activated by different odorant molecules found
(c) 1/4 and 1/10 in the air passing through the nostrils
(d) 1/2 and 1/6 (c) the part of the brain that processes the sense
109. The crystal field stabilization energies (CFSE) of of smell has many different receptors for
high spin and low spin d6 metal complexes in odorant molecules
terms of 0, respectively, are- (d) a large number of different chemoreceptors
(a) –0.4 and –2.4 are present in the nasal cavity that binds a
(b) –2.4 and –0.4 variety of odorant molecules
(c) –0.4 and 0.0 114. Although blood flows through large arteries at
high pressure, when the blood reaches small
(d) –2.4 and 0.0
capillaries the pressure decreases because-
110. Emulsification of 10 ml of oil in water produces
(a) the valves in the arteries regulate the rate of
2.4 × 1018 droplets. If the surface tension at the
blood flow into the capillaries
oil-water interface is 0.03 Jm–2 and the area of
each droplet is 12.5 × 10–16 m2, the energy spent (b) the volume of blood in the capillaries is much
in the formation of oil droplets is- lesser than that in the arteries
(a) 90 J (c) the total cross-sectional area of capillaries
arising from an artery is much greater than
(b) 30 J
that of the artery
(c) 900 J
(d) elastin fibers in the capillaries help to reduce
(d) 10 J the arterial pressure
115. E.coli about to replicate was pulsed with tritiated 118. For a human male what is the probability that all
thymidine for 5 min and then transferred to the maternal chromosomes will end up in the
normal medium. After one cell division which one same gamete?
of the following observations would be correct? (a) 1/23 (b) 223
(a) both the strands of DNA will be radioactive (c) 246 (d) (1/2)23
(b) one strand of DNA will be radioactive 119. Nocturnal animals have retinas that contain-
(c) none of the strands will be radioactive (a) a high percentage of rods to increase
(d) half of one strand of DNA will be radioactive sensitivity to low light conditions
116. Selection of lysine auxotroph (bacteria which (b) a high percentage of cones so that nocturnal
requires lysine for growth) from a mixed color vision can be improved in low light
population of bacteria can be done by growing the conditions
bacterial population in the presence of- (c) an equal number of rods and cones so that
(a) lysine vision can be optimized
(b) penicillin (d) retinas with the photoreceptor layer present
(c) lysine and penicillin in the front of the eye to increase light
sensitivity
(d) glucose
120. The length of one complete turn of a DNA double
117. Increasing the number of measurements of an
helix is-
experimental variable will-
(a) 34 Å
(a) increase the standard error of the sample
(b) 34 nm
(b) increase the mean of the sample
(c) 3.4 Å
(c) decrease the standard error of the sample
(d) 3.4 µm
(d) result in all of the above

1. (c) 2. (c) 3. (d) 4. (a) 5. (a) 6. (c) 7. (a) 8. (b) 9. (c) 10. (a)
11. (c) 12. (c) 13. (b) 14. (b) 15. (b) 16. (d) 17. (c) 18. (a) 19. (b) 20. (c)
21. (b) 22. (b) 23. (a) 24. (a) 25. (b) 26. (d) 27. (b) 28. (b) 29. (c) 30. (c)
31. (b) 32. (a) 33. (a) 34. (b) 35. (d) 36. (a) 37. (d) 38. (a) 39. (b) 40. (a)
41. (c) 42. (a) 43. (a) 44. (c) 45. (b) 46. (d) 47. (a) 48. (d) 49. (b) 50. (c)
51. (b) 52. (a) 53. (c) 54. (c) 55. (b) 56. (c) 57. (a) 58. (d) 59. (c) 60. (c)
61. (d) 62. (d) 63. (c) 64. (a) 65. (a) 66. (b) 67. (d) 68. (d) 69. (c) 70. (c)
71. (d) 72. (d) 73. (d) 74. (c) 75. (a) 76. (a) 77. (b) 78. (d) 79. (c) 80. (b)
81. (b) 82. (c) 83. (c) 84. (b) 85. (a) 86. (b) 87. (d) 88. (a) 89. (b) 90. (c)
91. (b) 92. (c) 93. (d) 94. (d) 95. (c) 96. (b) 97. (a) 98. (b) 99. (d) 100. (a)
101. (b) 102. (b) 103. (b) 104. (a) 105. (c) 106. (c) 107. (*) 108. (d) 109. (a) 110. (a)
111. (d) 112. (c) 113. (d) 114. (c) 115. (b) 116. (d) 117. (c) 118. (d) 119. (a) 120. (a)
5. (a) x2 + 2 x sin (xy) + 1 = 0

2. (c) S i 2i2 3i 2 1
ni n 2 sin (xy) = – x
x
Multiplying both the side by (i)
R.H.S. 2 or 2
iS i2 2i3 (n 1)i n nin 1

S(1 i) i i2 i3 in nin 1 L.H.S. R.H.S. 2 sin(xy) 1


(x 1)
i(1 i n )
S(1 i) ni n 1 x 1
1 i
sin( y) 1 sin y 1
n n 1 sin y 1
1 i ni
S
2i 1 i
y 2n , n I
z1 (let) z2 (let) 2
Hence pair of straight lines.
1
z1 or 0 6. (c)
2
n n
z2 2
2 2
n
18 n 36
2
3. (d) P2 = P
Now multiplying P–1 on both the side,
P–1 P2 = P–1 P
1 x y
P=I 1
1 0 12 18
(I + P)n = (2P)n = 2n Pn 2
1 4 0
= 2nP
1(– 48) –x(–12) + y(4) = ±36
= P + (2n – 1) P
12x + 4y – 48 = ± 36
= I + (2n – 1) P
3x + y – 12 = ± 9
4. (a) f(x) = x3 + ax2 + bx + c
(3x + y – 12)2 = 81
Now different w.r.t ‘x’ we get
7. (a)
f '(x) 3x 2 2ax b

Now D b 2 4ac 4a 2 4 3 b 4 a2 3b

Now, if a2 2b

a3 3b
D 0
f '(x) 0 has non real roots

4
Slope of AB is =2
2
1
Hence f(x) = 0, has 1 real and two imaginary and slope of BC = –
2
roots
(AB) = (6 2)2 (3 1)2 13. (b)

= 4 16 2 5
Now distance between 2x – y + 4 = 0 and
2x – y = 0
4
5
8
Area = 2 5 = 16
5
8. (b) Any normal
IInd curve
3
y = mx – 2am – am3 Here a = 1
2 y'
x
through ( , 0)
x=1 point (1, 0)
0 = m – 2am – am3
similarly Ist point (0, 1)
m = 0, = 2a + am2
distance = 2
m2 = –2>0 8
a f ( x)dx 2 3 2 5 3 7 22
14. (b)
> 2a >3 2
9. (c) Clearly f( + x) + f( – x) (every term contain 1
ecos x
cosine) 15. (b) I = 2012 dx
0 ecos x
e cos x
9 2 8
f f , f f , 1 cos x
5 5 5 5 e
I = 2012 dx
cos x
3 7 0e ecos x
f f
5 5 2I 2012 I 1006
3 n
1 1 n 1
T f (0) 2 f f 16. (d) lim lim
5 5 nr 1
n r 1 4 n2 r2 n
r
2

2 4 4
2 f f f( ) n
5 5 1 1
dx 1 x
f(0) – f( ) = 2(1 + B + D) sin
4 x2 0 0 6 2
3 5
f f f f 17. (c) For A to win, A can draw either 3, 6 or 5, 6. If
5 5 5 5
A draws 3, 6 then B can draw only 8 & 9
3
1 B cos D cos 1 1 1
5 5 Prob. =
3 3 9
2 4 2 3 If A draws 5, 6 then B can draw, any two
f f f f
5 5 5 5 1 1
Probability = 1
6 2 3 3
1 B cos D cos
5 5 1 1 4
Probability =
T contains only B, D terms 9 3 9
11. (c) x I & [x] > 1 18. (a) a b c b 0
x (2, 3) only option satisfy. similarly b c 2a
12. (c) (2011) n n n 1 n n 2
C1 (2011) C2 (2011) a c 1b b c 3c
n n
Cn 1 2011 Cn 1 Hence a b c 0
= (2011 + 1)n – 1
only 1 position of centroid
19. (b) Tn = (n2 – n + 1)n! kh2 3m 2 g 2
= (n2 – 1)n! – (n – 2)n! mgh 0
2 2k
Tn = (n – 1)(n + 1)! – (n – 2)n!
mg m2g 2 3m 2 g 2
Sum = 1 + (n – 1)(n + 1)! h
k
1, if x A B
20. (c) f ( x, A B) mg 2mg 3mg mg
0, if x A B ,
k k k
if x A, x B 3mg
h
if x A, x B f x, A B 1 k
if x A, x B W + mg = kh
W + mg = 3mg
None of the option (A, B, D) satisfy
W = 2mg
if x A, x B f x, A B 0
23. (a) In SHM
c (only C satisfy) a = – 2x
21. (b)
v A2 x2

v2 2
A2 x2

a2
v2 2
A2 2
2

aB > aC > aA
a2
aB = g v2 2
2
A2

a A /CM aA aCM
v2 a2
a B/CM aB aCM 2
1
A2 4
A2
22. (b) For lower block +ve lift, kx mg
i.e. ellipse
mg
x 24. (a)
k

expansion is against vacuum


W=0
Insulated container
Q=0
First law of thermodynamics
Q= W+ U
0=0+ U
0=0+ U
U=0
W/E theorem 25. (b) Mole conservation
1 2 1 2 n1 + n 2 = n
mg h x kh kx 0 0
2 2
m2g 2 1 2 1 m2 g 2
mgh kh 0
k 2 2 k
n 233
Initial no. of moles = n1 = n2 = T
2 2
0.28 7
finally when temp of 1 vessel is T & another
is 2T T is coming
more than 298 K or 25ºC
PV
n1 T is more than 25°C
RT
so to cool it an extra ac is required.
PV n1 2
n2 28. (b)
R2T n2 1
n1 n2 n f0 v
f1
v u
2n n
n1 ; n2
3 3
n1
v u
mass of gas f2 f0
v
n
M2 2 u v v
3 f2 f1 f0
M1 n 3 v v u
2 u 2 f0
f2 f1 ve
100 T T 0 (v)(v u)
26. (d)
R1 R2
f1 f2
100 T R1 29. (c) I = I0 (cos2 )4
T R2 4
3
L = I0 × = 30% of I0
R 4
KA
30. (c) f = v
R1 k2
f v
R2 k1
f1 v1
100 T 50 10
f2 v2
T 385 77
7700 – 77T = 10T 450 v1
7700 = 87T f2 1.04v1

7700 f 2 1.04 450 = 468Hz.


T 88 C
87 31. (b) Separation
D
Bright fringe =
d
27. (b) PV = C f =c
P1– T = C If f is doubled
(0.28)1– × (233) = 11– × T become halved
become half
7
= 1
5 = = 0.5 mm
2
1 7 7 1 7 7
0.28 5 233 5 1 5 T5 32. (a) R.M.S. value = 220 V
7 7 2 Peak value = 220 2
T5 233 5 0.28 5
= 2 n = 2 × 50 = 100
2
T 233 0.28 7
V(t) 220 2 cos 100 t
33. (a) 38. (a) P = AT4
= × 4 R2 × T4
4
T
P’ = × 4 (2R)2 ×
2
1
Power dissipate in R 1 is maximum as its P’ = × 4 R2 T4 × 4 ×
16
current is maximum and its resistance is also 1
40 which is higher than R5R4. P'
4
34. (b) Collision of e– lead to excitation of molecules V V i P
so Collision is inelastic 39. (b) R 2
i i i2
K ' K and loss of kinetic energy go for h
excitation of molecules. Momentum remain energy = h =
t
conserved during collision.
energy
Power =
P P' t
R h
V 1 P
35. (d) e e L
t2
R
e
i
t
P h
i2 e 2
36. (a) 40. (a) Precession mean
every time reading is coming nearly same.
41. (c) Both CO and H2O being strong field ligands
pair-up unpaired electrons so hybridisation
are sp3 and d2sp3 respectively.
outside the nucleus electric potential
decreases 42. (a) Ag 2S CN Ag CN 2
(X)
e– is negativity charged
Zn(Y)
its PE is negative even outside the nucleus 2
where nuclear attractive force is negligible Zn CN 4 Ag
(3) e– 43. (a) For ideal solution, both H mixing 0 and
outside the nucleus Vmixing 0.
neutron will not 1 C1 0.01
44. (c) 10
experience electric force 2 C2 0.001
as it is neutral. So no potential energy 45. (b) With increase of temp., more molecules attain
associated with activation energy.
it outside nucleus. 46. (d) Radical formed will not be stable on Carbon
1 neutron adjacent to Oxygen.
37. (d) 47. (a) As the given molecule have a chiral carbon so
enantiomers exists.
48. (d) It is second law of thermodynamics.
Surface Area d2 6
49. (b) 3
Volume d /6 d
hc hc hc Now d1 = 30 nm and d2 = 10 nm
1 2 3 6
1 1 1 So, their ratio is = 10 3
6
1 2 3
30
O (d)
OH CH3
CH3
C – CH3 + (a)
H
50. (c) CH3 (a)
OH Ring
Expansion
Major Product
(b)
51. (b) H3PO4 is the best reagent for diazonium Salt
to remove the group (diazonium salt) (d)
(c)
Br Br (c)
H3PO2/H2O (a)
+ H3PO3
+ –
N2Cl

52. (a) CH3 COOH CH3COONa HCl


m m m
(b)
NaCl (b)
m

= 91 + 425.9 – 126.4 = 390.5


53. (c) n 81. (b) Ak = I, B = 0 (det (B) = 0)
2d sin
det (AB) = 0
2 sin x
82. (c) 1.01
1 2n(2n 1)(4 n 1)
OR sin 30 x
2 6
2n(2n 1)(4 n 1)
2.01 x (1.01)
54. (c) G nFE0cell 6

4 96500 1.1 4 n(n 1)(2n 1) 2n(2n 1)(4 n 1)


424.6 kJ 2.01 (1.01)
6 6
1000
2.01 4 n 1
n 150.5
55. (b) NaBH4 I2 NaI B2 H6 H2 1.01 2n 2
4
83. (c) Let roots be d, , + d product
56. (c) Mn CN 6 oxi. No. of Mn = +2 a
Sum 3 = – a =– 2
d2 a
Mn +2 = 3d 5 4s 0 , As CN – pair-unpaired 3
electrons, pair product b = 2 – d + 2 + d + 2 – d2

so, 1(1 2) 3 1.732 B.M. 2– d2 = 3


b=2 2+ 3
2
and for MnBr4 , oxi. No. of Mn = +2 2 2 9
b–3= a locus x2 = ( y 3) parabola
9 2
and no. of unpaired electrons = 5
x2 y2
so, 5(5 2) 35 5.92 B.M. 84. (b) ellipse 1
16 9
a x cos y sin
57. (a) t1/ 2 Any tangent 1
2K 4 3
3
2a y intercept = 5 sin = ; ,
If a = 2a, t1/2 = or doubled. 5 2
2K 4
cos
60. (c) (iii) > (ii) > (i) 5
Because (iii) has intermolecular Hydrogen x y
tangent – 1 slope = 1
Bond (ii) has intramolecular Hydrogen Bond 5 5
85. (a) for A B | x + y | + | x – y | = 4 represent a square
cos sin 1 or –1 & sin(cos ) 0 x2 + y2 – 4x – 6y = (x – 2)2 + (y – 3)2 – 13

which is not possible = (distance point on square from (2, 3))2 – 13


Maximum = (– 2 – 2)2 + (– 2 – 3)2 – 13 = 28
or cos sin 0 & sin(cos ) 1 or –1
x y
also not possible 90. (c) 10 a b, xy 10 b a a, b N
2
so A B is an empty set
2
86. (b) Differentiating f(x) w.r.t ‘x’ we get xy 10 b a

2 1 2
Now x y 4(11a 11b)(9 a 9 b)
f '( x) 3 x2 x 3 x2 2x 1
3 3
4 11 (a b) 9(a b)
f ( x) x3 x2 x a b 11, a b 1

Now f (2) 8 4 2 0 2 a 6, b 5
((x – y)2 is perfect square of an integer)
f ( x) x3 x2 x 2 x + y = 130

1 1 r
2 91. (b) When r < R E =
f ( x)dx 2 x 2 dx 3 0
1 0
Q
When r > R E =
1 14 4 0 r2
2 2
3 3

87. (d) f ( x) x9 x 3 1 x x3 1 1 positive for


x 1 or x 0
= 1 – x + x4 – x9 + x12 positive for x (0, 1)
f(x) is always positive
1/ 2 1 92. (c) For uncharged particle
xn (1 x)n
88. (a) In dx dx
n! n!
0 1/ 2 u2 sin 2
L … (i)
n g
1
n 1 n 1
1 1 1 2 Range for particle of mass m and charge q.
(n 1)! 2 2 (n 1)!
Lu2 sin 2
1/2 (1 / 2) 2 … (ii)
In 2 e 3 2 qE
g
n 1 2! 3! m
From (i) and (ii)
89. (b)
u 2 sin 2 u 2 sin 2
2g qE
g
m
mg qE
Range of particle of mass m & charge 2q.

u 2 sin 2 u 2 sin 2 L
R
2qE 2qE 3
g g 1
m mg
95. (c)
93. (d)

WC A 0
Process BC T = Constant
Radical acceleration PCVC = PB VB
500 × 2 = 200 × VB
V2 2g sin
= 2g sin VB = 5m3
R
WA B= 200 [VB – VA]
tangential acceleration = g cos
= 200[5 – 2] = 600
total acceleration
WB C > WA B
2 2 2 2 2 Net work done is –ve
4g sin g cos g 1 3sin
WB C< 1200 KJ
94. (d) U = aV3
Total W < –600 KJ
fnRT 96. (b) 1.5 × sini = 1.2 sinr
aV 3
2 1.5
sinr = sin i
PV = RT 1.2
T/R should not take place
PV
aV 3 sinr < 1
R
1.5
sin i 1
P CV 2 1.2
12
2V 2V sin i
W PdV CV 2 dV 15
V V
sini < 0.8
1
sin 45 0.707
C 7V 3 C 2
8V 3 V3
3 3 i max 45

fRT 97. (a)


aV 3
2
PV = RT

f PV
aV 2
2

2a 2 energy conservation at A & B


P V
f 1 1
qVA mu 2 qVB m 2u 2
2 2
2a
C 1
f q VA VB mu 2
2
7 2a 7 7RT
W V3 fnRT 1
3 f 3f 3 q ln 2 mu 2
2 0 2
energy conservation at A & C
Veq
1 1 I
qVA mu 2 qVC mv 2 R R eq
2 2
In each case Req & R is same only V1 & V2 is
1 1
q VA VC mu 2 mv 2 changing
2 2
Veq is changing
q 1 1
ln 4 mu 2 mv 2 2 R2 0 R1
2 0 2 2 Veq V1 2, V2 0
R1 R2
2q 1 1
ln 2 mu 2 mv 2
2 0 2 2 2R 2
Veq
1 1 R1 R2
mu 2 mu 2 mv 2
2 2 Case-2
3 2 1 2 4R1
u v v 3u Veq V1 0, V2 4
2 2 R1 R2
98. (b) Flux is inward and it is decreasing as I1 3 2R 2 R2 3
loop is going away from wire I2 4 4R1 R1 2
Case-3
10R1 10R 2
Veq
R1 R2
3 2R 2 3 2 1.5R1
I' 10 R1 R 2 I' 10 2.5R1
or I ' 25mA
2 2
direction of induced current is clockwise x y
100. (a) 2
1
a b2
ux = u at (0, b)
uy = 0

2x dx 2y dy
0
a 2 dt b2 dt
Again diff. w.r.t. to time
Force on left side is in left and force on right
2
side is in right. 2x d 2 x 2 dx 2y d 2 y 2 dy
0
2 2 2 dt 2 2 2 dt
a dt a b dt b
99. (d) acceleration at (0, b) is
b
ay u2
a2

F
F F
V1 V2 101. (b) XeF6 Xe (distorted octahedral)
F F
R1 R2 V1R 2 V2 R1
Veq ; F
1 1 R1 R2
R1 R2

R1 R 2 Xe
R eq XeO3 O O (Pyramidal)
R1 R2
O
102. (b) MnO4– C2 O42
H
Mn 2 CO2 108. (d) PT PA0 PB0 X A PB0
n 5 n 2
300 = (–400)XA + 500
So moles of MnO4 = 2
1
XA =
and moles of C2O = 5 2
4
2

and hence ratio = 2 : 5 Now


H
MnO 4 Cl Mn 2 Cl 2 PA PA0 X A
X 'A
Balanced reaction is: PT PA0 X A PB0 X B

8H 2MnO4 10Cl 2Mn2 5Cl2 4H2O


1
So, required ratio = 1 : 8 100
2 50 1
1 1 300 6
103. (b) Fe3 e Fe 2 E10 0.770 100 500
2 2
Fe 2 2e Fe E20 0.44
Fe3 3e Fe E3 0
? 110. (a) Energy spent = total area covered by droplets
X (Cross) Surface Tension
2 0.44 1 0.770 Energy spent = 0.03 × 12.5 × 2.4 × 100
E30 0.037 V
3 = 90 Joule
1 1 (d)
E1st case 12 32 32 (b)
104. (a) E2nd case 1 1 27
12 22
(c)
O 23
1
CN CH3 – C = NH C – CH3 (d)
2
+
(i) CH 3MgBr H3 O
105. (c) (a)

Iodoform
NaOH/I2
Test

COOH COONa
+
H 3O

34A° 3.4A°

OH OH O OH
NO N
HNO2
106. (b)
(X)
OH

, H2SO 4
O
N
OH
6 . Let ABC be an equilateral triangle, let KLMN be
a rectangle with K, L on BC, M on AC and N on
0 i AB. Suppose AN/NB = 2 and the area of triangle
1 . Let A denote the matrix , where i2 = –1, BKN is 6. The area of the triangle ABC is-
i 0
(a) 54
1 0 (b) 108
and let I denote the identity matrix 0 1 Then
(c) 48
I + A + A2 + … + A2010 is- (d) not determinable with the above data
0 0 0 i 7 . Let P be an arbitrary point on the ellipse
(a) (b)
0 0 i 0 x2 y2
2
1 , a > b > 0. Suppose F and F are the
a b2 1 2
1 i 1 0
(c) (d) foci of the ellipse. The locus of the centroid of the
i 1 0 1 triangle PF1F2 as P moves on the ellipse is-
2. Suppose the sides of a triangle form a geometric (a) a circle (b) a parabola
progression with common ratio r. Then r lies in (c) an ellipse (d) a hyperbola
the interval- 8 . The number of roots of the equation
cos7 – sin4 = 1 that lie in the interval [0, 2 ] is-
1 5 1 5 2 5
(a) 0, (b) , (a) 2 (b) 3
2 2 2
(c) 4 (d) 8
9 . The product (1 + tan 1º) (1 + tan 2º) (1 + tan 3º) ….
1 5 1 5 2 5 (1 + tan 45º) equals-
(c) , (d) ,
2 2 2 (a) 221
3. The number of rectangles that can be obtained (b) 222
by joining four of the twelve vertices of a 12-sided (c) 223
regular polygon is- (d) 225
(a) 66 (b) 30 10. Let f : R R be a differentiable function such
(c) 24 (d) 15 that f (a) = 0 = f (b) and f (a) f (b) > 0 for some
4. Let I, and 2 be the cube roots of unity. The a < b. Then the minimum number of roots of
least possible degree of a polynomial, with real f (x = 0 in the interval (a, b) is-
coefficients, having 2 2, 3 + 4 , 3 + 4 2 and (a) 3
5 – – 2 as roots is- (b) 2
(a) 4 (b) 5 (c) 1
(c) 6 (d) 8 (d) 0
5 . A circle touches the parabola y2 = 4x at (1, 2) and 11. The roots of (x – 41)49 + (x – 49)41 + (x – 2009)2009
also touches its directrix. The y-coordinates of = 0 are -
the point of contact of the circle and the directrix
(a) all necessarily real
is-
(b) non-real except one positive real root
(a) 2 (b) 2 (c) non-real except three positive real roots
(c) 2 2 (d) 4 (d) non-real except for three real roots of which
exactly one is positive
12. The figure shown below is the graph of the 2iˆ ˆj k,
ˆ v 3ˆj 2kˆ be vectors in R3
17. Let u
derivative of some function y = f (x).
and w be a unit vector in the xy-plane. Then the
maximum possible value of u v w is-
(a) 5 (b) 12
(c) 13 (d) 17
Then-
18. How many six-digit numbers are there in which
(a) f has local minima at x = a, b and a local
no digit is repeated, even digits appear at even
maximum at x = c
places, odd digits appear at odd places and the
(b) f has local minima at x = b, c and a local number is divisible by 4 ?
maximum at x = a
(a) 3600 (b) 2700
(c) f has local minima at x = c, a and a local
maximum at x = b (c) 2160 (d) 1440
(d) the given figure is insufficient to conclude any 19. The number of natural numbers n in the interval
thing about the local minima and local maxima [1005, 2010] for which the polynomial 1 + x + x2 +
of f x3 + …. xn–1 divides the polynomial 1 + x2 + x3 +
x4 + …. + x2010 is-
13. The following figure shows the graph of a
continuous function y = f(x) on the interval [1, 3]. (a) 0 (b) 100
The points A, B, C have coordinates (1, 1), (3, 2), (c) 503 (d) 1006
(2, 3) respectively, and the lines L1 and L2 are 20. Let a0 = 0 and ax = 3an–1 + 1 for n 1. Then the
parallel, with L1 being tangent to the curve at C.
remainder obtained dividing a2010 by 11 is-
If the area under the graph of y = f(x) from x = 1
to x = 3 is 4 square units, then the area of the (a) 0 (b) 7
shaded region is- (c) 3 (d) 4

21. A pen of mass ‘m’ is lying on a piece of paper of


mass M placed on a rough table. If the coefficient
of friction between the pen and paper, and, the
paper and table are µ1 and µ2, respectively, then
the minimum horizontal force with which the
(a) 2 (b) 3 paper has to be pulled for the pen to start slipping
(c) 4 (d) 5 is given by-
1 (a) (m + M) (µ1 + µ2) g
14. Let In = (log x)n dx , where n is a non-negative (b) (mµ1 + Mµ2)g
0
integer. Then I2001 – 2011 I2010 is equal to- (c) {mµ1 + (m + M) µ2} g
(a) I1000 + 999 I998 (b) I890 + 890 I889 (d) m(µ1 + µ2) g
(c) I100 + 100 I99 (d) I53 + 54 I52 22. Two masses m1 and m2 connected by a spring of
15. Consider the regions A = {(x, y) | x2 + y2 100} spring constant k rest on a frictionless surface. If
and = {(x, y) | sin (x + y) > 0} in the plane. Then the masses are pulled apart and let go, the time
the area of the region A B is- period of oscillation is-
(a) 10 (b) 100 1 m1 m 2
(a) T = 2
(c) 100 (d) 50 k m1 m 2
16. Three vertices are chosen randomly from the
seven vertices of a regular 7-sided polygon. The m1 m2
(b) T = 2 k
probability that they form the vertices of an m1 m2
isosceles triangle is-
m1
1 1 (c) T = 2
(a) (b) k
7 3
3 3 m2
(c) (d) (d) T = 2
7 5 k
23. A bead of mass m is attached to the mid-point of a 27. A charge +q is placed somewhere inside the cavity
taut, weightless string of length and placed on of a thick conducting spherical shell of inner
a frictionless horizontal table. radius R1 and outer radius R2. A charge – Q is
placed at a distance r > R2 from the centre of the
shell. Then the electric field in the hollow cavity-
(a) depends on both +q and – Q
(b) is zero
(c) is only that due to – Q
Under a small transverse displacement x, as (d) is only that due to +q
shown, if the tension in the string is T, then the 28. The following travelling electromagnetic wave
frequency of oscillation is- Ex = 0, Ey = E0sin(kx + t), Ez = –2E0sin(kx – t) is-
1 2T 1 4T (a) elliptically polarized
(a) (b)
2 m 2 m (b) circularly polarized
1 4T 1 2T (c) linearly polarized
(c) (d)
2 m 2 m (d) unpolarized
24. A comet (assumed to be in an elliptical orbit 29. A point source of light is placed at the bottom of a
around the sun) is at a distance of 0.4 AU from vessel which is filled with water of refractive index
the sun at the perihelion. If the time period of µ to a height h. If a floating opaque disc has to be
the comet is 125 years, what is the aphelion placed exactly above it so that the source is invisible
distance ? AU : Astronomical Unit. from above, the radius of the disc should be-
(a) 50 AU (b) 25 AU h h
(c) 49.6 AU (d) 24.6 AU (a) (b) 2
1 1
25. The circuit shown consists of a switch (S), a battery
h h
(B) of emf E, a resistance R, and an inductor L.
(c) 2 (d) 2
1 1
30. Three transparent media of refractive indices µ1,
µ2, µ3 respectively, are stacked as shown. A ray
of light follows the path shown. No light enters
the third medium.
The current in the circuit at the instant the switch
is closed is-
(a) E/R (b) E/R(1 – e)
(c) (d) 0
26. Consider a uniform spherical volume charge
distribution of radius R. Which of the following
graphs correctly represents the magnitude of the
electric field E as a distance r from the center of Then-
the sphere?
(a) µ1 < µ2 < µ3 (b) µ2 < µ1 < µ3
(c) µ1 < µ3 < µ2 (d) µ3 < µ1 < µ2
31. A nucleus has a half-life of 30 minutes. At 3 PM its
decay rate was measured as 120,000 counts/sec.
(a) (b)
What will be the decay rate at 5 PM?
(a) 120,000 counts/sec
(b) 60,000 counts/sec
(c) 30,000 counts/sec
(d) 7,500 counts/sec
(c) (d)
32. A book is resting on shelf that is undergoing 36. Velocity of sound measured at a given
vertical simple harmonic oscillations with an temperature in oxygen and hydrogen is in the
amplitude of 2.5 cm. What is the minimum ratio -
frequency of oscillation of the shell for which the (a) 1 : 4 (b) 4 : 1
book will lose contact with the shelf? (Assume
(c) 1 : 1 (d) 32 : 1
that g = 10 m/s2)
37. In Young’s double slit experiment, the distance
(a) 20 Hz (b) 3.18 Hz
between the two slits is 0.1 mm, the distance
(c) 125.6 Hz (d) 10 Hz between the slits and the screen is 1 m and the
33. A van der Waal’s gas obeys the equation of state wavelength of the light used is 600 nm. The
n 2a intensity at a point on the screen is 75% of the
P (V nb) nRT . Its internal energy is maximum intensity. What is the smallest distance
V2
of this point from the central fringe ?
n 2a
given by U = CT – . The equation of a (a) 1.0 mm (b) 2.0 mm
V
quasistatic adiabat for this gas is given by- (c) 0.5 mm (d) 1.5 mm
(a) TC/nRV = constant 38. Two masses m1 and m2 are connected by a massless
spring of spring constant k and unstreched length
(b) T(C+nR)/nRV = constant
. The masses are placed on a frictionless straight
(c) TC/nR (V – nb) = constant channel – which we consider our x-axis. They are
(d) P(C+nR)/nR(V – nb) = constant initially at rest at x = 0 and x = , respectively. At
34. An ideal gas is made to undergo a cycle depicted t = 0, a velocity of v0 is suddenly imparted to the
by the PV diagram alongside. The curved line first particle. At a later time t0, the centre of mass
from A to B is an adiabat. of the two masses is at-
m2
(a) x =
m1 m2

m1 m 2 v0 t
(b) x =
m1 m2 m1 m2

m2 m 2 v0 t
(c) x =
m1 m2 m1 m2
Then-
m2 m1 v 0 t
(a) The efficiency of this cycle is given by unity (d) x =
as no heat is released during the cycle m1 m2 m1 m2
(b) Heat is absorbed in the upper part of the straight 39. A charged particle of charge q and mass m, gets
line path and released in the lower part deflected through an angle upon passing
(c) If T1 and T2 are the maximum and minimum through a square region of side ‘a’ which contains
temperatures reached during the cycle, then a uniform magnetic field B normal to its plane.
T2 Assuming that the particle entered the square at
the efficiency is given by 1 – right angles to one side, what is the speed of the
T1
particle?
(d) The cycle can only be carried out in the reverse
of the direction shown in figure qB
(a) a cot
35. A bus driving along at 39.6 kmph is approaching m
a person who is standing at the bus stop, while qB
honking repeatedly at an interval of 30 seconds. (b) a tan
m
If the speed of the sound is 330 m/s, at what
interval will the person hear the horn? qB
(c) a cot2
(a) 31 seconds m
(b) 29 seconds qB
(c) 30 seconds (d) a tan2
m
(d) the interval will depend on the distance of the
bus from the passenger
40. A piece of hot copper at 100ºC is plunged into a 48. The order of acidity of compounds I-IV, is-
pond at 30ºC. The copper cools down to 30ºC, while
the pond, being huge, stays at its initial
temperature. Then-
(a) copper loses some entropy, the pond stays at
the same entropy
(b) copper loses some entropy, and the pond gains
exactly the same amount of entropy
(c) copper loses entropy, and the pond gains more
than this amount of entropy (a) I < III < II < IV
(d) both copper and the pond gain in entropy (b) IV < I < II < III
(c) III < I < II < IV
(d) II < IV < III < I
41. The number of isomers of Co (diethylene triamine) 49. The most stable conformation for n-butane is-
Cl3 is-
(a) 2 (b) 3
(c) 4 (d) 5
42. Among the following, the -acid ligand is- (a) (b)
(a) F (b) NH3
(c) CN– (d) I–
43. The bond order in O22– is-
(a) 2 (b) 3
(c) 1.5 (d) 1 (c) (d)
44. The energy of a photon of wavelength k = 1 meter
is (Planck’s constant = 6.625 × 10–34 Js, speed of
light = 3 × 108 m/s) 50. In the nuclear reaction 234
94 Th
234
91 Pa X . X is-
(a) 1.988 × 10–23 J (b) 1.988 × 10–28 J
(a) 0 (b) 0
(c) 1.988 × 10–30 J (d) 1.988 × 10–25 J 1e 1e

45. The concentration of a substance undergoing 2


(c) H (d) 1H
a chemical reaction becomes one-half of
its original value after time t regardless of 51. A concentrated solution of copper sulphate, which
the initial concentration. The reaction is an is dark blue in colour, a mixed at room
example of a- temperature with a dilute solution of copper
(a) zero order reaction sulphate, which is light blue. For this process-
(b) first order reaction (a) Entropy change is positive, but enthalpy
change is negative
(c) second order reaction
(b) Entropy and enthalpy changes are both
(d) third order reaction
positive
46. The shape of the molecule ClF3 is-
(c) Entropy change is positive and enthalpy does
(a) trigonal planar not change
(b) pyramidal (d) Entropy change is negative and enthalpy
(c) T-shaped change is positive
(d) Y-shaped 52. Increasing the temperature increases the rate of
47. Friedel-Crafts acylation is- reaction but does not increase the-
(a) -acylation of a carbonyl compound (a) number of collisions
(b) acylation of phenols to generate esters (b) activation energy
(c) acylation of aliphatic olefins (c) average energy of collisions
(d) acylation of aromatic nucleus (d) average velocity of the reactant molecules
53. In metallic solids, the number of atoms for the 57. The enontiomeric pair among the following four
face-centered and the body-centered cubic unit structures-
cells, are, respectively-
(a) 2, 4 (b) 2, 2
(c) 4, 2 (d) 4, 4 I.
54. From equations 1 and 2,
1
CO2 CO O2
2
II.
13
K cl 9.1 10 at 1000 C (eq. 1)

1
H2O H2 O2
2 III.
12
K cl 7.1 10 at 1000 C (eq. 2)
the equilibrium constant for the reaction CO2 +
IV.
H2 CO + H2O at the same temperature, is-
(a) 0.78 (b) 2.0 (a) I & II
(c) 16.2 (d) 1.28 (b) I & IV
55. For a first order reaction R P, the rate constant (c) II & III
is k. If the initial concentration of R is [R0], the (d) II & IV
concentration of R at any time ‘t’ is given by the
58. Consider the reaction : 2 NO2(g) 2 NO(g) + O2 (g).
expression-
In the figure below, identify the curves X, Y and Z
(a) [R0] ekt (b) [R0](1 – e–kt] associated with the three species in the reaction-
(c) [R0] e–kt (d) [R0] (1 – ekt)
56. The correct structure of PCl3F2 is-

(a)

(a) X = NO, Y = O2, Z = NO2

(b) (b) X = O2, Y = NO, Z = NO2


(c) X = NO3, Y = NO, Z = O2
(d) X = O2, Y = NO2, Z = NO
59. The aromatic carbocation among the following
is-

(c)

(a) (b)

(d) (c) (d)


65. If the total number of photons falling per unit
60. Cyclohexene is reacted with bromine in CCl4 in
area of a leaf per minute is kept constant, then
the dark. The product of the reaction is-
which of the following will result in maximum
photosynthesis?
(a) (b) (a) Shining green light
(b) Shining sunlight
(c) Shining blue light
(d) Shining ultraviolet light
(c) (d) 66. Path-finding by ants is by means of-
(a) Visually observing landmarks
(b) Visually observing other ants
(c) Chemical signals between ants
61. Ribonucleic Acids (RNA) that catalyze enzymatic
(d) Using the earth’s magnetic field
reactions are called ribozymes. Which one of the
following acts as a ribozyme ? 67. Sometimes urea is fed to ruminants to improve
their health. It works by-
(a) Ribosome
(a) Helping growth of gut microbes that break
(b) Amylase
down cellulose
(c) tRNA
(b) Killing harmful microorganisms in their gut
(d) Riboflavin
(c) Increasing salt content in the gut
62. In 1670, Robert Boyle conducted an experiment
where in he placed a viper (a poisonous snake) in (d) Directly stimulating blood cell proliferation
a chamber and rapidly reduced the pressure in 68. If you compare adults of two herbivore species of
that chamber. Which of the following would be different sizes, but from the same geographical
true ? area, the amount of faeces produced per kg body
(a) Gas bubbles developed in the tissues of the weight would be-
snake (a) More in the smaller one than the larger one
(b) The basal metabolic rate of the snake (b) More in the larger one than the smaller one
increased tremendously
(c) Roughly the same amount in both
(c) The venom of the snake was found to decrease
(d) Not possible to predict which would be more
in potency
69. Fruit wrapped in paper ripens faster than when
(d) The venom of the snake was found to increase
kept in open air because-
in potency
63. Bacteria can survive by absorbing soluble (a) Heat of respiration is retained better
nutrients via their outer body surface, but animals (b) A chemical in the paper helps fruit ripening
cannot, because (c) A volatile substance produced by the fruit is
(a) Bacteria cannot ingest particles but animals retained better and helps in ripening
can (d) The fruit is cut off from the ambient oxygen
(b) Bacteria have cell walls and animals do not which is an inhibitor to fruit ripening
(c) Animals have too small a surface area per 70. When a person is suffering from high fever, it is
unit volume as compared to bacteria sometimes observed that the skin has a reddish
(d) Animals cannot metabolize soluble nutrients tinge. Why does this happen?
64. A horse has 64 chromosomes and a donkey has (a) Red colour of the skin radiates more heat
62. Mules result from crossing a horse and a (b) Fever causes the release of a red pigment in
donkey. State which of the following is the skin
INCORRECT?
(c) There is more blood circulation to the skin to
(a) Mules can have either 64, 63 or 62 chromosomes keep the body warm
(b) Mules are infertile
(d) There is more blood circulation to the skin to
(c) Mules have well defined gender (male/female) release heat from the body
(d) Mules have 63 chromosomes
71. Bacteriochlorophylls are photosynthetic pigments 76. If you dip a sack full of paddy seeds in water
found in phototrophic bacteria. Their function overnight and then keep it out for a couple of
is distinct from the plant chlorophylls in that days, it feels warm. What generates this heat ?
they- (a) Imbibation
(a) do not produce oxygen (b) Exothermic reaction between water and seed
(b) do not conduct photosynthesis coats
(c) absorb only blue light (c) Friction among seeds due to swelling
(d) function without a light source (d) Respiration
72. Athletes often experience muscle cramps. Which 77. Restriction endonucleases are enzymes that
of the following statements is true muscle cleave DNA molecules into smaller fragments.
cramps? Which type of bond do they act on ?
(a) Muscle cramp is caused due to conversion of (a) N-glycosidic Bond
pyruvic acid into lactic acid in the cytoplasm
(b) Hydrogen bond
(b) Muscle cramp is caused due to conversion
(c) Phosphodiester bond
of pyruvic acid into lactic acid in the
mitochondria (d) Disulfide bond
(c) Muscle cramp is caused due to nonconversion 78. The fluid part of blood flows in and out of
of glucose to pyruvate in the cytoplasm capillaries in tissue to exchange nutrients and
(d) Muscle cramp is caused due to conversion of waste materials. Under which of the following
pyruvic acid into ethanol in the cytoplasm conditions will fluid flow out from the capillaries
into the surrounding tissue?
73. A couple went to a doctor and reported that
both of them are “carriers” for a particular (a) When arterial blood pressure exceeds blood
disorder, their first child is suffering from osmotic pressure
that disorder and that they are expecting (b) When arterial blood pressure is less than blood
their second child. What is the probability that osmotic pressure
the new child would be affected by the same (c) When arterial blood pressure is equal to blood
disorder ? osmotic pressure
(a) 100 % (d) Arterial blood pressure and blood osmotic
(b) 50 % pressure have nothing to do with the outflow
(c) 25 % of fluid from capilleries
(d) 75 % 79. The distance between two consecutive DNA base
74. Of the following combinations of cell biological pairs is 0.34 nm. If the length of a chromosome is
processes which one is associated with 1 mm, the number of base pairs in the
embryogenesis? chromosome is approximately-
(a) Mitosis and Meiosis (a) 3 million
(b) Mitosis and Differentiation (b) 30 million
(c) Meiosis and Differentiation (c) 1.5 million
(d) Differentiation and Reprogramming (d) 6 million
75. Conversion of the Bt protoxin produced by Bacillus 80. Estimate the order of the speed of propagation of
thuringiensis to its active form in the gut of the an action potential or nerve impulse -
insects is mediated by- (a) nm/s
(a) acidic pH of the gut (b) micron/s
(b) alkaline pH of the gut (c) cm/s
(c) lipid modification of the protein (d) m/s
(d) cleavage by chymotrypsin
n
1/ 2 1
81. Arrange the expansion of x in (a) (b)
2x1/ 4
decreasing powers of x. Suppose the coefficient of
the first three terms form an arithmetic
progression. Then the number of terms in the
expansion having integer powers of x is-
(a) 1 (b) 2 (c) (d)
(c) 3 (d) more than 3
82. Let r be a real number and n N be such that
the polynomial 2x2 + 2x + 1 divides the polynomial (a) Fig. 1 (b) Fig. 2
(x + 1)n – r. Then (n, r) can be- (c) Fig. 3 (d) Fig. 4
1 86. Let V1 be the volume of a given right circular
(a) (4000, 41000) (b) 4000, cone with O as the centre of the base and A as its
41000
apex. Let V2 be the maximum volume of the right
1 1 circular cone inscribed in the given cone whose
(c) 41000 , (d) 4000,
4 1000
4000 apex is O and whose base is parallel to the base
of the given cone. Then the ratio V2/V1 is-
83. Suppose a, b are real numbers such that ab 0.
Which of the following four figures represents the 3 4
(a) (b)
curve (y – ax – b)(bx2 + ay2 – ab) = 0? 25 9
4 8
(c) (d)
27 27
87. Let f : R R be a continuous function satisfying
(a) (b)
x
f(x) = x + f (t)dt , for all x R. Then the number
0
of elements in the set S = {x R ; f(x) = 0} is-
(c) (d) (a) 1 (b) 2
(c) 3 (d) 4
2
84. Among all cyclic quadrilaterals inscribed in a circle
of radius R with one of its angles equal to 120º. 88. The value of min x , cos 1 (cos x) dx is-
0
Consider the one with maximum possible area.
2 2
Its area is-
(a) (b)
4 2
(a) 2 R2 (b) 3 R2 2
(c) (d) 2
(c) 2 R2 (d) 2 3 R2 8
89. Let ABC be a triangle and P be a point inside
85. The following figure shows the graph of a
differentiable function y = f(x) on the interval ABC such that PA 2PB 3PC 0 . The ratio of
[a, b] (not containing 0). the area of triangle ABC to that of APC is-
3
(a) 2 (b)
2
5
(c) (d) 3
3
90. Suppose m, n are positive integers such that
6 m + 2 m+n 3 w + 2 n = 332. The value of the
expression m2 + mn + n2 is-
Let g(x) = f(x) / x which of the following is a possible
graph of y = g(x)? (a) 7 (b) 13
(c) 19 (d) 21
95. A narrow parallel beam of light falls on a glass
sphere of radius R and refractive index µ at normal
91. A ball is dropped vertically from a height of h incidence. The distance of the image from the
onto a hard surface. If the ball rebounds from the outer edge is given by-
surface with a fraction r of the speed with which R(2 ) R(2 )
it strikes the latter on each impact, what is the (a) (b)
2( 1) 2( 1)
net distance traveled by the ball up to the 10th
impact ? R(2 ) R(2 )
(c) (d)
2( 1) 2( 1)
1 r10 1 r 20 96. A particle of mass m undergoes oscillations about
(a) 2h (b) h
1 r 1 r2 1 2 x
x = 0 in a potential given by V(x) = kx V0 cos ,
22 20 2 a
1 r 1 r
(c) 2h h (d) 2h h where V0, k, a are constants. If the amplitude of
2
1 r 1 r2 oscillation is much smaller than a, the time period
92. A certain planet completes one rotation about its is given by-
axis in time T. The weight of an object placed at
ma 2 m
the equator on the planet’s surface is a fraction f (a) 2 (b) 2
(f is close to unity) of its weight recorded at a ka 2 V0 k
latitude of 60º. The density of the planet assumed
to be a uniform perfect sphere is given by- ma 2 ma 2
(c) 2 (d) 2
V0 ka 2 V0
4 f 3 4 f 3
(a) (b) 97. An ideal gas with heat capacity at constant volume
1 f 4GT 2 1 f 4GT 2
CV undergoes a quasistatic process described by
4 3f 3 4 2f 3 PV in a P-V diagram, where is a constant. The
(c) (d) heat capacity of the gas during this process is
1 f 4GT 2 1 f 4GT 2
given by-
93. Three equal charges +q are placed at the three
(a) CV (b) CV + nR
vertices of an equilateral triangle centered at the
origin. They are held in equilibrium by a restoring nR nR
force of magnitude F(r) = kr directed towards the (c) CV + (d) CV + 2
1 1
origin, where k is a constant. What is the distance 98. An ideal gas with constant heat capacity
of the three charges from the origin?
3
2 1/ 2 2 1/3 CV = nR is made to carry out a cycle that is
1 q 3 q 2
(a) (b)
6 0 k 12 0 k depicted by a triangle in the figure given below.

2/3 2/3
1 q2 3 q2
(c) (d)
6 0 k 4 0 k
94. Consider the infinite ladder circuit shown below.

The following statement is true about the cycle-


P1 V1
(a) The efficiency is given 1 –
For which angular frequency will the circuit P2 V2
behave like a pure inductance? 1 P1 V1
(b) The efficiency is given by 1 –
LC 1 2 P2 V2
(a) (b) (c) Net heat absorbed in the cycle is (P2 – P1)
2 LC
(V2 – V1)
2 2L (d) Heat absorbed in part AC is given by
(c) (d)
LC C 1
2(P2V2 – P1V1) + (P1V2 – P2V1)
2
99. Two identical particles of mass ‘m’ and charge q
are shot at each other from a very great distance
with an initial speed v. The distance of closest (c)
approach of these charges is-
q2 q2
(a) 2 (b) 2
8 0 mv 4 0 mv (d)
q2
(c) 2 (d) 0
2 0 mv
103.The number of possible enatiomeric pair(s)
100. At time t = 0, a container has N 0 radioactive produced from the bromination of I and II,
atoms with a decay constant . In addition, c respectively, are
numbers of atoms of the same type are being
added to the container per unit time. How many
atoms of this type are there at t = T ?
c
(a) exp(– T) – N0 exp(– T)

c
(b) exp(– T) + N0 exp(– T)
(a) 0, 1
c
(c) {1 – exp(– T)} + N0 exp(– T) (b) 1, 0
(c) 0, 2
c
(d) {1 + exp(– T)} + N0 exp(– T) (d) 1, 1
104.For the reaction A B, Hº = 7.5 mol–1 and
Sº = 2.5 J mol –1. The value of Gº and the
101. 2.52 g of oxalic acid dehydrate was dissolved in temperature at which the reaction reaches
100 ml of water, 10 mL of this solution was diluted equilibrium are, respectively,
to 500 mL. The normality of the final solution (a) 0 kJ mol–1 and 400 K
and the amount of oxalic acid (mg/mL) in the (b) –2.5 kJ mol–1 and 400 K
solution are respectively-
(c) 2.5 kJ mol–1 and 200 K
(a) 0.16 N, 5.04 (b) 0.08 N, 3.60
(d) 0 kJ mol–1 and 300 K
(c) 0.04 N, 3.60 (d) 0.02 N, 10.08
105.The solubility product of Mg(OH) 2 is
102. Two isomeric compounds I and II are heated with
1.0 × 10–12. Concentrated aqueous NaOH solution
HBr –
is added to a 0.01 M aqueous solution of MgCl2.
The pH at which precipitation occur is-
(a) 7.2
(b) 7.8
(c) 8.0
(d) 9.0
The products obtained are- 106.A metal with an atomic radius of 141.4 pm
crystallizes in the face centred cubic structure.
The volume of the unit cell in pm is-
(a) (a) 2.74 × 107
(b) 2.19 × 107
(c) 6.40 × 107
(d) 9.20 × 107
(b)
107.Identify the cyclic silicate ion given in the figure
below:
111.A bust cell has intracellular bacteria symbionts.
If the growth rate of the bacterial symbiont is
always 10% higher than that of the host cell, after
10 generations of the host cell the density of
bacteria in host cells will increase -
(a) by 10 %
(b) two-fold
(c) ten-fold
(d) hundred-fold
112.In a diploid organism, there are three different
alleles for a particular gene. Of these three alleles
one is recessive and the other two alleles exhibit
co-dominance. How many phenotypes are possible
with this set of alleles ?
(a) 3 (b) 6
(c) 4 (d) 2
(a) [Si4O25]24– (b) [Si6O18]18– 113.Two students are given two different double
(c) [Si4O12] 12 (d) [Si6O24]12– stranded DNA molecules of equal length. They
108.Diborane is formed the elements as shown in are asked to denature the DNA molecules by
equation (1) heating. The DNA given to student A has the
2B(s) + 3H2(g) B2H6 (g) … (1) following composition of bases (A : G : T : C : 35 :
15 : 35 : 15) while that given to student B is
Given that
(A : G : T : C :: 12 : 38 : 12 : 38). Which of the
H2O(l) H2O (g) H1º = 44 kJ following statements is true?
2B(s) + 3/2O2(g) B2O3(s) H2º = – 1273 kJ
(a) Both the DNA molecules would denature at
B2H6(g) + 3O2(g) B2O3(s) + 3H2O (g) the same rate
H3º = – 2035 kJ (b) The information given is insufficient to draw
H2(g) + 1/2 O2(g) H2O(l) H4º = – 286 kJ any conclusion
the Hº for the reaction (1) is- (c) DNA molecule given to student B would
(a) 36 kJ (b) 509 kJ denature faster than that of student A
(c) 520 kJ (d) –3550 kJ (d) DNA molecule given to student A would
109.The Crystal Field Stabilization Energy (CPSE) and denature faster than that given to student B
the spin-only magnetic moment in Bohr 114. The amino acid sequences of a bacterial protein
Magneton (BM) for the complex K3[Fe(CN)6] are, and a human protein carrying out similar function
respectively- are found to be 60% identical. However the DNA
sequences of the genes coding for these proteins
(a) 0.0 s and 35 BM
are only 45% identical. This is possible because-
(b) –2.0 s and 3 BM (a) Protein sequence does not depend on DNA
sequence
(c) –0.4 s and 24 BM
(b) DNA codons having different nucleotides in
(d) –2.4 s and 0 BM the third position can code for the same amino
110.A solution containing 8.0 g of nicotine in 92 g of acids
water freezes 0.925 degrees below the normal
(c) DNA codons having different nucleotides in
freezing point of water. If the molal freezing point
the second position can code for the same
depression constant Kf = – 1.85ºC mol–1 then the
amino acids
molar mass of nicotine is-
(a) 16 (b) 80 (d) Same DNA codons can code for multiple amino
acids
(c) 320 (d) 160
115.The following DNA sequence (5’ 3’) specifies 119.Which of the following graphs accurately
part of a protein coding sequence, starting from represents the insulin levels (Y-axis) in the body
position I. Which of the following mutations will as a function of time (X-axis) after eating sugar
give rise to a protein that is shorter than the full- and bread/roti?
length protein?
1 2 3 4 5 6 7 8 9 10 11 12 13 14 15
AT GCAAGAT A T A G C T
(a) Deletion of nucleotide 13
(b) Deletion of nucleotide 8 (a) (b)
(c) Insertion of a single nucleotide between 3
and 4
(d) Insertion of a single nucleotide between 10
and 11
116.Which of the following correctly represents the
results of an enzymatic reaction ? Enzyme is E,
substrate is S and products are P1 & P2. (c) (d)
(a) P1 + S P2 + E (b) E + S P1 + P2
(c) P1 + P2 + E S (d) E + S P1 + P2 + E
117.Four species of birds have different egg colors :
[1] white with no markings, [2] pale brown with 120.You marked two ink-spots along the height at
no markings. [3] grey-brown with dark streaks the base of a coconut tree and also at the top of
and spots, [4] pale blue with dark blue-green spots. the tree. When you examine the spots next year
Based on egg color, which species is most likely when the tree has grown taller, your will see-
to nest in a deep tree hole? (a) the two spots at the top have grown more apart
(a) 1 (b) 2 than the two spots at the bottom
(c) 3 (d) 4 (b) the top two spots have grown less apart then
118.Consider a locus with two alleles, A and a. If the the bottom two spots
frequency of AA is 0.25, what is the frequency of (c) both sets of spots have grown apart to the same
A under Hardy-Weinberg equilibrium? extent
(a) 1 (b) 0.25 (d) both sets of spots remain un-altered
(c) 0.5 (d) 0

1. (c) 2. (c) 3. (d) 4. (b) 5. (c) 6. (b) 7. (c) 8. (a) 9. (c) 10. (b)
11. (b) 12. (c) 13. (a) 14. (c) 15. (d) 16. (d) 17. (d) 18. (d) 19. (c) 20. (a)
21. (a) 22. (a) 23. (b) 24. (c) 25. (d) 26. (a) 27. (d) 28. (b) 29. (b) 30. (d)
31. (d) 32. (b) 33. (c) 34. (b) 35. (b) 36. (a) 37. (a) 38. (d) 39. (a) 40. (c)
41. (a) 42. (c) 43. (d) 44. (a) 45. (b) 46. (c) 47. (d) 48. (a) 49. (a) 50. (a)
51. (c) 52. (b) 53. (c) 54. (d) 55. (b) 56. (a) 57. (b) 58. (a) 59. (c) 60. (a)
61. (a) 62. (a) 63. (c) 64. (a) 65. (c) 66. (a) 67. (a) 68. (a) 69. (c) 70. (d)
71. (a) 72. (a) 73. (c) 74. (b) 75. (b) 76. (d) 77. (c) 78. (a) 79. (a) 80. (d)
81. (c) 82. (b) 83. (b) 84. (b) 85. (b) 86. (c) 87. (a) 88. (b) 89. (d) 90. (c)
91. (d) 92. (a) 93. (b) 94. (c) 95. (a) 96. (a) 97. (c) 98. (b) 99. (b) 100. (c)
101. (c) 102. (a) 103. (a) 104. (d) 105. (d) 106. (c) 107. (b) 108. (a) 109. (b) 110. (d)
111. (b) 112. (c) 113. (d) 114. (b) 115. (b) 116. (d) 117. (a) 118. (c) 119. (a) 120. (a)
5 . (c)

0 i 1 0
1 . (c) A ; A2 ;
i 0 0 1

0 i 1 0
A3 ; I
i 0 0 1

0 0
Now, I + A + A2 + A3 = 0 0 y2 = 4x
The given equation of parabola is
differentiating above equation w.r.t to 'x' we
1 0 get
A4 I
0 1
dy
2y 4
I+A+ A2 + ….. A3 + A2010 dx
(I + A + A2 + A3) + A4(I + A + A2 + A3) + ... 2 2
Slope of tengent (mT) = 1
+ A2008(I + A + A2) y 2
1 i Equation of Circle S+ L=0
= (x + 1)2 +
(y – )2 +
(x + 1) = 0 … (1)
i 1
Differentiate Equation (1) w.r.t (x) we get
2 . (c)
dy
2(x + 1) + 2(y – ) + =0 ...(2)
dx
x = 1, y = 2
Putting the value of x and y in equation we
get
a + ar > ar2 4 + 2(2 – ) mT + =0
a(1 + r) > ar2
=2 –8 … (3)
1 + r > r2
Now (1, 2) satisfies eq.(1)
2
r –r+<0
22 + (2 – )2 + 2 = 0
2– 4 + 8 + 2(2 – 8) = 0
1 5 1 5
r , … (1) 2=
2 2 8
ar2 + ar > 1 = 2 2
r2 + r – 1 > 0 6 . (b) A

1 5 1 5 2x 2x
r , r … (2) 60°
2 2
N M
ar2 + a > ar ; r2 – r + 1 > 0 always true 2x
Solving equation (1) and (2)
x y x
y
5 1 5 1 60° 60°
r , C
2 2 B Z K 2x L Z
3 . (d) Number of diagonals passing through centre 3x
y In NKB,
=6 2
Z
6! Sin 30° =
Number of rectangles = 6C2 = = 15 2
4! 2! x
and z
2
1 48 12. (c) f (a) = f (b) = f (c) = 0
Now, yz 6 x2
2 3 f '(a ) 0 f '(a ) 0
Now, Area of ABC minima at a and c
f '(c ) 0 f '(c ) 0
1
= 6 + 6 + 2xy + (2x)(2x)sin 60 f (b–) > 0 f (b+) < 0 ] maximum at b.
2
2 1
3x 3 13. (a) Equation of Line L2 is given as y – 1 = (x
12 2x 2x 2 3 1
2 2 – 1)
2 3 2y – 2 = x – 1
12 + 4x 2y – x = 1
2
483 1
12 4 Hence slope of line L2 is
3 2 2
= 12 + 96 = 108
1
7 . (c) Let the co-ordination of point P be (a cos , b Equation of line L1 is given as = y – 3 = (x –
2
sin ) 2)
xi yi 2y – 6 = x – 2
G , 2y – x = 4
3 3
F1 (ae, 0) F2 (–ae, 0) 5 7
Now Co-ordinate of D is (1, ) and E(3, )
a cos ae ae 3h 2 2
h ; cos
3 a
b sin 3k
k ; sin
3 b
Now as we know that cos2 + sin2 = 1

9h 2 9k 2 x2 y2
1 1
a2 b2 a2 / 9 b2 / 9
Area under f(x) = 4
(ellipse)
Now Shaded area = Area of trapezium DEFG
8 . (a) cos7 = 1 + sin4 1 but cos7 1
– Area under f(x)
so cos = 1 ; sin =0
1 5 7
0, 2 2 4 =6–4=2
2 2 2
10. (b) f (a) . f (b) > 0 15. (d) x2 + y2 100 inside of a circle
so either both are positive or both are sin(x + y) > 0 (given)
negative x + y (0, ) (2 , 3 ) …..
Now f(a) = f(b) = 0 (given) x+y=c equation of a line

2 roots
11. (b) (x – 41)49 + (x – 49)41 + (x – 2009)2009 = 0
f(x) = (x – 41)49 + (x – 49)41 + (x – 2009)2009
Now differention w.r.t 'x' we get
f (x) = 49(x – 41)48 + 41(x – 49)40
+ 2009 (x – 2009)48 > 0 required area = shaded region
Hence f(x) will cut x-axis only once. 1
= (10)2 = 50
I real root. 2
16. (d) 21. (a)

For pen to start slipping maximum horizontal


force on it is f = µ1mg
a = µ 1 g is the maximum common
acceleration for both pen and paper
AGB, AFC & AED are isosceles
7
C1 3 7 3 3
P=
7
C3 7 6 5 5
3 2
ˆi ˆj kˆ
f1 N1 & N1 W1
17. (d) u v 2 1 1
0 3 2 f1 1 mg

u v ˆi 4 ˆj 6kˆ
Let w aiˆ bjˆ
a2 + b2 = 1
a = cos and b = sin
u v w a 4b cos 4 sin
max. value = 2 2 N2 N1 W1
1 ( 4) 17
18. (d) 3 ways 3 ways 4 ways 4 ways 5 ways 2 ways F f1 f2 Ma
= 3 × 3 × 4 × 4 × 5 × 2 = 1440 also f2 = µ2N2 = µ2(m + M)g
19. (c) 1 + x2 + x4 + ….. x2010 F = f1 + f2 + Ma
2012 1006 1006 F = µ1mg + µ2(m + M)g + M(µ1g)
11 x 1 x 1 x
= 2
1 x (1 x)(1 x) F (m M) g
1 2

1006 1 x503 1 x503 22. (a)


= 1 x
(1 x) (1 x)
= (1 + x1006)(1 + x + x2 + …. x502)(1 – x + x2
– x3 + …. x502)
this is divisible by 1 + x + x2 + ….. xn–1 Let the masses be slightly displaced by x1 and
if n – 1 = 502 x2 from this equilibrium position in opposite
n = 502 + 1 direction so net stretch in spring is x = x1 +
n = 503 x2. Because of this a restoring force kx will
act on each mass and therefore equation for
20. (a) an = 3an–1 + 1
m1 & m2 will be
a2010 = 3a2009 + 1
= 3(3a2008 + 1) + 1 = 32 a2008 + 3 + 1 d 2 x1 d2x2
m1 kx and m 2 kx
= 33 a2007 + 3 + 3 + 1 dt2 dt2
… but as x = x1 + x2

d2x d 2 x1 d2 x2

32010 a0 + (3 3 3) + 1 dt2 dt2 dt2
2009 times d 2 x1 d2 x 2
= 0 + 6027 + 1 = 6028 replacing values of and from
dt2 dt2
Now remainder obtained by dividing a2050 by acceleration equations we get
dividing a2050 by 11 is '0'
24. (c)
d2 x2 1 1
kx
dt 2 m1 m2
1 1 1
also if then m is the effective
m1 m 2 m
mass in this case therefore 0.4 y
r
2 2
d x 2 kx 2 k 2
2
x or and T
m m 2 3
dt T1 r1
also by Kepler’s law of time-
m m1 m 2 T2 r2
T 2 2 periods (T1, r1 are taken for earth)
k m1 m2 k
2 3
1y 1 AU
23. (b) 125 y 0.4 y
AU
2
solving we get y = 49.6 AU
t / tL
25. (d) Using the equation I = I0 1 e
Let the angle of T with the vertical be then
Put t = 0 we get I = 0
Just when the battery is closed inductor
provides infinite resistance to the current flow
F.B.D. current is zero initially.
26. (a)
2T cos = ma
2
also cos =
2
x2
2
r
given >>x Er for 0 r<R
3 0
2 2
x2 kQ
4 4 Er for r R
r2
2Tx 1
a
E r for r < R and E for r R
m r2
2
curve is
(negative sign for restoring force)
4T
or a = x
m
also this is similar to the equation of SHM
i.e. a = – 2x
27. (d)
4T
=
m

and f =
2

1 4T For a conductor electric field inside its cavity


f=
2 m is only due to inside charge and not due to
outside charge.
28. (b) From the equation of Ey & Ez it is evident dN 1
that wave is circularly polarized. th of the initial activity
dt at 5 P.M. 16
29. (b) dN 1 dN
dt 5 P.M. 16 dt 3 P.M.

dN
7500 counts/sec
dt 5 P.M.

32. (b) Book will loose contact with the shelf when a
=g
r should be such that rays beyond it got totally
2
internally reflected Now a x
For this > C or sin > sin C g= 2A (A Amplitude)
1 g
also µ = 2= A
also f =
2
sin C
r 1 1 g
f=
h 2
r 2 2 A
replacing g = 10 m/s2 and A = 2.5 × 10–2 m
r 1
In limiting case We get f 3.18 Hz
h2 r2
33. (c) For adiabatic process
h
solving we get r dQ = 0 and –dU = dW
2
1 –nCV T = P V or –nCVdT = PdV
30. (d) At first incidence light is deviated towards the when change is very small
normal therefore µ 2 > µ 1. Also at second
n 2a
incidence TIR takes place therefore µ2 > µ3, now given U = CT –
also µ1 > µ3 because for the same angle in V
medium µ2, angle in µ1 medium is less. n 2a
dU = CdT + dV
V2
put this value of dU in – dU = dW
n2a
CdT dV PdV … (1)
V2
nRT n 2a
also P = replace it in (1)
µ3 < µ1 < µ2 V nb V2
31. (d) Given T = 30 minutes. n2a nRT n2a
CdT dv dV
dN counts V2 V nb V2
120K
dt sec
nRT
After each half life, activity is reduced to half CdT dV
V nb
therefore after n half lives activity reduces to
n C dT dV
1
. nR T V nb
2
Integrating we get
dN
Also N – n TC/nR = n (V – nb) + k
dt
(k constant of integration)
dN
at 5 P.M. will be equal to activity n (TC/nR)(V – nb) = – k
dt
remaining after four half lives.
or T C/ nR (V nb) constant
4
1 1
i.e. th of the initial activity
2 16
34. (b) From the analysis of P-V diagram we can 39. (a)
easily say that B is the correct option.

35. (b)

d
v = 39.6 km/hr = 11 m/s, t 1 = and
30
d 11 30
t2 =
330
Now t = t1 – t2 = 1
now t1 = 30 sec a mv
t2 = 29 sec. Now sin = ,R=
r qB
RT qBa cot
36. (a) v = v
M m
1 40. (c) Using theory of entropy it is evident that
v
M answer is (C). ( S)univ > 0
V0 MH 2 1
41. (a) Two types of complexes are possible for Ma3b3
VH M0 32 16
type i.e. Cis and Trans.
V0 1
42. (c) CN– is known as -acid ligand because its
VH 4
antibonding orbitals acts as acid.
37. (a) d = 0.1 mm, D = 1 m, = 600 nm
IP = 75% of maximum or IP = 3I0 2 10 8
43. (d) Bond order of O2 =1
Where I0 is the intensity of a single wave 2
now IP = 3I0
2 2 hc
= I0 I0 2 I0 I0 cos 44. (a) We know that E =

yd
cos cos , also x = 34
3 D 6.6 10 Js 3 108 m/s
E
1m
now x =
2 3 6
23
9 1.988 10 J
D 600 10 1
y 3
or y 1m
6d 6 0.1 10 45. (b) It is known as half life of reaction.

38. (d)
46. (c) F – Cl – F T-shaped.

m1 v0 0 m1 (0) m 2 ( ) F
VCOM , x iCOM
m1 m2 m1 m2
O CH3
also x COM x iCOM vCOM t
O C
Anhyd.
m2 m1 v0 t 47. (d) + CH3 – C – Cl
x COM AlCl3
m1 m2 m1 m2 (Acetophenone)
48. (a) Because Benzene Sulphonic acid being 68. (a) Smaller animals produce more waste material.
strongest acid. 69. (c) Concentration of ethylene gas es
49. (a) The most stable form is anti-form because of 70. (d) As blood flows in superficial blood vessels to
maximum distance in methyl groups. get rid of heat produced in the body.
71. (a) Because they use H2O in place of H2O.
50. (a) 234
Th 234
Pa 0
e 72. (a) Lactic acid is produced when muscles respire
90 91 1
in absence of oxygen/deficiency of oxygen.
51. (c) Entropy change will be positive due to dilution 75. (b) Bt Prototoxin alkalinepH Bt -toxin
and enthalpy remains same. 76. (d) Respiration produce heat.
77. (c) Nucleotides are joined by phosphodiester
52. (b) Rate of reaction increases with increase in
bonds.
temp. but Activation energy remains constant.
1 mm
79. (a) 9 3 ×106.
1 1 0.34 10
53. (c) In F.C.C., Z = 8 × ×6=4
8 2

1
In B.C.C., Z = 8 × +1=2 81. (c) Tr 1 nCr x n r
ar
8
n 2n 3r
54. (d) Reversing equation 2 and adding to equation n
n r 1 Cr
Tr 1 Cn x1/ 2 r
x 4
1, we get the desired equation:
2x1/ 4 2r

1 K1 9.1 10 12
9.1 T1, T2, T3 are in A.P
Hence K = K1 × 12
K2 K2 7.1 10 7.1
2 n C1 n
n
C2
C0
= 1.28 2 22

F n(n 1)
Cl n 1 n 8
56. (a) Cl P 8
Cl
F 16 3r
Integers r 0, 4, 8
4
57. (b) Structure I & IV have chiral carbon so can
exist as enantiomers, while other are achiral. 82. (b) 2x2 + 2x + 1 = 0
Hence roots are
58. (a) As rate of disappearance of NO2 is equal to
the rate of appearance of NO, so (X) is NO 1 i 1 i
x ,
and Z is NO2. 2 2
Now x satisfies (x + 1)n – r = 0
59. (c) (a) Option Anti-aromatic
n
1 i
(c) Option Aromatic 1 r 0
2
Br n
CCl4 1 i
60. (a) + Br2 r 0
dark 2
Br
n n
61. (a) Ribozyme = 235 RNA 1 1 i
r
62. (a) 2 2
63. (c) Surface area to volume ratio is large in case
of Bacteria. n 1 n
64. (a) 1 4
e r
65. (c) Blue and Red light regions show maximum 2
photosynthesis.
67. (a) Ruminants require gut microbes for the RHS = real
digestion of cellulose.
and LHS = real only if n is multiple of 4
86. (c)
n = 4000
4000
1 1
r 1000
2 4

x2 y2
83. (b) y = ax + b and 1
a b
Here slope = a for the line and y intercept is b
From Fig.1 for line, a < 0 , b > 0 hence the From Figure,
other fig. cannot be an ellipse ABC and AOP are similar
and from Fig.2, a > 0 , b < 0 hence the fig. is h H rH
H
a hyperbola r R R
1 2 r
84. (b) Now, V2 r (H h) r 2H 1
3 3 R
3
H 3 r
r
3 R
Now differentiating V2 w.r.t 'r' we get
dV2 3r 2 2R
2r 0 r
dr R 3
8 R2H
Maxm Possible Area V2 max
81
1
A 2 3R R 3R 2 R2H
2 and V1
85. (b) 3
V2 4
V1 27
87. (a) f (x) = 1 + f(x) f(x) = ex – 1
f (x) = 0 ex = 1 = 0
f (c) = 0 f (c–) >0f (c+) <0 x 0 is one solution
f (x) /2
g(x)
x 88. (b) I xdx ( x)dx
Now differentiating w.r.t 'x' we get 0 /2

xf '(x) 1 3 /2 2
g '(x) 2 (x )dx 2 x dx
x
3 /2
(c h)f '(c h) 1 2 2 2 2 2
g '(c) lim 2
0
h 0 (c h)
8 8 8 8 2
[ f’(c + h) < 0]
Fig (2) is correct 89. (d)

PA 2PB 3PC 0
a p 2 b p 3 c p 0 M 3 2 (f 4)
now
4 16 G (f 1)
p
a 2b 3c R e3
3
6
2
a 2b 3c 6p also T
1
a b b c c a
Area ABC 2 3 (f 4)
Now, Area APC 1 2
4T G(f 1)
a p p c c a
2
93. (b)
a 2b 3c
Now putting the value of p we
6
get

Area of ABC 3
Area of APC 1
90. (c) 6m + 2m+n 3w + 2n = 332
maximum possible value of m is 3
checking for m = 3, 2 and 1
we get m = 2, n = 3, w = 2 F(r) = kr
m2 + mn + n2 = (2)2 + 2(3) + (3)2 now Fnet on a particle is 2Fq cos 30º due to
=4 + 6 + 9 = 19 the other two charges
91. (d) Total distance 2kq 2 3
Fnet =
v02 2
2 v0
2
4 v0 a 2 2
r r upto 10 th terms
g g g 2 3
also r = a
v02 3 2
–h= 1 r2 r4 10 th term h
g a 3r replacing it in Fnet we get
also v0 2gh
2kq 2 3 kq 2
10 Fnet
2
1 r2 3r 2 3r 2
Total distance = 2h h
1 r2 this is balanced by F(r)

2h 1 r 20 1 q2
or total distance h F(r) = Fnet kr =
1 r2 4 0 3r 2
1/3
GM 2 r r3 3 q2
92. (a) V also T = or T = 2 when r
r v GM 12 0 k
GM 94. (c) Let the equivalent impedance of the circuit
v is replaced by
r be Z
now geff = g – 2 Re cos2 So Z = L + Z
2 ZX C
g Re cos0
now f = now Z =
g 2
Re cos60 Z XC

4g(f 1) 1
Z
solving we get Re = 2 or C
f 4) Z L
1
2 Z
GM Re C
f 1 (f 4)
R 2e 4
LC ( LC)2 4LC
on solving we get Z =
4GM f 1 2C
R e3
2 f 4
for Z to be purely inductive 5 5
Q = nCP T = n R T P1 V1 P2 V2
2 2 2 2 2
L C2 4LC 0 or
LC For AC
1
95. (a) QAC = P1 P2 V2 V1 nC v T
2
3
now nCV T = (P V – P1V1)
2 2 2
now
1
V V P P
1 1 R =
W 2 2 1 2 1
now V1
V1 R 1 Q 1 3
P1 P2 V2 V1 PV PV
2 2 2 2 1 1
1 1 using formula for heat we can calculate heat
now V 2R V1 R
f absorbed in AC.
R 99. (b) Using law of conservation of mechanical
replace V1 by and solving for Vf energy
1
Initial K.E. = Final P.E.
R( 2)
we get Vf =
2( 1) 1 1 kq 2
mv 2 mv2
2 2 r
First image is real and second is virtual.
q2
1 2 x r
96. (a) V(x) = kx V0 cos 4 2
2 a 0 mv

100.(c) N0 – initial nucleon


dV x 1
E kx V0 sin at t = 0, N0
dx a a
since x < < a Addition is at a constant rate

x x V0 dN
sin or E k x ( N – C) = –
a a a2 dt
k N
This resembles F = –kx dN
dt
0 N0
N C
m ma 2
T=2 2 Integrating we get
k ka 2 v0
t
C e
N N0 C
ma 2
T 2
ka 2 v0 C t t
N 1 e N0 e
97. (c) Direct formula is to be used
nR 2.52 1000
C CV 101.(c) N 0.4
1 126
100
3 5R
2
98. (b) CV = R, CP = CV + R =
2 2 Now N1V1 = N2V2
f=3
0.4 × 10 = N2 × 500
1
W V2 V1 P2 P1 N2 = 0.08
2
For BA
2
OH OH 105. (d) K SP Mg 2 OH

HBr 12
2
102. (a) + H2O 10 (0.01) OH
CH2Br
CH2OH OR OH 10 5

OR P OH 5 and P H 9
OH OH
106. (c) In F.C.C., 4 2a
HBr
+ CH3Br a
OH r 141.4
OCH3 2 2
and a 2 2 141.4
103.(a) CH3 CH3 CH3 2 1.414 141.4 400 Pm
|
C CH3 – C – Br Now volume = a3 = (400)3
|| + Br2 | = 64 × 106
C CH3 – C – Br
| = 6.4 × 107
CH3 CH3 CH3
108. (a) 2B 3H2 B2 H6 H ?
Meso-Compound
H 1273 ( 286) 3 44 3 ( 2035)
= 36 kJ
CH3
CH3 | 110. (d) Tf Kf m
C = CH2 + Br2 Br – C – CH2Br
| 8 1000
C2H5 C2H5 0.925 1.85
M 92
(Chiral Center)
(d & ) 1.85 8000
M 174
0.925 92
104.(d) At equilibrium, Closest answer is 160.
113.(d) Depends on AT or GC rich regions.
G 0 114.(b) Only first two nucleotide are required for
identification of an amino acid.
H 7.5 1000 116.(d) E + S P1 + P2 + E.
T 300
S 25 118.(c) 2 × 0.25 0.5
9. Foot of the perpendicular drawn from origin to the
plane x + y + z = 1 is
1. Total number of positive integral solutions of x1 + x2
1 1 1 1 1 1
+ x3 = 24 and x 21 x 22 x 23 is equal to (a) , , (b) , ,
3 3 3 2 4 4
(a) 1 (b) 2
(c) 4 (d) None of these 1 1 1 1 1 1
(c) , , (d) , ,
4 2 4 4 4 2
2. Let a = ˆi + ˆj, b = 2iˆ - k.
ˆ Then the position vector of
10. A bag A contains 2 white and 3 red balls and bag B
the point of intersection of the lines r × a = b × a and contains 4 white and 5 red balls. One ball is drawn
r × b = a × b is at random from a randomly chosen bag and is found
to be red. The probability that it was drawn from B is
(a) 3iˆ + ˆj – kˆ (b) 3iˆ – ˆj + kˆ 5
5
(a) (b)
(c) 3iˆ – ˆj – kˆ (d) None of these 14 16
5 25
3. If f(x + f(y)) = f(x) + y x, y R and f(0) = 1, then (c) (d)
value of f(7) is 18 52
(a) 1 (b) 7 dy
11. If y = sin –1 x 1 x x 1 x2 and
(c) 6 (d) 8 dx
4. The exhaustive set of values of ‘a’ such that 1
= p, then p =
x2 + ax + sin–1 (x2 – 4x + 5) + cos–1 (x2 – 4x + 5) = 0 2 x(1 x)
has atleast one solution is
1
(a) 0 (b)
(a) –2 – (b) , 2 1 x
4 4 1
(c) sin–1 x (d)
1 x2
(c) – , –2– (d) 2 ,
4 4 12. The minimum value of the function defined by
f(x) = max(x, x + 1, 2 – x) is
5. The combined equation of straight lines that can be
obtained by reflecting the lines y = |x – 2| in the 1
y-axis, is (a) 0 (b)
2
(a) y2 + x2 + 4x + 4 = 0 (b) y2 + x2 – 4x + 4 = 0
3
(c) y2 – x2 + 4x – 4 = 0 (d) y2 – x2 – 4x – 4 = 0 (c) 1 (d)
2
6. If the circle x2 + y2 + 2ax + 2by + c = 0 passes
through exactly three quadrants and does not pass 13. The area of the region (s) enclosed by the curves
through origin, then y = x2 and y | x | is
(a) c > 0, a2 > b2 (b) c > 0, a2 + b2 > 2c
(c) c(a – c)(b – c) > 0 (d) c > 0, a2 > c, b2 > c
2 2 1 2
(a) (b)
7. The coefficient of x7 in the expansion of (1 – x4)(1 + x)9 3 3
is equal to 1
(a) –48 (b) 48 (c) (d) 1
6
(c) 120 (d) –120
14. If coefficients of the equation ax 2 + bx + c = 0,
8. Lim
x 0
{(1 + x)2/x}, where {,} denotes the fractional part a 0 are real and roots of the equation are non-real
of x, is equal to complex and a + c + b < 0, then
(a) e2 – 7 (b) e2 – 8 (a) 4a + c > 2b (b) 4a + c < 2b
(c) e2 – 9 (d) e2 – 10 (c) 4a + c = 2b (d) None of these
n r
21. An open pipe is suddenly closed at one end with the
15. If n k
Cr x
Cy then result that the frequency of third harmonic of the
k 1 closed pipe is found to be higher by 100 Hz than the
(a) x = n + 1; y = r fundamental frequency of the open pipe. The
fundamental frequency of the open pipe is:
(b) x = n; y = r + 1
(c) x = n; y = r (a) 200 Hz (b) 300 Hz
(d) x = n + 1; y = r + 1 (c) 240 Hz (d) 480 Hz
22. A sphere of mass M and radius R2 has a concentric
cavity of radius R1 as shown in figure. The force F
16. When an electron in the hydrogen atom in ground exerted by the sphere on a particle of mass m located
state absorbs a photon of energy 12.1 eV, its angular at a distance r from the centre of sphere varies as
momentum:
0 r :
(a) decreases by 2.11 × 10–34 J–s
(b) decreases by 1.055 × 10–34 J–s
(c) increases by 2.11 × 10–34 J–s
(d) increases by 1.055 × 10–34 J–s
R1
17. A particle of specific charge (charge/mass) starts R2
moving from the origin under the action of an electric
field E = E 0 i and magnetic field B = B0 k. Its velocity

at (x0, 0, 0) is 4iˆ + 3jˆ . The value of x0 is:

13 E0 16 B0
(a) (b) F F
2 B0 E0

25 5
(c) 2 E (d) 2B
0 0
(a) (b)
18. A hollow sphere of radius 2R is charged to V volts
and another smaller sphere of radius R is charged to r r
V F F
volts. Now the smaller sphere is placed inside
2
the bigger sphere without changing the net charge
on each sphere. The potential difference between
the two spheres would be:
(c) (d)
3V V
(a) (b)
2 4 r r

V 23. A uniform flexible chain of mass m and length 2l


(c) (d) V hangs in equilibrium over a smooth horizontal pin of
2
19. In Young’s double slit experiment the y co-ordinates negligible diameter. One end of the chain is given a
of central maxima and 10th maxima are 2 cm and 5 small vertical displacement so that the chain slips
cm respectively. When the YDSE appratus is over the pin. The speed of chain when it leaves pin
immersed in a liquid of refractive index 1.5 the is:
corresponding y coordinates will be:
(a) 2gl (b) gl
(a) 2 cm, 7.5 cm (b) 3 cm, 6 cm
4 10 (c) 4gl (d) 3gl
(c) 2 cm, 4 cm (d) cm, cm 24. A string of mass 0.2 kg/m and length l = 0.6 m is
3 3
20. A spherical black body with a radius of 12 cm fixed at both ends and stretched such that it has a
radiates 450 W power at 500 K. If the radius were tension of 80 N. The string vibrates in 3 segments
halved and the temperature doubled, the power with maximum amplitude of 0.5 cm. The maximum
radiated in watt would be: transverse velocity amplitude is:
(a) 225 (b) 450 (a) 1.57 m/s (b) 6.28 m/s
(c) 900 (d) 1800 (c) 3.14 m/s (d) 9.42 m/s
25. The potential energy of a particle of mass 1 kg is, p
(a) (b) 2p
U = 10 + (x – 2)2. Here, U is in joule and x in metres. 2
On the positive x-axis particle travels upto x = + 6 m.
p 2p
Choose the wrong statement: (c) (d)
2
(a) On negative x-axis particle travels upto x = – 2m
(b) The maximum kinetic energy of the particle is
16 J 31. Which one of the following has S° greater than zero?

(c) The period of oscillation of the particle is 2 (a) CaO(s) + CO2(g) CaCO3(s)
seconds (b) NaCl(aq) NaCl(s)
(d) None of the above
(c) NaNO3(s) Na+(aq) + NO3 (aq)
26. Frequency of a particle executing SHM is 10 Hz.
The particle is suspended from a vertical spring. At (d) N2(g) + 3H2(g) 2NH3(g)
the highest point of its oscillation the spring is 32. The reaction, X Product follows first order kinetics.
unstretched. Maximum speed of the particle is : In 40 minutes the concentration of X changes from
(g = 10 m /s2) 0.1 M to 0.025 M. Then the rate of reaction when
(a) 2 m / s (b) m/s concentration of X is 0.01 M
(a) 1.73 × 10–4 M min–1 (b) 3.47 × 10–5 M min–1
1 1
(c) m/s (d) m/s (c) 3.47 × 10–4 M min–1 (d) 1.73 × 10–5 M min–1
2
33. Aniline when diazotized and then treated with
27. A ball suspended by a thread swings in a vertical
plane so that its acceleration in the extreme position dimethyl aniline gives a coloured product. Its structure
would be
and lowest position are equal. The angle of thread
deflection in the extreme position will be:
(a) (CH3)2N N=N
(a) tan–1 (2) (b) tan 1 2

1 1
(c) tan 1 (d) 2 tan 1 (b) (CH3)2N NH
2 2
28. A particle moves in x-y plane. The position vector of
(c) CH3NH N=N NHCH3
particle at any time t is x 2t i 2t 2 j m. The
rate of change of at t = 2s. (where is the angle
which its velocity vector makes with positive x-axis) (d) CH3 N=N NH2
is:
34. The correct order of basicities of following compounds
2 1
(a) rad / s (b) rad / s is:
17 14
NH
4 6 CH3 – C
(c) rad / s (d) rad / s CH 3CH 2NH 2
7 5 NH2
(2)
29. A point charge q is placed at a distance of r from the (1)
centre of an uncharged conducting sphere of radius O
R(< r). The potential at any point on the sphere is:
(CH3)2NH CH3 – C – NH2
1q
(a) zero (b) . (3)
4 0 r (4)
(a) 2 > 1 > 3 > 4 (b) 1 > 3 > 2 > 4
1 qR 1 qr 2
(c) . (d) . (c) 3 > 1 > 2 > 4 (d) 1 > 2 > 3 > 4
4 0 r2 4 0 R
35. If the threshold wavelength ( 0) for ejection of electron
30. A charged particle enters a uniform magnetic field from the metal is 330 nm, then the work function for
with velocity vector at an angle of 45° with the the photoelectric emission is
magnetic field. The pitch of the helical path followed
(a) 1.2 × 10–20 J (b) 1.2 × 10–26 J
by the particle is P. The radius of the helix will be:
(c) 6 × 10 J –9
(d) 6 × 10–19 J
36. Which curve does not confirm Boyle's law? 41. During transformation of ac X to bd Y by and -decay,,
the number of -particles emitted are
a b a b
(a) P (a) (b) d c
4 2
a b
(c) d c (d) 2c – d + a – b
V 2
42. How many unit cells are present in a cube shaped
ideal crystal of NaCl of mass 1.00 g?
(b) P (Atomic masses : Na = 23, Cl = 35.5)
(a) 1.28 × 1021 (b) 1.71 × 1021
(c) 5.14 × 1021 (d) 2.57 × 1021
V
43. At 300 K, 36 g of glucose present per litre in its
solution has an osmotic pressure of 4.98 bar. If the
(c) Log P osmotic pressure of a urea solution is 1.52 bar at
the same temperature, its concentration is
(a) 1.6 M (b) 0.61 M
Log V (c) 0.061 M (d) None of these
44. The root mean square velocity of one mole of a
monoatomic gas having molar mass M is Urms. The
(d) Log V relation between the average kinetic energy (E) of
gas and Urms is

3E 2E
Log P (a) Urms (b) Urms
2M 3M
37. Element "M" reacts with oxygen to form the oxide
M2O3. If 0.359 g of M react with oxygen to give 0.559 2E E
(c) Urms (d) Urms
g of the oxide, the atomic weight of M is M 3M
(a) 34 (b) 43 45. The dissociation constants of two weak acids are K1 and
(c) 86 (d) 68 K2. The relative strength of the two acids is given by
K1 K1 K2
38. Among NO3 , AsO33 , CO 32 , ClO3 , SO32 , and BO33 (a) (b)
K2 K2 K1
ions, the non-planar species are
1/ 2 3/2
(a) AsO33 , SO 32 and ClO3 K1 K1
(c) (d)
K2 K2
(b) NO3 , SO23 and ClO3
(c) CO23 , AsO33 and SO32
46. Biodiversity of a geographical region represents:
(d) NO3 , CO32 and ClO3
(a) Genetic diversity present in the dominant species
39. When BaCrO4 is used as oxidizing agent in acidic
of the region
medium, the equivalent weight of barium chromate
is [At. wt. of Ba = 137.34; Cr = 52.0; O = 16] (b) Species endemic to the region
(a) 137.34 (b) 84.45 (c) Endangered species found in the region
(c) 114.45 (d) 68.67 (d) The diversity in the organisms living in the region
40. The following compounds have been arranged in the 47. Bryophytes are amphibians because:
order of increasing thermal stabilities. Identify the (a) They require a layer of water for carrying out
correct order. sexual reproduction
K2CO3(I), MgCO3(II), CaCO3(III), BeCO3(IV) (b) They occur in damp places
(a) I < II < III < IV (b) IV < II < III < I (c) They are mostly aquatic
(c) IV < II < I < III (d) II < IV < IIII < I
(d) All the above
48. An alga, very rich in protein, is: 56. The term ‘Test-Tube Baby’ implies that
(a) Chlorella (b) Nostoc (a) fertilisation of ovum takes place in the uterus but
(c) Spirogyra (d) Ulothrix develops in the test-tube
49. Homeostasis is: (b) fertilisation of ovum takes place in the test-tube,
but it develops in test-tube itself
(a) Tendency to charge with change in environment
(c) fertilisation of ovum takes place in the test-tube,
(b) Tendency to resist change but it develops in the uterus
(c) Disturbance in regulatory control (d) fertilisation of ovum takes place in the uterus and
(d) Plants and animal extracts used in homeopathy embryo develops in the uterus
50. A larval stage occurs in the life history of all members 57. Which of the following hormones are secreted in large
of the group: quantities during pregnancy in women?
(a) Frog, lizard and cockroach (a) hCG, progesterone, oestradiol and FSH
(b) Ascaris, housefly and frog (b) hCG, hPL, progesterone, oestrogen and LH
(c) Housefly, earthworm and mosquito (c) LH, oestrogen and oestradiol
(d) Butterfly, frog and mosquito (d) hCG and hPL
51. How many ATP molecules produced by Aerobic 58. An organic substance that can withstand
oxidation of one molecule of glucose: environmental extremes and cannot be degraded by
any enzyme is
(a) 2 (b) 4
(a) cuticle (b) sporopollenin
(c) 38 (d) 34
(c) lignin (d) cellulose
52. Brunner’s glands occur in:
59. What would be the number of chromosomes of the
(a) Submucosa of duodenum aleurone cells of a plant with 42 chromosomes in its
(b) Submucosa of stomach root tip cells?
(c) Mucosa of oesophagus (a) 63 (b) 84
(d) Mucosa of ileum (c) 21 (d) 42
53. Which of the following induces morphogenesis in 60. Seminal plasma in human males is rich in
tissue culture: (a) fructose and calcium
(a) Gibberellin (b) glucose and calcium
(b) Cytokinin (c) DNA and testosterone
(c) IAA (d) ribose and potassium
(d) Ethylene
54. Which one of the following statements about human
sperm is correct?
(a) Acrosome has a conical pointed structure used 61. An ellipse has eccentricity 1/2 and a focus at the
for piercing and penetrating the egg, resulting in point P 1/ 2, 1 . One of its directrix is the common
fertilisation tangent near to the point P, to the circle x 2 y2 1
(b) The sperm lysins in the acrosome dissolve the and the hyperbola x 2
y 2
1 , the equation of the
egg envelope facilitating fertilisation ellipse is
(c) Acrosome serves as a sensory structure leading
(a) 3x 2 4y2 6x 8y 4 0
the sperm towards the ovum 2
(b) 3x 4y2 2x 8y 4 0
(d) Acrosome serves no particular function
55. The permissible use of the technique amniocentesis (c) 4x 2 3y 2 8x 6y 4 0
is for (d) 4x 2
3y 2
8x 2y 4 0
(a) detecting sex of the unborn foetus 3 2
x dx yx dy
(b) artificial insemination 62. The solution of ydx xdy is
2
x y2
(c) transfer of embryo into the uterus of a surrogate
mother (a) x2 y2 Cx (b) x2 y2 y x C
(d) detecting any genetic abnormality 2
(c) x2 y2 y x2 C (d) x2 y2 xy 2 C
2 3 3b b 21
63. If c 0 , then the equation z 2iz 2c 1 i 0,
(a) (b)
(z is complex) has 8 8
(a) infinitely many solutions if c 2 1 b 19
(c) b (d)
8
(b) has unique solution if c 2 1
68. A beam of light has three wavelengths 310 nm, 400
(c) finite number of solutions if c 2 1 nm and 800 nm with a total intensity of 3.6×10–3.
W -m –2 equally distributed amongst the three
(d) no solutions if c 2 1
wavelengths. The beam falls normally on an area 1.0
2
64. On the ellipse 4x 9y 2 1 , the points at which cm2 of a metallic surface of work function 2.8 eV.
the tangents are parallel to the line 8x 9y are Assume that there is no loss of energy by reflection
and that each energetically capable photon ejects
2 1 2 1 one electron. Calculate the number of photoelectrons
(a) , (b) ,
5 5 5 5 liberated in two seconds.
(a) 3.2 1011 (b) 4.3 1011
2 1 2 1
(c) , (d) , (c) 1.6 1011 (d) 8.6 1011
5 5 5 5
65. The first two terms of an infinitely decreasing GP are 69. The position (x) of a particle is plotted versus time (t)
for a particle performing simple harmonic motion of
2
3 and . Then the amplitude 4mm and period 2 seconds. For each of
( 3 1) the graphs, their equation is shown in the options,
( 3 1) choose the correct option:
(a) common ratio is
3 Fig-1 Fig-2
X X
(b) sum to infinity of the GP is 3 3
4 4
1
(c) common ratio is 2 2
3
(d) sum of infinity is 3 t t

-4 -4

66. Two balls of equal masses are projected upward Fig-3 Fig-4
simultaneously, one from the ground with speed 50 X X
m/s and other from a 40 m high tower with initial 4 4
speed 30 m/s. Choose the correct velocity(V) vs
time(t) graph of their centre of mass. [Take
t t
acceleration due to gravity=10m/s2] 2 2

V V -4 -4
(m/s) 40 (m/s) 40
2 4
(a) t(s)
(b) t(s) (a) Fig-1: x 4 sin t

(b) Fig-2: x 4 sin t


V
6
V
(m/s)
(m/s) 5
(c)
40
(d)
40
(c) Fig-3: x 4 sin 2 t
t(s) t(s) 6
11
(d) Fig-4: x 4 sin t
67. A beam of parallel rays of width b propagating in 6
glass (refractive index 1.5), strikes the plane glass- 70. In a Young’s double slit experiment, the central fringe
air boundary at an angle (= 53°), as shown in figure. of the interference pattern produced by the light of
The beam width after it goes over to air through this wavelength 6000 Å is found to shift to the position of
face will be: 4th bright fringe after a glass sheet of refractive index
1.5 is introduced. Find the thickness of glass sheet.
(a) 4.8 m (b) 1.2 m
b Glass
(c) 3.6 m (d) 6 m
Air
CH2 +
71. Which of the following compound, on reaction with 75. H
excess of RMgX, gives three moles of alkane (RH) ? X
OH
(a) HC C CH COOH X would be
|
OH
CH2
(b) CH3 C C CH COOH (a)
|
OH O
(c) HC C CH COOH
|
OCH3 (b)

(d) CH2 O Me
CH CH COOH
|
OH
(c) CH2CH3
72.
O

A B C (d)
O CH3
V = 2 lit V = 1.5 lit V = 2.5 lit
P = 1 atm P = 2 atm P = 1.5 atm
What will be the final pressure if we open the stop
cork. (Assume that the volume of connecting tubes 76. Study the cycle shown below and select the option
which gives correct words for all the four blanks A,
are negligible)
B, C and D.
(a) 1.458 (b) 2.507
(c) 3.913 (d) None Atmospheric Nitrogen

73. For a certain first order gas phase reaction A(g)


B(g) + C(g), the time required for half the initial Biological Industrial Electrical A
amount to be composed is 10.4 minutes. If the initial N2 Fixation N2 Fixation N2 Fixation
pressures of A is 400 mm of Hg, the time required to
reduce the partial pressure of A to 50 mm of Hg NH3 NO2 NO3
is:(log2 = 0.3010)
(a) 15.6 min (b) 20.8 min
B (Uptake)
(c) 31.2 min (d) 41.6 min
74. Zn(s) + Cu (aq.) Cu(s) + Zn2+ (aq.)
2+
Decyaing Biomass C
2
Zn
Reaction quotient Q , variation of Ecell with
Cu2
log Q is of the type with OA = 1.10 volt. D

Ecell = 1.1591 volt when : A B C D


a. Nitrification Ammonification Animals Plants
A b. Denitrification Ammonification Plants Animals
c. Nitrification Denitrification Animals Plants
d. Denitrification Nitrification Plants Animals
Ecell 77. The figure below shows the structure of a
O log Q mitochondrion with its four parts labeled (A), (B), (C)
and (D). Select the part correctly matched with its
(a) [Cu2+]/[Zn2+] = 0.01 function.
(b) [Zn2+] / [Cu2+] = 0.01
(c) [Zn2+]/[Cu2+] = 0.1
(d) [Zn2+]/[Cu2+] = 1
(a) Part (C): Cristae – possess single circular DNA (c) Felis tigris-the Indian tiger, well protected in Gir
molecule and ribosomes Forests.
(b) Part (A): Matrix – major site for respiratory chain (d) E.coli-full name Entamoeba coli, a commonly
enzymes occurring bacterium in human intestine.
(c) Part (D): Outer membrane – gives rise to inner 80. Given below are four statements, a - d, regarding
membrane by splitting human blood circulatory system
(d) Part (B): Inner membrane – forms in foldings (A) Arteries are thick walled and have narrow lumen
called cristae as compared to veins
78. Many blue-green algae occur in thermal springs (hot (B) Angina is acute chest pain when the blood
water springs). The temperature tolerance of these circulation to brain is reduced
algae have been attributed to their: (C) Persons with blood group AB can donate blood
(a) Mitochondrial structure to any person with any blood group under ABO
(b) Importance of homopolar bonds in their proteins system
(c) Cell wall structure (D) Calcium ions play a very important role in blood
clotting
(d) Moderm cell organization
Which two of the above statements are correct
79. Which of the following organisms is scientifically
correctly named, correctly printed according to (a) A and B
International Rules of Nomenclature and correctly (b) A and D
described? (c) B and C
(a) Musca domestica-the common house lizard, a (d) C and D
reptile
(b) Plasmodium falciparum-a protozoan pathogen
causing the most serious type of malaria

1. (b) 2. (a) 3. (a) 4. (a) 5. (d) 6. (d) 7. (a) 8. (a) 9. (d) 10. (a)
11. (d) 12. (d) 13. (b) 14. (b) 15. (b) 16. (c) 17. (c) 18. (b) 19. (c) 20. (d)
21. (a) 22. (b) 23. (b) 24. (a) 25. (d) 26. (d) 27. (d) 28. (a) 29. (b) 30. (c)
31. (c) 32. (c) 33. (a) 34. (b) 35. (d) 36. (a) 37. (b) 38. (a) 39. (b) 40. (b)
41. (c) 42. (d) 43. (c) 44. (c) 45. (c) 46. (d) 47. (a) 48. (a) 49. (b) 50. (d)
51. (c) 52. (a) 53. (b) 54. (b) 55. (d) 56. (c) 57. (b) 58. (b) 59. (a) 60. (a)
61. (b) 62. (b) 63. (b, d) 64. (b, d) 65. (a, d) 66. (b) 67. (d) 68. (d) 69. (d) 70. (a)
71. (a) 72. (a) 73. (c) 74. (b) 75. (d) 76. (b) 77. (d) 78. (c) 79. (b) 80. (d)
1. (b) x 21 x 22 x 32 and x1 + x2 + x3 = 24 6. (d) It is possible if origin lies outside the circle and
These given informations give us the idea that x1, circle intersects both co-ordinate axis in two
x2 and x3 can be treated as the sides of a right distinct points.
angled triangle whose hypotenuse is x 3 and c > 0, a2 > c, b2 > c
perimeter is 24. The obvious solution is x1 = 8, x2 7. (a) (1 – x4) (1 + x)9 = (1 + x)9 – x4(1 + x)9
= 6, x3 = 10 or x1 = 6, x2 = 8, x3 = 10.
Thus, required coefficient
Thus there are two sets of positive integral
= 5C7 – 9C3 = 9C2 – 9C3 = –48
solutions.
8. (a) {(1 + x)2/x} = (1 + x)2/x – [(1 + x)2/x]
2. (a) r a b a
Now, Lim
x 0
(1 + x)2/x = e2
r b a 0
Lim {(1 + x)2/x} = e2 – {e2} = e2 – 7
x 0

r b 1 a 2iˆ kˆ 1
ˆi ˆj
9. (d) Let E1 be the event that the ball is drawn from
bag A, E2 the event that it is drawn from bag B
r b a b r a 2 b and E that the ball is red.
ˆi ˆj 2iˆ kˆ E2
2
We have to find P .
E
At the point of intersection;
Since both the bags are equally likely to be
2iˆ kˆ 1
ˆi ˆj ˆi ˆj
2 2iˆ kˆ selected,

2+ =1+2 + = 1, =1 1
1 2 1 2 we have P E1 P(E 2 ) .
= 1, =1 2
1 2

Thus P.V. of point of intersection is E 3 E 5


Also P E 5 and P
E2 9
1
i 2iˆ kˆ ˆi ˆj 3iˆ ˆj kˆ
Hence by Baye's theorem, we have
3. (a) f(x + f(y)) = f(x) + y, f(0) = 1 E2 P(E2 )P(E / E2 )
P
Putting y = 0, we get E P(E1 )P(E / E1 ) P(E2 )P(E / E2 )
f(x + f(0)) = f(x) + 0 f(x + 1) = f(x) x R
1 5
Thus f(x) is periodic with 1 as one of it’s period. .
5 9 25
=
Thus f(7) = f(6) = f(5) = .......= f(1) = f(0) = 1 1 3 1 5 52
. .
4. (a) We must have, –1 x2 – 4x + 5 1 2 5 2 9
If x2 – 4x + 5 1, then x2 – 4x + 4 0 10. (a) Direction ratios of the normal vector of the plane
(x – 2)2 0 x=2 are 1, 1, 1 respectively. Let P be the foot of the
altitude drawn from the origin to the plane,
If x2 + 4x + 5 –1, then x2 – 4x + 6 0 x R. equation of the line OP will be
Thus x = 2 is the only solution.
x y z
r say .
1 1 1
4 2a 0, a 2
2 4 Any point on this line can be taken as (r, r, r).
5. (d) If we reflect y = |x – 2| in y-axis, it will become Putting this point in the equation of the plane, we
y = |–x –2| = |x + 2| get

Hence, reflected lines are y = x + 2, y = –x – 2 1


r .
Their combined equation is (y – x – 2)(y + x + 2) 3
=0 1 1 1
Thus required coordinates are , ,
y2 – x2 – 4x – 4 = 0 3 3 3
11. (d) y Applying work-energy theorem
sin–1(x 1 x x 1 x2 )
1 25
dy 1 qE0 x 0 mv 2 m;
2 2
dx 2 x(1 x) p
25m 25
x0
y = sin (x) + sin
–1 –1 ( x) 2qE0 2 E0
dy 1 1 V 4 0RV
dx 18. (b) Charge on smaller sphere q C
1 x 2
2 x 1 x 2 2
13. (b) y=x
2 Potential difference
y = |x| q 1 1 VR 1 V
(1, 0) = 4 R 2R 2 2R 4
0

19. (c) Fringe width . Therefore, and hence w


will decrease 1.5 times when immersed in liquid.
The distance between central maxima and 10th
1 2
A 2 ( |x| x 2 )dx maxima is 3 cm in vacuum. When immersed in
0 3
liquid it will reduce to 2 cm. Position of central
14. (b) ax2 + bx + c = 0, a 0, maxima will not change while 10th maxima will
be obtained at y = 4 cm.
Given
20. (d) Power radiated (surface area) (T)4. The radius
a + b + c < 0& D < 0
f(x) < 0 1
x R is halved, hence, surface area will become
4
f(–2) < 0
times. Temperature is doubled, therefore, T 4
4a – 2b + c < 0 becomes 16 times.
4a + c < 2b
1
New power = (450) (16) = 1800 W..
n r 4
n k x
15. (b) Cr Cy
k 1
21. (a) Length of the organ pipe is same in both the
cases. Fundamental frequency of open pipe is
L.H.S = n–1Cr + n – 2Cr + n–3Cr + ... + rCr
v
= rCr + r+1Cr +...n–2Cr + n–1Cr f1 and frequency of third harmonic of closed
2l
= r+1
Cr+1 + r+1
Cr +...+ r+2Cr + ...+ n–1
Cr v
pipe will be f2 3
= r+2
Cr+1 + r+2
Cr +...+ n–1
Cr 4l
= r+2
Cr+1 +...+ n–1
Cr Given that f2 f1 100 or f2 f1 100
= n–1
Cr+1 + n–1
Cr
3 v 1 v
= Cr+1 = Cy
n x
x = n, y = r + 1 or 100
4 l 2 l
16. (c) After absorbing a photon of energy 12.1 eV
electron jumps from ground state (n = 1) to v
100 Hz ;
second excited state (n = 3). Therefore change 4l
in angular momentum v
or f1 200 Hz
h h h 2l
L = L3 – L 1 3
2 2 Therefore, fundamental frequency of the open pipe
is 200 Hz.
6.6 10 34
J s 22. (b) The whole space can be divided into three regions
3.14
= 2.11 × 10–34 J-s (i) 0 < r < R1 F(r) = 0

17. (c) Let q be the charge and m the mass of the 4 R13
particle. At (x0, 0, 0) speed of the particle is 5 (ii) R1 < r < R2 F r G m r
3 r2
units.
4 R3 R3 mg
(iii) R2 < r < F r G m 2 2 1 26. (d) Mean position of the particle is distance
3 r k
below the unstretched position of spring.
Here, is the density of material of the sphere. Therefore, amplitude of oscillation is
23. (b) Decrease in potential energy = increase in kinetic mg
energy A
k
1 mgl 1 mgl k
K mg l or mv
2
; 2 f 20 (f = 10 Hz)
2 2 2 2 m
V gl m 1
3 k 400 2
24. (a) l Therefore, the maximum speed of particle will be:
2
2l 2 0.6 g 1
m 0.4m v max A 2
20 m/s
3 3 400 2
27. (d) h = l(1 – cos )
T 80
v 20m / s
0.2 vB2 2gh 2gl 1 cos

N N N N aA aB
v B2
g sin 2g 1 cos
l
3
l=
2 or 2sin cos 4 sin2
2 2 2
v 20
f Hz 50 Hz 1
0.4 tan
2 2
Vmax amax
1 1
tan
0.5 10 2
m 2 50 m / s m/ s 2 2
2
1 1
= 1.57 m/s 2 tan
2
25. (d) U(6 m) = 10 + (6 – 2)2 = 26 J
dx
U(–2 m) = 10 + (– 2 – 2)2 = 26 J 28. (a) x = 2t vx 2
dt
As U(6 m) = U(– 2 m), dy
vy 4t
On negative x-axis particle travels upto y = 2t2 dt
x =–2m vy 4t
tan 2t
Mean position of the particle is x = 2 m vx 2
U(2 m) = 10 J
Differentiating with respect to time we get,
K(2 m) = (26 – 10)J = 16J = Kmax d
sec 2 2
Put x – 2 =X dt
U = 10 + X2
d
or 1 tan2 2
dU dt
F 2X
dX
d
or 1 4t 2 2
F dt
a 2X (m = 1 kg)
m
d 2
2 or
2 dt 1 4t 2
T
d d 2 2
T 2 at t 2s is 2
rad / s
dt dt 1 4 2 17
29. (b) Since, potential V is same for all points of the Weight of the element M = 0.359 g
sphere. Therefore, we can calculate its value at weight of oxygen = 0.200 g
the centre of the sphere. 0.359 8
1 q Eq. wt. of element M = 14.36
V . V 0.2
4 0 r Atomic weight of M = Eq. wt × valency
V = potential at centre due to induced charges = 0 = 14.36 × 3 = 43.08 43
(because net induced charge will be zero) 3 2
38. (a) AsO – tetrahedral; SO
3 3
– tetrahedral
1 q
V . . ClO 3 – tetrahedral; CO 32 – trigonal planar
4 0 r

2 m 2 m NO3 – trigonal planar; BO33 – trigonal planar


30. (c) p (v cos 45 ) (v sin 45 )
Bq Bq 39. (b) In acidic medium, CrO 24 is converted into
mv sin 45 P [Cr2O7] , which undergoes reduction to Cr3+ ion
2–

= radius of helix
Bq 2 2CrO24 2H Cr2O72 H2 O
2.303 0.1
32. (c) k log CrO24 14H 6e 2Cr 3 7H2O
40 min 0.025
Thus adding both equations,
2.303
log 4 2CrO24 16H 6e 2Cr 3 8H2 O
40 min
Therefore, 2BaCrO4 = 6e– or BaCrO4 = 3e–
2.303 0.6021
min–1 equivalent weight of Barium chromate
40
2.303 0.6021 253.34
Rate = k[X] 10 2
Mmin –1 = 84.45
40 3
a
2.303 6.021 41. (c) c X b Y x24He y 1
10 4 Mmin–1 d

4 Here x = -particles emitted and y = -particles


= 3.47 × 10–4 M min–1 emitted.
a = b + 4x
33. (a) NH2
NaNO2
HCl, 273 K C6H5N2Cl a b
x=
4
c = d + 2x – y
N(CH3)2
y = d – c + 2x
C6H5–N = N N(CH3)2
a b
y=d–c+
2
35. (d) Work function = Energy required to just dislodge
a b
the electron in the ground state y=d+ –c
2
hc
hv 0 42. (d) Mass of one unit cell (m) = Volume × density
0
MZ MZ
6.63 10 34 3 108 = a3 × d = a3 × N a3 N0
0
330 10 9 58.5 4
m g
19.89 10 26
6.02 1023
= 6 × 10–19 J
3.3 10 7 1
Number of unit cells in 1 g =
36. (a) According to Boyle’s Law m
1 cons tan t 6.02 1023
P at constant temperature; P
2.57 1021
V V 58.4 4
37. (b) Since the formula of the oxide is M2O3 the valency 43. (c) 1V = n1RT; 2V = n2RT
of the element M is 3 1 n1 36 1
; n = mol
Weight of oxide = 0.559 g 2 n2 1 180 5
1
= 4.98 bar; 2
= 1.52 bar Therefore
4.98 1/ 5 x2 y2 2y i 2c 2x 2c 0
1.52 n2
x2 y2 2y 2c 0 ... (I)
1.52 and 2c 2x 0 or x c ... (II)
n2 0.061M
4.98 5
Substituting x = c in Eq. (I), we get that
44. (c) For 1 mole of the gas
c2 y2 2y 2c 0 ... (III)
1 2
PV = Mu 2
3 Equation (III) has solutions if 4 4 c 2c 0,
3PV 3RT that is 1 c 2
2c 0 . Therefore
or Ur.m.s. =
M M 2
c 1 2 or 2 c 1 2
3
Kinetic energy, E RT
2 It is given that c 0 . Therefore 0 c 2 1.
or 3 RT = 2E
(i) If c 2 1 , then
2E
Ur.m.s. = .
M z c 1 1 2c c 2 i
45. (c) The dissociation of a weak acid is related to its
Ka value as
(ii) If c 2 1 , then z 2 1 i

HA H A (iii) If c 2 1 , the equation has no solutions.


Initial conc. (M) 1.0 0 0 64. (b,d)
Eqm. conc. C(1– ) C C 8 x2 y2
Let y x c be tangent to 1
C .C C 2
9 14 19
or Ka
C(1 ) (1 ) 1 64 1 5
If a < < 1 and C = 1 mol then where c a 2 m2 b2
4 81 9 9
Ka = 2
or = (Ka)1/2
a2m b 2 2 1
Ratio of relative strengths of two acids depend So, points of contact are , ,
c c 5 5
on degree of dissociation i.e.,
2 1
K1
1/ 2
or ,
1 5 5
2 K2 65. (a,d)
61. (b) Common tangent of x 2
y 2
1& x 2
y 2
1 is 2 3 1
The commom ratio is 3 1 3 3
x 1 Equation of ellipse is given by
2
1 1 2 The sum to infinity is
x y 1 x 1
2 4 3 3
3
3x 2 4y 2 2x 8y 4 0 1 3 1 3 1 3
x 3 dx yx 2 dy
62. (b) ydx xdy 66. (b) v 1 50 10 t and v 2 30 10 t .
x2 y2
Hence, v cm 40 10 t .
xdx ydy ydx xdy
67. (d) Refer the figure:
x2 y2 x2

y
x2 y2 c
x b
63. (b,d) B Glass
A r
Air
Let z x iy , then r d
x2 y2 2i x iy 2c 1 i 0
ABcosr d ....... 1 y
70. (a) x 1t d
D
ABsin b.......... 2
1 tD 4 D
sin 90 sinr.......... 3 Therefore, y (given)
d d
4
b 19 Hence, t 4.8 m
Solving for d , we get d 1
8
68. (d) Energy of each wavelength is: 71. (a) Girgnard reagent + Acidic-Hydrogen = Alkane
1-alkynes are acidic
hc
E1 4eV ; Alcohols are acidic
310
carboxylic acids are acidic
hc
E2 3.1 eV ;
400 P1V1 P2 V2 P3 V3
72. (a) PT V1 V2 V3
hc
E3 1.55eV
800 1 2 2 1.5 2.5 1.5
2 1.5 2.5
As E1 & E2 are more than the work-function, they
PT 1.458
will eject photoelectrons.
Intensity of each wavelength = 1.2 10 3
Wm 2 2.303 P0
73. (c) K t
log
P0 x
Hence, no. of photo-electrons generated per sec
given t1/2 = 10.4 min, initial pressure P0 = 400 mm
1.2 10 3 1.2 10 3 4 of Hg pressure remaining P0 - x = 50 mm of Hg
= 1 10
4 1.6 10 19 3.1 1.6 10 19

0.693 2.303 400


log
4.3 10 . Hence in 2 sec, the number of
11
t1 2 t 50
photo-electrons generated is 8.6 × 1011.
0.0591 Zn 2
74. (b) Ecell E0cell log
69. (d) Fig-1: x 4 sin t n Cu 2
6
5
Given Ecell 1.1591 & E0cell 1.10
Fig-2: x 4 sin t ( from graph)
6
2
7 0.0591 Zn
Fig-3: x 4 sin t 1.1591 1.10 log
6 2 Cu 2

2
11 Zn
Fig-4: x 4 sin t 0.01
6 Cu 2

+
75. (d)
CH2
+
H CH3

OH O H O CH3

H+

O CH3
7. P(t2, 2t), t (0, 1] is any arbitrary point on y2 = 4x.
‘Q’ is the foot of perpendicular drawn from focus ‘S’
1. If n N > 1, then sum of real part of roots of zn = (z + 1)n to the tangent drawn at ‘P’. Maximum area of triangle
is equal to PQS is
n n 1 (a) 1 sq. units (b) 2 sq. units
(a) (b)
2 2 1
(c) sq. units (d) 4 sq. units
n 1 n 2
(c) (d)
2 2 8. If the line x + y = a, touches the parabola y = x – x2,
n then the point of contact is
n n
2. r Cr Cr 1 is equal to (a) (1, 0) (b) (0, 0)
r 1
(c) (2, –2) (d) (–2, –6)
(a) 2n + n + 1 (b) 2n – n + 1
9. The number of positive integral solutions of
(c) n – 2n + 1 (d) n – 2n – 1
x4 – y4 = 4721326 is
3. a, b are two mutually perpendicular unit vectors and c (a) 1 (b) 2
be a unit vector that is inclined at an angle to both a (c) 4 (d) 0
and b. If c x1 a x 2 b x3 a b , then 10. If {x} denotes the least integer, not less then x,
then total number of solutions of the equation
(a) x1 = x2 + 1 (b) x22 cos (x – 1)2 + {x} = 4, is equal to
(c) x32 1 2 x12 (d) x22 1 2 x32 (a) 1 (b) 2
(c) Zero (d) None of these
4. Range of f(x) = [cos–1{x}], where [.] and {.} denotes
the greatest integer function and fractional part 11. The number of solutions of the equation
respectively, is [sin–1x] = x – [x] is
(a) {0, 1} (b) {0, 1, 2} (where [*] denotes the greatest inteter function)
(c) {0, 1, 2, 3} (d) None of these (a) 0 (b) 1
(c) 2 (d) infinitely many
1/ x 2
e , x 0 12. If y = f(x) satisfies the condition
5. If (x) = , then f(x) is
0 , x 0 1 1
f x x2 (x 0) then f(x) equals
(a) Differentiable at x = 0 x x2
(b) Continuous but not differentiable at x = 0 (a) –x2 + 2 (b) –x2 – 2
(c) Discontinuous at x = 0 (c) x2 + 2 (d) x2 – 2
(d) None of these 1 1
x

6. The side of a triangle ABC are in A.P. (order being 13. xlim sin cos is
x x
2! 2! 1 8a (a) e (b) e2
a, b, c) and satisfy, , then
1!9! 3!7! 5!5! 2b ! 1
(c) (d) does not exist
the value of cosA + cosB is e
12 13 14. f(x) = 1 + 2x2 + 4x4 + 6x6 + ............... + 100x100 is
(a) (b) polynomial in a real variable x, the f(x) has
7 7
(a) neither a maximum nor a minimum
11 10
(c) (d) (b) only one maximum
7 7
(c) only one minimum
(d) one maximum and one minimum
1 22. A rigid circular loop of radius r and mass m lies in
15. If f(x) is a function satisfying f x 2 f(x) 0 for all the x-y plane on a flat table and has a current i flowing
x
cos ec in it. At this particular place, the earth’s magnetic
non-zero x, then f ( x )dx equals field is B B x i B z k. The value of i so that one edge
sin

(a) sin + cosec (b) sin2 of the loop lifts from the table is:
(c) cosec 2
(d) None of these mg mg
(a) (b)
1 1 r B 2
B 2 rB z
16. tan cos–1 x tan cos –1 x , x 0 x z

4 2 4 2 mg mg
is equal to (c) (d)
rB x r B xBz
(a) x (b) 2x
d
2 x 23. In Young’s double slit experiment 10 4 (d = distance
(c) (d) D
x 2 between slits, D = distance of screen from the slits).
17. If two roots of the equation x3 – px2 + qx – r = 0 are At a point P on the screen resulting intensity is equal
equal in magnitude but opposite in sign, then to the intensity due to individual slit I0. Then the
(a) pr = q (b) qr = p distance of point P from the central maximum is:
(c) pq = r (d) None of these ( = 6000 Å)
(a) 2 mm (b) 1 mm
18. If a1, a2...an are in A.P. with common difference d 0,
(c) 0.5 mm (d) 4 mm
then the sum of the series
24. Four equal charges of magnitude q each are placed
(sin d) [cosec a1 cosec a2 + cosec a2 cosec a3 + ...+
at four corners of a square with its centre at origin
cosec an–1 cosec an]
and lying in y-z plane. A fifth charge +Q is moved
(a) sec a1 – sec an (b) cosec a1 – cosec an along x-axis. The electrostatic potential energy (U)
(c) cot a1 – cot an (d) tan a1 – tan an varies on x-axis as:
19. If a1, a2, ..., an are in HP, then the expression U U
a1a2 + a2a3 + ... + an – 1 an is equal to
(a) (n – 1)(a1 – an) (b) na1an
(a) (b)
(c) (n – 1)a1an (d) n(a1 – an)
2 3 4
20. cos 0 cos cos cos cos x O x x O x
7 7 7 7
5 6 U U
cos cos
7 7
1 1 (c) (d)
(a) (b)
2 2
(c) 0 (d) 1 x O x x O x
25. Equations of a stationary and a travelling waves are as
21. A conducting circular loop of radius 'a' and resistance follows; y1 a sinkx cos t and y2 asin t kx
R is kept on a horizontal plane. A vertical time varying
magnetic field B = 2t is switched on at time t = 0. The phase difference between two points x1
3k
Then : 3
and x 2 are 1 and 2 respectively for the two
(a) power generated in the coil at any time t is 2k
constant 1
waves. The ratio is:
(b) flow of charge per unit time from any section of 2
the coil is constant 5
(c) total charge passed through any section between (a) 1 (b)
6
4 a2
time t = 0 to t = 2 is 3 6
R (c) (d)
4 7
(d) all of the above
26. A disc of radius 0.1 m rolls without sliding on a horizontal 30. Two particles of equal mass have velocities
surface with a velocity of 6 m/s. It then ascends a v1 2i m / s and v 2 2j m / s. First particle has an
smooth continuous track as shown in figure. The height
upto which it will ascend is : (g = 10 m/s2) acceleration a1 3i 3j m / s2 , while the acceleration

of the other particle is zero. The centre of mass of


the two particles moves in a:
(a) circle (b) parabola
(c) straight line (d) ellipse
31. The maximum kinetic energy of photoelectrons
emitted from a surface when photons of energy 6eV
6 m/s fall on it is 4eV. The stopping potential in volts is:
(a) 2 (b) 4
(a) 2.4 m (b) 0.9 m (c) 6 (d) 10
(c) 2.7 m (d) 1.8 m 32. A 100 W bulb B1, and two 60 W bulbs B2 and B3, are
27. A particle moves in the x-y plane with velocity connected to a 250 V source, as shown in the figure.
Now, W 1, W 2 and W 3 are the output powers of the
a 3 bulbs B1, B2 and B3 respectively. Then:
v ai bt j . At the instant t the magnitudes
b
of tangential, normal and total accelerations are: B1 B2

3 b
(a) b, and b respectively
2 2 B3
b b
(b) , and b respectively
2 2 250 V
(c) 2b, 2b and b respectively
(a) W 1 > W 2 = W 3 (b) W 1 > W 2 > W 3
(d) none of the above
(c) W 1 < W 2 = W 3 (d) W 1 < W 2 < W 3
28. In the arrangement shown in figure, coefficient of
33. A capacitor is filled with an insulator and a certain
1
friction between the two blocks is . The force potential difference is applied to its plates. The energy
2 stored in the capacitor is U. Now the capacitor is
of friction acting between the two blocks is:
disconnected from the source and the insulator is
F1 = 2N
2 kg pulled out of the capacitor. The work performed against
the forces of electric field in pulling out the insulator is
F2 = 20N 4U. Then dielectric constant of the insulator is:
4 kg
(a) 4 (b) 8
(a) 8 N (b) 10 N (c) 5 (d) 3
(c) 6 N (d) 4 N 34. In the circuit shown in figure, a conducting wire HE
29. A uniform sphere of radius R is placed on a rough is moved with a constant speed v towards left. The
horizontal surface and given a linear velocity v0 and complete circuit is placed in a uniform magnetic
angular velocity 0 as shown. The sphere comes to rest field B perpendicular to the plane of circuit inwards.
after moving some distance to the right. It follows that: The current in HKDE is:
× × H × × ×
A K

× × × × ×
v0 R C

0
× × × × ×

B D
× × E × × ×
(a) v 0 0R (b) 2v0 5 0R (a) clockwise (b) anticlockwise
(c) 5v0 2 0R (d) 2v 0 0R (c) alternating (d) zero
35. In the formula X = 3YZ2, X and Z have dimensions of 39. In the circuit shown in figure potential difference
capacitance and magnetic induction respectively. between A and B is:
What are the dimensions of Y in MKSQ system? E = 190V
(a) [M–3L–1T3Q4] (b) [M–3L–2T4Q4]
(c) [M–2L–2T4Q4] (d) [M–3L–2T4Q1]
36. A uniform disc of radius R lies in x-y plane with its C 3C
centre at origin. Its moment of inertia about the axis
x = 2R and y = 0 is equal to the moment of inertia
about the axis y = d and z = 0, where d is equal to: A
B
4 C 3C
R
17
(a) (b) R (a) 30 V (b) 60 V
3 2
(c) 10 V (d) 90 V
15 40. A radioactive substance is being produced at a
(c) 13 R (d) R
2 constant rate of 200 nuclei/s. The decay constant of
37. One mole of a monoatomic ideal gas undergoes the the substance is 1 s–1. After what time the number of
process radioactive nuclei will become 100. Initially there are
A B in the given P-V diagram. The specific heat no nuclei present
for this process is: 1
(a) 1s (b) s
ln 2
P
(c) ln (2) s (d) 2 s
B
6P0
41. The values of van der Waal's constant 'a' for the gases
O2, N2, NH3 and CH4 are 1.360, 1.390, 4.170 and
3P0 2.253 L atm.mol–2 respectively. The gas which can
A most easily be liquefied is
V (a) O2
V0 5V0 (b) N2
(c) NH3 (d) CH4
3R
(a) 42. The correct product of mono-nitration of
2
13R
(b) CH2–COO is
6
5R
(c) (a) CH2–COO
2
(d) 2R
38. Refraction takes place at a concave spherical NO2
boundary separating glass air medium. For the image
(b) CH2COO
3
to be real, the object distance g :
2
(a) should be greater than three times the radius of NO2
curvature of the refracting surface
(b) should be greater than two times the radius of (c) NO2 CH2COO
curvature of the refracting surface
(c) should be greater than the radius of curvature of
the refracting surface
(d) CH2COO NO2
(d) is independent of the radius of curvature of the
refracting surface
(c) I is more basic than II because six membered
43.
HCl
Major product: ring is more stable than five membered ring.
CCl4
(d) In III lone pair of electrons is present in sp orbital
Cl Cl but in I, II and IV lone pair of electrons is present
in sp2 orbital.
(a) (b)
49. Hydrogen bonding is maximum in
(a) ethyl chloride (b) triethylamine
Cl
(c) ethanol (d) diethyl ether

Cl 50. The wavelength of radiation emitted when an electron


(c) (d) in H-atom makes a transition from an energy level
with n = 3 to a level with n = 2 is
(a) 0.1527 Å (b) 6560 Å
44. The amount of water that should be added to 500
mol of 0.5 N NaOH solution to give a concentration (c) 6.56 Å (d) 1.527 Å
of 10 mg per ml is 51. Na2B4O7.10H2O Heat (X) + NaBO2 + H2O; (X) +
(a) 500 ml (b) 1000 ml
Cr2O3 Heat (Y) (Green coloured)
(c) 200 ml (d) 250 ml
(X) and (Y) are
45. For the reaction NH3 + OCl– N2H4 + Cl– occurring
in basic medium, the co-efficient of N2H4 in the (a) Na3BO3 and Cr(BO2)3
balanced equation is (b) Na2B4O7 and Cr(BO2)3
(a) 1 (b) 2 (c) B2O3 and Cr(BO2)3
(c) 3 (d) 4 (d) B2O3 and CrBO3

46. If Ka1 and K a2 are the ionization constants of 52. The edge length of a face centred cubic cell of an
ionic substance is 508 pm. If the radius of the cation
H3N+CHICOOH and H3N+CHICOO–, respectively, the is 110 pm, the radius of the anion is
pH of the solution at the isoelectric point is
(a) 144 pm (b) 288 pm
(a) pH pK a1 pK a2 (b) pH pK a1 pK a2 (c) 398 pm (d) 618 pm
1 (pK a1 pK a2 ) 53. The limiting molar conductivities for NaCl, KBr and
(c) pH (pK a1 pK a2 ) 2 (d) pH
2 KCl are 126, 152 and 150 S cm2 respectively. The
47. For the redox reaction, Zn(s) + Cu2+(0.1 M) for NaBr is
Zn2+(1 M) + Cu(s) taking place in a cell, Ecell is 1.10 (a) 128 S cm2 mol–1 (b) 302 S cm2 mol–1
RT (c) 278 S cm2 mol–1 (d) 176 S cm2 mol–1
volt. Ecell for the cell will be 2.303 0.0591
F 54. 1.520 g of hydroxide of a metal on ignition gave 0.995
g of its oxide. The equivalent weight of the metal is
(a) 2.14 volt (b) 1.80 volt
(a) 1.52 (b) 0.995
(c) 1.07 volt (d) 0.82 volt
(c) 190 (d) 9
48. The correct order of basicity is
55. Select the law that corresponds to data shown for
I II III IV the following reaction
N A+B Products
> > > Exp. [A] [B] Initial rate
N N N 1 0.012 0.035 0.1
H 2 0.024 0.035 0.1
H
3 0.012 0.070 0.8
Identify the correct statement. (a) rate = k[B]3
(a) In III, lone pair of electrons is involved in (b) rate = k[B]4
delocalisation but not in II.
(c) rate = k[A][B]3
(b) In IV, lone pair of electrons is involved in
(d) rate = k[A]2[B]2
delocalisation but not in III.
56. 2 moles of acetaldehyde are warmed in the presence 62. Which of the following characteristic features always
of 10% Ba(OH)2 to give a product which dehydrates holds true for the corresponding group of animals?
to a compound X. The compound on reduction with (a) Cartilaginous endoskeleton Chondrichthyes
NaBH4 will give (b) Viviparous Mammalia
(a) CH3CH = CHCHO Possess a mouth with
(c) Chordata
(b) CH3CH = CHCH2OH an upper and a lower jaw
3-chambered heart with
(c) CH3CH2CH2CH2OH (d) one incompletely divided Reptilia
ventricle
(d) CH3CH CH2CH2OH 63. You are given a tissue with its potential for
|
OH differentiation in an artificial culture. Which of the
following pairs of hormones would you add to the
57. The standard e.m.f. of a cell involving one electron
medium to secure shoots as well as roots?
charge is found to be 0.591 V at 25°C. The equilibrium
constant of the reaction is (F = 96500C mol –1; (a) Auxin and Abscisic acid
R = 8.314 JK–1 mol–1) (b) Gibberellin and Abscisic acid
(a) 1.0 × 101 (b) 1.0 × 1030 (c) IAA and Gibberellin
(c) 1.0 × 1010 (d) 1.0 × 105 (d) Auxin and Cytokinin
58. In a compound, element X occupy the corners, Y 64. Select the correct statement from the ones given
atoms occupies the body centred positions and Z below with respect to Periplaneta americana:
atoms at the centres of faces of the cubic unit cell. (a) Grinding of food is carried out only by the mouth
What is the empirical formula of the compound? parts
(a) XY2Z3 (b) Nervous system located dorsally, consists of
(b) XYZ3 segmentally arranged ganglia joined by a pair of
longitudinal connectives
(c) X2Y2Z3
(c) Males bear a pair of short thread like anal styles
(d) X8YZ6
(d) There are 16 very long Malpighian tubules present
59. The volumes of 4N HCl and 10N HCl required to make
at the junctions of midgut and hindgut
1 litre of 6N HCl are
65. When cell has stalled DNA replication fork, which
(a) 0.75 litre of 10N HCl and 0.25 litre of 4N HCl
checkpoint should be predominantly activated?
(b) 0.25 litre of 4N HCl and 0.75 litre of 10N HCl (a) M (b) Both G2/M and M
(c) 0.67 litre of 4N HCl and 0.33 litre of 10N HCl (c) G1/S (d) G2/M
(d) 0.80 litre of 4N HCl and 0.20 litre of 10N HCl 66. A and B cells are contiguous. Cell A has OP = 10 atm,
60. If is the degree of dissociation of Na2SO4 the van’t TP = 7 atm and DPD = 3 atm. Cell B has OP = 8 atm.
Hoff’s factor (i) used for calculating the molecular TP = 3 atm and DPD = 5 atm. The result would be:
mass is (a) No movement of water
(a) 1 + (b) Equilibrium between the two
(b) 1 – (c) Movement of water from A to B
(c) 1 + 2 (d) Movement of water from B to A
(d) 1 – 2 67. Which one of the following processes during
decomposition is correctly described?
(a) Leaching: Water soluble inorganic nutrients rise
61. Select the wrong statements: to the top layers of soil
(a) W.M. Stanley showed that viruses could be (b) Fragmentation: Carried out by organisms such
crystallized as earthworm
(b) The term ‘Contagium vivum fluidum’ was coined (c) Humification: Leads to the accumulation of a dark
by M.W. Beijerinek colored substance humus which undergoes
(c) Mosaic disease in tobacco and AIDS in human microbial action at a very fast rate
being are caused by viruses (d) Catabolism: Last step in the decomposition under
(d) The viroids were discovered by D.J. Ivanowsky fully anaerobic condition
68. Which one of the following statements is incorrect? 74. Restriction endonucleases:
(a) The competitive inhibitor does not affect the rate (a) Are used in genetic engineering for ligating two
of breakdown of the enzyme substrate complex DNA molecules
(b) The presence of the competitive inhibitor (b) Are used for in vito DNA synthesis
decreases the Km of the enzyme for the substrate (c) Are synthesized by bacteria as part of their
(c) A competitive inhibitor reacts with the enzyme defense mechanism
to form an enzyme inhibitor complex (d) Are present in mammalian cells for degradation
(d) In competitive inhibition, the inhibitor molecule of DNA when the cell dies
is not chemically changed by the enzyme 75. Lenticels are involved in:
69. A person with unknown blood group under ABO (a) Photosynthesis
system has suffered much blood loss in an accident
(b) Transpiration
and needs immediate blood transfusion. His one
friend, who has a valid certificate of his own blood (c) Gaseous exchange
type, offers for blood donation without delay. What (d) Food transport
would have been the type of blood group of the donor 76. The pyrenoids are made up of
friend? (a) Proteinaceous centre and starchy sheath
(a) Type A (b) Type B (b) Core of nucleic acid surrounded by protein sheath
(c) Type AB (d) Type O (c) Core of protein surrounded by fatty sheath
70. Emerson’s enhancement effect and Red drop have (d) Core of starch surrounded by sheath of protein
been instrumental in the discovery of:
77. Haemoglobin of human foetus:
(a) Photophosphorylation and non-cyclic electron
(a) Has two protein subunits instead of four
transport
(b) Has higher affinity of oxygen than that of the adult
(b) Two photosystem operating simultaneously
(c) Has lower affinity of oxygen than that of the adult
(c) Photophosphorylation and cyclic electron
transport (d) Its affinity for oxygen is the same as that of an adult
(d) Oxidative phosphorylation 78. Which of the following pairs of hormones are not
antagonistic (having opposite effects) to each
71. Which one of the following may require pollinators,
other?
but is genetically similar to autogamy?
(a) Apogamy (b) Cleistogamy a. Parathormone – Calcitonin
b. Insulin – Glucagon
(c) Geitonogamy (d) Xenogamy
c. Aldosterone – Atrial Natriuretic
72. When a neuron is in resting state i.e. not conducting factor
any impulse, the axonal membrane is:
d. Relaxin – Inhibin
(a) Comparatively more permeable to K+ ions and
79. A biologist studied the population of rats in a barn.
nearly impermeable to Na+ ions
He found that the average natality was 250, average
(b) Comparatively more permeable to Na+ ions and mortality 240, immigration 20 and emigration 30. The
nearly impermeable to K+ ions net increase in population is:
(c) Equally permeable to both Na+ and K+ ions (a) Zero
(d) Impermeable to both Na+ and K+ ions (b) 10
73. Satellite DNA in important because it: (c) 15
(a) Shows high degree of polymorphism in population (d) 05
and also the same degree of polymorphism in an 80. How many hot spots of biodiversity in the world have
individual, which are heritable form parents to been identified till date by Norman Myers?
children. (a) 34
(b) Does not code for proteins and is same in all (b) 43
members of the population (c) 17
(c) Codes for enzymes needed for DNA replication (d) 25
(d) Codes for proteins needed in cell cycle.
87. Two numbers x and y are chosen at random(without
replacement) from the numbers 1, 2, 3, ...2004. The
dx dx probability that x3 + y3 is divisible by 3 is
81. If Q = ax2 + bx + c, then I , , Qdx
Q Q 2 1
(a) (b)
Rule: Make the coefficient of x2 unity in Q and then 3 3
express it as the sum or difference of two squares. 1 1
(c) (d)
1 1 6 4
The value of the integral dx is
0 x2 2x cos 1 2 sin2
equal to 88. Let z cos i , then equation whose roots
9 9
(a) sin (b) sin are z + z3 + z5 + z7 and z2 + z4 + z6 + z8, is

(c) 2 sin sin


(d) (a) 4x2 + 4x + sec2 0
2 9
82. Given two circles intersecting orthogonally having
length of common chord 24/5 units. Radius of one of (b) 4x2 + 4x + cos2 0
the circle is 3 units. 9
Angle between direct common tangent is
(c) x2 + x + sec2 0
9
1 24 4 6
(a) sin (b) sin 1
25 25 (d) x2 + x + cos2 0
9
(c) sin 14 (d) sin 1 12
5 25 89. Let z = a + ib = (a, b) be any complex
83. Let f : N R be a function satisfying the following number, a, b R and –1 i. Let z 0 0i, arg z =
conditions, f(1) = 1 and f(1) + 2, f(2) + 3, f(3) + ... + n
lmz
f(n) = n(n + 1), f(n) for n 2. tan 1 , where – arg(z) and arg (z) +
Re z
1 if arg(z) 0
The value of f(1003) = , where K equals
K arg(–z)
– if arg(z) 0
(a) 1003 (b) 2003
On the basis of above information answer the following
(c) 2005 (d) 2006
questions.
/4 4
84. The value of the integral sin x + tan–3 xdx, If arg(z) < 0 then arg(z) – arg(–z) = 3 1
and if arg(z) >
4 0 then arg (–z) – arg(z) = 2 2 then
where sinx [ 1, 1] and tan x [0, 1] 5
(a) 1– 2 (b) 3 1 – 2 2 0
8 4 6
(a) (b)
3 3
(c) 2 – 1 (d) 2 1 – 3 2 0
4 8 6
(c) (d)
3 3 x 1
90. If I dx , then I equals
85. The number of terms common between the series x 3
1 + 2 + 4 + 8 + ... to 100 terms and 1 + 4 + 7 + 10 + ...
to 100 terms is x 1
(a) 2 x 1 4 tan 1 C
(a) 6 (b) 4 2
(c) 5 (d) None of these
1 x 1
86. The roots of equation ( C0 + 16C2 + 16C4 + ... + 16C16)x2
16 (b) 2 x 1 2 tan C
2
+ 2(16C1 + 16C3 + ... + 16C15)x + 32768 = 0 are
(a) –1, 1 x 1
(c) 2 x 1 4 tan 1 C
(b) –1, –1 2
(c) 1, 0 (d) None of these
(d) 1, 1
R2
A C

91. Figure shows a 2.0 V potentiometer used for the


determination of internal resistance of a 1.5 V cell.
The balance point of the cell in open circuit is 75
cm. When a resistor of of 0.5 is used in the external
circuit of the cell, the balance point shifts to 60 cm
length of the potentiometer wire. Length of wire AB
is 100 cm. B R1 D
2.0 V R1 is
9
(a) (b) 0.2
19
A B (c) 5 (d) 0.3

1.5 V G 95. In the figure mA = mB = 1 kg. Block A is neutral while


G qB = – 1C. Sizes of A and B are negligible. B is
released from rest at a distance 1.8 m from A. Initially
0.5
spring is neither compressed nor elongated.
When 0.5 W is used in the external resistance then N
terminal voltage of cell is E = 10
C
(a) 1.5 V (b) 1.2 V K = 18 N/m
A B
(c) 1.0 V (d) 1.5 V
92. The convex surface of a thin concavo-convex lens of smooth x = 0 x = 1.8 m x-axis
glass of refractive index 1.5 has a radius of curvature If collision between A and B is perfectly inelastic,
of 20 cm. The concave surface has a radius of what is velocity of combined mass just after collision?
curvature of 60 cm. The convex side is silvered and
(a) 6 m/s (b) 3 m/s
placed on a horizontal surface as shown in the figure.
(c) 9 m/s (d) 12 m/s
r =60 cm 96. In the figure shown a conducting wire PQ of length
l = 1 m, is moved in a uniform magnetic field B = 4T
r =20 cm with constant velocity v = 2 m/s towards right.
Given: R 2 , C = 1 F and L = 4H
The focal length of the combination has the magnitude
P
(a) 8.6 cm (b) 7.5 cm
(c) 1.5 cm (d) 15 cm
93. If various elements, i.e., resistance, capacitance and
inductance which are in series and having values R C L V
1000 , 1 F and 2.0 H respectively. Given emf as,
V = 100 2 sin1000 t volts ×B
Voltage across the inductor is
(a) 70.7 Volts (b) 100 Volts Q
(c) 141.4 Volts (d) 270.7 Volts Currents through resistance, capacitor and inductor
at any time t are I1, I2 and I3 respectively, Current
94. Two parallel vertical metallic rails AB and CD are
through wire PQ is I.
separated by 1m. They are connected at the two
ends by resistances R1 and R2 as shown in figure. Find the force required to move the wire with the
Horizontal metallic bar of mass 0.2 kg slides without given constant velocity of 2 m/s at t = 2s:
friction, vertically down the rails under the action of (a) 8 N
gravity. There is a uniform horizontal magnetic field (b) 16 N
of 0.6 T perpendicular to the plane of the rails. It is
(c) 24 N
observed that when the terminal velocity is attained,
the power dissipated in R1 and R2 are 0.76 W and (d) 32 N
1.2 W respectively
97. Consider the meter bridge shown in figure 102. Oxidation of amines gives different products depending
X 9 on the nature of amines and oxidising agent. Amines
are oxidised by KMnO4, H2O2 and H2SO5.

G
Secondary amine is also oxidised either by KMnO4
or H2O2
KMnO4
10 cm R – NH – R [P]
The resistance X has temperature coefficient 1 and R – NH – R H2 O2 [Q]
from RB [9 shown] has 2. For shown situation [P] and [Q] are respectively
balance point is at 10 cm from left end, if temperature
R R R
of system increases by T due to joule heating than
the shift in the balance point is [Assume that only (a) N–N and N – OH
the resistance of X and RB changes due to change
in temperature and there is no other effect] R R R
(a) 9( 1 2) T (b) 9( 1 2) T
R R R
1 1 (b) N – OH and N–N
(c) ( 2) T (d) ( 1 2) T
9 1 9
98. What is the radius of a steel sphere that will float on R R R
water with exactly half the sphere submerged? O
Density of steel is 7.9 × 103 kg/m3 and surface tension
(c) R – N – OH and R – NH
of water is 7 × 10–2N.
(a) 2.6 cm (b) 4.6 mm R R
(c) 1.2 mm (d) 6.5 mm O
99. A thin rod of length L and mass M is bent at its (d) R – N – H and R – N – OH
midpoint into two halves so that the angle between
them is 90°. The moment of inertia of the bent rod R R
about an axis passing through the bending point and 103. The gas phase decomposition 2N2O5 4NO2 + O2
perpendicular to the plane defined by the two halves follows I order rate law. At a given temperature the
of the rod is rate constant of the reaction is 23.03 × 10–3 s–1. The
initial pressure of N2O5 is 0.09 atm.
ML2 2 ML2
(a) (b) The total pressure after 200 seconds if the initial
6 24
2
pressure is 0.1 atm is
ML ML2
(c) (d) (a) 0.154 atm (b) 0.248 atm
24 12
(c) 0.174 atm (d) 0.114 atm
100. Heat is flowing through two cylinderical rods of the
same material. The diameters of the rods are in the 104. Five isomeric para-disubstituted aromatic compounds
ratio 1 : 2 and the lengths in the ratio 2 : 1. If the (A) to (E) with molecular formula C8H8O2 were given
temperature difference between the ends is same, then for identification. Based on the following observations
ratio of the rate of flow of heat through them will be give structures of the compounds :
(a) 2 : 1 (b) 8 : 1 (i) Both (A) and (B) form a silver mirror with Tollen's
reagent; also, (B) gives a positive test with FeCl3
(c) 1 : 1 (d) 1 : 8
solution but (A) not.
(ii) (C) gives positive iodoform test.
(iii) (D) is readily extracted in aqueous NaHCO 3
101. Given that the normal energy of the reactant and
solution.
product are 40J and 20J respectively and threshold
energy of the uncatalysed reaction is 120J. If the (iv) (E) on acid hydrolysis gives 1, 4-dihydroxybenzene.
rate of uncatalysed reaction at 400K becomes equal The compound (A) is:
to the rate of catalysed reaction at 300K, then what
CHO CHO
will be the activation energy of the catalysed forward
and backward reactions respectively?
(I) (II)
(a) 80J, 60J (b) 60J, 80J
(c) 80J, 100J (d) 50J, 70J CH2OH OCH3
CH2CHO COCH3 107. Following mechanism has been proposed for a
reaction,
(III) (IV)
2A + B D+E
OH OH A+B C+D ...... (Slow)
(a) I, II, III (b) I, II, III, IV A+C E ...... (Fast)
(c) I, II (d) I, IV
The rate law expression for the reaction is:
105. Some organic compounds contain double bonds. The
(a) r = K[A]2 [B] (b) r = K[A] [B]
simplest example is ethene, C2H4. Such compounds
are said to be unsturated, they contain less than the (c) r = K[A] 2
(d) r = K[A] [C]
maximum amount of hydrogen. Ethene for example 108. 1.1 mol of A is mixed with 2.2 mol of B and the mixture
can be converted into ethane C2H6. The reaction of is kept in one litre flask till the equilibrium is reached.
adding hydrogen to a double bond is known as At equilibrium, 0.2 mol of C is formed. If the
hydrogenation. The heat change in a hydrogenation equilibrium reaction is A + 2B 2C + D, the value of
reaction is the enthalpy of hydrogenation. equilibrium constant is
H H H (a) 0.002 (b) 0.004
H H
H H H2 H H (c) 0.001 (d) 0.003
H 109. For the reaction
H H H H
H H H H H H 2A + B + C A2B + C
Cyclohexene Cyclohexane Rate = k[A][B]2 where k = 2.0 × 10–6 M–2 s–1
Initial concentrations of A, B and C are 0.1 M,
H H
0.2 M and 0.8 M respectively. If the rate of reverse
3H2 reaction is negligible, the rate of reaction when [A]
H H has reached 0.06 M is

H (a) 2.59 × 10–9 Ms–1


H
(b) 3.89 × 10–9 Ms–1
Benzene (c) 1.3 × 10–9 M s–1
Calculate the enthalpy of the following reaction (d) none of these
CH2 CH2 g H2 g CH3 CH3 g 110. The correct order of increasing basicity of the following
is
The bond energies of C H,C C,C C and H H
H2O, OH–, –C2H5OH, –C2H5O–
are 414, 347, 615 and 435 kJ mol–1 respectively.
(a) –C2H5OH < H2O < OH– < –C2H5O–
(a) –125 kJ mol–1 (b) 125 kJ mol–1
(b) H2O < –C2H5OH < –C2H5O– < OH–
(c) –733 kJ mol–1 (d) +733 kJ mol–1
(c) H2O < –C2H5OH < OH– < –C2H5O–
106. The colligative properties of the dilute solutions used
for the calculation of molecular weight of the solute (d) –C2H5OH < H2O < –C2H5O– < OH–
which is often called observed molecular mass. It
may be same as the theoretical molecular mass if
the solute behaves normally in solution. In case it 111. If two persons with ‘AB’ blood group marry and have
undergoes association or dissociation the observed sufficiently large number of children, these children
molecular mass gives different results. The nature of could be classified as ‘A’ blood group: ‘AB’ blood
solute in solution is expressed in terms of van't Hoff group : ‘B’ blood group in 1 : 2 : 1 ratio. Modern
factor (i) which may be 1, less than 1 or more than 1. technique of protein electrophoresis reveals presence
of both ‘A’ and ‘B’ type proteins in ‘AB’ blood group
0.004 M solution of Na2SO4 is isotonic with a 0.01 M
individuals. This is an example of:
solution of glucose at the same temperature.
(a) Complete dominance
The apparent degree of dissociation of Na2SO4 is
(b) Co dominance
(a) 25% (b) 50%
(c) Incomplete dominance
(c) 75% (d) 85% (d) Partial dominance
112. In a normal pregnant woman, the amount of total (c) The typhlosole greatly increases the effective
gonadotropin activity was assessed. The result absorption area of the digested food in the
expected was: intestine
(a) High level of circulating HCG to stimulate (d) The S-shaped setae embedded in the integument
estrogen and progesterone synthesis are the defensive weapons used against the enemies
(b) High level of circulating FSH and LH in the uterus 117. Match column-I with column-II and select the correct
to stimulate implantation of the embryo option using the codes given below:
(c) High level of circulating HCG to stimulate Columnn-I Columnn-II
endometrial thickening (A) Pistiles fused together (i) Gametogenesis
(d) High levels of FSH and LH in uterus to stimulate (B) Formation of gametes (ii) Pistillate
endometrial thickening Hyphae of higher
(C) (iii) Syncarpous
113. Select the correct matching of hormone, its source Ascomycetes
and function: (D) Unisexual female flower (iv) Dikaryotic
Hormone Source Functions (a) (A) – (iii), (B) – (i), (C) – (ii), (D) – (iv)
a. Vasopressin Posterior Increased loss of (b) (A) – (iii), (B) – (i), (C) – (iv), (D) – (ii)
pituitary water through
(c) (A) – (i), (B) – (iii), (C) – (iv), (D) – (ii)
urine
b. Norepine- Adrenal Increased heart (d) (A) – (iii), (B) – (ii), (C) – (i), (D) – (iv)
phrine medulla beat, respiration 118. Which one of the following preceeds re-formation
and alertness of the nuclear envelope during M phase of the cell cycle:
c. Glucagon -cells of islet Stimulates
(a) Transcription from chromosomes and reassembly
of langerhans glycogenolysis
of the nuclear lamina
d. Prolactin Posterior Regulates growth
pituitary of mammary (b) Formation of the contractile ring and formations
glands and milk of the phragmoplast
formation in (c) Formation of the contractile ring and transcription
females from chromosomes
114. Geitonogamy involves: (d) Decondensation from chromosomes and
(a) Fertilisation of a flower by the pollen from a reassembly of the nuclear lamina
flower of another plant belonging to a distant 119. The eye of octopus and eye of cat show different
population patterns of structure, yet they perform similar
(b) Fertilisation of a flower by the pollen from another function. This is an example of:
flower of the same plant (a) Analogous organs that have evolved due to
(c) Fertilization of a flower by the pollen from the divergent evolution
same flower (b) Homologous organs that have evolved due to
(d) Fertilization of a flower by the pollen from a flower convergent evolution
of another plant in the same population (c) Homologous organs that have evolved due to
divergent evolution
115. Which is incorrect?
(d) Analogous organs that have evolved due to
(a) Presence of non respiratory air sacs increases
convergent evolution
efficiency of respiiration in birds
120. Common cold differs from pneumonia in, that:
(b) In insects, circulating body fluids serve to
distribute oxygen to tissues (a) Pneumonia pathogen infects alveoli whereas the
common cold affects nose and respiratory
(c) Principle of counter-current flow facilitates efficient passage but not the lungs
respiration in gills at fishes
(b) Pneumonia is a communicable disease whereas
(d) Residual air in lungs slightly decreases the the common cold is a nutritional deficiency disease
efficiency of respiration in mammals
(c) Pneumonia can be prevented by a live attenuated
116. One very special feature in the earthworm (Pheretima) bacterial vaccine whereas the common cold has
is that: no effective vaccine
(a) It has a long dorsal tubular heart (d) Pneumonia is caused by a virus while the
(b) Fertilization of eggs occurs inside the body common cold is caused by the bacterium
Haemophilus influenzae
1. (d) 2. (c) 3. (c) 4. (a) 5. (a) 6. (a) 7. (c) 8. (a) 9. (d) 10. (a)
11. (b) 12. (d) 13. (a) 14. (c) 15. (d) 16. (c) 17. (c) 18. (c) 19. (c) 20. (d)
21. (d) 22. (c) 23. (a) 24. (b) 25. (d) 26. (d) 27. (a) 28. (a) 29. (c) 30. (c)
31. (b) 32. (d) 33. (c) 34. (d) 35. (b) 36. (b) 37. (b) 38. (a) 39. (c) 40. (c)
41. (c) 42. (d) 43. (a) 44. (a) 45. (a) 46. (d) 47. (c) 48. (b) 49. (c) 50. (b)
51. (c) 52. (a) 53. (a) 54. (d) 55. (a) 56. (b) 57. (c) 58. (b) 59. (c) 60. (c)
61. (d) 62. (a) 63. (d) 64. (c) 65. (c) 66. (c) 67. (b) 68. (b) 69. (d) 70. (b)
71. (c) 72. (a) 73. (a) 74. (c) 75. (c) 76. (a) 77. (b) 78. (d) 79. (a) 80. (a)
81. (d) 82. (b) 83. (d) 84. (d) 85. (c) 86. (b) 87. (b) 88. (a) 89. (b) 90. (c)
91. (b) 92. (b) 93. (c) 94. (a) 95. (b) 96. (d) 97. (a) 98. (c) 99. (d) 100. (d)
101. (b) 102. (a) 103. (b) 104. (c) 105. (a) 106. (c) 107. (b) 108. (c) 109. (b) 110. (c)
111. (b) 112. (a) 113. (c) 114. (b) 115. (b) 116. (c) 117. (b) 118. (b) 119. (d) 120. (a)

2
| a |2 | b |2 2a .b cos2 x 32 1 a .b
1. (d) zn = (z + 1)n, this equation will have exactly
(n – 1) roots. 1 = 2 cos2 + x 32

z 1
n z 1
1 x 32 = 1 – 2 cos2 = 1 – 2 x12
We have, 1 z |z + 1| = |z|
z

‘z’ lies on the right bisector of the segment 4. (a) Range of cos–1{x} is 0, Range of [cos–1{x}]
2
connecting the points (0, 0) and (–1, 0) is {0, 1}
1 e 1/ h2
0 e 1/ h
2 1
Thus, Re(z) = . Hence roots are collinear and 5. (a) f' Lim Lim Lim e h2
lnh
2 h 0 h h 0 elnh h 0
1
will their real part equal to . 1
2 as h 0, ends to at a much faster rate
h2
1 1
Hence, sum of real parts of roots n 1.
as compared to –lnh h2
lnh
=0
2 Lim e
h 0
n n n

r. n Cr n
Cr r. nCr r 1 1 nCr
Thus, f(x) is differentiable at x = 0
2. (c) 1 1
r 1 r 1 r 1 2! 2 1 2 10 2 10 1 10
6. (a) C1 C3 C5
n n n 1!9! 3!7! 5!5! 10! 10! 10!
r. nCr r 1 nCr n
Cr
1 1
1 10 10 10 10 10
r 1 r 1 r 1 C1 C9 C3 C7 C5
10!
n 1

k. nCk 2n 1 1 10 29 8a
= n.2n–1 – 2 1 given .
k 0 10! 10! 2b !
= n.2n–1 – (n.2n–1 – n.nCn) – (2n – 1) = n – 2n + 1 3a = 9, 2b = 10 a = 3, b = 5
3. (c) a .b 0, a.c b.c cos Also a, b, c are in A.P.
c x1 a x 2b x3 a b c.a x1a.a x1 cos c = 7.
b2 c 2 a2 25 49 9 13
Also, c .b x 2 b .b x2 cos cos A
2bc 2.5.7 14
c a b cos x3 a b a2 c 2 b2 9 49 25 11
cosB
2ac 2.3.7 14
Now, | c |2 a b cos x3 a b 13 11 24 12
cosA + cosB =
14 14 7
. a b cos x3 a b
y
7. (c) ‘Q’ will clearly lie on y-axis.
y
4

3
P
2
Q

x
O S(1, 0) x
–4 0
–3 –2 –1 1 2 3 4
–1

–2

–3

Equation of PQ is
yt = x + t2 11. (b) [sin–1x] = x –[x]
[sin–1x] = {x}
Q = (0, t) PQ t4 t2 t 1 t2
–2, –1, 0, 1
1 t2 1 1 1
QS = PQS = 2 PQ.QS 2 t 0 {x} 1
1 t2 1 t2
which is increasing function of ‘t’ possible only if [sin–1x] = 0 & {x} = 0

1 0 sin–1 x 1; x=0
PQS|max = sq. unit
2 0 x sin1
8. (a) a – x = x – x2 should have equal roots common solution is x = 0
x2 – 2x + a = 0 should have equal roots 12. (d) y = f(x)
x2 – 2x + a = 0 should have equal roots 1 1
f x x2 (a 0)
4 = 4a x x2
2
a=1 1 1
f(x) = x2 – 2
f x x 2
x x
and that equal root is
2 13. (a) 1 From
x 1
2 1 1
l lim x sin cos 1
x = 1, y = 0 x x x

9. (d) Given equation is x4 – y4 = 4721326 1


put x = t
Here RHS is an even integer
1 1 t t t
LHS is also even integer. So, either both x, y = lim [sin t cos t 1] = lim
t 0 t
2 sin cos 2sin2
t 0 t 2 2 2
are even or both are odd integers.
1 t t 1
Now, x4 – y4 = (x – y)(x + y)(x2 + y2) = lim
t 0 t
2 sin cos
2 2
sin
2
= 1 = e1
Here, x – y, x + y, x2 + y2 are all even integers. 14. (c) f(x) = 1 + 2x2 + 4x4 + 6x6 + ... + 100x100
(when both x, y are even or both x, y are odd) f (x) = 4x + 16x3 + 36x5 + ... + (100)2 x99
Thus, (x – y)(x + y)(x + y ) must be divisible by 8.
2 2
f (x) = x(4 + 16x2 + 36x4 + ...+ (100)2 x98)
But RHS is not divisible by 8. f (x) = 0
Given equation has no solution. x=0
10. (a) Let x (I –1, I), I integer
Then {x} = I x = 0 is minima point
0
We have; {x} = 4 – (x – 1)2 1 1
15. (d) f(x) = f
Adjacent figure shows that graphs of y = 4 – (x – 1)2 x2 x
and y = {x} meet exactly once. Thus there is cos ec cos ec
1 1
I f(x)dx f dx
only one solution. sin sin
x2 x
1 1 2 3 3 3 2
t dx dt 1 cos cos cos cos cos cos
x x2 7 7 7 7 7 7 7
sin cos ec
=1
I f(t)dt f(t)dt
cos ec sin dB
21. (d) e = ( a2) =( a2)(2); Induced emf e = 2 a2
2I = 0 I=0 dt
Induced current or flow of charge per unit time
1 cos 1 x
16. (c) tan cos 1
x tan e
4 2 4 2 through any section of the coil is i R or
Let cos–1x x cos
2 a2
2 2 i = constant
R
1 tan 1 tan
2 2
tan tan Charge passed through any section between
4 2 4 2 1 tan 2
2 t = 0 and t = 2s is

2 sec 2 2 a2 4 a2
2 2 2 2 q it 2 or q
R R
cos x
1 tan2 cos2 sin2
2 2 2 Further P = i2R = constant (as i is constant)
17. (c) x3 – px2 + qx – r = 0 22. (c) The torque on the loop must be equal to the
p Satisfy given equation gravitational torque exerted about an axis tangent
p3 – p3 + pq – r = 0 to the loop.
pq = r The gravitational torque:
= mgr ...(1)
18. (c) d = (a2 – a1) = (a3 – a2) = ... = (an – an–1 0) 1
Only Bx, causes a torque. Therefore torque to
sind[cosec a1 cosec a2 + coseca2 cosec a3 + ...
the magnetic field
+ cosec an–1 cosec an]
2 |M B| = MB sin 90° = r2iBx ...(2)
sind sind sin d
= .. mg
sina1 sina2 sina2 sina3 sinan 1 sinan i
Eqs. (1) and (2), we get, rB x
n n
sin(ar ar 1) sin(ar cosar 1 cosar sinr 1
r 2 sina r 1 sinar r 2
sinar 1 sinar sinar 1 sinar 23. (a) I 4I0 cos2 ; I0 4I0 cos2
2 2
n
(cot ar cot ar ) cot a1 cot a n 1
r 2
1 cos
2 2
or 2 2
19. (c) a1, a2, ..., an in H.P.
2 2 1 1 d yd
1 1 1
or x or y x
, ,..., 3 3 D D
a1 a2 an in A.P..
7
6 10 3
a1 a2 a 2 a3 a an y 2 10 m 2 mm
d ... n 1 d 3 10 4
a1a2 a 2a 3 an 1an 3
D
a1a2 + a2a3 + ...+ an–1an 24. (b) At the centre of square net force on +Q is zero.
d
But at x = 0, it is in unstable equilibrium position
[a a a2a3 ... an 1an ] i.e., potential energy is maximum.
d 1 2
1 a1 an 3
[(a a2 ) (a2a3 ) ... (an 1an )] 25. (d) At x1 and x2
d 1 d 3k 2k

1 1
sinkx1 or sinkx2 is not zero.
(n 1)d
an a1 Therefore, neither of x1 or x2 is a node
(a1 an ) 3 1 7
a1 an = (n – 1)a1an x x2 x1
(n 1)d 2 3 k 6k
a1an d
2 2
20. (d) cos0 cos cos
2
cos
3
cos
4
cos
5
cos
6 Since x ; x k
k k 2
7 7 7 7 7 7
7 1 6
1 cos cos
2
cos
3
cos
3
cos
2 Therefore, 1 and 2 k x ; 7
6 2
7 7 7 7 7
NOTE: In case of a stationary wave phase
cos difference between any two points is either zero
7
or .
26. (d) Let m be the mass of the disc. Then translational v0 a v0
Now, t 0
or
kinetic energy of the disc is: a 0

1 2R v0
Kr mv 2 (1) ; 5v 0 2 0R
2 5 0

When it ascends on a smooth track its rotational m1v1 m2 v 2


kinetic energy will remain same while 30. (c) v com
m1 m2
translational kinetic energy will go on decreasing.
v1 v2
At highest point. m1 m2
2
2
v2 6
Kr = mgh or 1 mv 2 mgh or h 1.8 m (i j)m / s
2 2g 2 10

27. (a) v aj bt j ; a bj ; Similarly acom a1 a2 3


(i j)m / s2
2 2
1/ 2
Total acceleration = b; speed, v a2 b2 t 2 Since, v com is parallel to acom the path will be a
straight line.
Tangential acceleration
31. (b) Stopping potential is the negative potential applied
dv 1 2 1/ 2
a b2 t 2 2b2t to stop the electrons having maximum kinetic
dt 2
energy. Thereore, stopping potential will be 4V.
3a 3b
At t ; Tangential acceleration V2 V2
b 2
32. (d) P ; So, R
R P
3b2 b
and normal acceleration b2 V2 V2
4 2 R1 and R2 R3
100 60
28. (a) Free body diagram of the two bodies are as
follows (250)2
Now, W1 .R1
F1 = 2N (R1 R 2 )2

(250)2 2
f W2 2
.R2 and W3 (250)
(R1 R2 ) R3
f
F2 = 20N W 1 : W 2 : W 3 = 15 : 25 : 64 or W 1 < W 2 < W 3
33. (c) Let C0 be the capacity of capacitor without
Let acceleration of both the blocks towards left
insulator and C its capacity with insulator. Then
is a. Then
C = kC0 (k = dielectric constant)
f 2 20 f
a or 2f – 4 = 20 – f or f = 8N charge q remains constant.
2 4
Maximum friction between the two blocks can 1 q2 ...(1)
U
be: 2 C
2 2
fmax mg (m = 2 kg) and U 4U 1 q or 5U 1 q ...(2)
2 C0 2 C0
= (0.5) (2) (10) = 10 N
From Equation (1) and (2), we get
Now since f fmax C
5 or K 5
Therefore, friction force between the two blocks C0
is 8N. 34. (d) Potential difference across capacitor
29. (c) f mg, a g V = Bvl = constant
mgR 5g Therefore, charge stored in the capacitor is also
2 2 R constant.
mR2
5
Thus, current through the capacitor is zero.

X Capaci tance
35. b Y
Z2 Magnetic induction
2

a
M 1L 2Q2T 2
M 3L 2Q4 T 4
M2Q 2T 2

f = mg
36. b An axis passing through x = 2R, y = 0 is in E = 190V
direction as shown in figure. Moment of inertia 15
C C
about this axis will be: 4
1 2 9
I1 mR2 m 2R mR2 ...(1)
2 2 V1 V2
Axis passing through y = d, z = 0 is shown as 15
C 4
dotted line in figure. Moment of inertia about this V1 4 15 ; V2 190 40V
19
axis will be: V2 C 4
Now this 40V is distributed in C and 3C in the
Y = d, z = 0 3
ratio of
1
VAB = 10 V
40. (c) Let N be the number of nuclei at any time t. Then
x dN
200 N;
dt
x = 2R, N t
dN 200
dt or N 1 e t
y=0 200 N
0 0

Given that N = 100 and = 1s–1


2R r
t 1
100 = 200(1 – e–t) or e 2 ;
1 (2)
I2 mR2 md 2
4 t = 1n (2) sec.
1 9 44. (a) Conc. of final solution (NaOH) = 10 mg per ml
Eqs. (1) and (2), we get, 4 mR2 md2 2
mR2 or
= 10 g per litre = 10 equivalent per litre = 0.25 N
17 40
d R
2 Conc. of initial solution of NaOH = 0.5 N
Q 1 W V1N1 = V2N2
37. (b) Specific heat C = U W CV
T T T
V1N1 500 0.5
9P0 or V2 = N2 0.25
1000 ml
For the given process W 4V0 18P0 V0
2
V2 – V1 = 1000 – 500 = 500 ml
(6P0 )(5V0 ) (3P0 )V0 27P0 V0
Also, T T2 T1
Hence 500 ml of water should be added to 500
R R R
3 3R 2R 13R ml of 0.5 N NaOH to get the NaOH solution with
and CV R; C
conc of 10 mg/ml.
2 2 3 6
38. (a) 45. (a)
–3 –2
2 1 NH3 N2H4(Oxidation)
2NH3 N2H4 + 2e–
Object 2NH3 + 2OH– N2H4 + 2H2O + 2e–

+1
OCl Cl–1 (Reduction)
Applying, 2 1 2 1
+1
v u R OCl + 2e– Cl–1
1 1.5 1 (1.5) 1 3 1 OCl– + H2O + 2e Cl– + 2OH–
or
v ( u) R v 2u 2R (in basic medium, OH– will be either in reactant
1 3 side or product side)
For v to be positive 2R 2u or u > 3R
Adding (1) and (2),
39. (c) E = 190V E = 190V
2NH3 + OCl– N2H4 + H2O + Cl–
Thus the coefficient of N2H4 is one.
C 3C C 3C
0.0591 [Zn2 ]
47. (c) E = E° – log [Here n = 2]
2 [Cu2 ]
A B
B
C 3C 3 E = 1.1 V – 0.0591 V log 1
C 2 0.1
4
= (1.1 – 0.02955)V = 1.07045V 1.07 V
48. (b) IV is least basic because lone pair of electrons 83. (d) Here, f(1) + 2f(2) + 3f(3) + ...
is involved in delocalisation. In III, lone pair is + nf(n) = n(n + 1) f(n), for n 2 ... (1)
present in sp2 hybridised nitrogen but in I and II
Replacing n by n + 1, we get
lone pair is present in sp3 hybridised nitrogen.
Six membered ring has greater +I effect than five f(1) + 2f(2) + 3f(3) + ... + (n + 1) f(n + 1)
membered ring. = (n + 1)(n + 2) f(n + 1) ... (2)
49. (c) Strength of H-bond depends upon the From (2) – (1), we get
electronegativity of the atom carrying the H-atom. (n + 1) f(n + 1) = (n + 1) {(n + 2) f(n + 1) – nf(n)}
Since O is more electronegative than N, H- nf(n) = (n + 1) f(n + 1)
bonding is maximum in ethanol.
Putting n = 2, 3, 4, ..., we get
50. (b) 1 1 1
v R 2f(2) = 3f(3) = 4f(4) = ... = nf(n)
n12 n22
From (1), f(1) + 2f(2) + 3f(3) + ... + nf(n) = n(n +
1 1
= 1.0974 ×107 22 32 = 0.1525 × 107 m–1 1)f(n)
f(1) + (n – 1).nf(n) = n(n + 1)f(n)
1
6.56 10 7 m
0.1525 107 f(1) = 2nf(n)
= 6560Å [1 Å = 10–10 m] f(n) = f(1) 1
2n 2n
1 1
f(1003) =
1 dx 2(1003) 2006
81. (d) I 2
1
x cos sin2
/4 /4 1 1
84. (d) [sin–4 x tan–3 x]dx dx 4 dx
1 x cos
1
4 4 sin4 x tan3 x
1 4
I tan
sin sin 0 1 1
[Here is even function and is odd
1 1 cos1 cos 1
sin4 x tan3 x
tan tan function]
sin sin sin
/4 1 /4
1 Now, cos ec 2 x dx cot3 (x)dx 0
tan 1 cot tan 1 cot sin x 2
sin 2 4 4
/4
1 2 (cos ec 2 x).cosec 2 xdx
0
sin 2 2 2 2sin
a a a
/2 From property, f(x)dx f(x)dx f(–x)dx
82. (b) B
a 0 0
A cot x = t
r1 r2
/2 –cosec2x + dx = dt
C1 C2
cosec2x dx = –dt
lim t to 1
d d /4
We have sin ,cos , (where 2d = length of 2 (1 cot 2 x)cosec 2x dx
r1 r2 0
common chord) 3 1
= –2 (1 t2 )dt –2 t t
1
d d r1r2 2r1r2 24 3
1 d 2d ,
r12 r22 r 2
r2
r2
r2 5
1 2 1 2 1 8
–2 1 0
3 3
where r1 3
85. (c) For GP tn = 2n–1
6r 24
d r2 4
9 r22 5 for AP tn = 1 + (m – 1)3 = 3m – 2

r2 r1
For common terms, we are required to choose
From the figure sin value of m such that 3m – 2 = 2n–1
2 C1C2

Where C12C22 r12 r22 i.e. 3m – 2 = 1, 2, 4, 8, 16, 32, 64, 128, 256
C1C2 5
4 10 34 130
1 24 1 24 4 6 m 1, , 2, , 6, , 22, , 86
sin cos sin 2 3 3 2 3
2 5 2 5 5 5 5
[we are not required to go further as t100(AP) = 298)]
4 6 1
sin
5 m = 1, 2, 6, 22, 86 (possible values)
Also AB C1C 2
2 r1 r2
2
25 1 24 Hence, there are five common terms.
86. (b) 16
C0 + 16C2 + ... + 16C16 = 216 – 1 = 215 = 32768 3 1 –
16
C1 + 16C3 + ... + 16C15 = 216 – 1 = 215 = 32768
For arg z < 0 we have arg(z) + arg(–z) =
Thus, the given quadratic equation become
and 2 2 = arg(–z) – arg(z) = arg(–z) + arg(z)
32768x2 + 2(32768)x + 32768 = 0
2 –
x2 + 2x + 1 = 0 2

(x + 1)2 = 0 3 1 –2 2 0...

x = –1, –1 whereas, 2 1 – 3
5
0
2
6
87. (b) Let P1 = 1, 4, 7, ..., (n elements), where 3n =
2004 5
1 – 2 and 2 – 1
6 6 6
P2 = 2, 5, 8, ...(n numbers)
x 1
P3 = 3, 6, 9, ...(n numbers) 90. (c) I dx
x 3
If X and Y belong (P1, P2), (P2, P1) or (P3, P3)
Let x 1 t 2 dx 2t dt
Favourble outcomes = nC2 + n × n
t t2 4 4
2
2t dt 2 dt
n(n 1) 2 3n n t2 4 t2 4 t2 4
n
2 2
1 t
Possible outcomes = 3nC2 2t 4 tan
2
c

(3n)(3n 1) 9n2 3n 3(3n2 n) 91. (b) When the circuit is closed, the balance point is
2 2 2 at 60 cm
3n2 n 60
1 terminal voltage of the cell 2 1.2V
Required probability 2 100
2 3
1 1 1
3n2 n 92. (b) Use the equation .....
3 f f1 f2
2
88. (a) The equation whose roots are and is to find the equivalent power of the combination,
with proper sign. Then use the equation of the
x2 – sx + p = 0 lens/ mirror as appropriate.
n
8
2 2 93. (c) P.D. across inductor
sum cos isin –1
n 1 9 9
VL irms XL 0.0707 1000 2 141.4 volts
8 n
2 2 94. (a) At terminal velocity magnetic force = weight
cos isin 0
n 0
9 9
BIl mg i.e. I 0.2 9.8 9.8 A
p = (z + z3 + z5 + z7)[z2 + z4 + z6 + z8] 0.6 1 3
Now if e is the emf induced in the rod.
= z.z2(1 + z2 + z4 + z6)2 = z3(1 + z2 + z4 + z6)2 0.76 1.20
2 e I P P1 P2 ; e 0.6V
3 1 z8 z 9.8 3
z
1 z2 z2 2z 1 Further, as in case of Joule heating
1 1 1 V2 V2
1 2 2 P i.e., R ; And as here, V1 = V2 = e
z 2 2cos 2 2 1 cos R P
z 9 9 2
e2 0.6 9
1 1 1 So, R1
sec 2 P1 0.76 19
4 9
2 2cos2 4 cos2
9 9 95. (b) Electrostatics force on A is zero, while on B is
Equation is x – sx + p = 0 2 |F B| = |qB| E = (1) (10) = 10 N (along negative
1 x-direction)
i.e., x2 + x +4 sec2 =0 4x2 + 4x + sec2 0
9 9 FB
aB = 10 m/s2
89. (b) To solve this problem use properties of argument mB

such as Just before collision,


arg(z) + arg(z) = 0 for arg z < 0. vB 2aBs 2 10 1.8 = 6 m/s
Now, from conservation of linear momentum,
3 1 = arg(z) – arg(z), were arg(z) < 0
velocity of combined mass will be,
= – arg(z) – arg(–z) {arg(z) + arg(–z)} (mA + mB)v = mBvB or v = 3 m/s
96. (d) Fapplied (rightwards) = Fm (leftwards) 103. (b) After 200 s, log p0 kt
= IlB = (I1 + I2 + I3)lB p 2.303

23.03 10 3 200
I3 we have already calculated at 2 which is 4A. = 200 × 10–2 = 2 i.e., p0 100
2.303 p
dq d d
I2 (CVc ) (8) 0 p = p0 × 0.01
dt dt dt
VR 8 pN2O5 (p0 2x) p0 0.01
and I1 4A
R 2
1
x p0 (1 0.01) = 0.495 p0 = 0.0495
Fapplied = (4 + 4) (1) (4) = 32 N 2
X l Total pressure = (p0 + 3x)
97. (a) From balance condition,
Y 100 l = p0 + 3 × 0.0495 = 0.1 + 0.1485 = 0.2485 atm
for given situation, Y 9 and l = 10 cm
CHO CHO
dX dY dl dl
X Y l 100 l
104. (c) , presence of aldehyde group
As error sign is known or we can say these are
systematic error we will substitute them with sign.
CH 2OH O CH 3
1 1
1 T 2 T dl dl 9( 1 2) T
10 90 105. (a) – 125 kJ
98. (c) Surface tension force + upthrust = weight 106. (c) Isotonic solutions 1 = 2
2 3 4 i1c1 = i2c2
(2 rT) r wg r3 sg
3 3
i1(0.004) = 1 × (0.01)
Substituting the values, we get
0.01
r 1.2 mm i1 2.5
0.004
101. (b) In absence of catalyst = In presence of catalyst i 1 2.5 1 1.5
0.75
Ea Ea 80 Ea n 1 3 1 2
T1 T2 400 300 % = 75%
Ea 60J Eb 60 20 80J 107. (b) Slow step is RDS; r = K[A][B]
R
102. (a) 2R – NH – R KMnO4
N–N–R
R
R
[P] tetra alkyl hydrazine
R
R – NH – R H2O2
N – OH
R
dialkyl hydroxyl amine

You might also like